Download as pdf or txt
Download as pdf or txt
You are on page 1of 273

Preface

It requires a great deal of time patience and perseverance to understand

thoroughly the intricacies of any subject of law particularly the law of Income Tax.

Therefore in these notes every care and effort has been taken to present the subject in

the simplest possible language with illustrations and test cases.

It took a great deal of time to prepare these notes as it was really challenging to

present such a complicated subject in a very concise, compact & Crisp manner.

The response for the notes has been super-duper positive from the students and few of

my Students recently secured AIR – 7th,8th, 11th, 12th, 14 , 17 and 19th in CS


th th

Executive. The plus point of this edition is that almost all questions of the institute’s

material have been covered and also the questions of other professional institutes

have been covered.

I wish to place on record that following my passion of mentoring would not have been

possible without the constant love and support of My Parents, Siblings, and

friends.

I would also like to convey special Thanks to Unique Academy & my beloved

students for believing in me.

Happy studying!!!

“Bottoms Up Tax with Saumil Sir”


About the Author
Saumil Sir has been Mentor for various ALL INDIA

RANKERS at different stages of CA & CS stream.

CA Saumil Manglani carries the charm of being a First

Attempt Chartered Accountant and has been Indore topper

in CA- CPT and IPCC stage. He is pursuing CS as well. Saumil

Sir has been Mentor for various ALL INDIA RANKERS at

different stages of CA & CS stream and has also been

teaching to MBA, BBA & B.Com Students.

CA Saumil Manglani (ACA, B. Com)

There’s a case law of Sir named as Saumil Manglani vs The Board of

Secondary Education on 12 September, 2012

He has earned a huge fame among the students in a very short span because of his exceptional way of

teaching (with Sher – o - shayari’s), making the concepts crystal clear and understandable in a very

simplified language.

The Social media pages of Saumil Sir are flooded with the appreciation of the students and his friends.

He is also the author of the India’s first ever “Last Day Revisionary Buddy – DT Bullet & GST Bullet ”

designed with a sole view that whole subject should get covered in just 12 – 15 hours before the day

of exam.

Saumil Sir is a faculty for CA and CS Students and also visiting faculty at various prestigious institutions of

Pune. He holds a record of conducting 5 days continuous marathon lectures of around 10 hours daily. He is

Blessed with the skill of Pin Pointing the accidental points in the subject.

Saumil Sir can be reached at manglanisaumil@gmail.com or at Facebook.com/saumil.manglani or at Instagram

ca_saumil_manglani or at his contact Number 9921051593


Income Tax – Volume 1 for June 21 and Dec 21 exams CS Executive

Index

Page
Topics Topic Name
Numbers

1 Direct Taxes at a Glance 1.1 – 1.16

2 Basics & Residential Status 2.1 – 2.45

3 Incomes Exempt from Tax 3.1 – 3.23

4 Income from Salaries 4.1 – 4.53

5 Income from House Property 5.1 – 5.20

Profits and Gains from Business &


6 6.1 – 6.65
Profession

7 Capital Gains 7.1 – 7.46

Unique Academy – 8007916622 CA Saumil Manglani - 9921051593


1. Direct Taxes at a Glance 1.1

Unique Academy - 8007916622 CA Saumil Manglani - Contact: 9921051593


1. Direct Taxes at a Glance 1.2

Introduction
The word tax is based on the Latin word “taxo” which means “Charge”.
To tax means to impose a financial charge or other levy upon a taxpayer,
an individual or legal entity, by a state or the functional equivalent of a
state such that failure to pay is punishable by law.

Taxation is a payment from natural persons or legal entity and it is levied


by Government for which no goods or services is received directly in
return, so taxes is that amount of money, the people pay which is not
related directly to the benefit of people obtained from the provision of a
particular goods or services.

Until the early 1930s, it was universally accepted in principle that


governments should balance their budgets. Thus, the principle reason for
taxation was to pay for government expenditures.

The taxes collected have been used by the government to carry out many
functions. Some of these include:

• expenditures on war,

• the enforcement of law and public order,

• protection of property,

• economic infrastructure (roads, legal tender, enforcement of contracts,


etc.),

• public works,

• social engineering, and

• the operation of government itself.

Governments also use taxes to fund welfare and public services. These
services can include

• education systems,

• health care systems,

• pensions for the elderly,

• unemployment benefits, and


• public transportation, energy, water and waste management systems,
common public utilities, etc.

Unique Academy - 8007916622 CA Saumil Manglani - Contact: 9921051593


1. Direct Taxes at a Glance 1.3

Definitions

There is no precise and accurate definition for the tax and the concept of tax has been defined
differently by different economists. Some definitions are as follows.
According to Prof Seligman – A tax is compulsory contribution from the person to the
government to defray the expense incurred in the common interest of all without reference to
special benefits conferred.
According to Bastable – A tax as a compulsory contribution of the wealth of a person, or body of
persons for the service of public powers.
Deviti. De Marco defines – A tax as a share of the income of citizens which the state
appropriate in order to procure for itself the means necessary for the production of general public
services.
Hugh Dalton – A tax is a compulsory charges imposed by a public authority irrespective of the
exact amount of service rendered to the tax payer in return and not imposed as a penalty for legal
offence.
Jom Bouvier defined a tax as “A pecuniary burden imposed for support of the government, the
enforced proportional contribution of persons and property of the government and for all public
needs”
According to Trussing, “The essence of Tax as distinguished from other charges by government is
the absence direct quid pro quo- tit for tat between the tax payers and the public authority”.

From the above definitions we may conclude that a tax is compulsory contribution, levied by
government from owner of income without direct benefit but for public benefit, and taxes should be
arranged by the law.

Unique Academy - 8007916622 CA Saumil Manglani - Contact: 9921051593


1. Direct Taxes at a Glance 1.4

Characteristics of Taxes

1. Tax is compulsory – A tax is imposed by law. So tax is compulsory


payment to the governments from its citizens. Tax is duty from every
citizen to bear his share for supporting the government

2. Tax is contribution – Contribution means in order to help or provide


something. Tax is contribution from members of community to the
Government. A tax is the duty of every citizen to bear their due share
for support to government to help it to face its expenditures.

3. Tax is for public benefit – Tax is levied for the common good of society
without regard to benefit to special individual.

4. No direct benefit – Government is compulsorily collecting all types of


taxes and does not give any direct benefits to the tax payer for taxes
paid. The essence of tax as distinguished from other charges by
governments is the absence of a direct quid-pro-que between the
taxpayer and the public authority. Tax is different from another
government charges which may give direct benefits to payers such as
prices, fees, fines etc where the direct benefits are available. Taxes are
for common benefits to all the members of the society.

5. Tax is paid out of income of the tax payer – Income means money
received, especially on regular basis, for work or through investment.
Tax is paid out of income as long as the income becomes realized, here
the tax is imposed. Income owner has profit from any business, so he
should pay his share for support to the government.

6. Government has the power to levy tax – Governments are practicing


sovereign authority upon its citizens through levying of taxes. Only Govt.
can collect tax from the people.

7. Tax is not the cost of the benefit – Tax is not the cost of benefit
conferred by the government on the public. Benefit and taxpayer are
independent of each other, and payment of taxation is of course
designed for conferring of benefits on general public.
8. Tax is for the economic growth and public welfare – Major
objectives of the government are to maximize economic growth and
social welfare.

Unique Academy - 8007916622 CA Saumil Manglani - Contact: 9921051593


1. Direct Taxes at a Glance 1.5

Canons of Taxation

Canons of taxation refer to the administrative aspect of a tax.

Canon of
Equity

Canon of Canon of Canon of


economy Taxes Certainty

Canon of
Convenie
nce

Unique Academy - 8007916622 CA Saumil Manglani - Contact: 9921051593


1. Direct Taxes at a Glance 1.6

1. Canon of equity: This canon implies that any tax system should be based on the
principle of social justice. Equity refers to both horizontal and vertical equity.
Horizontal equity describes the concept that, taxpayers with equal abilities to pay
should pay the same amount of tax. Vertical equity means that taxpayers with a
greater ability to pay should pay more tax.

2. Canon of Certainty : The tax rules should clearly specify when the tax is to be paid, how
it is to be paid, and how the amount to be paid is to be determined. Objective of this
canon is to create trust between two parties, first party taxpayer who is to pay the tax and
second party the authority whom receipt tax. If taxpayers have difficulty measuring the
tax base or determining the applicable tax rate or the tax consequences of a transaction,
then certainty doesn‘t exist. Certainty might also be viewed as the level of confidence a
person has that the tax is being calculated correctly.

3. Canon of Convenience: A tax should be due at a time or in a manner that is


most likely to be convenient for the taxpayer. Convenience in paying a tax helps
to ensure compliance. The appropriate payment mechanism depends on the amount
of the liability and the how easy or difficult it is to collect. Discussion of this principle
in designing a particular rule or tax system would focus on whether it is best to collect
the tax from the manufacturer, wholesaler, retailer or customer, as well as the
frequency of collection.

4. Canon of Economy: This canon implies that decreasing the administrative cost
of collection of the tax at the lowest level. The costs to collect a tax should be kept to
a minimum for both the government and taxpayers. This principle considers the
number of revenue officers needed to administer a tax. Compliance costs for
taxpayers should also be considered. This principle is closely related to the principle
of simplicity.

Unique Academy - 8007916622 CA Saumil Manglani - Contact: 9921051593


1. Direct Taxes at a Glance 1.7

Objectives

Objectives of taxes have been developed when the functions of the


Government are developed. The Objectives of taxation in brief are as
under:-

• Revenue -This objective is the oldest, uppermost and primary objective.


Taxes are imposed so as to produce the necessary amount of revenue to
meet the requirement of the government, as the public expenditures is
increasing in scope and size day by day. Therefore, the main objective of
taxes is to raise revenue to meet the Govt. expenditures adequately.

• Social objectives – Taxes became a main goal for some of social


objectives.

a) Redistribution of income and wealth

b) Social welfare

c) Safety of society from bad and injurious customs(tobacco, alcohol)

• Economic significance of taxes- Taxes are used from economic point


of view, so taxation helps to encourage some economic activities, and as
a tool to solve some economics problems. Tax is also a means for
directing of scarce economic activities. Taxation helps to accelerate
economic growth, and taxation plays very important role in case of
economic stability.

• Economic growth: Taxes are considered as a tool for economic growth


and it helps to accelerate growth of economic development.

• Enforcing government policy: The Government can encourage


investment, saving, consumption, export, protection of home industry,
employment, production, protection of society from harmful customs, and
economic stability through suitable tax policy. Therefore, the government
gives tax exemption to the investment and saving.

• Directing limited scarce resources into effective and essential


channels: Tax policy plays crucial role for directing scarce resources into
essential commodities. This is achieved by giving tax exemption to certain
industries and imposition of heavy duties on other industries, so with the
adoption of suitable tax policy, economic resources may be diverted to

Unique Academy - 8007916622 CA Saumil Manglani - Contact: 9921051593


1. Direct Taxes at a Glance 1.8

• the production of necessary articles and investors will go to the


exemption industries.

• Economic stability: Maintaining economic stability is one of the tax


objectives. Economic stability is a very important factor for the sustained
economic growth. Government can effectively use taxes in the case of
inflation and depression.

Direct Vs Indirect Tax

GST

Unique Academy - 8007916622 CA Saumil Manglani - Contact: 9921051593


1. Direct Taxes at a Glance 1.9

Direct and Indirect taxes

Point of Direct Indirect Tax


Tax
difference
Incidence & A tax is said to be direct when If impact of tax is on one person and
Impact impact and incidence of a tax incidence (burden) on the another, the
are on one and same person. tax is called ‘indirect’
Burden Direct tax is imposed on the Indirect tax is imposed on commodities
individual organisation and and allows the tax burden to shift.
burden of tax cannot be
shifted to others.
Viability of Direct taxes are lesser burden Indirect taxes are borne by the
payment then Indirect taxes to people as consumers of commodities and services
direct taxes are based on Income Irrespective of financial ability as the
earning ability of people. MRP Includes all taxes.

Administrative The administrative cost of Cost of collecting Indirect taxes is very


viability collecting direct taxes is more less as indirect taxes are wrapped up in
and Improper administration may prices of goods and services and
result in tax evasion. cannot be evaded.

Penalty It is levied on the assessee. It is levied on supplier of Goods &


Services.

Unique Academy - 8007916622 CA Saumil Manglani - Contact: 9921051593


1. Direct Taxes at a Glance 1.10

Merits of DT Demerits of DT

1. Equity: - Direct taxes have equity of 1. Evasion: - Direct tax is lump sum
sacrifice, depend upon the volume of therefore tax payers try evasion.
income. They are based on the principle of
2. Uneconomical.:-Expenses of collection
progressive, so rates of tax increase as the
are larger in the case of direct taxes,
level of income of a person rises.
because they require
2. Elasticity and productivity: - Direct taxes
widely- spread staff for collection
have elasticity because when the
government faces some Emergency, like 3. Unpopular:-Direct tax is required to be
earthquake, floods and famine the paid in lump sum for the whole year, so
government can collect money for facing the tax payers feel the painful payment,
those Problems by direct tax. these taxes are therefore unpopular.

3. Certainty: - Direct taxes have certainty on 4. Little incentive to work and save:-In
both sides‘tax-payer and government. The direct taxes, rates are of progressive
tax- payers are aware of the quantity of tax. nature. A person with higher earning is
They have to pay and rate, time of payment, taxed more; in turn he is left little with
manner of payment, and punishment from amount. So the tax payer feels
the side of government is also certain about disincentive to work hard and save
the total amount they are getting. money after reaching a certain level of
income.
4. Reduce inequality: - Direct taxes follow
progressive principles so it is taxing the rich 5. Not suitable to a poor country: -
people with higher of taxation and the poor Direct taxes are not enough to meet its
people with a lower level of taxation. expenditure.

5. Good instrument in the case of inflation. -


Tax policy as fiscal instrument plays 6. Arbitrary:-Due to absence of logical or
important role in the case of the inflation, so scientific principle to determine the
government can absorb the excess money degree of progression in the taxation,
by arising in the rate of existing taxes or the direct taxes are arbitrary.
imposition of new taxes.

6. Simplicity: - Direct taxes are simplicity, while


levy the rules, procedures, regulations of
income tax are very clear and simple

Unique Academy - 8007916622 CA Saumil Manglani - Contact: 9921051593


1. Direct Taxes at a Glance 1.11

Merits of Indirect Tax Demerits of Indirect Tax

1. High revenue production: - Indirect taxes 1. Regressive in effect:-Essential


cover a large number of essential goods and commodities are used from all members of
luxurious goods which are consumed by the community. No distinction is made between
mass both rich and poor people, these help in the rich and poor people.
collecting large revenue.
2. Uncertainty in collection- Discourage
2. No evasion. Nature of indirect tax is that, it is savings and Increase inflation:-Indirect
included in the price of commodity, so tax taxes are payable when people spent their
evasion or tax avoidance is difficult. income or when people buy goods and
services, so tax authorities cannot
3. Convenient: -Indirect taxes are small amount
accurately estimate the total yield from
and indirect taxes are hidden in the price of
different indirect taxes.
goods and service, hence the burden of these
taxes is not felt very much by the tax-payers, 3. Discourage savings- Increase inflation:-

and not lump sum like direct taxes. Indirect taxes are included in the price of
commodity, so people have to spend more
4. Economy - Indirect taxes are economical in
money on essential commodities, when
collection and the administrations costs of
levied indirectly. In this case that means the
collection are very low, also the procedure of
customers cannot save some of their
collection of these taxes is very simply.
money.
5. Wide coverage:-Indirect taxes cover almost
4. Increased inflation:-Indirect taxes
all commodities like essential commodities,
increase the cost of input and output,
luxuries, and harmful ones.
increase in production cost, push the
6. Elasticity:-Since a large number of commodities price of goods. These reflect an increase
and services are covered by indirect taxation in the wages of the workers.
there is great scope for modification of taxes,
goods and tax rate, much depends on nature of
goods and on its demands.

Taxation in India during Ancient Time

In India, the system of direct taxation as it is known today, have been in force in one form or another
even from ancient times. there are references both in Manu Smriti and Arthashastra to a variety of tax
measures

Manu, the ancient sage and law-giver He laid down that traders and artisans should pay 1/5th of their
profits in silver and gold, while the agriculturists were to pay 1/6th, 1/8th and 1/10th of their produce
depending upon their circumstances.

Unique Academy - 8007916622 CA Saumil Manglani - Contact: 9921051593


1. Direct Taxes at a Glance 1.12

Background of Income tax in India


Income Tax Act, 1860
Consequent upon the financial difficulties created by the events of 1857. Income Tax was
introduced in India
for the first time by the British in the year 1860. The Act of 1860 was passed only for five
years and therefore it lapsed in 1865. It was replaced 1867 by a licence tax on professions
and trades and the latter was converted into a certificate tax in the following year. It was latter
abolished in 1873. Licence tax traders remained in operation till 1886 when it was merged in
the income tax Act of that year.

Income Tax Act, 1886

The Act of 1886 levied a tax on the income of residents as well as non residents in India.
The Act defined agricultural income and exempted it from tax liability in view of the already
existing land revenue a kind of direct taxes. The Act of 1886 exempted life insurance
premiums paid by assesse policies of his own life. Another important provision of this Act
Hindu undivided family was treated as a distinct taxable entity.

Income Tax Act, 1918


The Act of 1918 brought under change also receipts of casual or non recurring nature
pertaining to business or professions. Although income tax in India has been a charge on net
income since inception, it was in the Act of 1918 that specific provisions were inserted for the
first time pertaining to business deductions for the purpose of computing net income.
The Act of 1918 remained in force for a short period and was replaced by new Act (Act XI of
1922) in view of the reforms introduced by the Govt. of India Act, 1919.

Income Tax Act, 1922

The organizational history of the income tax department dates back to the year 1922. “one of the
important aspects of the 1922 Act was that, it laid down the basis, the mechanism of administering
the tax and the rates at which the tax was to be levied would be laid down in annual finance acts.
This is procedure brought in the much needed flexibility in adjusting the tax rates in accordance with
the annual budgetary requirements and in securing a degree of elasticity for the tax system. Before
1922 the tax rate were determined by the Income tax act itself and to revise the rates, the act
itself had to be amended. The Income tax Act, 1922 gave for first time a specific nomenclature to
various income tax authorities and laid the foundation of a proper system of administration as per
provisions of income tax act 1922 thus, it is the income tax act 1961, which is currently operative in
India.

Unique Academy - 8007916622 CA Saumil Manglani - Contact: 9921051593


1. Direct Taxes at a Glance 1.13

Income Tax Act, 1961

The present law of income tax in India is governed by the Income Tax Act, 1961 which is amended
from time to time by the annual finance Act and other legislations pertaining to direct tax. The act
which came into force on April 1, 1962, replaced the Indian income tax Act, 1922, which had
remained in operation for 40 years. Furthermore, A set of rules known as Income Tax Rules, 1962
have been framed for implementing the various provisions of the Act.

Changes in administrative set up since the inception of the department

The rapid changes in administration of direct taxes, during the last decades, reflect the history of socio-
economic thinking in India.
The organizational history of the income-tax department starts in the year 1922. The income-tax act,
1922, gave, for the first time, a specific nomenclature to various Income-tax authorities. The foundation
of a proper system of administration was thus laid. in 1924, Central Board of revenue act constituted the
Board as a statutory body with functional responsibilities for the administration of the income-tax act.
Commissioners of Income- tax were appointed separately for each province and Assistant
Commissioners and Income-tax Officers were provided under their control. The amendments to the
income tax act, in 1939, made two vital structural changes: (i) appellate functions were separated from
administrative functions; a class of officers, known as appellate assistant Commissioners, thus came
into existence, and (ii) a central charge was created in Bombay.
In 1940, with a view to exercising effective control over the progress and inspection of the work of
Income-tax Department throughout India, the very first attached office of the Board, called Directorate of
Inspection (Income tax) - was created. As a result of separation of executive and judicial functions, in
1941, the appellate tribunal came into existence. In the same year, a central charge was created in
Calcutta also.
In order to improve the quality of work, in 1977, a new cadre known as IAC (assessment) and in 1978
another cadre known as CIT (Appeals) were created. The Commissioners’ cadre was further
reorganized and five posts of Chief Commissioners (administration) were created in 1981.
Certain important policy and administrative reforms carried out over the past few years are as follows :-
(a) the policy reforms include :-
• Lowering of rates;
• Withdrawals/reduction of major incentives;
• introduction of measures for presumptive taxation;
• simplification of tax laws, particularly relating to capital gains; and
• Widening the tax base.
(b) the administrative reforms include :--
• Computerization involving allotment of a unique identification number to tax payers which is
emerging as a unique business identification number; and

• Realignment of the available human resources with the changed business needs of the
organization.

Unique Academy - 8007916622 CA Saumil Manglani - Contact: 9921051593


1. Direct Taxes at a Glance 1.14

Tax Structure in India

Constitution of India

The roots of every law in India lies in the Constitution, therefore understanding the provisions of
Constitution is foremost to have clear understanding of any law. Let us first understand what it
talks about tax:

• Article 265– No tax shall be levied or collected except by the Authority of Law.

• Article 246- Distributes legislative powers including taxation, between the Parliament of India
and the state legislature

Schedule VII- Enumerates powers under three lists

o Union List – Powers of Central Government

o Legislative List- Powers of State Government

o Concurrent List- Both Central and state Government have powers, in case of conflict; law
made
By Union Government prevails.

Some of the major taxes under respective lists are:-

Central Government

• Customs including export duties

• Excise on Tobacco and other goods manufactured in India except alcoholic liquors for human
consumption, opium, narcotic drugs

• Corporation Tax

• Taxes on interstate trade of goods

• Taxes on interstate consignment of goods

• Any other matter not included in List II or III

Unique Academy - 8007916622 CA Saumil Manglani - Contact: 9921051593


1. Direct Taxes at a Glance 1.15

State Government

• Excise duty on alcoholic liquors, opium and narcotics

• Octroi or Entry Tax

• Tax on intra state trade of goods other than newspapers

• Tax on advertisements other than that in newspapers

• Tax on goods and passengers carried by road or inland waterways

• Tax on professionals, trades, callings and employment

Administration

The Central Board of Revenue or Department of Revenue is the apex body charged with the
administration of taxes. It is a part of Ministry Of Finance which came into existence as a result of
the Central Board of Revenue Act, 1924.

Initially the Board was in charge of both direct and indirect taxes. However, when the
administration of taxes became too unwieldy for one Board to handle, the Board was split up into
two, namely the Direct Taxes (CBDT) and Central Board of Excise and Customs (CBEC) now
CBIC Central Board of Indirect Taxes and Customs. This bifurcation was brought about by
constitution of the two Boards under Section 3 of the Central Boards of Revenue Act, 1963.
CBDT

The Central Board of Direct Taxes (CBDT) provides essential inputs for policy and planning of
direct taxes in India and is also responsible for administration of the direct tax laws through
Income Tax Department. The CBDT is a statutory authority functioning under the Central Board
of Revenue Act, 1963. It is India’s official Financial Action Task Force (FATF) unit.

Organizational Structure

The CBDT is headed by CBDT Chairman and also comprises six members. The Chairperson
holds the rank of Special Secretary to Government of India while the members rank of Additional
Secretary to Government of India.

• Member (Income Tax)

• Member (Legislation and Computerization)

• Member (Revenue)

• Member (Personnel & Vigilance)

Unique Academy - 8007916622 CA Saumil Manglani - Contact: 9921051593


1. Direct Taxes at a Glance 1.16

• Member (Investigation)

• Member (Audit & Judicial)

The CBDT Chairman and Members of CBDT are selected from Indian Revenue Service (IRS), a
premier civil service of India, whose members constitute the top management of Income Tax
Department.
Income Tax Department

Income Tax Department functions under the Department of Revenue in Ministry of Finance. It is
responsible for administering following direct taxation acts passed by Parliament.

• Income Tax Act, 1961

• Various Finance Acts (Passed Every Year in Budget Session)

Income Tax Department is also responsible for enforcing Double Taxation Avoidance
Agreements and deals with various aspects of international taxation such as Transfer Pricing.
Income Tax Department has powers to combat aggressive Tax avoidance by enforcing General
Anti Avoidance Rules.

CBIC

Central Board of Indirect Taxes and Customs (CBIC) is a part of the Department of Revenue
under the Ministry of Finance, Government of India. It deals with the tasks of formulation of policy
concerning levy and collection of Customs & Central Excise duties and GST, prevention of
smuggling and administration of matters relating to Customs, Central Excise, GST and Narcotics
to the extent under CBIC’s purview.

GST Council

A GST Council consisting of representatives from the Centre as well as State has been
formulated under the GST Law of indirect taxes. The Council will make recommendations to the
Union and the States on Goods and Service Tax laws, on any other matter relating to GST.

Till date, numerous conclusive meetings of GST Council have been undertaken. Decisions have
been taken regarding rates, Composition Scheme, exemption schemes to North-Eastern and
hilly areas, compensation method for loss of revenue to states etc. Rules regarding return,
refund, registration, payment, invoicing and the like have been finalized by the same. However,
various other issues and modalities regarding the GST are constantly being discussed at the
GST Council Meetings for smoothening the law and making it easy to implement for society at
large.

Unique Academy - 8007916622 CA Saumil Manglani - Contact: 9921051593


2. Basics & Residential Status 2.1

Unique Academy - 8007916622 CA Saumil Manglani - Contact: 9921051593


2. Basics & Residential Status 2.2

Introduction

Tax is the financial charge imposed by the Government on income, commodity or activity. Government
imposes two types of taxes namely Direct taxes and Indirect taxes. Direct tax is one where burden of
tax is directly on the payer e.g. income tax, whereas Indirect tax is paid by the person other than the one
who utilizes the product or service e.g Custom duty, Goods and Service tax (GST).
Article 265 of the Constitution provides that no tax shall be levied or collected except by authority of
law. Thus, the tax proposed to be levied or collected must be within the legislative competence of the
legislature imposing the tax. Further, the law imposing the tax, like other laws, must not violate any
fundamental right.
Income tax being direct tax happens to be the major source of revenue for the Central Government.
The responsibility for collection of income-tax vests with the Central Government. This tax is leviable
and collected under Income-tax Act, 1961 (hereinafter referred to as ‘the Act’).

The Income tax Act contains the provisions for determination of taxable income, determination of tax
liability, procedure for assessment, appeal, penalties and prosecutions. It also lays down the powers
and duties of various income tax authorities.

Overview of Income Tax Law in India


 Finance Act: Every year a Budget is presented before the parliament by the Finance Minister.
One of the important components of the Budget is the Finance Bill. The Bill contains various
amendments in the Income-Tax Act and prescribes the rates of taxes. When the Finance Bill is
approved by both the houses of parliament and receives the assent of President, it becomes the
Finance Act.
Finance Bill Presented by Finance Minister


Recommendation / Suggestion considered and changes made to Finance Bill


Parliament Approval


President’s approval


Finance Act

Effective date of amendments in Finance Bill


Direct Tax

Indirect Tax


Usually Date if Notification

Usually midnight of date
in Official gazette or in of Presentation of Bill
the Finance Act.

Unique Academy - 8007916622 CA Saumil Manglani - Contact: 9921051593


2. Basics & Residential Status 2.3

Basic Concepts of Income Tax

Income Tax is levied on the Total Income of the previous year of every person. It is governed by the:
• Income tax Act, 1961
• Income Tax Rules, 1962
• Relevant Finance Act
• Notifications, Circulars and Clarification issued by CBDT
• Judicial pronouncements
To levy income tax, one must have an understanding of the various concepts related to the charge of tax
like previous year, assessment year, Income, total income, person etc.
Computation of Tax Liability include following steps:
1. Determine the category of person
2. Determine the residential status of the person as per section 6
3. Calculate the Total income as per the provisions
4. Calculate the tax on income

Charge of Income Tax (Section 4)


Section 4 of the Act is the charging section. It lays down the basis on which tax is
imposed. Accordingly, the section provides that:
a) Income tax shall be charged at the rate or rates prescribed in the finance act for the
relevant previous year,
b) the charge of tax is on various persons specified u/s 2(31);
c) the income sought to be taxed is that of the previous year and not of the of
assessment year
d) the levy of tax on the assessee is on his total or taxable income computed in
accordance with and subject to the appropriate provisions of the income tax Act,
including provisions for the levy of additional income-tax
Provided that where by virtue of any provision of this Act income-tax is to be charged in respect of the
income of a period other than the previous year, income- tax shall be charged accordingly.

Unique Academy - 8007916622 CA Saumil Manglani - Contact: 9921051593


2. Basics & Residential Status 2.4

Determining category of persons

PERSON [SECTION 2(31)]: Income-tax is charged in respect of the total income of the previous year of
every person. Hence, it is important to know the definition of the word person. As per section 2(31), Person
includes:

Individual

Hindu Undivided Family (HUF)

Companies

Firms

AOP

BOI

Local Authorities

Every Artificial Juridical Person not covered above

Unique Academy - 8007916622 CA Saumil Manglani - Contact: 9921051593


2. Basics & Residential Status 2.5

 Individual: An individual is a natural human being i.e. male, female, minor or a person of sound or unsound
mind.
 HUF: It consists of all persons lineally descended from a common ancestor and includes their wives and
unmarried daughters and also a stranger who has been adopted by the family.

Types of HUF

No. Dayabhaga Mitakshara

1 West Bengal and Assam Rest of India i.e. except West


Bengal and Assam

2 Right to family property not by birth Right to family property by birth

Company: It include Domestic company, Foreign company, company in which public are substantially
interested. Section 2(17) defines the term company to mean:
(a) any Indian company, or
(b) any body corporate incorporated by or under the laws of a country outside India i.e. a foreign
company, or
(c) any institution, association or body, whether incorporated or not and whether Indian or non- Indian,
which is declared by general or special order of the Board to be a company only for such
assessment year or assessment years.
Firm: It includes a partnership firm whether registered or not and shall include a Limited Liability
Partnership as defined in the Limited Liability Partnership Act,2008.

According to Section 4 of the Partnership Act, 1932 persons who have entered into partnership with one another
are called individually, ‘partners’ and collectively ‘a firm’.

 Association of Person: Two or more persons join in for a common purpose or common action to produce
income, profits or gains.

HUF, companies, firms, etc. as members

The object must be to produce income. It is not enough that the persons receive the income jointly.

 Body of Individuals denote the status of persons who are assessable in like manner and to the same extent as
the beneficiaries individually.

Only individuals can be the members Individuals join together for common purposes

The difference between Association of persons and body of individuals is that whereas an association implies a
voluntary getting together for a definite purpose, a body of individuals would be just a body without an intention to
get-together. Moreover, the members of body of individuals can be individuals only whereas the members of an
association of persons can be individual or non-individuals (i.e. artificial persons)

Unique Academy - 8007916622 CA Saumil Manglani - Contact: 9921051593


2. Basics & Residential Status 2.6

 Local Authority

It means a municipal committee, district board, body of port commissioners, or other authority legally entitled to
or entrusted by the Government with the control and management of a Municipal or local fund.

 Artificial Juridical person


This is a residuary clause. If the assessee does not fall in any of the first six categories, he is assessed under
this clause. Generally, a statutory corporation, deity or charitable institution or an endowment for charitable or
religious purposes falls under artificial juridical person.

Assessee [Section 2(7)]


• In layman language tax payer is assessee, he may be the person of any of the above
categories
• As per Section 2(7) of Income Tax Act, assessee is a person
– by whom any tax or

– any other sum of money


is payable under Income Tax Act. It includes every person:
(a) in respect of whom any proceeding under Income Tax Act has been taken for assessment of
– his income or
– fringe benefits or
– the income of any other person in respect of which he is assessable or
– the loss sustained by him or by such other person or
– the amount of refund due to him or to such other person
(b) who is deemed to be an assessee under any provision of Income Tax Act;
(c) who is deemed to be an assessee in default under any provision of Income TaxAct;
Accordingly, assessee is a person by whom tax or any other sum is payable under the
Act. The expression “other sum of money” includes
– fine, interest, penalty and tax or
– person to whom any refund of tax etc. is due under the Act or
– if any proceeding under the Act has been taken against any person, he is also an assessee.
Remember, the proceedings must be initiated under the provisions of the Act. In other words,
a single enquiry letter issued by the Income-tax Department without reference to any specific
provision of the Act does not constitute proceeding under the Act and, as such, till
proceedings are initiated under the Act, the person may not become an assessee within the
ambit of Section 2(7) of the Act.

Unique Academy - 8007916622 CA Saumil Manglani - Contact: 9921051593


2. Basics & Residential Status 2.7

Previous Year [Section 3]


Previous year means the financial year immediately preceding the assessment year. Income earned in a year is
taxable in the next year. The year in which income is earned is known as previous year.

Examples for previous year in the case of newly set-up business/profession.

Example 1: Y sets up a new business on May 15, 2020. What is the previous year for the
assessment year 2021-2022

Answer: Previous year for the assessment year 2021-22 is the period commencing on May 15,
2020 and ending on March 31, 2021.

Example 2: A joins an Indian company on February 17, 2019. Prior to joining this Indian company
he was not in employment nor does he have any other source of income. Determine the previous
year of A for the Assessment Years 2020-21 and 2021-22

Answer: Previous years for the assessment years 2020-21 and 2021-22 will be as follows.

Previous year Assessment year

Feb. 17, 2019 to March 31, 2020 2020-21


April 1, 2020 to March 31, 2021 2021-22

Assessment year [Section 2(9)]


“Assessment year” means the period of twelve months commencing on 1st April every
year. Thus, it is normally period beginning on 1st April of one year and ending on 31st
March of the next year. Income of previous year of an assessee is taxed during the
following assessment year at the rates prescribed by the relevant Finance Act.
Exception to the General Rule: In the following situation, the Income of previous year of an assessee is taxed in the
previous year itself:
1. Income of Non-Resident from Shipping (172)
2. Income of persons leaving India either permanently or for long duration (174)
3. Income of bodies formed for short duration (174A)
4. Income of person trying to alienate his assets with a view to avoid tax (175)
5. Income of discontinued business (176)

Certain cases when income of a previous year will be assessed in the previous year itself

Unique Academy - 8007916622 CA Saumil Manglani - Contact: 9921051593


2. Basics & Residential Status 2.8

Previous year for undisclosed sources of income

Amount
borrowed or
repaid on
hundi
[Section 69D]

Unexplained
Cash Credits expenditure
[Section 68] [Section 69C]

Undisclosed
sources of
Unexplained Investment
Investments etc not fully
disclosed
[Section 69]
[Section 69B]

Unexplained
money
[Section 69A]

(a) Cash Credits [Section 68]


Where any sum is found credited in the books of the assessee and
the assessee offers no explanation about the nature and source or the
explanation offered is not satisfactory in the opinion of the Assessing
Officer, the sum so credited may be charged as income of the assessee
of
that previous year.

(b) Unexplained Investments [Section 69]


Where in the financial year immediately preceding the assessment
year, the assessee has made investments which are not recorded in
the boo s of account and the assessee offers no explanation about the
nature and the source of investments or the explanation offered is not
satisfactory, the value of the investments are taxed as income of the
assessee of such
financial year.

(c) Unexplained money etc. [Section 69A]

Where in any financial year the assessee is found to be the owner of


any money, bullion, Jewellery or other valuable article and the same is
not recorded in the boo s of account and the assessee offers no
explanation about the nature and source of acquisition of such money,
bullion etc. or the explanation offered is not satisfactory, the money
and the value of bullion etc. may be deemed to be the income of the
assessee for such financial year. Ownership is important and mere
possession is not enough

Unique Academy - 8007916622 CA Saumil Manglani - Contact: 9921051593


2. Basics & Residential Status 2.9

(d) Amount of investments etc., not fully disclosed in the books of account [Section
69B]
here in any financial year the assessee has made investments or is found to be the
owner of any bullion, jewellery or other valuable article and the Assessing Officer finds
that the amount spent on ma ing such investments or in acquiring such articles
exceeds the amount recorded in the books of account maintained by the
assessee and he offers no explanation for the difference or the
explanation offered is unsatisfactory, such excess may be deemed to
be the income of the assessee for such financial year.

(e) Unexplained expenditure [Section 69C]


Where in any financial year an assessee has incurred any expenditure
and he offers no explanation about the source of such expenditure or the
explanation is unsatisfactory the Assessing Officer can treat such
unexplained expenditure as the income of the assessee for such
financial year. Such unexplained expenditure which is deemed to be the
income of the assessee shall not be allowed as deduction under any
head of income.
(f) Amount borrowed or repaid on hundi [Section 69D]

Where any amount is borrowed on a hundi or any amount due


thereon is repaid other than through an account-payee cheque drawn
on a ban , the amount so borrowed or repaid shall be deemed to be
the income of the person borrowing or repaying for the previous year in
which the amount was borrowed or repaid, as the case may be.

However, where any amount borrowed on a hundi has been deemed to be the income of
any person, he will not be again liable to be assessed in respect of such amount on
repayment of such amount. The amount repaid shall include interest paid on the
amount borrowed.

Unique Academy - 8007916622 CA Saumil Manglani - Contact: 9921051593


2. Basics & Residential Status 2.10

Income [Section 2(24)]


The definition of Income as given in Section 2(24) of the Act starts with the word includes
therefore the list is inclusive not exhaustive. The definition enumerates certain items,
including those which cannot ordinarily be considered as income but are treated
statutorily as such.
As per section 2(24), the term income includes:
1. Profits and gains;
2. Dividend;
3. Voluntary contributions
4. The value of any perquisite or profit in lieu of salary taxable.
5. Any special allowance or benefit specifically granted to the assessee to meet
expenses wholly, necessarily and exclusively for the performance of the duties of an
office or employment of profit.
6. City Compensatory Allowance/ Dearness allowance
7. Benefit or Perquisite to a Director
8. Any Benefit or perquisite to a Representative Assessee
9. Any sum chargeable under section 28, 41 and 59 –
10. Capital Gain
11. Insurance Profit
12. Banking income of a Co-operative Society
13. Winnings from Lottery
14. Employees Contribution Towards Provident Fund
15. Amount Received under Keyman Insurance Policy
16. Amount received for not carrying out any activity
17. Any sum referred to in clause (v) or (vi) of sub-section (2) of section 56;
18. Gift received for an amount exceeding Rs. 50,000
19. Consideration received for issue of shares
20. Amount received as an advance or otherwise in the course of negotiation for transfer
of a capital asset referred to in clause (ix) of section 56(2).
21. Assistance in the form of a subsidy or grant or cash incentive or duty drawback or
waiver or concession or reimbursement (by whatever name called) by the Central
Government or a State Government or any authority or body or agency in cash or
kind to the assessee other than the subsidy or grant or reimbursement which is taken
into account for determination of the actual cost of the asset in accordance with the
provisions of Explanation 10 to clause (1) of section 43.
22. Fair market value of inventory which is converted into, or treated as a capital asset
[Section 28(iva)].
23. Any compensation or other payment, due to or received by any person, in
connection with termination of his employment or the modification of the term and
conditions relating thereto [Section 56(2)(xi)].

Unique Academy - 8007916622 CA Saumil Manglani - Contact: 9921051593


2. Basics & Residential Status 2.11

Note: Legal or illegal source


The income-tax law does not make any distinction between income accrued or
arisen from a legal source and income tainted with illegality. In CIT v. Piara Singh
(1980) 3 Taxman 67, the Supreme Court has held that if smuggling activity can be
regarded as a business, the confiscation of currency notes by customs authorities
is a loss which springs directly from the carrying on of the business and is,
therefore, permissible as a deduction

Capital Vs Revenue Receipt


A receipt is taxable if it is of the nature of income. But receipts which are of
capital nature are generally not taxable. The basic scheme of income-tax is to
tax income not capital, and similarly to allow revenue expenditure. But this general
rule is subject to certain exceptions.
An amount referable to fixed capital is a capital receipt whereas a receipt
referable to circulating capital would be a revenue receipt. While the latter is
chargeable to tax, the former is not subject to income-tax unless otherwise
expressly provided.
The Income-tax Act does not define the term “Capital receipt” & “Revenue
receipt”.

Whether a particular receipt is of the nature of income or capital is explained


below by the following examples. The following test can be applied to determine
the nature of particular receipt

Factors that do not determine the nature or character of receipt


The capital or revenue nature of a receipt must be determined with reference to each
receipt on the basis of the facts and circumstances of each case, the ultimate
conclusion as to the capital or revenue character of the receipt would be of the High
Court or the Supreme Court and the principles laid down by the Court must be
followed for the purpose. However, while determining the question whether a
particular receipt is capital or revenue in nature, care must be taken to ensure that
the following are not taken as the basis for determination although these factors
may, to a certain extent, be helpful to arrive at the conclusion
a. Character and Source of Income b) Application of Income c) Allowance –
Disallowance of amount d) Treatment given in books e) Magnitude and method pf
payment f) Basis for measurement of receipt

Unique Academy - 8007916622 CA Saumil Manglani - Contact: 9921051593


2. Basics & Residential Status 2.12

Question1: State whether the following are capital or revenue receipts/expenses and give your reasons:
1. ABC & Co. received Rs. 5,00,000 as compensation from XYZ & Co. for premature termination of
contract of agency.
2. Sales-tax collected from the buyer of goods.
3. PQR Company Ltd. instead of receiving royalty year by year, received it in advance in lump sum.

4. An amount of Rs. 1,50,000 was spent by a company for sending its production manager abroad to
study new methods of production.
5. Payment of Rs. 50,000 as compensation for cancellation of a contract for the purchase of machinery
with a view to avoid an unnecessary expenditure.
6. An employee director of a company was paid Rs. 3,50,000 as a lump sum consideration for not resigning
from the directorship.
Solution
1. Receipt in substitution of a source of income is a capital receipt. Therefore, the amount received by
ABC & Co. from XYZ & Co. for premature termination of an agency contract is a capital receipt though
the same is taxable under Section 28.
2. Sales-tax is the liability of a seller to pay to the Government on the sale of goods made by him, which
is allowed as deduction as revenue expenditure. If any part of Sales-tax is collected from the buyer of
goods that may be treated as a revenue receipt. Thus the sales-tax collected from the buyer of goods
is a revenue receipt.
3. Receipt of lump sum royalty in lieu of future royalties is a revenue receipt, as it is an income from
royalty.
4. Amount spent by a company for sending its production manager abroad to study new methods of
production is revenue expenditure to be allowed as a deduction. Because the new knowledge and
exposure of that manager will assist the company in improving its existing methods of production etc.
5. This is a capital expenditure, as any expenditure incurred by a person to free himself from a
capital liability is a capital expenditure. In the given case, the payment of Rs. 50,000 for canceling
the order for purchase of the machinery, has helped the assessee to become free from an
unnecessary capital liability.
6. The amount of Rs. 3,50,000 received for not resigning from the directorship is a reward received from
the employer. Therefore it is a revenue receipt.

Now Moving ahead let us first roughly understand the different sources of
Income in a broader sense

 Income accruing or arising in India


 Income deeming to accrue or arise in India
 Income received in India
 Income deemed to be received in India
 Income accruing or arising outside India

Unique Academy - 8007916622 CA Saumil Manglani - Contact: 9921051593


2. Basics & Residential Status 2.13

Residential Status (Section 6)

Total income of an assessee cannot be computed unless the person’s residential


status in India during the previous year is known. Thus, determining residential
status of a person is important for calculating tax liability of a person.
Section 6 of the Income tax Act prescribes the tests to be applied to determine the
residential status of all tax payers for purposes of income-tax. An assessee’s
residential status must be determined with reference to the previous year in respect
of which the income is sought to be taxed. Residential status of a person could be:

Person

Individual Individual/HUF Others

In One
exceptional case Resident Non Resident Resident Non Resident
Individual can be
directly
considered as
RNOR

Resident & Resident & Not


AY 21 - 22 Ordinarily Ordinarily
Resident Resident

There are different test to be applied for different types of person, let us understand
test for each category of person:

1. Individuals

 First of all, an individual is classified as resident or non-resident and again a resident


individual may further be categorized as Ordinarily Resident or Not Ordinarily Resident in
India and In One exceptional case Individual can be directly considered as RNOR

Resident in India [section 6(1)]


An individual is said to be a resident in India, if he satisfies any one of the following
conditions -
(i) He is in India in the previous year for a period of 182 days or more [Sec. 6(1)(a)];
or
(ii) He is in India for a period of 60 days or more during the previous year and
for 365 or more days during 4 previous years immediately preceding the relevant
previous year [Sec. 6(1)(c)]
Tax point: Given Conditions are alternative in nature i.e. assessee needs to satisfy
any one condition.
Non-Resident in India
An assessee who is not satisfying sec. 6(1) shall be treated as a non-resident in India
for the relevant previous year.

Unique Academy - 8007916622 CA Saumil Manglani - Contact: 9921051593


2. Basics & Residential Status 2.14

EXCEPTIONS - In the following cases, condition (ii) of sec. 6(1) [i.e. sec. 6(1)(c)] is irrelevant:
1. An Indian citizen, who leaves India during the previous year for employment purpose.

Note: A person going abroad in connection with his employment in India, is not
covered by above exception
E.g. X is having business in India. During the previous year 2020-21, he visited to
Japan for purchasing raw-material for his business, from there he went to USA for
attending a business meeting. Since, X was outside India in connection with his
employment in India, he is not covered by the exception.
2. An Indian citizen, who leaves India during the previous year as a member of crew of an
Indian ship.

For the above exception of Crew member of an Indian Ship while calculating the number of days of
stay –
Period to be excluded

Period commencing from Period ending on


the date entered into the Continuous and the date entered into the Continuous
Discharge Certificate in respect of Discharge Certificate in respect
of
joining the ship by the said individual signing off by that individual from
for the eligible voyage the ship in respect of such voyage.

3. An Indian citizen or a person of Indian origin, who normally resides outside India,
comes on a visit to India during the previous year.

Tax point for all 3 above exceptions: Above assessee shall be treated as resident in India only if he
resides in India for 182 days or more in the relevant previous year.

#Person of Indian origin: A person is deemed to be of Indian origin if he or either of his parents or
grandparents were born in undivided India. Here, grand parents may be paternal or maternal.

Resident and ordinarily resident


If a resident individual satisfies the both the following two additional conditions,
he will be treated as resident & ordinarily resident in India -
(a)He has been resident in India [as per sec. 6(1)] in at least 2 out of 10 previous
years immediately preceding the relevant previous year; and
(b)He has stayed in India for a period of 730 days or more during 7 previous
years immediately preceding the relevant previous year.

Resident but not ordinarily resident


If a resident individual does not satisfy any or both of the additional conditions as
given u/s 6(6), he is “Resident but not ordinarily resident in India”.

Unique Academy - 8007916622 CA Saumil Manglani - Contact: 9921051593


2. Basics & Residential Status 2.15

Now let’s understand the amendments made by Finance Act 2020 i.e. Applicable from AY 21-22

A. Amendment added in the exceptional point of Indian Citizen or Person of Indian Origin coming to
India for the purpose of Visit

 In case of the citizen or person of Indian origin


 having total income,
AY 21 - 22
 other than the income from foreign sources
 Exceeding Rs. 15,00,000 during the previous year
Would be considered a resident if he stays in India for less than 182 days but minimum
120 days during the Relevant Previous Year
and
Stayed in India for minimum 365 days during the preceding 4 years

Now if such a person as stated above becomes a resident then he will be directly considered as
RNOR. Here preceding 7 and 10 years data would not be required to be checked

Note – "income from foreign sources" means income which accrues or arises outside India (except
income derived from a business controlled in or a profession set up in India)].

B. Newly added concept of Deemed Resident (Clause (1A) shall be inserted after clause (1) of section 6 by
the Finance Act, 2020, w.e.f. 1-4-2021) AY 21 - 22

Clause (1A)
 Notwithstanding anything contained in clause (1)
 An individual, being a citizen of India
 Having total income, other than the income from foreign sources,
 Exceeding fifteen lakh rupees during the previous year
 Shall be deemed to be resident in India in that previous year, if
 He is not liable to tax in any other country or territory by reason of his domicile or residence or any
other criteria of similar nature
 Such a person will be directly considered as RNOR [Sec 6(6)]

Important Points
• The fact that an assessee is resident in India in respect of one year does not automatically

mean that he would be resident in the preceding or succeeding years as well. Consequently,

the residential status of the assessee should be determined for each year separately.

This is in view of the fact that a person resident in one year may become non-resident or not

ordinarily resident in another year and vice versa.

• The period of stay required in each of the conditions need not necessarily be
continuous or consecutive nor it is stipulated that the stay should be at the usual place of
residence, business or employment of the individual. Purpose of stay is immaterial in
determining the residential status.

Unique Academy - 8007916622 CA Saumil Manglani - Contact: 9921051593


2. Basics & Residential Status 2.16

• The stay may be anywhere in India and for any length of time at each place in cases
where the stay in India is at more places than one, what is required is the total period of stay
should not be less than the number of days specified in each condition.

• Where the exact arrival and departure time is not available then the day he comes to India and
the day he leaves India is counted as stay in India.
• India means (Section [2(25A)] territory of India, its territorial waters, continental shelf,
Exclusive Economic Zone (up to 200 nautical miles) and airspace above its territory and
territorial waters.

Unique Academy - 8007916622 CA Saumil Manglani - Contact: 9921051593


2. Basics & Residential Status 2.17

Unique Academy - 8007916622 CA Saumil Manglani - Contact: 9921051593


2. Basics & Residential Status 2.18
Question 2

Mr. A, an Indian Citizen, is living in Mumbai since 1950, he left for China on July 1, 2016 and comes
back on August 7, 2020. Determine his residential status for the assessment year 2021-22.

Solution:
Stay in India for a minimum period of 182 days in the previous year:

Mr. A has stayed in India for 237 (viz. 25 + 30 + 31 + 30 + 31 + 31 + 28 + 31) days in the
Previous year 2020-21. So, this test is satisfied.
So, Mr. A shall be a resident in India during the previous year 2020-21. (Assessment year
2021-22). Keeping in view the facts of the given case, Mr. A satisfies the two additional conditions
also namely: He is resident in two out of ten previous years preceding the relevant previous year.

PY Stay in PY (days) Stay during Basic Condition Resident/Non-


Preceding Year Satisfied Resident
(days)
2019-20 Nil - None Non-Resident
2018-19 Nil - None Non-Resident
2017-18 Nil - None Non-Resident
2016-17 30+31+30+1= 92 366 days Second Resident
2015-16 366 365 First Resident

His stay in India is also more than 730 days in 7 previous years preceding the relevant previous year.
As he left for Japan on 1st July 2016.

PY Stay (days)
2019-20 Nil
2018-19 Nil
2017-18 Nil
2016-17 92
2015-16 366
2014-15 365
2013-14 365
Total Stay in preceding 7 Previous Years 1188

Hence, Mr. A is resident and ordinary resident in India for the assessment year 2021-22.

Unique Academy - 8007916622 CA Saumil Manglani - Contact: 9921051593


2. Basics & Residential Status 2.19

Question 3

Mr. Steve Waugh, the Australian cricketers comes to India for 100 days every year. Find out his
residential status for AY 2021-22.

Solution:

Step 1: The total stay of Steve Waugh in the last 4 preceding years is 400 days and his stay in India
during the previous year is 100 days. Since, he satisfied the second condition in section 6(1), he is
resident.

Step 2: Since his total stay in India in the last 7 years preceding the previous years is 700 days, he does not
satisfy the minimum requirements of 730 days in 7 years.

Therefore the residential status of Mr. Steve Waugh for the previous year 2020-21 is Resident but not
ordinarily resident in India.

Question 4

Dr. A, an Indian Citizen and a Professor in IIM, Luc now, left India on September 15, 2020 for USA to
take up Professors job in MIT, USA. Determine his residential status for the assessment year 2021-22.

Solution:

Dr. A being a citizen of India and who has gone out of the country for employment, will be governed by
182 days test only and therefore the second condition under section 6(1), i.e. 60 days during
relevant previous year shall not be applicable.

Dr. A stayed in India for 168 (viz. 30 + 31 + 30 + 31 + 31 + 15) days only in the relevant
previous year.

Hence, Dr. A shall be a non-resident in India for the assessment year 2021-22.as condition by stay of
182 days in relevant previous year is not satisfied.

Question 5
Basic Data
Mr. X is a foreign citizen. His father was born in Mumbai in 1960 and mother was born in USA in 1965. His
grandfather was born in Chennai in 1935. Mr. X is coming to India to see Taj Mahal and visit other historical
places in India. He comes to India on 1st November, 2020 for 200 days. He has never come to
India before. Determine his residential status for A.Y. 2021-22.
(Note – Part 2,3, and 4 are amendment based which got introduced in AY 21-22)
Part 2 of the question (Independent case taking above data as base)– What if the stayed for 300 days
during preceding 4 years and the Total income of the person is Rs. 16 Lacs
Part 3 of the question (Independent case taking above data as base) – What if the person stayed for 400
days during preceding 4 years and the Total Income of the individual is Rs. 14 Lacs
Part 4 of the question (Independent case taking above data as base) – What if the person stayed for 400
days during preceding 4 years and the Total Income of the individual is Rs. 14 Lacs
Unique Academy - 8007916622 CA Saumil Manglani - Contact: 9921051593
2.Basics & Residential Status 2.20

Solution:

Mr. X falls in exception to basic conditions as he is a Person of Indian Origin (as his grandfather was born
in undivided India) and he comes on a visit to India during relevant Previous year. Therefore, only first basic
condition of 182 days during relevant previous year would be checked.

Stay during relevant PY 2020-21 = 1st Nov, 2020 to 31st March, 2021 = 30+31+31+28+31 = 151
days Mr. X is Non-resident in India for PY 2020-21 as he does not satisfy first basic condition.

Part 2 – Person is still a Non-resident as the individual does not satisfy the condition of staying
minimum 365 days during the preceding 4 Years.

Part 3 - Person is still a Non-resident as the individual does not satisfy the condition of Total
income being more than 15 Lacs.

Part 4 – The person is a RNOR as all the conditions which have been mentioned in the amendment
have been satisfied.(You can refer back the table given on page 2.20)

Question 6

Mr. Anil, an Indian citizen, leaves India on 22nd September, 2020 for the first time to work as an Engineer
in France. Determine his residential status for A.Y. 2021-22.

Solutions:

During the previous year 2020-21, Mr. Anil, an Indian citizen, was in India for 175 days
(i.e. 30+31+30+31+31+22). He does not satisfy the minimum criteria of 182 days. Also since he is an Indian
citizen leaving India for the purpose of employment outside India, the second condition u/s 6(1) is
not applicable to him.
Therefore Mr. Anil is non-resident for the P.Y 2020-21

2. Hindu Undivided Families


The test to be applied to determine the residential status of a HUF is based upon the control and
management of the affairs of the assessee concerned. The tests based on the period of stay in India
applicable to individuals cannot be applied to these assessees for obvious reasons.

Meaning of place of control and management:


Control & management means -
• controlling & directive power;
• actual control & management (mere right to control & manage is not enough);
• central control & management and not the carrying out of day to day affairs.
• In other words, the Control and Management means taking policy decisions relating to business.
Policy decisions are concerning finance, marketing, production, advertising, personnel etc. It does not
mean day to day operations of the concern/assessee. The control and management is situated at
that place where policy decisions are taken.
• The business may be done from outside India and yet its control and management may be wholly
within India. Therefore, control and management of a business is said to be situated at a place where
the head and brain of the business is situated

Unique Academy - 8007916622 CA Saumil Manglani - Contact: 9921051593


2. Basics & Residential Status 2.21

An HUF can be “not ordinarily resident”


If manager/Karta has been a not ordinarily resident in India in the previous year in accordance with the
tests applicable to individuals.

Where, during the last ten years the kartas of the H.U.F. had been different from one another, the total
period of stay of successive kartas of the same family should be aggregated to determine the residential
status of the Karta and consequently the H.U.F.
In other words, if Karta of Resident HUF satisfies both the following additional conditions (as applicable
in case
of Individual) then Resident HUF will be ROR, otherwise it will be RNOR :

Additional Conditions:
(1)Karta of Resident HUF should be resident in at least 2 previous years out of 10 previous year
immediately preceding relevant previous year.
(2) Stay of Karta during 7 previous year immediately preceding relevant previous year should be 730
days or more.

Unique Academy - 8007916622 CA Saumil Manglani - Contact: 9921051593


2.Basics & Residential Status 2.22

[6(2)]HUF would

Resident Non-Resident

If control and management of the affairs is If control and management of the affairs is
Situated wholly or partly in India. Situated wholly outside India.

If the HUF is resident, then the status of the


Karta would determine whether the HUF is

ROR RNOR

If Karta is ROR, then If Karta is RNOR, then the


the HUF would be ROR HUF would be RNOR

Important Note:
• It is immaterial whether Karta is Resident or Non-Resident during relevant previous year, for the
purpose of determining whether HUF is ROR or RNOR. If Karta satisfies both the additional
conditions, then HUF will be ROR, otherwise RNOR.
• Firms, association of persons, local authorities and other artificial juridical persons can be either resident
ordinarily resident or non-resident in India but they cannot be not ordinarily resident in India.
• Even if negligible portion of the control and management of the affairs is exercised from India, it
will be sufficient to make the family, firm or the association resident in India for tax purposes.
A Hindu Undivided Family would generally be presumed to be resident in India unless the assessee
proves to the tax authorities that the control and management of its affairs is situated wholly outside India
during the relevant accounting year.

Unique Academy - 8007916622 CA Saumil Manglani - Contact: 9921051593


2. Basics & Residential Status 2.23

Question 7
An HUF, whose affairs of business are completely controlled from India. Determine its Residential status for A.Y.
2021-22 (a) if Karta is ROR in India for that year (b) If Karta is NR in India but he satisfies both the additional
conditions (c) If Karta is RNOR in India.
Solution:
HUF would be Resident in India as Control and Management is wholly situated in India. Determination of
whether HUF is ROR or RNOR:
a) HUF is ROR in India as Karta would be satisfying both the additional conditions (because he is ROR).
b) HUF is ROR in India as Karta is satisfying both the additional conditions. Karta’s Residential status
during relevant previous year (i.e. resident/non-resident) is irrelevant.
c) HUF is RNOR as Karta does not satisfy both the additional conditions.
Question 8

Karta of an Hindu Undivided Family comes to India for decision making every year for a minimum period of 60
days and maximum 91 days. Determine residential status of the HUF for the previous year 2020-21.

Solutions:

Hindu Undivided Family is resident since the Karta has come to India for decision ma ing at least 60 days but the stay
of Karta during seven years can be maximum 637 days. Hence HUF can be considered as resident but not
ordinarily resident.

Question 9

Hindu Undivided Family is being managed partly from Mumbai and partly from Japan. The Karta of HUF is a
foreign citizen and comes to visit in India every year since 1980 in the month of April for 105 days. Determine
residential status of HUF for PY 2020-21.
Solutions:
Since the control and management of the affairs of HUF is partly managed from Mumbai and therefore HUF is
resident in India. Further, the Karta of HUF is also satisfying both of additional conditions of section 6 (6)(b)
and therefore HUF is resident and ordinarily resident in India during the previous year 2020-21.

3. Companies [6(3)]
 All Indian companies within the meaning of Section 2(26) of the Act are
always resident in India regardless of the place of control and management
of its affairs.
 In the case of a foreign company the place of control and management of its
affairs is the basis on which the company’s residential status is determinable.
Accordingly a company shall be said to be resident in India in any previous
year, if –
(i) it is an Indian company; or

(ii) its place of effective management, in that year, is in India.


From Assessment Year 2017-18 a foreign company will be resident in India if its
Place of Effective Management (POEM) during the previous year is in India.
For this purpose, the Place Of Effective Management means a place where Key
management and commercial decisions that are necessary for the conduct of
the business of an entity as a whole are, in substance are made
Unique Academy - 8007916622 CA Saumil Manglani - Contact: 9921051593
2.Basics & Residential Status 2.24

Company

Indian Company Foreign Company

T.Over/ Gross Receipt > 50 T.Over/ Gross Receipt is upto


Always Resident
Crores during the year 50 Crores during the year

Non
POEM is in India ?
Resident

Yes No

Resident Non Resident

We need to understand the guiding principles for POEM but before it we need to understand the Phrase
Active Business Outside India.

Active Business outside India - A company shall be said to be engaged in “active business outside
India”
(i) if the passive income is upto 50% of its total income; and
(ii) less than 50% of its total assets are situated in India; and
(iii) less than 50% of total number of employees are situated in India or are resident in
India; and
(iv) the payroll expenses incurred on such employees is less than 50% of its total payroll
expenditure.
Explanation : For the aforesaid purpose :

the number of employees the number of employees shall be the average of the
number of employees as at the beginning and at the
end of the year and shall include temporary employees
(Contractors/ Labors)

Passive income “Passive income” of a company shall be aggregate of,


income from the transactions where both the purchase
and sale of goods is from / to its associated
enterprises; and income by way of royalty, dividend,
capital gains, interest or rental income;
Note: any income by way of interest shall not be
considered to be passive income in case of a
company which is engaged in the business of
banking or is a public financial institution,

Unique Academy - 8007916622 CA Saumil Manglani - Contact: 9921051593


2. Basics & Residential Status 2.25

Active Business Outside India

Example 1: Company A Co. is a sourcing entity, for an Indian multinational group, incorporated in country X
and is 100% subsidiary of Indian company (B Co.). The warehouses and stock in them are the only assets
of the company and are located in country X. All the employees of the company are also in country X. The
average income wise breakup of the company’s total income for three years is,

i. 30% of income is from transaction where purchases are made from parties which are non-associated
enterprises and sold to associated enterprises;

ii. 30% of income is from transaction where purchases are made from associated enterprises and sold to
associated enterprises;

iii. 30% of income is from transaction where purchases are made from associated enterprises and sold to
non-associated enterprises; and

iv. 10% of the income is by way of interest.

Interpretation : In this case passive income is 40% of the total income of the company. The passive
income consists of :

i. 30% income from the transaction where both purchase and sale is from/to associated enterprises; and

ii. 10% income from interest.

The A Co. satisfies the first requirement of the test of active business outside India. Since no assets or
employees of A Co. are in India the other requirements of the test is also satisfied. Therefore company is
engaged in active business outside India.

Example 2 : The other facts remain same as that in example 1 with the variation that A Co. has a total of 50
employees. 47 employees, managing the warehouse, storekeeping and accounts of the company, are
located in country X. The Managing Director (MD), Chief Executive Officer (CEO) and sales head are
resident in India. The total annual payroll expenditure on these 50 employees is of Rs. 5 crore. The annual
payroll expenditure in respect of MD, CEO and sales head is of Rs. 3 crore.

Interpretation: Although the first condition of active business test is satisfied by A Co. as only 40% of its total
income is passive in nature. further, more than 50% of the employees are also situated outside India. All
the assets are situated outside India. However, the payroll expenditure in respect of the MD, the CeO and
the sales head being employees resident in India exceeds 50% of the total payroll expenditure. Therefore, A

Unique Academy - 8007916622 CA Saumil Manglani - Contact: 9921051593


2.Basics & Residential Status 2.26

Co. is not engaged in active business outside India.

Example 3 : The basic facts are same as in example 1. further facts are that all the directors of the A Co.
are Indian residents. During the relevant previous year 5 meetings of the Board of Directors is held of which
two were held in India and 3 outside India with two in country X and one in country Y.

Interpretation : The A Co. is engaged in active business outside India as the facts indicated in example 1
establish. The majority of board meetings have been held outside India. Therefore, the POEM of A Co. shall
be presumed to be outside India.
Now Determination of POEM if -

Determination

ABOI Outside India ABOI Not outside India

Majority Board Meetings outside India Majority Board Meetings Not outside India Identify persons who make the key
management and key commercial
Resident decisions
Non Resident
and
Determine the place where decisions are
being made

 Business and the whole of it may be done outside India and yet the control
and management of that business may be wholly within India.
 It is entirely irrelevant where the business is done and where the income has
been earned. What is relevant and material is from which place has that
business been controlled and managed.

Unique Academy - 8007916622 CA Saumil Manglani - Contact: 9921051593


2. Basics & Residential Status 2.27

Question 10

AB & Co. is a partnership firm whose operations are carried out in India. However, all meetings of
partners take place outside India as all the partners are settled abroad. Determine Residential status of
firm for A.Y. 2021-22.

Solution:

AB & Co. is Non-Resident in India during relevant previous year as Control and Management (place where
policy decisions are ta en, here it is the place where meetings are held) is wholly situated outside India.

Once the residential status of an individual is identified, the rule of taxation could be applied to determine
whether an income is to be taxed or not.
Meaning and Scope of Total Income

The income is bifurcated into 5 categories


a. Income Received in India
b. Income accrued in India
c. Income deemed to be received in India
d. Income deemed to be accrued in India
e. Income accrued outside India

Explanation 1. - Income accruing or arising outside India shall not be deemed to be


received in India within the meaning of this section by reason only of the fact that it is
taken into account in a balance sheet prepared in India.

Income received in India


Tax point: Receipt is different from remittance. The receipt of income refers to the
first occasion when the recipient gets the money under his control. Once the amount
is received as income (at any place outside India), any subsequent remittance or
transmission of the amount to India does not result to receipt in India
Example: Mr. X, a non-resident, received dividend from an Italian company in Japan
on 15/12/2020. On 17/12/2020 he remitted such income in India. Such income shall
not be taxable in India as income has neither received in India nor accrued in India.

Income Deemed to be received in India


Following incomes shall be deemed to be received in India and taxable in hands of
all assessee irrespective of their residential status -
(a) The annual accretion in the previous year to the balance at the credit of an
employee participating in a recognized provident fund
• Employer’s contribution to the recognized provident fund in excess of
12% of salary.
• Interest credited on the above balance by a rate exceeding 9.5% [Sec.
7(i)]
(b) The transferred balance in recognized provident fund, to the extent liable to
income tax [Sec. 7(ii)]

Unique Academy - 8007916622 CA Saumil Manglani - Contact: 9921051593


2.Basics & Residential Status 2.28
(c) The contribution made, by the employer in the previous year, to the account of
an employee under a pension scheme notified u/s 80CCD [Sec. 7(iii)]
(d) Tax Deducted at source [Sec. 198]
(e) Income from undisclosed sources

Following incomes are deemed to accrue or arise in India.

Section 9(1) The following incomes shall be deemed to accrue or arise in India :—

(i)Income accruing or arising through -

a) Any asset or source of income in India.


b) Transfer of a capital asset situated in India.
c) Any Business connection in India
(Only for Understanding - In proportion to operations carried out in India)

Following Explanation 3A shall be inserted after Explanation 3 to clause (i) of sub-section (1) of
section 9 by the Finance Act, 2020, w.e.f. 1-4-2021 :
Explanation 3A.— Income attributable to the operations carried out in India shall include income
from—
AY 21 - 22 (i) such advertisement which targets a customer who resides in India or a customer who
accesses the advertisement through internet protocol address located in India;
(ii) sale of data collected from a person who resides in India or from a person who uses
internet protocol address located in India; and
(iii) sale of goods or services using data collected from a person who resides in India or from a
person who uses internet protocol address located in India.

Unique Academy - 8007916622 CA Saumil Manglani - Contact: 9921051593


2. Basics & Residential Status 2.29

Amendment in section 9 (AY 19-20)

(I) in Explanation 2, for clause(a) i.e. business connection shall include any business activity
carried out through a person who, acting on behalf of the non-resident –

“(a) has and habitually exercises in India, an authority to conclude contracts on behalf of
the non-resident or habitually concludes contracts or habitually plays the principal role
leading to conclusion of contracts by that non-resident and the contracts are––
(i) in the name of the non-resident; or
(ii) for the transfer of the ownership of, or for the granting of the right to use, property
owned by that non-resident or that non-resident has the right to use; or
(iii) for the provision of services by the non-resident
(b) Maintains Stock and delivers on behalf of the NR
(c) Secures orders on behalf of the NR

 Exceptions to the business connection, In case of Non Residents (Means in the following cases
the business connection is not formed for purpose of taxing the income)–
1. Operations confined to purchase of goods from India for Export.
2. Person engaged in the business of running a news agency or of publishing newspapers,
magazines or journals and the activities are confined to the collection of news and views in
India for transmission out of India.
3. operations which are confined to the shooting of any cinematograph film in India provided if –
a) Assessee -> An Individual -> Not Citizen of India
b) Assessee -> A Firm -> No partner is Citizen of India
c) Company -> No Shareholder citizen of India or Resident of India
4. In the case of a foreign company engaged in the business of mining of diamonds no income
shall be deemed to accrue or arise in India to it through or from the activities which are confined
to the display of uncut and unassorted diamond in any special zone notified by the Central
Government in the Official Gazette in this behalf.

(ii) Income from Salary which is earned in India.


Amount earned for the rest period or leave period which is preceded and succeeded by services
rendered in India and forms part of the service contract of employment.

(iii) Income chargeable under the head "Salaries" payable by the Government to a citizen of India for
service outside India (Other than Perquisites and allowances)

(iv) Dividends paid by an Indian company outside India.

Unique Academy - 8007916622 CA Saumil Manglani - Contact: 9921051593


2.Basics & Residential Status 2.30

(v) In Case of interest, royalty and technical fees following things should be kept in mind –
a) In case it is paid by Government of India, it shall always accrue in India.
b) In case it is paid by resident, it shall always accrue in India except where money borrowed is
used for
- The purpose of business or profession carried outside India or
- For making or earning income from any source outside India.
c) In case it is paid by the Non Resident person, where such person uses the money borrowed
for a business or profession carried on or in India.

viii) income arising outside India, being any gifts paid on or after the 5th day of July, 2019 by a person
resident in India to a non-resident, not being a company, or to a foreign company.”

Residential Status and Scope of Total Income (Section 5)


Scope of total Income Resident and Resident but not Non-
Ordinarily Resident Ordinarily Resident Resident
Income received or deemed to be Yes Yes Yes
received in India during the previous
year
Income accruing or arising or Yes Yes Yes
deeming to accrue or arise in India
during the previous year
Income accruing or arising outside Yes, even if such income Yes, if income is derived from No
India is not received or brought a business controlled from or
into India during the profession set up in India;
previous year Otherwise, No.

Unique Academy - 8007916622 CA Saumil Manglani - Contact: 9921051593


2. Basics & Residential Status 2.31

Practice Question

Compute the total income in the hands of an individual, being a resident and ordinarily resident,
resident but not ordinarily resident, and non-resident for the A.Y. 2021-22
Particulars Amount (₹)
Interest on Uk Development Bonds, 50% of interest received in India 10,000
Income from a business in Chennai (50% is received in India) 20,000
Profits on sale of shares of an Indian company received in London 20,000
Dividend from British Company received in London 5,000
Profits on sale of plant at Germany 50% of profits are received in India 40,000
Income earned from Business in Germany which is controlled from Delhi 70,000
(₹40,000 is received in India)
Profits from a business in Delhi but managed entirely from London 15,000
Income from House Property in London deposited in a Indian Bank at London, 50,000
brought to India (computed)
Interest on debentures in an Indian company received in London. 12,000
Fees for technical services rendered in India but received in London 8,000
Profits from a business in Bombay managed from London 26,000
Pension for services rendered in India but received in Burma 4,000
Income from property situated in Pakistan received there 16,000
Past foreign untaxed Income brought to India during the previous year 5,000
Income from agricultural land in Nepal received there and then brought to India 18,000
Income from profession in Kenya which was set up in India, received there but 5,000
spent in India
Gift received on the occasion of his wedding 20,000
Interest on Savings Bank Deposit in State Bank of India 12,000
Income from a business in Russia, controlled from Russia 20,000
Dividend from Reliance Petroleum Limited, an Indian company 5,000
Agricultural income from a land in Rajasthan 15,000

Solution:

Computation of total income for the A.Y. 2021-22

Particulars Resident Resident Non-


and but not resident
ordinarily ordinarily
resident resident
₹ ₹ ₹
Interest on UK Development Bonds, 50% of interest received in India 10,000 5,000 5,000
Income from a business in Chennai (50% is received in India) 20,000 20,000 20,000
Profits on sale of shares of an Indian company received in London 20,000 20,000 20,000
(assuming that they are in the nature of short-term capital gains)
Dividend from British company received in London 5,000 - -
Profits on sale of plant at Germany, 50% of profits are received in India 40,000 20,000 20,000
Income earned from business in Germany which is controlled from 70,000 70,000 40,000
Delhi, out of which₹40,000 is received in India
Profits from a business in Delhi but managed entirely from London 15,000 15,000 15,000
Income from property in London deposited in a Bank at London, later on 50,000 - -
remitted to India
Interest on debentures in an Indian company received in London 12,000 12,000 12,000

Unique Academy - 8007916622 CA Saumil Manglani - Contact: 9921051593


2.Basics & Residential Status 2.32

Fees for technical services rendered in India but received in London 8,000 8,000 8,000
Profits from a business in Bombay managed from London 26,000 26,000 26,000
Pension for services rendered in India but received in Burma 4,000 4,000 4,000
Income from property situated in Pakistan received there 16,000 - -
Past foreign untaxed income brought to India during the previous year - - -
Income from agricultural land in Nepal received there and then brought 18,000 - -
to India
Income from profession in Kenya which was set up in India, received 5,000 5,000 -
there but spent in India
Gift received on the occasion of his wedding [not taxable] - - -
Interest on savings bank deposit in State Bank of India 12,000 12,000 12,000
Income from a business in Russia, controlled from Russia 20,000 - -
Dividend from Reliance Petroleum Limited, an Indian Company [Exempt - - -
under section 10(34)]
Agricultural income from a land in Rajasthan [Exempt under section - - -
10(1)]
Gross Total Income 3,51,000 2,17,000 1,82,000
Less: Deduction under section 80TTA [Interest on savings bank account
subject to a maximum of₹10,000] 10,000 10,000 10,000
Total Income 3,41,000 2,07,000 1,72,000

Question 1: Mr. X earns the following income during the previous year ended 31st March, 2021. Determine the income
liable to tax for the assessment year 2021-22 if Mr. X is (a) resident and ordinarily resident in India,
(b) resident and not ordinarily resident in India, and (c) non-resident in India during the previous year ended 31st
March, 2021.
– Profits on sale of a building in India but received in Holland – Rs. 20,000
– Pension from former employer in India received in Holland – Rs. 14,000
– Interest on U.K. Development Bonds (1/4 being received in India) – Rs. 20,000
– Income from property in Australia and received in U.S.A. – Rs. 15,000
– Income earned from a business in Abyssinia which is controlled from Zambia (Rs. 30,000 received in
India) – Rs. 70,000
– Dividend on shares of an Indian company but received in Holland
– Profits not taxed previously brought into India – Rs. 40,000
– Profits from a business in Nagpur which is controlled from Holland – Rs. 27,000
Solution: Computation of income liable to tax:

Particular Resident & Resident but not Non-Resident


Ordinarily (Rs.) Ordinarily Resident (Rs.)
(Rs.)
Profits on sale of a building in India but 20,000 20,000 20,000
received in Holland (accrued in India received
outside India)
Pension from former employer in India 14,000 14,000 14,000
received in Holland (accrued in India, received
out of India)
Interest on U.K. Development Bonds (Accrued 20,000 5,000 5,000
out of India, 1/4th received in India)
Income from property in Australia and 15,000 Nil Nil
received in U.S.A. (Accrued and received out
of India)

Unique Academy - 8007916622 CA Saumil Manglani - Contact: 9921051593


2. Basics & Residential Status 2.33

Income earned from a business in Abyssinia 70,000 30,000 30,000


which is controlled from Zambia (Business
controlled outside India)
Dividend on shares of an Indian company 10,000 10,000 10,000
but received in Holland (Accrued in India)
Profits not taxed previously brought into India Nil Nil Nil
(Not an income so not taxable)

Profits from a business in Nagpur which is 27,000 27,000 27,000


controlled from Holland (Accrued in India)

Total 1,76,000 1,06,000 1,06,000

Question 2: A had the following income during the previous year ended 31st March, 2021:
– Salary Received in India for three Months - Rs. 9,000
– Income from house property in India(Computed)- Rs. 13,470
– Interest on Saving Bank Deposit in State Bank of India-Rs. 1,000
– Amount brought into India out of the past untaxed profits earned in Germany- Rs. 20,000
– Income from agriculture in Indonesia being invested there-Rs. 12,350
– Income from business in Bangladesh, being controlled from India- Rs. 10,150
– Dividends received in Belgium from French companies, out of which Rs. 2,500 were remitted to India-
Rs. 23,000
You are required to compute his Gross total income for the assessment year 2021-22 if he is: (1) a resident;
(ii) a not ordinarily resident, and (iii) a Non-resident.
Solution: Computation of total income of A is given below:

Particular Resident & Resident but Non-Resident


Ordinarily not Ordinarily (Rs.)
Resident (Rs.) Resident (Rs.)
Salary Received in India for three Months (Indian 9,000 9,000 9,000
received in India)
Income from house property in India (Income 13,470 13,470 13,470
accrue or arise in India)
Interest on Saving Bank Deposit in State Bank of 1,000 1,000 1,000
India (Income accrue or arise in India)
Amount brought into India out of the past untaxed Nil Nil Nil
profits earned in Germany ( not an income, hence
not taxable)
Income from agriculture in Indonesia being 12,350 Nil Nil
invested there (Income accrue or arise in outside
India)

Unique Academy - 8007916622 CA Saumil Manglani - Contact: 9921051593


2.Basics & Residential Status 2.34

Income from business in Bangladesh, being 10,150 10,150 Nil


controlled from India (it is supposed that the money
is not received in India) (Income accrued outside
India from a business controlled from India)
Dividends received in Belgium from French 23,000 Nil Nil
companies (Income accrue outside India India
remittance is irrelevant)

Total 68,970 33,620 23,470

Question 3: Mr. Y earns the following income during the previous year ended on 31st March, 2021 Determine
the Gross Total Income for the assessment year 2021-22 if Mr. Y is (a) resident and ordinary resident (b) resident
and not ordinary resident, and (c) non-resident in India during the previous year ended on 31st March, 2021.
(i) Honorarium received from Government of India (Travelling and other incidental expenses of Rs. 7,000
were incurred in this connection)- Rs. 20,000
(ii) Profits earned from a business in Tamilnadu controlled from Pakistan - Rs. 50,000
(iii) Profits earned from a business in U.K. controlled from Delhi-Rs. 30,000
(iv) Profits earned from a business in U.S.A. controlled from Pakistan and amount deposited in a bank
there- Rs. 40,000
(v) Income from a house property in France, received in India-Rs. 10,000
(vi) Past untaxed foreign income brought into India during the year-Rs. 25,000
(vii) Dividends from a German company credited to his account in Pakistan- Rs. 35,000
(viii) Dividends declared but not received from an Indian company- Rs. 20,000
(ix) Agricultural income from Burma not remitted to India-Rs. 40,000
(x) Pension for services rendered in India, but received in Pakistan-Rs. 30,000

Solution: Computation of Gross Total Income of Mr. Y is given below:

Particular Resident & Resident but Non-Resident


Ordinarily not Ordinarily (Rs.)
Resident (Rs.) Resident (Rs.)
Honorarium received from Govt. of India 20,000 20,000 20,000
Profits earned from a business in Tamilnadu 50,000 50,000 50,000
controlled from Pakistan (Income accrue or arise
in India)
Profits earned from a business in U.K. controlled 30,000 30,000 -
from Delhi (Income accrue or arise outside India
from a business controlled from
India)
Profits earned from a business in USA controlled 40,000 - -
from Pakistan and amount deposited in a Bank
there (Income accrue outside India)
Income from a house property in France, 10,000 10,000 10,000
received in India (Income received in India)
Past untaxed foreign income brought into India - - -
during the year (Not taxable as profit of past
years, also remittance is irrelevant)

Unique Academy - 8007916622 CA Saumil Manglani - Contact: 9921051593


2. Basics & Residential Status 2.35

Particular Resident & Ordinarily Resident but not Non-Resident (Rs.)


Resident (Rs.) Ordinarily
Resident (Rs.)
Dividends from a German 35,000 - -
company credited to his
account in Pakistan
Dividends declared but not 20,000 20,000 20,000
received from an Indian
company
Agricultural income from 40,000 - -
Burma not remitted to India
Pension for services 30,000 30,000 30,000
rendered in India, but
received in Pakistan
GTI 2,75,000 1,60,000 1,30,000

COMPUTATION OF TAXABLE INCOME AND TAX LIABILITY OF AN ASSESSEE

Income tax is a charge on the assessee’s income. Income Tax law lays down the provisions for computing
the taxable income on which tax is to be charged. Taxable income of an assessee shall be calculated in
the following manner.
1. Determine the residential status of the person as per section 6 of the Act.
2. For calculation of income, amount received is classified under 5 heads of income; it is then to
be adjusted with reference to the provisions of the Income Tax laws in the following manner.

Particulars Amount (Rs.)


Income under the head :
+ Income from Salaries XXX
+ Income from House Property XXX
+ Profits and gains of business or profession XXX
+ Capital gains XXX
+ Income from other sources XXX
Adjustment in respect of :
+ Clubbing of Income XXX
- Set off and carry forward of losses (XXX)

= Gross Total Income XXX


- Deductions under section 80C to 80U (or Chapter VIA) (XXX)
= Total Income XXX

TAX RATES for PY 2020-21 (AY 2021-22)

Calculation of Tax on Income


 Tax rate depends upon the category of person
 Amount of income
 Residential status of person
 Age of individual
 Type of Income

Unique Academy - 8007916622 CA Saumil Manglani - Contact: 9921051593


2.Basics & Residential Status 2.36

Components of Tax

Education Tax
Tax Surcharge Cess SHEC Payable

Tax Rates for Different types of person depending upon various parameters:
1. For :
 Resident Individual of the age below 60 years
 Non Residents Individual
 Hindu undivided family
 Association of Persons
 Body of Individuals (other than Co-operative society)
 Artificial Juridical Person

Total Income (Rs.) Tax Rate Tax liability (Rs.)


up to 2,50,000 Nil Nil
2,50,001 – 5,00,000 5% 5% of (Total Income – 2,50,000)
5,00,001 – 10,00,000 20% 20% of (Total Income – 5,00,000) + 12,500
Above 10,00,000 30% 30% of (Total Income – 10,00,000) + 1,12,500
2. Applicable for:
Resident individual of the age of 60 years or more but less than eighty years at any time during the previous year

Total Income (Rs.) Tax Rate Tax liability (Rs.)


up to 3,00,000 Nil Nil
3,00,001 – 5,00,000 5% 5% of (Total Income – 3,00,000)
5,00,001 – 10,00,000 20% 20% of (Total Income – 5,00,000) + 10,000
Above 10,00,000 30% 30% of (Total Income – 10,00,000) + 1,10,000
3. Applicable for:
Resident Individual of the age of 80 years or more at any time during the previous year

Total Income (Rs.) Tax Rate Tax liability (Rs.)


up to 5,00,000 Nil Nil
5,00,001 – 10,00,000 20% 20% of (Total Income – 5,00,000)
Above 10,00,000 30% 30% of (Total Income – 10,00,000) + 1,00,000

Clarification regarding attaining prescribed age of 60 years/80 years on 31st March itself, in case of
senior/very senior citizens whose date of birth falls on 1st April [Circular No. 28/2016, dated 27-07-
2016]
An individual who is resident in India and of the age of 60 years or more (senior citizen) and 80 years or more (very
senior citizen) is eligible for a higher basic exemption limit of Rs. 3,00,000 and Rs. 5,00,000, respectively.
The CBDT has, vide this Circular, clarified that a person born on 1st April would be considered to have attained a
particular age on 31st March, the day preceding the anniversary of his birthday. In particular, the question of
attainment of age of eligibility for being considered a senior/very senior citizen would be decided on the basis of
above criteria.
Therefore, a resident individual whose 60th birthday falls on 1st April, 2018, would be treated as having attained the
age of 60 years in the P.Y. 2017-18, and would be eligible for higher basic exemption limit of Rs. 3 lakh in computing
his tax liability for A.Y. 2018-19. Likewise, a resident individual whose 80th birthday falls on 1st April, 2018, would be
treated as having attained the age of 80 years in the P.Y. 2017-18, and would be eligible for higher basic exemption
limit of Rs. 5 lakh in computing his tax liability for A.Y. 2018-19.

Unique Academy - 8007916622 CA Saumil Manglani - Contact: 9921051593


2. Basics & Residential Status 2.37

New Section 115BAC inserted for Individuals & HUF – AY 21-22 (These rates Will be discussed after
chapter 11)

115BAC. (1) Notwithstanding anything contained in this Act but subject to the provisions of this
Chapter, the income-tax payable in respect of the total income of a person, being an individual or a
Hindu undivided family, for any previous year relevant to the assessment year beginning on or after
the 1st day of April, 2021, shall, at the option of such person, be computed at the rate of tax given in
the following Table, if the conditions contained in sub-section (2) are satisfied, namely:—
TABLE
1. Up to Rs. 2,50,000 Nil

2. From Rs. 2,50,001 to Rs. 5,00,000 5 per cent.


3. From Rs. 5,00,001 to Rs. 7,50,000 10 per cent.
4. From Rs. 7,50,001 to Rs. 10,00,000 15 per cent.
5. From Rs. 10,00,001 to Rs. 12,50,000 20 per cent.
6. From Rs. 12,50,001 to Rs. 15,00,000 25 per cent.
7. Above Rs. 15,00,000 30 per cent.:

Note - The above option of rates is subject to various conditions which can be discussed only after you
have studied with the major part of Income Tax. So please have patience. “Sabra ka Fal Meetha hota hai”

4. For others:

Types of person Tax Rates


i. Firms (including LLP) 30% of total Income
ii. Local Authorities 30% of total Income
iii. Domestic Companies • A domestic company, whose Gross turnover or gross
receipt in the previous year 18 - 19 does not exceed Rs
400 crore, shall be taxable at rate of 25% for
assessment year 2021-22. (Amended by Finance Act
2020)

• 30% of total Income in all other cases


iv. Companies other than a domestic 40% of total income;
company

Special Tax rates for Companies (To be discussed in detail Chapter 15)

Section 115BA – Certain manufacturing companies may chose to pay taxes @ 25%
Section 115BAA - Certain manufacturing companies may chose to pay taxes @22% (plus
surcharge@10% and HEC@4%)
Section 115BAB - Certain manufacturing companies may chose to pay taxes @15% (plus
surcharge@10% and HEC@4%)

Unique Academy - 8007916622 CA Saumil Manglani - Contact: 9921051593


2.Basics & Residential Status 2.38

Surcharge

Sr. Rate of
No. Person Total Income Surcharge
upto Rs. 50 Lakhs No Surcharge
> Rs. 50 lakhs upto
Rs. 1 10%
crore
1 Individual/ HUF/ AOP/ BOI/ AJP > Rs. 1 crore upto Rs. 15%
2 crore
> Rs. 2 crore upto Rs. 25%
5 crore
> Rs. 5 crore 37%
Firm/ LLP/ Co-operative Society/ Local upto Rs. 1 Crore No Surcharge
2
Authority > Rs. 1 crore 12%
upto Rs. 1 Crore No Surcharge
> Rs. 1 crore upto Rs.
3 Domestic Company 10 7%
crore
> Rs. 10 crore 12%
upto Rs. 1 Crore No Surcharge
> Rs. 1 crore upto Rs.
4 Foreign Company 10 2%
crore
> Rs. 10 crore 5%

Surcharge is an additional tax imposed on certain cases. It is imposed over the basic tax rate calculated
on the income.
For example : Suppose total taxable income of an individual of 45 years is Rs. 1,30,00,000, then Base tax
will be : Rs. 1,12,500 + 30% of (1,20,00,000)= Rs. 37,12,500.
Surcharge @15% of Rs. 37,12,500 = Rs. 5,56,875. There are different rate of surcharge prescribed in the
Following manner:

Marginal Relief in Surcharge: When an assesses taxable income exceeds Rs.50 lakhs, 1 crore, 2 crore, 5
crore / 10 crore , he is liable to pay Surcharge at prescribed rates mentioned above on Income Tax payable
by him. However, the amount of Income Tax and surcharge on total income shall not exceed the amount of
income that exceeds Rs. 50 lakhs, 1 crore, 2 crore, 5 crore , 10 crore.

Example : Suppose Mr. Ram an individual assessee of 42 years is having taxable income of Rs.
1,00,01,000/-
1. Income Tax on 1,00,01,000/- Rs. 28,12,800
2. Surcharge @15% of Income Tax Rs. 4,21,920
3. Income Tax including Surcharge Rs. 32,34,720
4. Income Tax on income of Rs. 1 crore Rs. 30,93,750
including 10% Surcharge
5. Increase in Tax – Rs. 1,39,970
Increase in income
6. Income Tax after marginal relief ( Rs. Rs. 30,94,750
32,34,720 - Rs. 1,39,970)

Unique Academy - 8007916622 CA Saumil Manglani - Contact: 9921051593


2. Basics & Residential Status 2.39

Thus, in the above case, though the surcharge @15% is Rs. 421920. However, since the income of Mr. Ram
exceeds Rs. 1 crore by just Rs. 1,000, Ram will be eligible for marginal relief and maximum surcharge will be
restricted to Rs. 1,000 only.

Health and Education cess


 Governments resort to imposition of cess for meeting specific expenditure
 Health and Education cess is an additional levy on the basic tax liability + surcharge, if applicable.
 Rate of HEC is 4% (Amended by Finance Act 2018).

Rebate u/s 87A

Rebate u/s 87A Applicable to: Resident Individual Conditions to be satisfied: Total income of the
assessee does not exceed ₹
5,00,000.

Quantum of Rebate: Lower of the following: (a) 100% of tax liability as computed above; or (b) ₹
12,500/-

Steps involved in calculation of Tax on Total Income

Unique Academy - 8007916622 CA Saumil Manglani - Contact: 9921051593


2.Basics & Residential Status 2.40

Particulars Amount Rs.

Tax on Special Incomes @ specified tax rates (Long term capital gains @ 20%; Casual Income @
XXX
30% and Short term capital gains (on Securities transaction tax paid securities) @ 15%;

Add : Tax on Balance Income @ Slab Rate/Flat Rate (as applicable) XXX
Total Tax XXX

XXX
Add : Surcharge, if any

Less : Marginal Relief, if applicable


(XXX)

Tax including Surcharge XXX

Add : HEC @ 4% on tax including surcharge XXX

Tax liability XXX

Add : Interest under Section 234A/234B/ 234C XXX


Net tax liability XXX
Less : Taxes paid by way of :

– Tax deducted at source (TDS) (XXX)


– Advance tax (XXX)
– Self Assessment Tax (XXX)
– Double Taxation Relief (XXX)
Tax Payable/Refundable
XXX

Question
Mr. Raghav aged 26 years, has a total income of ₹ 4,80,000, comprising his salary income and
interest on bank fixed deposit. Compute his tax liability for A.Y.2021-22.
Solution
Computation of tax liability of Mr. Raghav for A.Y.2021-22
Tax on total income of ₹ 4,80,000
@5% of ₹ 2,30,000 (₹ 4,80,000 – ₹ 2,50,000) ₹11,500
Less: Rebate u/s 87A ₹ 11,500
Nil

Rounding Off of :

Income Tax payable (Including refund if applicable)


Section 288A Section 288B
Nearest multiple of ₹ 10 Nearest multiple of ₹ 10
Example: ₹4,52,554.94 To ₹4,52,550. Example: ₹52,554.94 To ₹52,550.
>=5 is rounded off to next multiple of 10

Application Vs Diversion of Income

Sno Application of Income Diversion of income

1. Obligation to apply the income Obligation to apply the income before it is received or accrued.
after it is received or accrued or
arisen.
2. Treated as income, hence taxable. Not treated as income. Hence, not taxable.

Unique Academy - 8007916622 CA Saumil Manglani - Contact: 9921051593


2. Basics & Residential Status 2.41

CASE LAWS

1. What is the nature of liquidated damages received by a company from the supplier of plant for failure
to supply machinery to the company within the stipulated time – a capital receipt or a revenue receipt?
CIT v. Saurashtra Cement Ltd. (2010) 325 ITR 422 (SC)
Facts of the case: One of the conditions in the agreement was that if the supplier failed to supply the machinery
within the stipulated time, the assessee would be compensated at 0.5% of the price of the respective portion of the
machinery without proof of actual loss. The assessee received ₹ 8.50 lakhs from the supplier by way of liquidated
damages on account of his failure to supply the machinery within the stipulated time. The Department assessed the
amount of liquidated damages to income-tax. However, the Appellate Tribunal held that the amount was a capital
receipt and the High Court concurred with this view.

Supreme Court’s Decision: The Apex Court affirmed the decision of the High Court holding that the damages were
directly and intimately linked with the procurement of a capital asset i.e., the cement plant, which lead to delay in coming
into existence of the profit-making apparatus. It was not a receipt in the course of profit earning process. Therefore, the
amount received by the assessee towards compensation for sterilization of the profit earning source, is not in the ordinary
course of business, hence it is a capital receipt in the hands of the assessee.

2. Can capital contribution of the individual partners credited to their accounts in the books of the firm
be taxed as cash credit in the hands of the firm, where the partners have admitted their capital
contribution but failed to explain satisfactorily the source of receipt in their individual hands?

CIT v. M. Venkateswara Rao (2015) 370 ITR 212 (T & AP)

High Court’s Decision: The High Court, accordingly, held that the view taken by the Assessing Officer that the
partnership firm has to explain the source of income of the partners as regards the amount contributed by them towards
capital of the firm, in the absence of which the same would be treated as the income of the firm, was not tenable.

ICSI Module Questions


FILL IN THE BLANKS
(1) The residential status of an assessee is determined for the relevant ....................................................
.........
(2) The incidence of tax on any assessee depends upon his .............................................................. under
the Act.
(3) If control and management of the affairs HUF is situated wholly outside India it would become a ........
.....................................................
TRUE AND FALSE
(1) An Indian company is always resident in India no matter where and to what extent its place of effective
management is situated.
(2) Sandeep Ltd. is a company registered in Japan. The place of effective management of its affairs is in
India. Sandeep Ltd. is non-resident company in India.

Unique Academy - 8007916622 CA Saumil Manglani - Contact: 9921051593


2.Basics & Residential Status 2.42

FILL IN THE BLANK


1. Previous year;
2. Residential status;
3. Non-Resident
TRUE AND FALSE
1 True; 2 False

Questions for practice

Solved Questions (Residential Status Individual)


QUESTION 1
Sam came to India first time during the P.Y. 2020-21. During the previous year, he stayed in India for (i) 50 days;
(ii) 183 days; & (iii) 153 days. Determine his residential status for the A.Y. 2021-22.

Solution
 Since Sam resides in India only for 50 days during the P.Y. 2020-21, he does not satisfy any of the conditions
specified in sec. 6(1). He is, therefore, a non-resident in India for the P.Y. 2020-21.
 Since Sam resides in India for 183 days during the P.Y. 2020-21, he satisfies one of the conditions specified in
sec. 6(1). He is, therefore, a resident in India for the P.Y. 2020-21.
 Sam resides in India only for 153 days during the P.Y. 2020-21. Though he resided for more than 60 days
during the previous year but in 4 years immediately preceding the previous year (as he came India first time), he
did not reside in India. Hence, he does not satisfy any of the conditions specified in sec. 6(1). Thus, he is a non-
resident for the P.Y. 2020-21.
QUESTION 2 (Residential Status Individual)
Andy, a British national, comes to India for the first time during 2016-17. During the financial years 2016- 17, 2017-
18, 2018-19 and 2019-20, he was in India for 55 days, 60 days, 80 days, 160 days and 70 days respectively.
Determine his residential status for the AY 2021-22.

Solution
During the P.Y. 2020-21, Andy was in India for 70 days & during 4 years immediately preceding the previous year,
he was in India for 355 days as shown below:

Year 2016-17 2017-18 2018-19 2019-20 Total


No. of days stayed in India 55 60 80 160 355

Thus, he does not satisfy Sec.6(1) & consequently, he is a non-resident in India for the P.Y. 2020-21.

QUESTION 3 (Residential Status Individual)


Miss Pal, an Indian citizen, left India for first time on 1st April, 2019 for joining job in Tokyo. She came to
India on 11th October, 2020 for only 190 days. Determine her residential status for P.Y. 2020-21.

Solution
Number of days Miss Pal stayed in India can be calculated as under:

P.Y. Apr May June July Aug Sep Oct Nov Dec Jan Feb Mar Total
19-20 1 - - - - - - - - - - - 1
20-21 - - - - - 21 30 31 31 28 31 172
Since she left India for employment purpose, hence for becoming resident she has to stay in India for at least 182
days. However, she is in India for only 172 days during the previous year, thus she is a non-resident for the P.Y.
2020-21.

Unique Academy - 8007916622 CA Saumil Manglani - Contact: 9921051593


2. Basics & Residential Status 2.43

QUESTION 4 (Residential Status Individual)


Mr. X, aged 19 years, left India for first time on May 31, 2020. Determine his residential status for the P.Y.
2020-21 if:
(i) He left India for employment purpose (ii) He left India on world tour.
Solution
During the P.Y. 2020-21, Mr. X was in India for 61 days as shown below –

P.Y. Apr May June July Aug Sep Oct Nov Dec Jan Feb Mar Total

20-21 30 31 - - - - - - - - - - 61

During the P.Y. 2020-21, X stayed in India for 61 days. Further, he was in India for more than 365 days during 4
years immediately preceding the relevant previous year (as he left India for first time).

(i) Since he left India for employment purpose, condition of sec. 6(1)(c) shall not be applicable on such
assessee. He will be treated as resident in India, if and only if, he resided in India for at least 182 days during the
previous year. Hence, Mr. X is a non-resident in India for the P.Y. 2020-21.

(ii) Since he left India on world tour, which is not an exception of sec. 6(1), satisfaction of any one condition
of sec. 6(1) makes him resident in India for the P.Y. 2020-21. As he satisfies 2nd condition of sec. 6(1) [shown
above], he is resident in India. Further, he also satisfies dual conditions specified u/s 6(6) (since he left India for first
time). Therefore, he is an ordinarily resident for the P.Y. 2020-21.

QUESTION 5 (Residential Status Individual

X, a foreign citizen, resides in India during the P.Y. 2020-21 for 83 days. Determine his residential status
for P.Y. 2020-21 assuming his stay in India during the last few previous years are as follows –

Year Days Year Days Year Days


2009-10 36 days 2013-14 137 days 2017-18 175 days
2010-11 115 2014-15 265 days 2018-19 15 days
days
2011-12 123 2015-16 310 days 2019-20 67 days
days
2012-13 65 days 2016-17 121 days

Solution
During P.Y. 2020-21, X was in India for 83 days & during 4 years immediately preceding the previous year, he was
in India for 378 days as shown below:

Year 2016-17 2017-18 2018-19 2019-20 Total


No. of days stayed in 121 175 15 67 378
India

Thus, he satisfies one of the conditions specified u/s 6(1) & consequently, he becomes resident in India in the P.Y. 2020-
21. Further, to determine whether X is an ordinarily resident or not, he needs to satisfy both conditions laid down u/s 6(6).

Unique Academy - 8007916622 CA Saumil Manglani - Contact: 9921051593


2.Basics & Residential Status 2.44

Condition
Presence in India satisfied to
Year (In Days) Resident or Non resident become a
resident
2019-2020 67 Resident 6(1)(c)
2018-2019 15 Non-Resident None
2017-2018 175 Resident 6(1)(c)
2016-2017 121 Resident 6(1)(c)
2015-2016 310 Resident Both
2014-2016 265 Resident Both
2013-2014 137 Non Resident None
65 Non Resident None
2012-2013
2011-2012 123 Resident 6(1)(c)
2010-2011 115 Resident 6(1)(c)

Condition (i) of sec. 6(6) requires that an individual should be resident in India for at least 2 out of 10 years preceding
the relevant previous year. X was resident in India for 8 out of 10 years immediately preceding the previous year. Thus,
he satisfies this condition.
Condition (ii) of sec. 6(6) requires that an individual should be present in India for at least 730 days during 7 years
preceding to relevant previous year. X was in India for 1090 days during 2013-14 to 2019-20. Hence, he satisfies this
condition also.
X satisfies condition (ii) of sec. 6(1) as well as both the conditions of sec. 6(6). Thus, he is a resident and ordinarily
resident in India for the P.Y. 2020-21

QUESTION 6
Miss Monica, a foreign national, comes India every year for 90 days since 2003-04.
(a) Determine her residential status for the P.Y. 2020-21.
(b) Will your answer differ, if she comes India for 100 days instead of 90 days every year?

Solution
(a) Since Miss Monica stayed for 90 days during the P.Y. 2020-21 and for 360 days (90 days * 4 years) during the 4
years immediately preceding the previous year, hence, she is not satisfying any of the conditions of sec. 6(1). Thus,
she is a non-resident for the P.Y. 2020-21.
(b) Since Miss Monica stayed for 100 days during the P.Y. 2020-21 and for 400 days (100 days * 4 years) during the
4 years immediately preceding the previous year, hence, she is satisfying sec. 6(1)(c). Thus, she is resident for the
P.Y. 2020-21. Further, she resides for only 700 days (100 days * 7 years) during the 7 years immediately preceding
the previous year. Hence, she does not satisfy one of the conditions of sec. 6(6). Thus, she is resident but not
ordinarily resident for the P.Y. 2020-21.

QUESTION 7
Mr. Sid, a British national, joined XYZ Co. Ltd. as an engineer in India on 1st May, 2010. On 31st December, 2011, he
went to Sri Lanka on deputation. On 1st April, 2016, he came back to India and left for Sri Lanka again on 31st May,
2016. He returned to India and joined his original post on 1st July, 2020. Determine his residential status for the A.Y.
2021-22.

Solution
Number of days Mr. Sid stayed in India in past few years can be calculated as under:

Unique Academy - 8007916622 CA Saumil Manglani - Contact: 9921051593


2. Basics & Residential Status 2.45

SN P.Y. Apr May June July Aug Sep Oct Nov Dec Jan Feb Mar Total
0 20-21 - - - 31 31 30 31 30 31 31 28 31 274
1 19-20 - - - - - - - - - - - - 0
2 18-19 - - - - - - - - - - - - 0
3 17-18 - - - - - - - - - - - - 0
4 16-17 30 31 - - - - - - - - - - 61
5 15-16 - - - - - - - - - - - - 0
6 14-15 - - - - - - - - - - - - 0
7 13-14 - - - - - - - - - - - - 0
8 12-13 - - - - - - - - - - - - 0
9 11-12 30 31 30 31 31 30 31 30 31 - - - 275
10 10-11 - 31 30 31 31 30 31 30 31 31 28 31 335

On the basis of data drawn, residential status of Mr. Sid in last few years can be decided as under:

Presence in India Resident (R) or Condition satisfied


Year Previous
(In days) Nonresident to become a
Year
(NR) resident
1 19-20 0 NR None
2 18-19 0 NR None
3 17-18 0 NR None
4 16-17 61 NR None
5 15-16 0 NR None
6 14-15 0 NR None
7 13-14 0 = 61 NR None
8 12-13 0 NR None
9 11-12 275 R 6(1)(a)
10 10-11 335 R 6(1)(a)

Since assessee resided in India for 274 days in the P.Y. 2020-21, hence he satisfies sec. 6(1)(a). Therefore, he is
resident in India.
Further, since he is resident in India for 2 years out of 10 years preceding the previous year (as shown in above
working), but resided in India for less than 730 days out of 7 immediately preceding years, hence he does not satisfy
one of the conditions of sec. 6(6), therefore, he is resident but not ordinarily resident.
Conclusion: Resident but not ordinarily resident.

Unique Academy - 8007916622 CA Saumil Manglani - Contact: 9921051593


3.Exempt Income 3.1

Unique Academy - 8007916622 CA Saumil Manglani - Contact: 9921051593


3. Exempt Income 3.2

Background

Tax is calculated on the total income of an individual for the previous year. For
providing relief to the tax payer, income Tax laws provides for exemption,
deduction and rebate. The exempt income is often confused with the deductions
and rebate. However there is difference between these concepts. The same has
been explained in the table below:

• Exemptions are claimed on the basis of the source of


income.
Exemption • The exempted income is not included in the total Income
of the assessee.

• Deductions are allowed on the basis of the payments/


investments made.
Deductions • The tax deductions are allowed under different heads of
income as well as from the Gross Total Income.

• Rebate is a percentage amount reduced from total


income tax payable.
Rebate • Tax rebate is allowed as a reduction to the total tax
payable.

There are several incomes that do not form part of the total income of the
assessees, which are entailed u/s 10 of the Act. Being exempt, these do not
enter the computation of taxable incomes therefore.

Incomes exempt u/s 10


will NOT be included at
all for computing Gross Deductions under Chapter

Total Income VI-A would call for first


including the items in the
Gross Total Income and
then the deductions would
be allowable

Unique Academy - 8007916622 CA Saumil Manglani - Contact: 9921051593


3.Exempt Income 3.3

INCOMES WHICH DO NOT FORM PART OF TOTAL INCOME

Agricultural Income [Section 10(1)]

Section 10(1) states that agricultural incomes are not included in the total income

Broadly 3 sources

through agriculture
Rent or Revenue derived from
farm building required for
land situated in India and used or
agricultural operations
for agricultural purposes process ordinarily employed by a
cultivator or receiver of rent in
kind to render the produce fit
to be taken to the market
or

the sale of such agricultural


produce in the mar et.

Note – Ownership of land is not necessary. Thus, the rent received by the original tenant
from sub-tenant would also be agricultural income.
Agriculture includes 2 operations

Basic Operations Subsequent operations

Those operations by agriculturists which Operations to be performed after the produce


are absolutely necessary for the purpose of sprouts from the land (e.g., weeding, digging
effectively raising produce from the land etc.) are subsequent operations. These
are the basic operations subsequent opera- tions would be agricultural
operations only when taken in conjunction with
and as a continuation of the basic operations.

Income from farm building:


Income arising from the use of farm building would be considered as agricultural income if
the following conditions are satisfied –

(a) building should be on or in the immediate vicinity of the land; and


(b) It should be used as a dwelling house or as a store house for agricultural produce.

Unique Academy - 8007916622 CA Saumil Manglani - Contact: 9921051593


3. Exempt Income 3.4

Following conditions are also to be satisfied –


a. The land should either be assessed to land revenue in India or be subject to a local rate assessed
and collected by the officers of the Government as such
Or
b. It has to be within the limits mentioned below –

• Population upto 9999 – whole of the land is Rural agricultural land

Population upto 2 kms. Urban इसके आगे का Rural Land


>= 10K
upto 1 Lac

Population >
1 Lacs upto upto 6 kms Urban इसके आगे का Rural Land
10 Lacs

Population >
upto 8 kms Urban इसके आगे का
10 Lacs
Rural Land

Examples:
Area Shortest aerial Population according to the Would income
distance from the last preceding census of derived from farm
local limits of a which the relevant figures building situated in
municipality or have been published before this area be treated as
cantonment board the first day of the previous agricultural income?
referred to in item a. year

(i) A 1 km 9,000 Yes


(ii) B 1.5 kms 12,000 No
(iii) C 2 kms 11,00,000 No
(iv) D 3 kms 80,000 Yes
(v) E 4 kms 3,00,000 No
(v) F 5 kms 12,00,000 No
(vi) G 6 kms 8,000 Yes
(vii) H 7 kms 4,00,000 Yes
(viii) I 8 kms 10,50,000 No
(ix) J 9 kms 15,00,000 Yes

Unique Academy - 8007916622 CA Saumil Manglani - Contact: 9921051593


3.Exempt Income 3.5

Whether Income from Nursery Constitutes Agricultural Income?


Yes, As Per Explanation 3 To Section 2(1A), Income Derived from Saplings Or Seedlings Grown In A
Nursery Would Be Deemed To Be Agricultural Income, Whether Or Not The Basic Operations Were
Carried Out On Land.

In order to render the produce, fit for being taken to market for sale, the activities would
include all activities, like cleaning, drying, winnowing, crushing etc. Example, let’s assume
the process being referred to is obtaining rice from paddy, the process
ordinarily employed by the cultivator would include:

⚫ Removing hay from basic grains

⚫ Removing chaff from the grains

⚫ Filtering the grain to remove stones

⚫ Packing the rice in gunny bags

Therefore, all activities, manual or otherwise, all the processes would be included in and therefore
constitute the activities deployed to render the produce fit for being taken to the market.

Reference Case Law - Dy. CIT v. Best Roses Biotech (P) Ltd., 49 SOT 277.

• Assessee acquired land from agriculturist on lease and constructed a greenhouse


floriculture project on said land. It started growing of rose flowers
/ plants on bridge of plastic trays erected with help of a stand 2.3 ft. above
land.
• The assessee claimed the income from rose flowers as exempt.
• The Assessing Officer held that the rose plants were not planted on earth land and
no basis operation was carried out by assessee on land hence, not eligible for
exemption.
• According to assessee, for plantation of roses a very well treated soil was
required, manures were mixed in soil for preparing a base for growing rose plants
trays were filed with mixture of soil, insecticides were sprinkled on plants to save
plants from any disease, mother plant was otherwise reared on earth,
subsequently saplings were planted on plastic trays which were kept at height of
2-3 ft. placed on a stand,
Purpose of growing rose plants at a height was primarily to avoid pest and to develop in a controlled
atmosphere and green house was used for various benefits so that sunlight and humidity level both could be
maintained. The Tribunal held that the claim of exemption was justified.
Unique Academy - 8007916622 CA Saumil Manglani - Contact: 9921051593
3. Exempt Income 3.6

Manner of Calculating Agricultural Income

Rule 7 - Where the income is partially agricultural income and partially business income, the market
value of any agricultural produce so raised by the assessees, which has been further utilised /
processed in such business will be allowed as a deduction in such business.

Determination of market value - There are two possibilities here:


(I) The agricultural produce is capable of being sold in the market → either in its raw stage or after
application of any ordinary process to make it fit to be taken to the market → the value calculated
at the average price at which it has been so sold during the relevant previous year will be the market
value.
(II) It is possible that the agricultural produce is not capable of being ordinarily sold in the market in
its raw form or after application of any ordinary process. In such case the market value will be the
total of the following:—
• The expenses of cultivation;
• The land revenue or rent paid for the area in which it was grown; and
• Such amount as the Assessing Officer finds having regard to the circumstances in each case to
represent at reasonable profit.
Illustration 1

Mr. B grows sugarcane and uses the same for the purpose of manufacturing sugar in his
factory. 30% of sugarcane produce is sold for ₹ 10 lacs, and the cost of cultivation of such
sugarcane is ₹ 5 lacs. The cost of cultivation of the balance sugarcane (70%) is ₹ 14 lacs and
the market value of the same is ₹ 22 lacs. After incurring ₹ 1.5 lacs in the manufacturing
process on the balance sugarcane, the sugar was sold for ₹ 25 lacs. Compute B’s business
income and agricultural income
Solution: Income from sale of sugarcane gives rise to agricultural income and from sale of sugar gives
rise to business income.
Business income = Sales (–) Market value of 70% of sugarcane produce (–)
Manufacturing expenses
= Rs 25 lacs – Rs. 22 lacs – Rs. 1.5 lacs = Rs. 1.5 lacs.
Agricultural = Market value of sugarcane produce – Cost of cultivation
income

= [ Rs. 10 lacs – Rs. 5 lacs] + [ Rs. 22 lacs – Rs. 14 lacs]

= 5 lacs + 8 lacs = Rs. 13 Lacs

0 Lacs 14L 22L 25L

Unique Academy - 8007916622 CA Saumil Manglani - Contact: 9921051593


3.Exempt Income 3.7

Specified types of agricultural Incomes

Income tax Amount of


Amount of non-
Nature of income Rule
agricultural income agricultural
applicable income
Income from
sale of tea
Rule 8 40% of such
manufactured 60% of such income
income
or grown in India
Income from
growing and
Rule 7A 35% of such
manufacturing 65% of such income
income
of rubber
Income derived from sale of
Rule 7B(1) 25% of such
coffee 75% of such income
income
grown and cured in India
Income derived from sale of
coffee grown, cured, roasted
Rule 7B(1A) 40% of such
and grounded 60% of such income
income
is India

Illustration 2

Nikhil manufactures latex from rubber plants grown by him in India. These are subsequently sold in the
market at INR 50,00,000. The costs incurred are as under:

⚫ Manufacturing Latex: INR 12,00,000


⚫ Growing Rubber Plants: INR 18,00,000
The treatment is explained as under:
Particulars INR

Total Income 50,00,000

Total Costs 30,00,000

Profits 20,00,000

Business Income 7,00,000

Agricultural Income 13,00,000

Note: the business income chargeable to tax under the head “Profits and Gains from Business /
Profession” is taken @ 35% of the profits and the agricultural income is taken @ 65%, which is
subsequently exempt from tax.

Unique Academy - 8007916622 CA Saumil Manglani - Contact: 9921051593


3. Exempt Income 3.8
Scheme of Partial Integration

Calculation when there is a combination of Agricultural and Non - Agricultural


Income

This is applicable to Individuals, HUF, AOP, BOI & AJP and the conditions subject to the fulfilment
of which this is allowable are:

1) The net agricultural income must be > INR 5000/- p.a.; &

2) The non-agricultural income must be > the maximum amount not chargeable to tax (which is INR
250,000 for all individuals / HUF’s; INR 300,000 for senior citizens and INR 500,000 for very
senior citizens)

The manner of computation of Income in such cases would be as under:asasasa

Refer to the illustration below which will explain this better.

Particulars INR

Income from salary 2,50,000

Income from house 1,25,000


property

Net Agricultural Income 1,00,000

Total (A) 4,75,000

Tax Liability 11,250

Total (B) 3,50,000

Tax Liability 5,000

(A)-(B) 6,250

Less: Rebate 87A[12500 or 250


tax payable i.e. 6,250
whichever is lower]
Total tax payable Nil

Note:
1) The (A) would be the combined income and the tax liability is computed per the current slabs and
rates

2) The (B) would be the net agricultural income as increased by the minimum exemption amount and
the tax liability is computed per the current slabs and rates

3) The (A)-(B) would be the tax liability on which cess is added to determine the total tax liability
Unique Academy - 8007916622 CA Saumil Manglani - Contact: 9921051593
3.Exempt Income 3.9

Other Exempted Incomes

Amount received by a member of the HUF from the income of the HUF
[Section 10(2)]

As per section 10(2), amount received out of family income, or in case of impartible
estate, amount received out of income of family estate by any member of such HUF is
exempt from tax

Share of profit received by a partner from the firm [Section 10(2A)]

As per section 10(2A), share of profit received by a partner from a firm is exempt from tax
in the hands of the partner. Further, share of profit received by a partner of LLP from the
LLP will be exempt from tax in the hands of such partner. This exemption is limited only to
share of profit and does not apply to interest on capital and remuneration received by the
partner from the firm/LLP.

Interest to non-residents [Section 10(4)]

In the case of an individual, any income by way of interest on money standing to his credit in
a Non-Resident (External) Account in any bank in India in accordance with the Foreign
Exchange Management Act, 1999, and the rules made thereunder is exempt from tax.
Royalty Income or Fees for technical services received from National Technical Research
Organisation [Section 10(6D)] AY 19-20

Any income arising to a non-resident (not being a company), or a foreign company,


by way of royalty from, or fees for technical services rendered in or outside India to,
the National Technical Research Organisation.

Compensation for Bhopal Gas Leak Disaster [Section 10(10BB)]

Compensation in accordance with Bhopal Gas Leak Disaster (Processing of Claims) Act,
1985] received by victims of Bhopal gas leak disaster is exempt from tax. However,
compensation received for any expenditure which is allowed as deduction from taxable
income is not exempt

Compensation on account of any disaster [Section 10(10BC)]

Any amount received from the Central Government or State Government or a Local
Authority by an individual or his legal heirs as compensation on account of any disaster is
exempt from tax. However, no deduction is available in respect of the amount received or
receivable to the extent such individual or his legal heirs has been allowed a deduction under
the Act on account of loss or damage caused due to such disaster.

Unique Academy - 8007916622 CA Saumil Manglani - Contact: 9921051593


3. Exempt Income 3.10

Amount paid on life insurance policy [Section 10(10D)]

As per section 10(10D), any amount received under a life insurance policy, including
bonus is exempt from tax. Following points should be noted in this regard:

✓ Exemption is available only in respect of amount received from life insurance


policy. Exemption under section 10(10D) is unconditionally available in respect of sum
received for a policy which is issued on or before March 31, 2003.

✓ However, in respect of policies issued on or after April 1st, 2003, the exemption is
available only if the amount of premium paid on such policy in any financial year does not
exceed 20% of the actual capital sum assured

✓ 10% in respect of policy taken on or after 1st April, 2012) of the actual
capital sum assured.

✓ With effect from 1-4-2013, in respect of policy taken in the name of a person
suffering from diseases specified under section 80DDB or in the name of a person
suffering from disability specified under section 80U, the limit will be increased to 15% of
capital sum assured.

✓ Amount received on death of the person will continue to be exempt without any
condition.

✓ Insurance taken for disabled person but because of his death it is paid to the legal
guardian/family member → The amount received will be taxable.
Note : No exemption would be available in case of any sum received under Key man
insurance policy. Amount of Keyman Insurance policy is taxable under 3 heads, Salary,
PGBP or IOS depending upon the circumstances.
Explanation 1.—For the purposes of this clause, "Keyman insurance policy" means a life
insurance policy taken by a person on the life of another person who is or was the
employee of the first-mentioned person or is or was connected in any manner whatsoever
with the business of the first-mentioned person and includes such policy which has been
assigned to a person, at any time during the term of the policy, with or without any

Interest Incomes [Section 10(15)]

Interest, Redemption Premium, on notified securities, bonds, certificates, deposits etc., are
exempt for all assessees.
Educational scholarship [Section 10(16)]

Any amount received as educational scholarship (i.e., scholarship to meet the cost of
education) is exempt from tax in the hands of recipient.
Daily allowance to a Member of Parliament [Section 10(17)]
Are exempt from tax in the hands of a Member of Parliament and a Member of State
Legislature.

Unique Academy - 8007916622 CA Saumil Manglani - Contact: 9921051593


3. Exempt Income 3.11

Awards [Section 10(17A)]

Any payment received in pursuance of following (whether paid in cash or in kind) is


exempt from tax:

Pension to gallantry award winner [Section 10(18)]

Pension received by an individual who was employee of the Central Government or


State Government and who has been awarded Param Vir Chakra or Maha Vir
Chakra or Vir Chakra or any other notified gallantry award is exempt from tax.

After death, Family pension received by any member of such individual is also
exempt.
Family
a) Spouse and children
b) Parents, brothers and sisters - wholly or mainly dependent

Family pension received by the family members of armed forces [Section 10(19)]
Received by
⚫ the widow or children or nominated heirs,
⚫ of a member of armed forces (including paramilitary forces) of the Union,
is exempt from tax in the hands of such family members, if
⚫ the death of such member of armed forces has occurred in the course of operational duty
in prescribed circumstances and subject to such conditions as may be prescribed.

Exemption of disability pension granted to disabled personnel of armed forces who have
been invalided on account of disability attributable to or aggravated by such service
[Circular No. 13/2019, dated 24.6.2019]

• The entire disability pension,


• to members of naval, military or air forces

Unique Academy - 8007916622 CA Saumil Manglani - Contact: 9921051593


3. Exempt Income 3.12

• would be exempt from tax.


• available to all armed forces personnel (irrespective of rank)
• Bodily disability caused by such service

Income of a news agency [Section 10(22B)]

Any income of a notified news agency, set-up in India solely for collection and
distribution of news is exempt from tax provided that the news agency applies its
income or accumulates it for application solely for collection and distribution of news
and does not distribute its income in any manner to its members. (Ex. The Press Trust
of India Ltd. (New Delhi)

Income from fund or trust or institution or university or other educational institution


or hospital or other medical institution[Section 10(23C) subject to certain conditions

:
Income of mutual funds is exempt subject to certain conditions [Section 10(23D)]

Exemption in respect of certain income of wholly owned subsidiary of Abu Dhabi


Investment Authority and Sovereign Wealth Fund [Section 10(23FE)]
AY 21 - 22

• Incomes Exempt - Dividend, interest or long-term capital gains


• Investment made by it in India
• whether in the form of debt or equity
• In a company or enterprise carrying on the business of developing, or operating and maintaining,
or developing, operating or maintaining any infrastructure facility of section 80-IA

Explanation.—For the purposes of this clause, "infrastructure facility" means—


(a) a road including toll road, a bridge or a rail system;
(b) a highway project including housing or other activities being an integral part of the highway project;
(c) a water supply project, water treatment system, irrigation project, sanitation and sewerage system
or solid waste management system;
(d) a port, airport, inland waterway, inland port or navigational channel in the sea;
• Investment should be made between 1st April 20 upto 31st March, 2024 and
• is required to be held for at least three years
Income of a registered trade union [Section 10(24)]

Any income chargeable under the head “Income from house property” and “Income from
other sources” of a registered union formed primarily for the purpose of regulating the
relation between workmen and employers or between workmen and workmen is exempt from
tax.

Unique Academy - 8007916622 CA Saumil Manglani - Contact: 9921051593


3. Exempt Income 3.13

Specified income of a Sikkimese Individual [Section 10(26AAA)]


(i) The following income, which accrues or arises to a Sikkimese individual, would be exempt
from income- tax –
(a) income from any source in the State of Sikkim; or
(b) income by way of dividend or interest on securities.
However, this exemption will not be available to a Sikkimese woman who, on or after 1st
April, 2008, marries a non-Sikkimese individual.

Reverse Mortgage [Section 10(43)]

Any amount received by an individual as a loan, either in lump-sum or in


installment in a transaction of reverse mortgage referred in clause (xvi) of
Section 47 shall be exempted.

New Pension System Trust [Section 10(44)]

Any income received by any person for, or on behalf of the New Pension
System Trust shall be exempted.
Exemption of Income of a foreign company from sale of Crude Oil in India [Section
10 (48)]

Any income of a foreign Co. received in India in Indian currency on account of sale of crude oil
to any person in India shall be exempt if the following conditions are satisfied.

• Such Income is in pursuant to an agreement or an arrangement entered into by the Central


Govt. or approved by the Central Govt.;
• having regard to the national interest, the foreign company and the agreement or
arrangement are notified by the Central Govt. in this behalf; and
the foreign company is not engaged in any activity, other than receipt of such income, in
India.
• 10 (48B) Sale of leftover of such oil on termination if agreement is also exempt.

ncome accruing or arising to Indian Strategic Petroleum Reserves Limited


(ISPRL) [Section 10(48C)]
• Any income accruing or arising to
AY 21 - 22
• ISPRL
• a wholly owned subsidiary of Oil industry development Board
• under the Ministry of Petroleum and Natural Gas, as
• for replenishment (refilling) of crude oil stored in its storage facility
• Condition -That the crude oil is replenished in the storage facility within 3 years
from the end of the financial year in which the crude oil was removed from the
storage facility for the first time.

Unique Academy - 8007916622 CA Saumil Manglani - Contact: 9921051593


3. Exempt Income 3.14
Other important exemptions

Section 10AA provides for exemption in respect of income of newly established


units in Special Economic Zones

Special Provision in respect of Newly established Units in SEZ [Sec. 10AA]

Applicable to: All assessee

Conditions to be satisfied

1. The undertaking has begun or begins to manufacture or produce articles or


things or provide services on or after 01/04/2005 but upto 31st March 2021
in any SEZ.

2. Clarification on issues relating to export of computer software


AY 21 - 22 The profits and gains derived from on site development of computer software (including
services for development of software) outside India shall be deemed to be the profits
and gains derived from the export of computer software outside India

3. New Business: Business should not be formed by splitting up or reconstruction


of an existing business.

However, this condition is not applicable where the business has been revived
now due to the loss suffered in the past because of any natural calamities and
Such business is re-established, reconstructed or revived by the assessee at
any time before the expiry of 3 years from the end of previous year in which
such damage was caused.
3.Further Conditions

(i) It should not be formed by the transfer of machinery or plant previously used for any
purpose to a new business.
(ii) However, deduction under section 10AA will be available if total value of the machinery or
plant transferred does not exceed 20% of the total value of machinery or plant used in the
business.
(iii) For this purpose, any machinery or plant which was used outside India by any person
other than the assessee shall not be regarded as machinery or plant previously used for
any purpose if the following conditions are fulfilled:
(a) such machinery or plant was not at any time used in India;
(b) such machinery or plant is imported into India from any country outside India; and
(c) no deduction on account of depreciation has been allowed in respect of such
machinery or plant to any person earlier.
(iv) CA’s report is required

Unique Academy - 8007916622 CA Saumil Manglani - Contact: 9921051593


3.Exempt Income 3.15

Quantum of Deduction

Period Deduction
For first 5 years (Profits of the business of the undertaking × Export turnover)
from the ---------------------------------------------------------
commenceme
nt of operation Total turnover of the business carried on by the undertaking

50% of [Profits of the business of the undertaking × Export turnover]


For next 5
------------------------------------------------------------------
years
Total turnover of the business carried on by the undertaking

50% of [Profits of the business of the undertaking × Export turnover]

-----------------------------------------------------------------

Total turnover of the business carried on by the undertaking


Or
Amount invested in “SEZ Re-investment Allowance Reserve A/c”
(Whichever is Lower)

Conditions:

a) the amount credited to the Special Economic Zone Re-investment Reserve


For next 5
Account is to be utilised -
years
(i) for the purpose of acquiring machinery or plant which is first put to use before the expiry of
a period of three years following the previous year in which the reserve was created; and

(ii) until the acquisition of the machinery or plant as aforesaid, for the purposes of the
business of the undertaking other than for distribution by way of dividends or profits or for
remittance outside India as profits or for the creation of any asset outside India;

Misutilization of Reserve: Where any amount credited to such reserve -

(a) Has been misutilised; or

(b) Has not been utilised before the expiry of the specified period,

– then such amount shall be deemed to be the taxable profits of the previous year in

which the amount was so misutilised or in the year after the expiry of 3 years, as
the case may be.

Unique Academy - 8007916622 CA Saumil Manglani - Contact: 9921051593


3. Exempt Income 3.16

4.

Illustration to explain exemption u/s 10AA

Rajveer Turbines has 2 undertakings, one in a SEZ and one in a normal zone. The
summarised results are as under:

Particulars SEZ Normal

Domestic turnover 50 125

Export turnover 200 0

Gross Profit 75 25

Expenses & 15 10
Depreciation

Net profit 60 15

Compute the business income of the assessee. The treatment is as under:

Total profit 75

Less : Exempt u/s 48


10AA

Taxable profits 27

Calculation of Deduction

Step 1: Calculate the total income of the assessee as per the provisions of the act, but before allowing
deduction under section 10AA.

Step 2 : From the amount calculated in Step 1, allow the deduction under section 10AA, which is least
of the following;

– Amount calculated under step 1; or

– Amount deductible under section 10AA

In Case of Amalgamation/ Demerger – This allowance will be available to the new company from the
year in which the new company is formed.

Few Clarifications

Meaning of Export turnover: It means the consideration received in India or brought into India by the
assessee in respect of export by the undertaking being the unit of articles or things or services.
However, it does not include
• freight
• telecommunication charges
• insurance

Unique Academy - 8007916622 CA Saumil Manglani - Contact: 9921051593


3.Exempt Income 3.17

attributable to the delivery of the articles or things outside India or expenses incurred
in foreign exchange in rendering of services (including computer software) outside
India. These are to be excluded both from "export turnover" and "total turnover'

Miscellaneous Exemptions

1. Post Office Savings Bank Account be exempt from tax for any assessment year
only to the extent of:

i. Rs. 3,500 in case of an individual account.


ii. Rs. 7,000 in case of a joint account.

Questions for Concept Clarity 😉

1. Mr. C manufactures latex from the rubber plants grown by him in India. These are then sold in
the market for ₹ 30 lacs. The cost of growing rubber plants is ₹ 10 lacs and that of
manufacturing latex is ₹ 8 lacs. Compute his total income.

Solution

The total income of Mr. C comprises of agricultural income and business income. Total
profits from the sale of latex= ₹ 30 lacs – ₹ 10 lacs – ₹ 8 lacs = ₹ 12 lacs.
Agricultural income = 65% of ₹ 12 lacs. = ₹ 7.8 lacs

Business income = 35% of ₹ 12 lacs. = ₹ 4.2 lacs

2. Mr. X, a resident, has provided the following particulars of his income for the P.Y. 2020-21.
i. Income from salary (computed) - ₹ 1,80,000

ii. Income from house property (computed) - ₹ 2,00,000

iii. Agricultural income from a land in Jaipur - ₹ 2,80,000

iv. Expenses incurred for earning agricultural income - ₹ 1,70,000 Compute his tax
liability assuming his age is -
(a) 45 years

(b) 70 years

Solution

Computation of total income of Mr. X for the A.Y. 2021-22

(a) Computation of tax liability (age 45 years)

For the purpose of partial integration of taxes, Mr. X has satisfied both the conditions i.e.
1. Net agricultural income exceeds ₹ 5,000 p.a., and
2. Non-agricultural income exceeds the basic exemption limit of ₹ 2,50,000. His tax liability is
computed in the following manner

Unique Academy - 8007916622 CA Saumil Manglani - Contact: 9921051593


3. Exempt Income 3.18

Particulars ₹ ₹
Income from salary 1,80,000
Income from house property 2,00,000
Net agricultural income 1,10,000
[2,80,000 – 1,70,000]

Less: Exempt under section 10(1) (1,10,000) -


Gross Total Income 3,80,000
Less Deductions under Chapter VI-A -
Total Income 3,80,000

Step 1: ₹ 3,80,000 + ₹ 1,10,000 = ₹ 4,90,000

Tax on ₹ 4,90,000 = ₹ 12,000 (i.e., 5% of ₹2,40,000)

Step 2 : ₹ 1,10,000 + ₹ 2,50,000 = ₹ 3,60,000

Tax on ₹ 3,60,000 = ₹ 5,500 (i.e. 5% of ₹1,10,000)

Step 3: ₹ 12,000 – ₹ 5,500 = ₹ 6,500

Step 4 – Less Rebate u/s 87A = ₹ 6,500

Step 5: Total tax payable = Nil

(b) Computation of tax liability (age 70 years)

For the purpose of partial integration of taxes, Mr. X has satisfied both the conditions i.e.

1. Net agricultural income exceeds ₹ 5,000 p.a., and

2. Non-agricultural income exceeds the basic exemption limit of ₹ 3,00,000. His tax liability is
computed in the following manner:
Step 1: ₹ 3,80,000 + ₹ 1,10,000 = ₹ 4,90,000

Tax on ₹ 4,90,000 = ₹ 9,500 (i.e. 5% of ₹1,90,000)

Step 2: ₹ 1,10,000 + ₹ 3,00,000 = ₹ 4,10,000

Tax on ₹ 4,10,000 = ₹ 5,500 (i.e. 5% of ₹1,10,000)

Step 3: ₹ 9,500 – ₹ 5,500 = ₹ 4,000

Step 4 – Less Rebate u/s 87A = ₹ 6,500

Step 5: Total tax payable = Nil

3. Y Ltd. furnishes you the following information for the year ended 31.3.2021:

Unique Academy - 8007916622 CA Saumil Manglani - Contact: 9921051593


3. Exempt Income 3.19

Particulars ₹ (in lacs)

Total turnover of Unit A located in Special Economic Zone 100

Profit of the business of Unit A 30

Export turnover of Unit A 50

Total turnover of Unit B located in Domestic Tariff Area (DTA) 200

Profit of the business of Unit B 20

Compute deduction under section 10AA for the A.Y. 2021-22

Solution

100% of the profit derived from export of articles or things or services is eligible for
deduction under section 10AA,assuming that F.Y. 2020-21 falls within the first five year
period commencing from the year of manufacture or production of articles or things or
provision of services by the Unit in SEZ.

As per section 10AA(7), the profit derived from export of articles or things or services shall
be the amount which bears to the profits of the business of the undertaking, being the
Unit, the same proportion as the export turnover in respect of articles or things or services
bears to the total turnover of the business carried on by the undertaking.

Deduction under section 10AA


(Profit of the business of Unit A X Export Turnover of Unit A )/Total Turnover of Unit A
= (₹30Lakhs X50)/100
=15Lakhs

3. Examine whether the following are chargeable to tax, and if so, compute the amount
liable to tax:

(i) Arvind received ₹ 20,000 as his share from the income of the HUF.

(ii) Mr. Xavier, a ‘Param Vir Chakra’ awardee, who was formerly in the service of the
Central Government, received a pension of ₹ 2,20,000 during the financial year 2020-
21.

(iii) Agricultural income of ₹ 1,27,000 earned by a resident of India from a land situated
in Malaysia.

(iv) Rent of ₹ 72,000 received for letting out agricultural land for a movie shooting.

Answer

Unique Academy - 8007916622 CA Saumil Manglani - Contact: 9921051593


3. Exempt Income 3.20

S.No. Taxable/ Amount Reason


Not Taxable liable to
tax (₹)

(i) Not Taxable - Share received by member out of the income of the HUF
is exempt under section 10(2).
(ii) Not Taxable - Pension received by Mr. Xavier, a former Central
Government’s employee who is a ‘Param Vir Chakra’ awardee, is
exempt under section 10(18).
(iii) Taxable 1,27,000 Agricultural income from a land in any foreign country is taxable
in the case of a resident taxpayer as income under the head
“Income from other sources”. Exemption under
section 10(1) is not available in respect of such income.

(iv) Taxable 72,000 Agricultural income is exempt from tax as per section 10(1).
Agricultural income means, inter alia, any rent or revenue
derived from land which is situated in India and is used for
agricultural purposes. In the present case, rent is being
derived from letting out of agricultural land for a movie shoot,
which is not an agricultural purpose. In effect, the land is not
being put to use for agricultural purposes. Therefore, ₹ 72,000,
being rent received from letting out of agricultural land for
movie shooting, is not exempt under section 10(1). The same
is chargeable to
tax under the head “Income from other sources”.

5. Whether the income derived from saplings or seedlings grown in a nursery is


taxable under the Income-tax Act, 1961? Examine.

Solution

As per Explanation 3 to section 2(1A) of the Act, income derived from saplings or
seedlings grown in a nursery shall be deemed to be agricultural income and exempt
from tax, whether or not the basic operations were carried out on land.

6. Examine with reasons in brief whether the following statements are true or false
with reference to the provisions of the Income-tax Act, 1961:

(i) Exemption is available to a Sikkimese individual, only in respect of income from any
source in the State of Sikkim.

(ii) Pension received by a recipient of gallantry award, who was a former employee of
Central Government, is exempt from income-tax.

(iii) Mr. A, a member of a HUF, received ₹ 10,000 as his share from the income of the

Unique Academy - 8007916622 CA Saumil Manglani - Contact: 9921051593


3. Exempt Income 3.21

HUF. The same is to be included in his chargeable income.

Solution

False: Exemption under section 10(26AAA) is available to a Sikkimese individual not


only in respect of the said income, but also in respect of income by way of dividend or
interest on securities.

True: Section 10(18) exempts any income by way of pension received by individual
who has been in service of Central Government and has been awarded “ParamVir
Chakra” or “MahaVir Chakra” or “Vir Chakra” or such other gallantry award as the
Central Government, may, by notification in the Official Gazette, specify in this behalf.

False: Section 10(2) exempts any sum received by an individual as a member of a


HUF where such sum has been paid out of the income of the family. Therefore, ₹
10,000 should not be included in Mr. A’s chargeable income.

7. Rudra Ltd. has one unit at Special Economic Zone (SEZ) and other unit at Domestic
Tariff Area (DTA). The company provides the following details for the previous year
2020-21.

Particulars Rudra Ltd. (₹) Unit in DTA


(₹)
Total Sales 6,00,00,000 2,00,00,000

Export Sales 4,60,00,000 1,60,00,000

Net Profit 80,00,000 20,00,000

Unique Academy - 8007916622 CA Saumil Manglani - Contact: 9921051593


3. Exempt Income 3.22

Calculate the eligible deduction under section 10AA of the Income-tax Act, 1961, for the
Assessment Year 2021-22, in the following situations:

(i) If both the units were set up and start manufacturing from 22-05-2013.

(ii) If both the units were set up and start manufacturing from 14-05-2017.

Solution

Computation of deduction under section 10AA of the Income-tax Act, 1961

As per section 10AA, in computing the total income of Rudra Ltd. from its unit located in a Special
Economic Zone (SEZ), which begins to manufacture or produce articles or things or provide any
services during the previous year relevant to the previous year commencing on or after
01.04.2005 but before 30th June 2020, there shall be allowed a deduction of 100% of the profit and
gains derived from export of such articles or things or from services for a period of five consecutive
assessment years beginning with the assessment year relevant to the previous year in which the
Unit begins to manufacture or produce such articles or things or provide services, as the case may
be, and 50% of such profits for further five assessment years subject to fulfillment of other
conditions specified in section 10AA.

Computation of eligible deduction under section 10AA [See Working Note below]:

(i) If Unit in SEZ was set up and began manufacturing from 22-05-2013:

Since A.Y. 2021 - 22 is the 8th assessment year from A.Y. 2014-15, relevant to the
previous year 2013-14, in which the SEZ unit began manufacturing of articles or things, it
shall be eligible for deduction of 50% of the profits derived from export of such articles or
things, assuming all the other conditions specified in section 10AA are fulfilled.

= Profits of Unit in x Export turnover of Unit in x 50%


SEZ SEZ

Total turnover of Unit in SEZ

= 60 lakhs X 300 lakhs x 50% = ₹ 22.50 lakhs

400 lakhs

Unique Academy - 8007916622 CA Saumil Manglani - Contact: 9921051593


3.Exempt Income ` 3.23

(ii) If Unit in SEZ was set up and began manufacturing from 14-05-2017:

Since A.Y. 2021 – 22 is the 4th assessment year from A.Y. 2018-19, relevant to the
previous year 2017-18, in which the SEZ unit began manufacturing of articles or things, it
shall be eligible for deduction of 100% of the profits derived from export of such articles or
things, assuming all the other conditions specified in section 10AA are fulfilled.

= Profits of Unit in SEZ x Export turnover of Unit in x 100%


SEZ

Total turnover of Unit in SEZ

= 60 lakhs x 300 lakhs x 100% =₹ 45 lakhs

400 lakhs

The unit set up in Domestic Tariff Area is not eligible for the benefit of deduction under section
10AA in respect of its export profits, in both the situations.

Working Note:

Computation of total sales, export sales and net profit of unit in SEZ

Particulars Rudra Ltd. (₹) Unit in DTA (₹) Unit in SEZ (₹)
Total Sales 6,00,00,000 2,00,00,000 4,00,00,000
Export Sales 4,60,00,000 1,60,00,000 3,00,00,000
Net Profit 80,00,000 20,00,000 60,00,000

8. Kundan Lal grows sugarcane and uses the same for the purpose of manufacturing sugar in
his factory.40% of the sugarcane produce is sold for INR 15,00,000 and the cost of cultivation of
this part is INR 8,00,000. 60% of the sugarcane produce is further subjected to manufacturing
sugar and the Market Value (MV) of the same was INR 33,00,000 and the cost of cultivation of this
part was INR 21,00,000. Post incurring INR 3,00,000 in the manufacturing process for sugar, that
the sugarcane was subjected to, the sugar was sold for INR 40,00,000. You are required to advise
on his Agricultural and Business Income.

Hint – Agricultural Income 19 lacs, Business Income – 4 lacs

Unique Academy - 8007916622 CA Saumil Manglani - Contact: 9921051593


4. Salaries 4.1

Salary got due, but when it will be received…….

Unique Academy – 8007916622 CA Saumil Manglani - Contact: 9921051593


4. Salaries 4.2

Introduction
The provisions related to “Salaries” are contained as under:

(Section 15)

Deductions
(Section 16)

Constituents
(Section 17)

-
• Employer – Employee relationship should exist for charging income under the head
Salary
• Employment could be full time or part time, that really doesn’t matter.
• But, if an employee receives any money from his employer as part of the terms of
employment for not carrying on any profession, such income must be taxed as
salary income.
▪ For instance, the allowance given by employer to a doctor employed
by him for not carrying on a profession in addition to the employment
would be be taxed as salary income.
▪ If an employee gets money from persons other than his employer and if
such money is not in any way related to the contract of services with
the employer under whom he is working, the receipts, if taxable as
income, must be assessed under the head “Income from other
sources”.
Let’s examine the following cases, whether payments are chargeable under head salaries;

a. Professor: The professor of university would be receiving income by way of monthly salary from the
university which is chargeable to tax under this head. But this does not mean that every item of income
received by the employee from his employer would be taxable under this head. Thus, income by way of
examinership fees received by a professor from the same university in which he is employed would not
be chargeable to tax under this head but must be taxed as Income from other sources under Section 56.
This is because of the fact that the essential condition that the income in question must be received for
services rendered in the ordinary course of employment would not be fulfilled in the case of examinership
fees.

b. Director: A director of a company may, in some cases, be an employee of a company where there is a
specific contract of employment between him and the company. The fact that the same person has dual
capacity in his relationship with the company does not mean that he cannot be taxed under this head.
Every item of income arising to such a director who is also an employee of the company (e.g. a managing
director or other whole-time director) by virtue of his employment would be taxable as his income from
salary. Thus, income by way of remuneration received by a managing director would be taxable as
his salary income whereas the income

Unique Academy – 8007916622 CA Saumil Manglani - Contact: 9921051593


4. Salaries 4.3
received by him as director’s fees in his capacity as director for attending the meetings of the Board
would be assessable under the head “Income from other sources”.

c. Official Liquidator: An official liquidator appointed by the Court or by the Central Government would also
become an employee of the Central Government consequently the remuneration due to him would also be
assessable under the head ‘Salaries”.
d. Manager: Remuneration received by a manager of a company even if he is wrongly designated as a
director or by any other name would be chargeable to tax under this head regardless of the fact that the
amount is payable to him monthly or is calculated at a certain percentage of the company’s profits.
e. Partner of a firm: Salary paid to a partner by a firm is nothing but appropriation of profits. Any salary,
bonus, commission, or remuneration by whatever name called due to or received by partner of a firm shall
not be regarded as salary but has to be charged as income from business. It is because of the fact that
the relationship between the firm and its partner is not of employer and employee.
f. Member of Parliament: According to a circular of the Board dated 22-5-1967, the salary received by a
person as Member of Parliament will not be chargeable to income-tax under the head “Salaries” but as
“Income from other sources” because a Member of Parliament is not an employee of the
Government but only an elected representative of the people.
g. Treasurer of a bank: The income received by a treasurer of a bank would be taxable as his salary income
if the treasurer is an employee of the bank. If he does not happen to be an employee, the income
received by him would be taxable as “Income from other sources”.
h. Person carrying on a profession or vocation: Income derived by any person from carrying on a
profession or vocation must be taxed as business income and not as salary income because
employment is different from profession.

Proforma for computation of income under the head “Salaries”


Particulars Amt
(i) Basic Salary XXX
(ii) Fees/Commission XXX
(iii) Bonus XXX
(iv) Allowances:
(a) Dearness Allowance XXX
(b) House Rent Allowance (HRA) xx
Less: Least of the following is exempt [Section 10(13A)] xx XXX
HRA actually received xxx
Rent paid (-)10% of salary for the relevant period xxx

50% of salary, if accommodation is located in


Mumbai, Kolkata, Delhi or Chennai or 40% of salary xxx
in any other city for the relevant period

(c) Children Education Allowance xx


Less: Rs. 100 per month per child upto maximum of two xx XXX
children
(d) Children Hostel Allowance xx
Less: Rs. 300 per month per child upto maximum of two xx XXX

Unique Academy – 8007916622 CA Saumil Manglani - Contact: 9921051593


4. Salaries 4.4
children
(e) Transport allowance xx
Less: Rs. 3,200 per month in case of blind/ deaf and dumb/ XXX
orthopedically handicapped employee only xx
(f) Entertainment Allowance XXX
(g) Other Allowances including overtime allowance, city compensatory allowance etc. XXX
(v) Taxable
Perquisites
(a) Valuation of rent free accommodation* XXX
I) Where the accommodation is provided by the Govt. to its employees

License fee determined by the Govt. xx


Less: Rent actually paid by the employer xx
II) Where the accommodation is provided by any other employer

If accommodation is owned by the employer


(i) Cities having population > 25 lakh as per 2001
census
15% of salary in respect of the period of occupation (–)
rent recovered from employee xx

(ii) Cities having population >10 lakh < 25 lakh as per


2001 census
10% of salary in respect of the period of occupation (–)
rent recovered from employee xx

(iii) In other cities


7.5% of salary in respect of the period of occupation (–)
rent recovered from employee xx

If accommodation is taken on lease by the employer


Lower of lease rental paid or payable by the employer (or) xx
15% of salary
Less: Rent actually paid by the employee xx
(b) Obligation of employee discharged by employer. For XXX
e.g. Professional tax paid by the employer
(c) Any sum payable by the employer to effect an assurance on the life of the XXX
employee or to effect a contract for annuity: Actual expenditure incurred by the
employer

(d) Value of use of motor car XXX


(e) Any other perquisite: For example, XXX
(1) Provision of services of a sweeper, gardener, watchman or personal
attendant : Actual cost to employer by way of salary paid or payable for such
services (-) amount paid by the employee

(2) Gas, electricity, or water supplied by employer for household


consumption of the employee : Amount paid on that account by the
employer to the agency supplying gas etc. (-) amount paid by the employee

Unique Academy – 8007916622 CA Saumil Manglani - Contact: 9921051593


4. Salaries 4.5
Specified (3) Provision of free or concessional education facilities for any member of
employees – employee’s household : Sum equal to the expenditure incurred by the
a. Director employer (-) amount paid or recovered from the employee
b. Employee having
Where educational institution is maintained and owned by employer: Cost
substantial interest
(i.e. minimum 20% of such education in similar institution in or near the locality (-) amount paid
share holding or recovered from employee [However, there would be no perquisite if the
c. Income under the value of benefit per child does not exceed Rs. 1,000 p.m.]
head Salaries > Rs. Note: Above perquisites including Motor car are taxable only in case of specified employees.
50,000 excluding
perquisites (4) Interest-free or concessional loan exceeding
Rs. 20,000 : Interest computed at the rate charged by SBI as on 1st day of
relevant PY in respect of loans for similar purposes on the maximum
outstanding monthly balance (-) interest actually paid by employee

(5) Value of gift, voucher: Sum equal to the amount of such gift [If
value of gift, voucher is below Rs. 5,000, there would be no
perquisite]
(6) Use of moveable assets

Asset given Value of benefit


a) Use of laptops and computers Nil
b) Movable assets, other than - 10% p.a. of the actual cost of such
(i) laptops and computers; and assets, or the amount of rent or
(ii) assets already specified charge paid or payable by the
employer, as the case may be
(-)
amount paid by/recovered from an
employee
(7) Transfer of movable assets: Actual cost of asset to employer – cost of normal
wear and tear – Amount paid or recovered from employee.
Assets transferred mputation of cost of normal wear and tear
Computers and electronic items 50% on WDV for each completed year of usage
Motor cars 20% on WDV for each completed year of usage
Any other asset 10% of actual cost of such asset to employer for
each completed year of usage [on SLM basis]

(vi) Leave travel concession xxx


Less: Exempt u/s 10(5) xxx XXX
(vii) Gratuity
(a) Received during the tenure of employment (fully taxable) xxx
(b) Received at the time of retirement or otherwise xxx
Less: Exempt u/s 10(10) xxx XXX
(viii) Uncommuted pension (fully taxable) Commuted pension xxx
(ix) Less: Exempt u/s 10(10A) xxx
XXX
(x) Leave encashment
(a) Received during the employment (fully taxable) xxx
(b) Received at the time of retirement or otherwise xxx
Less: Exempt u/s 10(10AA) xxx XXX
(xi) Voluntary retirement compensation xxx
Less: Exempt u/s 10(10C) - Least of the following: xxx XXX

Unique Academy – 8007916622 CA Saumil Manglani - Contact: 9921051593


4. Salaries 4.6
(a) Compensation received/ receivable on voluntary xxx
retirement
(b) Rs. 5,00,000 xxx
(c) 3 months’ salary x completed years of service xxx
(d) Last drawn salary x remaining months of service left xxx

(xi) Retrenchment compensation etc. xxx


Less: Exempt u/s 10(10B)] – Least of the following: xxx XXX
(a) Compensation actually received xxx
(b) Rs. 5,00,000 xxx
(c) 15 days average pay x completed years of service and part
thereof in excess of 6 months xxx

Gross Salary XXX


Less: Deduction under section 16
Standard deduction u/s 16(ia) upto Rs. 50,000 XXX
Entertainment allowance u/s 16(ii) (only for Govt. employees) xxx
Least of the following is allowable as deduction: xxx XXX
(a) Rs. 5,000 xxx
(b) 1/5th of basic salary xxx
(c) Actual entertainment allowance received xxx
Professional Tax (paid by employer/ employee) under section 16(iii) XXX

Income under the head salary XXX

REGULATORY FRAMEWORK
Sections (Income Tax Act, 1961) Details

Section 15 Basis of Charge

Section 17(1) Salary

Section 10(13A) House rent allowance

Section 10(14) Special allowances

Section 10(10A) Commuted Pension

Section 10(10) Gratuity

Section 10(10AA) leave encashment

Section 17(2) Valuation of Perquisites

Section 17(3) Profits in lieu of Salary

Section 89 relief when salary is paid in arrears or advance

Section 16(ia) Standard deduction

Section 16(ii) entertainment allowance

Section 16(iii) Professional tax

Unique Academy – 8007916622 CA Saumil Manglani - Contact: 9921051593


4. Salaries 4.7

Basis of Charge

Section 15
• Salary is taxable on “due” or “paid” basis whichever is earlier.
• If it is due, it is included in taxable salary, irrespective of whether it is paid or not, and if it is paid, it is taxable,
irrespective of whether it is due or not.
• Therefore, it is only logical to note that if it has already been taxed on due basis, the same cannot be taxed again
when it is paid.
• Similarly, if a salary which was paid in advance, if it has already been taxed in the year of payment, it cannot
subsequently be taxed when it becomes due.

KEY POINTS

• Advance salary is taxable; however, an Advance against Salary is essentially loan which will be
recovered later from the Employee and therefore that is not taxable.

ILLUSTRATION
State whether the following receipts should be treated as salary or not?
• A teacher receives emoluments in kind from school in which he teaches.

Yes, it is immaterial whether salary has been received in cash or in kind.


• A teacher of a college receives fees from a University for checking answer sheets.
No, as employer – employee relationship does not exist between payer and payee. (College-teacher is
not the employee of the University). Such receipt shall be taxable under the head ‘Income from other
sources’.
• A payment made to the Member of the Parliament or the State legislature.

No, as employer-employee relationship does not exist.


A member of the Parliament or the State legislature is not treated as employee of the Government.
Payment received by them shall be taxable under the head “Income from other sources.

Computation of Salary in Grade system – varying Pay Scale


In this concept the person gets an increment in Salary after every completion of duration of 12 months in
employment.

Unique Academy – 8007916622 CA Saumil Manglani - Contact: 9921051593


4. Salaries 4.8

Retirement Benefits

1. Gratuity 10(10)
Gratuity is normally paid in lieu of the long-term service of an employee (usually > 5 years),
but is a voluntary payment by the employer, as an appreciation of the long- standing services.
The Gratuity so received, is exempt as under:

Gratuity

Received during Received at the


service time of

Employee/ Defense Non-Government


services Employees

Fully Exempt
Covered under Not covered
Payment of under payment
Gratuity Act, of Gratuity Act,
1972 1972

Least of the following Least of the following


would be exempt: would be exempt:
- ₹20 lakh (AY 19-20) - ₹ 20 lakh
Gratuity received
- Gratuity received
½ * Salary *
- 15/26 * Last drawn completed number years of
Salary * Number of service (ignore fraction of
years of service (where years
> 6 moths = 1 year)

Unique Academy – 8007916622 CA Saumil Manglani - Contact: 9921051593


4. Salaries 4.9
2. Annuity/ Pension 10 (10A)

Annuity is a yearly payment to an employee post his retirement on account of the funds that were
saved by him by way of subscription to the annuity fund vide his salary when he was in
employment.
Annuity received from the present employer is chargeable to tax as Salary and any amount
received from the past employer is chargeable to tax as Profits In lieu of Salary.
Pension however is generally paid by the Government or a Company to the employee for his
past service and this too is payable after the retirement.
This pension so received could be commuted / uncommuted, explained as under:

Sr Type of Remarks
no. Pension
1. Uncommuted 1. Received Periodically
2. Fully Taxable for all employees
2. Commuted 1. Received in Lumpsum (Whole/Part)
Pension
2. Future right to receive payments given up to receive
immediate lumpsum (refer below on the treatment)

Pension 10 (10A)
Pension

Commuted Uncommuted
Lumpsum)d (Monthly)

Employees of the Non-Government Fully taxable


Central Employees
Government/local
authorities/Statutory
Corporation/Members
of the Defence Services If the If the employee
employee is does not
in receipt of receive any
gratuity gratuity

1/3  (commuted ½  (commuted


pension received pension received
 commutation  commutation
%)  100, would %)  100, would
be exempt be exempt
Few pointers

Sr. No. Different Situations Tax Treatments


1 Pension is received from UNO by the employee Not chargeable to tax
or his family members
2 Family pension received by family members of Exempt u/s 10(19)
armed forces
3 Family pension received by family members Taxable in the hands of recipient u/s 56 under
after the death of employee the head ' income from other sources'.
Standard deduction is available u/s 57 which
is 1/3 rd of such pension or ₹ 15000 whichever
is less

Unique Academy – 8007916622 CA Saumil Manglani - Contact: 9921051593


4. Salaries 4.10

3. Leave Encashment 10 (10AA)


Leave Encashment Salary received by employees of the Government, is fully exempt from tax.
For the Non-Government employees, the Leave Encashment Salary so received is exempt from
tax to the extent of least of the following:
a) INR 3,00,000
b) Leave Salary actually received
c) 10 month’s Salary on the basis of average Salary drawn in the last 10 months
d) Cash Equivalent of Leave standing to the credit of the employee at the time of
retirement / death, based on last 10 month’s average salary drawn. Earned leave
entitlement per year cannot exceed 30.
Note: Here Salary would mean Basic + DA (only to the extent of forming part of the retirement benefits) +
Commission as a % of Turnover and number of days in the month to be taken at 30.

Leave Encashment [Section 10 (10AA)

Leave Encashment

Received during Received on


the period of retirement whether
Service on Superannuation
or otherwise

Full Taxable

By a Government By any other


employee employee

Full Exempt Least of the


following
is exempt

Rs. 3,00,000 Leave salary 10 months’ salary (Total leaves allowed/


actually received (on the basis of Earned – Total leaves 10
average salary of Availed)  months
last 10 months) average
30 monthly
salary

Earned leave entitlement cannot


exceed 30 days for every year of
actual service

Unique Academy – 8007916622 CA Saumil Manglani - Contact: 9921051593


4. Salaries 4.11

Allowances
An allowance is defined as a fixed amount of money given periodically in addition
to the salary for the purpose of meeting some specific requirements connected with the
service rendered by the employee or by way of compensation for some unusual
conditions of employment. It is taxable on due/accrued basis whether it is paid in
addition to the salary or in lieu thereon. These allowances are generally taxable and are
to be included in the gross salary unless a specific exemption has been provided in
respect of allowances provided under the Act.

to prescribed
limits

Allowances partially Taxable


House rent allowance [Section 10(13A)]: HRA is a special allowance
specifically granted to an employee by his employer towards payment of rent
for residence of the employee. HRA granted to an employee is exempt to the
extent of least of the following:
Metro Cities (i.e. Delhi, Kolkata,
Other
Mumbai, Chennai)
Cities
1) HRA actually received. 1) HRA actually received
2) Rent paid -10% of salary for the 2) Rent paid - 10% of
relevant period salary for the relevant
period
3) 50% of salary for the relevant 3) 40% of salary for the
period relevant period
Notes:

a. Exemption is not available to an assessee who lives in his own house, or in


a house for which he has not incurred the expenditure of rent.

b. Salary for this purpose means basic salary, dearness allowance, if provided
in terms of employment and commission as a fixed percentage of turnovers.

c. Relevant period means the period during which the said accommodation
was occupied by the assessee during the previous year.
Unique Academy – 8007916622 CA Saumil Manglani - Contact: 9921051593
4. Salaries 4.12
Illustration 1: (ICSI Module)
Vir is employed with Happiness Solutions Ltd. and during the FY 2020-21. He had a Basic Pay of INR
75000 per month. Owing to his good performance at workplace, he was given an annual increment in his
Salary of 20% effective Feb 2021. He also received during the year, a Dearness Allowance of 100% of
Basic (however only 50% of which was included per his terms of employment). The Company also gave
him HRA of INR 30000 per month, which was increased to INR 35000 per month effective Jan 2021. He
stayed at his parental home in Ahmedabad for
Apr and May 2020, post which he took up a Rented Accommodation at Surat at a monthly rental of INR
26000. Effective Nov 20 he was moved to Mumbai to the Corporate Office, and he took up an
accommodation at Mumbai at a monthly rental of INR 36000. You are required to compute the HRA
exempt from Tax and the Gross Salary.

Solution
The computation of HRA exempt from tax would have to be done on a monthly basis as the salary and rental
figures changed during the year, in the manner provided below.

Item Apr May Jun Jul Aug Sep Oct Nov Dec Jan Feb Mar Total
Basic 75,000 75,000 75,000 75,000 75,000 75,000 75,000 75,000 75,000 75,000 90,000 90,000 9,30,000
D.A. 37,500 37,500 37,500 37,500 37,500 37,500 37,500 37,500 37,500 37,500 45,000 45,000
Salary for 1,12,500 1,12,500 1,12,500 1,12,500 1,12,500 1,12,500 1,12,500 1,12,500 1,12,500 1,12,500 1,35,000 1,35,000
HRA
Accomm Own Own Surat Surat Surat Surat Surat Mumbai Mumbai Mumbai Mumbai Mumbai
oda- tion
40% or – – 45,000 45,000 45,000 45,000 45,000 56,250 56,250 56,250 67,500 67,500
50%
of Salary
HRA 30,000 30,000 30,000 30,000 30,000 30,000 30,000 30,000 30,000 35,000 35,000 35,000 3,75,000
Actually
Recd
Rent - – – 14,750 14,750 14,750 14,750 14,750 24,750 24,750 24,750 22,500 22,500
10%
Salary
Min – – 14,750 14,750 14,750 14,750 14,750 24,750 24,750 24,750 22,500 22,500 1,93,000

Note:
(1) For the stay at Surat, 40% of Salary and for stay at Mumbai, 50% of Salary is considered for one of
the parameters for exemption.
(2) DA, only to the extent of it being included per terms of employment, is included in Salary for the
purposes of HRA.

Please note the Gross Salary Computation below.

Basic Salary 9,30,000


Dearness Allowance 9,30,000
HRA 3,75,000
Less : Exempt 1,93,000 1,82,000
Gross Salary 20,42,000
Note: It is assumed that assessee has not opted u/s 115BAC, otherwise exemption u/s 10(13A) of rs.1,93,000
will not be available. in that case, Gross Salary would be Rs.22,35,000.

Unique Academy – 8007916622 CA Saumil Manglani - Contact: 9921051593


4. Salaries 4.13
2 Special Allowance u/s 10(14) (i) read with Rule 2BB- Exempt to the extent such
expenses are actually incurred
1. All special allowances specifically granted to meet expenses,
incurred, for the purposes of performance of duties -
a. Wholly
b. Exclusively &
c. Necessarily
These are exempt to the extent such expenses are actually incurred or the amount
received whichever is less.
Allowance exemption depends upon the actual expenditure

Sr. No. Allowance Purpose


To meet the ordinary daily charges incurred by an employee on
1 Daily allowance
account of absence from his normal place of duty
2 Uniform allowance. Given for Purchase, maintenace of uniform
3 Helper allowance Helper for official duties (But Servant allowance fully taxable)
Encouraging the academic research and training pursuits in research
4 Research allowance
institutions
5 Conveyance allowance Performance of duties of an office
6 Travelling Allowance Cost of travel on tour one city to another – Official tour
Transfer allowance –
7 Transfer, packing and shifting of personal effects on such transfer
Shifting city

1. Exemption subject to prescribed limits 10(14) (ii)


First Priorit

S. Name of Allowance Extent to which


No. allowance is exempt
1 Special Compensatory (Tribal Areas / Schedule Areas / Agency Areas) Rs. 200 per month.
Allowance
2 Any allowance granted to an employee working in any transport system 70% of such allowance upto
to meet his personal expenditure during his duty performed in the a maximum of
course of running such transport from one place to another, provided Rs. 10,000 per month.
that such employee is not in receipt of daily allowance

3 Children Education Allowance Rs. 100 per month per child


upto a maximum of two
children.
4 Any allowance granted to an employee to meet the hostel expenditure Rs. 300 per month per child
on his child upto a maximum of two
children.
5 Any transport allowance granted to an employee to meet his Rs. 1,600 per month
expenditure for the purpose of commuting between the place of his Withdrawn
from AY 19-20

Unique Academy – 8007916622 CA Saumil Manglani - Contact: 9921051593


4. Salaries 4.14
residence and the place of his duty

6 Any transport allowance granted to an employee who is blind or deaf Rs. 3,200 per month.
and dumb or orthopedically handicapped with disability of the lower
extremities of the body, to meet his expenditure for commuting
between his residence and place of duty
7 Underground Allowance would be granted to an employee who is Rs. 800 per month
working in uncongenial, unnatural climate in underground mines. This
is applicable to whole of India.

Second Priority
Sr.
No Name of Allowance
. Exemption Limit

Rs. 800 or Rs. 300 per month


Any Special Compensatory Allowance in the nature of Special
depending upon the specified
Compensatory (Hilly Areas) Allowance or High Altitude
1 locations
Allowance or Uncongenial Climate Allowance or Snow Bound
Rs. 7,000 per month in Siachen
Area Allowance or Avalanche Allowance
area of Jammu and Kashmir

Rs. 1,300 or Rs. 1,100 or Rs. 1,050


Any Special Compensatory Allowance in the nature of border
or Rs. 750 or Rs. 300 or Rs. 200
2 area allowance or remote locality allowance or difficult area
per month depending upon the
allowance or disturbed area allowance
specified locations

Compensatory Field Area Allowance [Specified areas in


3 Rs. 2,600 per month
Specified States]
Compensatory Modified Field Area Allowance [Specified
4 Rs. 1,000 per month
areas in Specified States]
Any special allowance in the nature of counter insurgency
5 allowance granted to the members of the armed forces Rs. 3,900 per month
operating in areas away from their permanent locations.
Any special allowance in the nature of high Altitude allowance
granted to the member of the armed forces operating in high
6 altitude areas For altitude of
Rs. 1,060 per month
9,000 to 15,000 feet
Rs. 1,600 per month Rs.
For above 15,000 feet
Any special allowance in the nature of special compensatory 4,200 per month
7 highly active field area allowance granted to the member of
the armed forces
Any special allowance in the nature of Island (duty) allowance Rs. 3,250 per month
8 granted to the member of the armed forces in Andaman &
Nicobar and Lakshadweep Group of Islands

2. Fully Taxable Allowance


a. Dearness Allowance, Additional Dearness Allowance and Dearness Pay:
This is a very common allowance these days on account of high prices. Sometimes
Additional Dearness Allowance is also given. It is included in the income from salary
and is taxable in full. Sometimes it is given under the terms of employment and
sometimes without it. When it is given under the terms of employment it is included
in salary for purposes of determining the

Unique Academy – 8007916622 CA Saumil Manglani - Contact: 9921051593


4. Salaries 4.15
exemption limits of house rent allowance, recognized provident fund, gratuity and
value of rent-free house and is also taken into account for the purposes of
retirement benefits.
b. Fixed Medical Allowance
c. Tiffin Allowance
d. Servant Allowance
e. Non-practicing Allowance
f. Hill Allowance
g. Warden Allowance and Proctor Allowance
h. Deputation Allowance
i. Overtime Allowance
j. Other Allowances like Family allowance, Project allowance, Marriage
allowance, City Compensatory allowance, Dinner allowance, Telephone
allowance etc. These are fully taxable.

3. Allowances Fully Exempt


• Allowance to High Court Judges and Supreme court Judges

• Allowance received from United Nations Organisation (UNO):


Allowance paid by the UNO to its employees is not taxable

• Allowances payable outside India [Section 10(7)] - By the Government to


a citizen of India for services rendered outside India.

Note: if the assessee opted concessional tax slab under section 115BAC of the income tax act, 1961, then
assessee is not eligible to claim exemption from any allowances except:
1. Travelling allowances
2. daily allowances AY 21-22
3. Conveyance allowance
4. Transport allowance (For blind, handicapped, deaf or dumb employee)

Profits in lieu of salary


Under this the following items are included:-
• The amount of any compensation due to or received by an assessee from the employer or
former employer at or in connection with the termination of his employment. The termination
of employment means retirement, premature termination of employment, termination by
death or voluntary resignation. Generally, under the Income- tax Act, the income that is
chargeable to tax is only a receipt which is revenue in nature; receipts of a capital nature
are not chargeable to tax but this provision constitutes an exception to this rule because
compensation received by an employee for termination of his employment would be a capital
receipt since it is received in replacement of the sources of income itself. Still it is chargeable
to tax because of the specific provision in the Act.

• However, relief under Section 89(1) would be available to the assessee in cases where he
gets money which represents a profit in lieu of salary

• The amount of any compensation due to or received by any assessee from his employer in
connection with the modification of the terms and conditions relating to employment.
For example, where an employer wants to cut down the salary payable to the employee, the
lump sum paid to compensate the employee shall be treated as profits in lieu of salary.

• In the same way, where the remuneration for services is paid at the end of the period of
employment or a lump sum remuneration is paid at the beginning of employment for a
number of years, such payment shall be treated as profits in lieu of salary.

Unique Academy – 8007916622 CA Saumil Manglani - Contact: 9921051593


4. Salaries 4.16
• Any amount due to or received, whether in lump sum or otherwise, by any assessee from any
person - before his joining any employment with that person; or after cessation of his
employment with that person.

• Any sum received under Keyman Insurance Policy


Perquisites

Perquisite may be defined as any casual emolument or benefit attached to an office or position in addition to
salary or wages. It also denotes something that benefits a man by going into his own pocket. Perquisites may be
provided in cash or in kind. However, perquisites are taxable under the head “Salaries” only if they are
a. allowed by an employer to his employee;
b. allowed during the continuance of employment;
c. directly dependent upon service;
d. resulting in the nature of personal advantage to the employee; and
e. derived by virtue of employer’s authority.
It is not necessary that a recurring and regular receipt alone is a perquisite. Even a casual and non-recurring
receipt can be perquisite if the aforesaid conditions are satisfied.

Taxable Perquisites
Rent Free Residential Accommodation
Interest Free / Concessional Loan
Use of movable assets by employee / any member of his household
Transfer of movable assets
Provision of gas / electricity / water
Provision of free / concessional educational facilities
Credit Card Expenses
Club expenditure
Health Club, Sports, Similar facilities
Sweat Equity

Tax-free Perquisites (in all cases)


Medical Facilities
Refreshment
Subsidized lunch or dinner
Recreational facilities
Telephone facility
Transport
Personal accident insurance
Refresher Course
Free rations – given to armed forces personnel
Computer/laptops
Rent free houses / conveyance to High Court & Supreme Court Judges
Employers’ Contribution to Group Insurance Schemes, to recognized Provident Funds

Unique Academy – 8007916622 CA Saumil Manglani - Contact: 9921051593


4. Salaries 4.17
Taxable Perquisites

Rent free accommodation (Reduce recovery from employer wherever done)

Valuation of accommodation only (Excluding furniture)

Rent Free Accommodation

Govt. Employee (Central/State) Non-Govt. Employee

License fee determined by


Govt. would be the value of Accommodation Accommodation not
perquisite Owned by employer Owned by employer

Population# of Population# of City Population# of 15% of Salary*


City upto 10 Lac > 10 lac to 25 lacs City > 25 Lac or
Rent paid
Whichever
7.5% of Salary* 10% of Salary* 15% of Salary* is lower

Note – Calculate for only that number of months for which the house is occupied by the employee.

*Salary means
Basic Salary
⬧ DA (forming part of the retirement benefits)
⬧ Bonus
⬧ Fee
⬧ Commission (also includes fixed commission)
⬧ Taxable allowances i.e. only taxable portion of allowances
⬧ Monetary payment not being perquisties (e.g. Leave encashment) i.e. “Ignore ALL types of
perquisites in this calculation”)

# Population of the city as peer 2011 census.


Accommodation may be provided:
(1) Rent free; or
(2) At concessional rate.

Note: In case the house is provided at concessional rate, the value determined above shall be reduced
by the rent, if any, actually paid by the employee.
Accommodation provided by the employer in a hotel - Where the accommodation is provided by the
employer in a hotel (except where the employee is provided such accommodation for a period not exceeding in
aggregate 15 days on the transfer from one place to another): The perquisites value would be 24% of salary
paid or payable for the previous year or the actual charges paid or payable to such hotel, which is lower, for
the period during which such accommodation is provided as reduced by the rent, if any, actually paid or
payable by the employees.

Unique Academy – 8007916622 CA Saumil Manglani - Contact: 9921051593


4. Salaries 4.18
Other Taxable Perquisites: We need to understand the valuation of perquisites. The table
appended below, summarizes the taxable value of various perquisites in the hands of the
employee assessees.

Taxable Value of
Nature of Perquisite Note
Perquisite
Free Domestic Servant Actual cost of the employer
Service of sweeper, gardener or Less: Amount paid by
watchman or personal employee
attendant
Supply of gas, electricity or The amount determined shall be
water for household reduced by the amount, if any
consumption recovered from the employee for
a) Procured from outside Amount paid to outside such benefit.
agency agency
b) Resources owned by Manufacturing cost per unit
employer himself
Education Facilities for 1. Amount paid for free training of
Children the employee is not taxable
a) Free education to Cost to the Employer 2. Payment or reimbursement of
employee's own children in school fee is taxable in all cases
Less: Rs. 1,000 per month
the school 3. No restriction on number of
Less: Amount recovered children
owned/maintained by the
from employee
employer or the school
sponsored by the employer
b) Other Schools Cost to the Employer
Less: Amount recovered
from employee
c) For others (other than Cost of education to
assessees children i.e., Employer
grandchildren and other Less: Amount recovered
household members) from employee
Interest free or Concessional outstanding Balance for Note taxable if -
Loan each loan on last day of 1. Loan < 20,000
Provided to Employee or each month  Rate of 2. Loan for diseases specified in
household members Interest charged by SBI on the rule 3A (Cancer, TB, AIDS,
1st day of the relevant PY. Disease requiring surgical
Less: Interest charged operation, mental disorder,
caesarean operation). However,
not applicable to so much of the
loan as has been reimbursed to
the employee under medical
insurance scheme.

Unique Academy – 8007916622 CA Saumil Manglani - Contact: 9921051593


4. Salaries 4.19
Taxable Value of
Nature of Perquisite Note
Perquisite
Travelling, Touring, Accommodation The amount
a) Where such facility is It will be the value at which determined shall be
maintained by the such facilities are offered by reduced by the amount,
employer, and is not other agencies to the if any recovered from
available uniformly to all public. the employee for such
employees. benefit

b) Where the employee is on The amount of expenditure


official tour and the so incurred.
expenses are incurred in
respect of any member of
his household
accompanying him.
c) Where any official tour is The value will be limited to
extended as a vacation. the expenses incurred in
relation to such extended
period of stay or vacation
d) In any other case, where A sum equal to the amount
such facility is given to the of expenditure incurred by
employee or any member the employer.
of his household. (Facility
available uniformly to all
employee)
Free food and Non-Alcoholic Working hours include
Beverages extended office hours
(like working on
holidays, over time)
a) Tea or snacks provided Nil
during working hours
b) Free food and non- Nil
alcoholic beverages during
working hours provided in
a:
(i) Remote area; or
(ii) An offshore installation.

Taxable Value of
Nature of Perquisite Note
Perquisite
c) Free food and non- Cost to the employer in excess of
alcoholic beverages Rs. 50 per meal
provided by the employer Less: Recovery from the employee
during working hours:
(i) at office or business
premises; or

Unique Academy – 8007916622 CA Saumil Manglani - Contact: 9921051593


4. Salaries 4.20
(ii) Through paid vouchers
which are not
transferable and usable
only at eating joints.
d) In any other case Actual amount of expenditure
incurred by the employer Less:
Recovery from the employee
Value of any Gift, Voucher or Gifts made in cash or
Token convertible into
The value of any gift, or Amount over and above of Rs. money (like gift
voucher, or token received by 5,000 only shall be taxable cheques) are not
the employee or by member of [CircularNo.15/2001 dated exempt.
his household on ceremonial 12.12.2001]
occasions or otherwise from the Alternative view → proviso to Rule
employers 3(7)(iv)
If amount > Rs. 5,000, full amount
taxable.
Where the value of such gift, The value of perquisite shall be
voucher or token, as the case nil. As per proviso to Rule 3(7)(iv)
may be, is upto Rs. 5,000 in the
aggregate during the previous
year.
Expenses on Credit Cards
Expenses including The amount paid for or reimbursed
membership fees and annual by the employer.
fees are incurred by the Less: Expenditure on use for official
employee or any member of his purpose
household, which is charged to Less: Amount recovered from
a credit card, provided by the employee
employer or otherwise are paid
for or reimbursed by the
employer
Nature of Perquisite Taxable Value of Note
Perquisite
Club Membership
The payment or reimbursement The actual amount of expenditure
by the employer of any incurred or reimbursed by the
expenditure incurred (including employer.
the amount of annual or Less: Expenditure on use for official
periodical fee) in a club by the purpose
employee or by any member of Less: Amount recovered from
his household employee
Use of Moveable Assets

Unique Academy – 8007916622 CA Saumil Manglani - Contact: 9921051593


4. Salaries 4.21
a) Use of laptops and Nil
computers
b) Moveable assets other than i) If owned by employer than 10% The amount
Laptops and computers per annum of the actual cost of determined shall be
such asset, or reduced by the
ii) If taken on hire by employer the reduced by the
amount of rent or charge paid, amount, if any paid
or payable by the employer as or recovered from the
the case may be. employee for such
benefit.
Transfer of any Moveable
Assets
a) Computers and Electronic Actual cost of such asset to the
Items employer as reduced by 50% for he amount determined
each completed year during which shall be reduced by the
such asset was put to use by the amount, if any paid or
employer, on the basis of reducing recovered from the
balance method. employee for such
b) Motor Cars Actual cost of such asset to the benefit.
employer as reduced by 20% for
Note – Completed year
each completed year during which
means ignore fraction
such asset was put to use by the
of the year)
employer, on the basis of reducing
balance method.
c) Any other Assets Actual cost of such asset to the
employer as reduced by 10% SLM
of the actual cost to the employer for
each completed year during which
such asset was put to use by the
employer.

Unique Academy – 8007916622 CA Saumil Manglani - Contact: 9921051593


4. Salaries 4.22

Solution :
Perquisite = MV on the date of Exercise - Amount recovered from employee
= 100 Shares X Rs. 175 - 100 Shares X Rs. 20
= Rs. 17,500 - Rs. 2,000
But taxable in the year of allotment (i.e. 5th year).
Note - Some perquisites are taxable only in the hands of specified employees for example: Provision
of Sweeper, Gardener, Watchman Or Personal Attendant, Facility Of Use Of Gas, Electricity
Or Water Supplied By Employer, Free Or Concessional Tickets, Use Of Motor Car, Free Or
Concessional Educational Facilities.

Specified employees: Meaning 17(2)(iii)


(i) Director employee: An employee of a company who is also a director is a specified
employee. It is immaterial whether he is a full-time director or part- time director. It also does
not matter whether he is a nominee of the management, workers, financial institutions or the
Government. It is also not material whether or not he is a
director throughout the previous year.
(ii) An employee who has substantial interest in the company: An employee of a company
who has substantial interest in that company is a specified employee. A person has a
substantial interest in a company if he is a beneficial owner of equity shares carrying 20% or
more of the voting power in the company.

(iii) An employee other than an employee described in (i) & (ii) above, whose income
chargeable under the head ‘salaries’ exceeds ₹ 50,000 is a specified employee. The above
salary is to be considered exclusive of non-monetary benefits.

Motor Car- perquisite Valuation

Unique Academy – 8007916622 CA Saumil Manglani - Contact: 9921051593


4. Salaries 4.23

Illustration 1A (ICSI Module)


1) ABC ltd. provided the following perquisites to its employee Srinivasan, for the Fy 2020-21.
2) leased accommodation provided to the employee. Hire Charges iNr 50000 pm; recovered
from employee iNr 20000 pm
3) accommodation was furnished and the actual hire charges paid by the employer was iNr 4050/-
pm
4) He was also provided a Hyundai Santro with Chauffer and a Gift Voucher worth INR 9000/-
5) Salary for the purposes of valuation of perquisites is iNr 25,00,000/-.
6) Compute the taxable value of the perquisites assuming assess had not opted for section 115BAC
Solution :

Accomodation on lease
Salary for the purposes of Valuation of Perquisites 25,00,000
actual lease Charges 6,00,000
15% of the above (Cap) 3,75,000
Hence, Gross taxable Value of the 3,75,000
Perquisite
less: amount recovered from the employee 2,40,000
taxable value of unfurnished leased 1,35,000
accomodation
add: actual hire charges of furniture hired 48,600
Taxable value of Furnished Accomodation 1,83,600
Car used partly for Official & partly for Personal
engine Capacity is within 1.6 l (Santro)
taxable Value of Perquisite @ 1800 pm 21,600
Chauffer @ 900 pm 10,800
taxable value of Motor Car provided 32,400
Gift Voucher 9,000
Total Value of Perquisites 2,25,000

Unique Academy – 8007916622 CA Saumil Manglani - Contact: 9921051593


4. Salaries 4.24
Note:
1) Refer to the valuation rules for perquisites – taxable value of perquisite for a hired accommodation is the actual
hire charges incurred by the employer subject to max. 25% of salary reduced by the amount recovered from
the employee.
2) Since the accommodation is furnished, the actual hire charges are added to the above.
3) Gift Vouchers are taxable as perquisites too if received by the employer and more than 5000/-

Exemption In Respect Of Leave Travel Concession [Section 10(5)]

(I) This Clause Exempts The Leave Travel Concession (LTC) received By Employees From Their
Employers For Proceeding To Any Place In India,
(A) Either On Leave Or
(B) After Retirement From Service Or
(C) After Termination Of His Service.
(ii) The Benefit Is Available To Individuals - Citizens As Well As Non-Citizens - In Respect Of
Travel Concession Or Assistance For Himself Or Herself And For His/Her Family- I.E., Spouse And
Children Of The Individual And Parents, Brothers And Sisters Of The Individual Or Any Of Them Wholly Or
Mainly Dependent On The Individual.
(iii) Limit Of Exemption - The Exemption In All Cases Will Be Limited To The Amount Actually Spent
Subject To Such Conditions As Specified In Rule 2B Regarding The Ceiling On The Number Of Journeys
For The Place Of Destination.
Under Rule 2B, Exemption Will Be Available In Respect Of 2 Journeys Performed In A Block Of 4 Calendar
Years Commencing From The Calendar Year 1986. Where Such Travel Concession Or Assistance Is Not
Availed By The Individual During Any Block Of 4 Calendar Years, One Such unavailed LTC Will Be Carried
Forward To The Immediately Succeeding Block Of 4 Calendar Years And Will Be Eligible For Exemption.
(iv) Monetary Limits - The Amount Exempted Under Section 10(5) In Respect Of The Value Of LTC
Shall Be The Amount Actually Incurred On Such Travel Subject To The Following Conditions:

S.No. Journey performed by Limit


1 Air Amount not exceeding the air economy fare of the
National Carrier by the shortest route to the place of destination
2 Any other mode:
(i) Where Rail Service Amount not exceeding the air- conditioned first class rail fare by the shortest
is available route to the place of destination
(ii)Where rail service is A) Recognised transport system exists amount not exceeding the 1st class or
not available deluxe class fare, as the case may be, on such transport by the shortest route
to the place of destination
B) Recognised transport system does not exists amount
equivalent to the air- conditioned first class rail fare, for the
distance of the journey by
the shortest route, as if the journey had been performed by rail

Note – In CIT Vs. L&T Ltd. Court gave the verdict that employer has no obligation to collect the evidence
regarding the actual leave taken by the employee.

Unique Academy – 8007916622 CA Saumil Manglani - Contact: 9921051593


4. Salaries 4.25

Tax Free Perquisites

The value of the following perquisites is not to be included in the salary income of an employee:
i)Medical Facilities
a. The value of any Medical facility provided to an employee or his family member in any
hospitals, clinics, etc. maintained by the employer.
b. Reimbursement of expenditure actually incurred by the employee on medical
treatment for self or for his family members in any hospitals, dispensaries etc.
maintained by the Government or local authority or in a hospital approved under
the Central Health Scheme or any similar scheme of the state Government or in a
hospital, approved by the chief commissioner having regard to the prescribed
guidelines for the purposes of medical treatment of the prescribed diseases or
ailments.
c. Group medical insurance obtained by the employer for his employees (including
family members of the employees) or all medical insurance payments made directly
or reimbursement of insurance premium to such employees who take such insurance.
d. Any expenditure incurred or paid by the employer on the medical treatment of the
employee or any family member of the employee outside India, the travel and stay
abroad of such employee or any family member of such employee or any travel or stay
abroad of one attendant who accompanies the patient in connection with such
treatment will not be included in perquisites of the employee. However, the travel
expenditure shall be excluded from the perquisites only when the employee’s gross
total income as computed before including the said expenditure does not exceed two
lakh rupees and further to such conditions and limits as the Board may prescribe
having regard to guidelines, if any, issued by the Reserve Bank of India.

Unique Academy – 8007916622 CA Saumil Manglani - Contact: 9921051593


4. Salaries 4.26

Family means – Spouse, Children, Dependent Parents, Brothers & Sisters

Provident Fund- Tax Treatment

Particulars Statutory Recognized PF Unrecognized PF Public PF


PF
Employer’s Fully Amount in excess of 12% Not taxable yearly N.A.
Contribution exempt of salary is taxable
Employee’s Eligible for Eligible for deduction u/s Not eligible for deduction Eligible
Contribution deduction 80C for
u/s 80C deduction
u/s 80C
Interest Credited Fully Amount in excess of 9.5% Not taxable yearly Fully
exempt p.a. is taxable exempt
Amount Fully Exempt from tax if Employer’s Fully
received on exempt u/s employee served a contribution and exempt u/s
retirement, 10(11) continuous period of 5 interest thereon is 10(11)
etc. years or more or retires taxable as salary.
before rendering 5 years Employee’s contribution
of service because of is not taxable. Interest
reason beyond the on employee’s
control of the employee. In contribution is taxable
other case, it will be under income from
taxable. other source.
10(12)

Note: Salary for this purpose means basic salary and dearness allowance -if provided in the terms of employment
for retirement benefits and commission as a percentage of turnover.

Unique Academy – 8007916622 CA Saumil Manglani - Contact: 9921051593


4. Salaries 4.27

Following sub-clauses (vii) and (viia) shall be substituted for the existing sub-clause (vii) of
clause (2) of section 17 by the Finance Act, 2020, w.e.f. 1-4-2021:
(vii) the amount or the aggregate of amounts of any contribution made to the account of the assessee by
the employer—
AY 21-22 (a) in a recognized provident fund;
(b) in the scheme referred to in sub-section (1) of section 80CCD – National Pension Scheme
; and
(c) in an approved superannuation fund,
to the extent it exceeds 7.5 Lacs rs. in a previous year;
(viia) the annual accretion by way of interest, dividend on the above amounts is also to be included
in value of 7.5 Lacs rupees.

Note – Previously, contribution of employer to Superannuation fund was exempt to employee upto Rs.
1.5 lacs.Now this clause has been removed and an overall limit of 7.5 lacs is given.

Illustration: Mr. X, working in MNO ltd., draws the following amount of emoluments from the company:

Particulars Amount (in lakhs)

Basic Pay 50

Commission 15

Employer's contribution to recognized provident fund 10

Employer's contribution to NPS 7

Employer's contribution to the superannuation fund 5

Total 87

Solutions:
Amount (in lakhs)

Particulars Before After


Amendment Amendment
Basic Pay 50 50
Commission 15 15
Employer's contribution to recognized provident fund (in 4 4
excess of 12% of basic pay) [Rs. 10 lakh (less) Rs.6 lakh (Rs.
50 lakh * 12%] [Section 17(1)]
Employer's contribution to NPS 7 7
Employer's contribution to the superannuation fund in excess 3.50 -
of rs. 1.5 lakhs [Old Section 17(2)(vii)]
Perquisite arising from Employer's contribution to the - 14.50
superannuation fund, rPF and NPS in excess of rs. 7.5 lakhs
[New Section 17(2)(vii)]
Income chargeable to tax under the head "Salary" 79.50 90.50

Unique Academy – 8007916622 CA Saumil Manglani - Contact: 9921051593


4. Salaries 4.28

Expert’s opinion - The above amendment seems to be illogical. Let’s see if any
amendment is introduced at a later part.

Payment from approved superannuation fund in specified circumstances and


subject to certain limits [Section 10(13)]

Approved superannuation fund means superannuation fund which is


approved by the Commissioner of Income- tax.
Payments made from the fund are exempt from tax under section 10(13) in
following cases:
✓ Payment on death of beneficiary; or

✓ Payment to employee in lieu of, or in commutation of an annuity on his retirement at


or after the specified age or on his becoming incapable prior to such retirement; or

✓ Refund of contributions on the death of a beneficiary

✓ Refund to employee on leaving job

Relief u/s 89(1)

Tax Liability in the PY in which adv / arrears are


received
a) Incl. adv / arrears A
b) Excl. adv / arrears B
Differential A-B
Tax Liability of the PY to which such addl. salary
relates
a) Incl. adv / arrears C
b) Excl. adv / arrears D
Differential C-D
Relief u/s 89 (A-B)-(C-D)

Deductions from Salary


Standard Deduction ( Section 16 (ia) (AY 20-21)
A standard deduction of ₹ 50,000 or the amount of salary, whichever is lower, is to
be provided to the employees
Entertainment Allowance – (Section 16(ii))
✓ First to be included in salary and then deduction to be made.

✓ In case of Government Employees, the deduction is available, which would be lower of:

⚫ 1/5th of Basic Salary Or


⚫ INR 5000/- Or
⚫ Actual Entertainment Allowance received

Profession Tax (Section 16 (iii))


Allowed as a deduction when paid by the employee (recovered from salary) during the
previous year.
Unique Academy – 8007916622 CA Saumil Manglani - Contact: 9921051593
4. Salaries 4.29

CASE LAWS
1. Can notional interest on security deposit given to the landlord in respect of residential
premises taken on rent by the employer and provided to the employee, be included in the
perquisite value of rent-free accommodation given to the employee?
CIT v. Shankar Krishnan (2012) (Bom.)
On appeal by the Revenue, the Bombay High Court held that the Assessing Officer is not right in
adding the notional interest on the security deposit given by the employer to the landlord in valuing the
perquisite of rent- free accomodation, since the perquisite value has to be computed as per Rule 3 and
Rule 3 does not require addition of such notional interest. Thus, the perquisite value of the residential
accommodation provided by the employer would be the actual amount of lease rental paid or payable
by the employer, since the same was lower than 10% (now 15%) of salary.

2. Can the limit of INR 1,000 per month per child be allowed as standard deduction, while
computing the perquisite value of free or concessional education facility provided to the
employee by the employer?
CIT (TDS) v. Director, Delhi Public School (2011) (Punj. & Har.)
The Punjab and Haryana High Court held that on a plain reading of Rule 3(5), it flows that, in case the
value of perquisite for free/concessional educational facility arising to an employee exceeds Rs. 1,000
per month per child, the whole perquisite shall be taxable in the hands of the employee and no
standard deduction of INR 1,000 per month per child can be provided from the same. It is only in case
the perquisite value is less than INR 1,000 per month per child, the perquisite value shall be nil.
Therefore, INR 1,000 per month per child is not a standard deduction to be provided while calculating
such a perquisite.

Impact of Section 115BAC under the head Salaries [Amendment vide Finance Act, 2020]
AY 21-22
• Finance act, 2020 has introduced a New Optional tax System for

• Individuals and HUFs u/s 115BAC of the income tax act, 1961 w.e.f. a/y 21-22 to provide for concessional
rate of Slab rates to be applied on total Income calculated without claiming specified deductions and
exemptions.

• Hence, from ay 2021-22 or FY 2020-21, there are two operative tax system –

1. One is the existing tax system where all the applicable deductions and exemptions are allowed and
the
tax rates are as per the Slab rates of tax specified in the Finance Act, 2020.
2. The second one is section 115BAC which is a Optional tax System and under which many
deductions and exemptions have not been allowed but lower slab tax rates are provided in the
section 115BAC itself.

• Individual and HUF opting for connectional tax regime under section 115BAC: the deduction under Chapter
Vi-a other than the provisions of sub-section (2) of section 80CCD or section 80JJAA; not available to the
individual and HUF opting to pay tax under connectional tax regime under section 115BAC of the income
tax act, 1961.

• Many exemptions & deduction are not allowed under the new tax system. the below chart
contains the exemptions and deduction not available under the new system related to income under
the head Salary. Similarly, deductions & exemptions not available under the new tax system and which are
related to other heads are provided in other chapters
Sr. Nature of Exemption/Deduction Relating to Head Salaries New System Existing
No. of Tax u/s system of
115BAC Tax
a RETIREMENT BENEFITS EXEMPTIONS

Unique Academy – 8007916622 CA Saumil Manglani - Contact: 9921051593


4. Salaries 4.30
leave Salary u/s 10(10aa) allowed allowed
Gratuity u/s 10(10) allowed allowed
Commutation of Pension u/s 10(10a) allowed allowed
retrenchment Compensation u/s 10(10B) allowed allowed
VRS Compensation u/s 10(10C) allowed allowed
leave travel Concession u/s 10(5) Not allowed allowed
B Allowances

Sr. Nature of Exemption/Deduction Relating to Head Salaries New System Existing


No. of Tax u/s system of
115BAC Tax
exemption u/s 10(13A) and rule 2A from House rent allowance Not allowed allowed
1. Exemption u/s 10(14)(i) and Rule 2BB
Travelling allowance allowed allowed
Conveyance allowance allowed allowed
daily allowance allowed allowed
Helper allowance Not allowed allowed
any allowance granted for encouraging the academic, research and Not allowed allowed
training pursuits in educational and research institutions
uniform allowance Not allowed allowed
2. Exemption u/s 10(14)(ii) and Rule 2BB
Children education allowance Not allowed allowed
Hostel expenditure allowance Not allowed allowed
tribal area allowance Not allowed allowed
transport allowance to Handicapped/deaf/dumb/Blind employee allowed allowed
transport allowance to other than above employees Not allowed Not allowed
c Perquisites
Free food and beverage through vouchers provided to the employee Not allowed allowed
upto 50/meal/tea & snacks
Other exemptions from perquisites e.g. use of Computers, laptops allowed allowed
etc
d Deductions u/s 16
Standard deduction u/s 16(ia) Not allowed allowed
entertainment allowance u/s 16(ii) Not allowed allowed
Professional tax u/s 16(iii) Not allowed allowed

Clarification in respect of option under section 115BAC of the Income-tax Act, 1961 [Notification No. 38/2020]
Section 115BAC of the income-tax act, 1961, inserted by the Finance act, 2020 w.e.f. the assessment year 2021-
22, inter alia, provides that a person, being
• An individual or a Hindu undivided family having income other than income from business or
profession”,
• may exercise option in respect of a previous year to be taxed under the said section 115BAC alongwith his
return of income to be furnished under sub-section (1) of section 139 of the act for each year.
• the concessional rate provided under section 115BAC of the act is subject to the condition that the total

Unique Academy – 8007916622 CA Saumil Manglani - Contact: 9921051593


4. Salaries 4.31
income shall be computed without specified exemption or deduction, set-off of loss and additional
depreciation.
Illustration 2: (ICSI Module)
Niteen is an employee of XYZ Ltd. He was appointed on 1st Mar 2020 at a scale of 50000 – 5000 – 70000.
He is paid DA (which form part of retirement benefits) @ 15% of Basic Pay and Bonus equivalent to 2
month’s salary at end of FY. He contributes 18% of his Basic + DA to a recognized provident fund, and the
contribution is matched by the employer.
He is provided rent free accommodation, hired by the employer, @ 25000 pm. He is also provided the
following benefits / amenities:
a) Medical Treatment of his dependant spouse INR 40000
b) Monthly salary to housekeeper INR 4000
c) Telephone Allowance INR 1200 pm
d) Gift Voucher of INR 4500 on account of his marriage anniversary
e) Medical Insurance Premium for Niteen, paid by his employer INR 15000
f) Motor Car owned and driven by Niteen, and engine capacity within 1.6 L; used partly for official
and partly for personal purposes. Running & maintenance expenses borne by the employer INR
36,600/-.
g) Lunch during office hours valued at INR 2200/-.
h) He was also allotted 2000 sweat equity shares in Sep 2020. The shares were allotted @ INR 227
per share against the FMV of INR 377 per share as on the date of exercise of the Option.
Compute the Salary Chargeable to tax.
Option 1 : assessee has not opted for Section 115BAC
Option 2 : assessee has opted for Section 115BAC

Solution
Particulars INR
Basic 6,05,000
DA 90,750
Bonus 1,10,000
Employers’ Contribution to PF > 12% 41,745
Taxable Allowances
Telephone 14,400
Taxable Perquisites
Medical Reimbursement 40,000
Housekeeper 48,000
Motor Car 15,000
Rent Free accommodation 1,23,023
Sweat Equity 3,00,000
Gross Salary 13,87,918
Standard Deduction (50,000)
Salary Chargeable to tax 13,37,918

Note:

1) Employer’s Contribution to Provident Fund in excess of 12% is chargeable to Income Tax.


2) Rent Free Accommodation is valued as under:

Unique Academy – 8007916622 CA Saumil Manglani - Contact: 9921051593


4. Salaries 4.32
a. Since the accommodation is hired, the actual hire charges subject to a cap of 15% of “salary”
is considered
b. “Salary” for this purpose is Basic + DA + Bonus + all Taxable Allowances = INR 8,20,150
3) Medical Treatment is chargeable to tax, as no more tax free perquisite.
4) Since the value of the gift voucher is below INR 5000, it is not taxable as perquisite.

5) Lunch during office hours is also not taxable as perquisite assuming cost of meal upto Rs.50/ meal.
6) Medical Insurance Premium paid by the employer on behalf of Niteen is also not taxable as
perquisite.
7) The motor car is chargeable as under:
If the If the Car is owned / hired by the employee; expenses met by the employer & is used by the
employee partly for Official and partly for Personal purposes, the taxable value of the perquisite
would be the actual expenditure incurred by the employer as reduced by the taxable value of the
perquisite determined basis the engine capacity, i.e., INR 36600 – INR (1800*12) = INR 15000

Option 2 : Assessee has opted for Section 115BAC

Particulars Amount (INR)


Basic 6,05,000
Da 90,750
Bonus 1,10,000
Employers’ Contribution to PF > 12% 41,745
taxable allowances telephone 14,400
taxable Perquisites Medical reimbursement (Fully taxable) 40,000
Housekeeper 48,000
Motor Car 15,000
rent Free accomodation 1,23,023
Sweat equity 3,00,000
Meals 2,200
Gross Salary 13,90,118
less: Standard deduction under section 16(ia) Not Available
taxable Salary 13,90,118

Note:
• Employer’s Contribution to Provident Fund in excess of 12% is chargeable to Income Tax.
• Rent Free Accommodation is valued as under:
a) Since the accommodation is hired, the actual hire charges subject to a cap of 15% of “salary” is
considered
b) “Salary” for this purpose is Basic + DA + Bonus + all Taxable Allowances = INR 8,20,150
• Medical Treatment is chargeable to Tax, as no more tax free perquisite.
• Since the value of the gift voucher is below INR 5000, it is not taxable as perquisite.
• Lunch during office hours is taxable as perquisite.
• Medical Insurance Premium paid by the employer on behalf of Niteen is also not taxable as perquisite.

• The motor car is chargeable as under:

Unique Academy – 8007916622 CA Saumil Manglani - Contact: 9921051593


4. Salaries 4.33
a) if the if the Car is owned / hired by the employee; expenses met by the employer & is used by the
employee partly for Official and partly for Personal purposes, the taxable value of the perquisite would
be the actual expenditure incurred by the employer as reduced by the taxable
• Deduction u/s 16 is not allowed
Illustration 3 : (ICSI Module)
Mr. Ram is employed at Bombay. His basic Salary is Rs. 5,000 per month. He receives Rs. 5,000 p.a. as
house rent allowance. Rent paid by him is Rs. 12,000 p.a. Find out the amount of taxable house rent
allowance assuming assessee has not opted u/s 115BAC
Solution:
As per Rule 2A, the least of the following is exempt from tax:
(i) the actual house rent allowance;
(ii) excess of rent paid over 10% of salary;
(iii) where the accommodation is situated at Bombay, Delhi, Calcutta or Madras, one-half of the
amount of salary due to the assessee for the relevant period;
(iv) Where the accommodation is situate at any other place, two-fifth of the salary due to the assessee
for the relevant period.
Accordingly, Mr. Ram would be entitled to the least of :
(i) Rs. 5,000 or
(ii) Rs. 6,000 being excess of rent over 1/10th of salary; or
(iii) Rs. 30,000 (being one-half of the salary of the assessee).
Rs. 5,000, being the least, would not be included in the total income of Mr. Ram. So the entire amount of
HRA would be exempt from tax.
Salary for this purpose includes basic salary as well as dearness allowance if the terms of employment so
provide. It also includes commission based on a fixed percentage of turnover achieved by an employee as
per terms of contract of employment but excludes all other allowances and perquisites and these are
determined on due basis for the period during which rental accommodation is occupied by the employee in
the previous year.
Illustration 4 : (ICSI Module)
Mr. Shyam, employed at Mumbai, receives the following from his employer during the previous year:
Particulars Rs.
Basic Salary 60,000
Bonus 1,800
Entertainment allowance (taxable) 6,000
Electricity expenses 2,000
Professional tax paid by the employer 2,000
Rent free house (owned by Employer):
Fair rent 48,000
Salary of gardener 2,400
Garden Maintenance 1,200
Salary of watchman 1,800

Determine the value of taxable perquisites in respect of rent free house assuming (a) Mr. Shyam is a
Government Officer and the fair rent as arrived at by the Government is Rs. 6,000 p.a (b) Mr. Shyam is a
semi-Government employee, and (c) Mr. Shyam is employed by a private company.

Unique Academy – 8007916622 CA Saumil Manglani - Contact: 9921051593


4. Salaries 4.34
Solution:
(a) If Mr. Shyam is a Government Officer: As per Rule 3(1) of Income-tax Rules, Rs. 6,000 p.a being
the rent of the house as per Government rules, will be the taxable value of the perquisite.
(b) If Mr. Shyam is a semi-Government employee : As per Rule 3(1) of the Income-tax Rules, the value
of the perquisite in respect of rent free accommodation is taken at 15% of salary of the employee (as
the house is owned by the Employer and provided in Mumbai).
Salary = Rs. 67,800 (‘Rs. 60,000 + 1,800 + 6,000)
15% of salary = Rs. 10,170 and
Therefore, Rs. 10,170 is taxable value of the perquisite.
Further, the value of Electricity expenses and Professional Tax paid by the employer, being
perquisites, are not included in the salary for valuation of Rent Free House Accommodation.
(c) If Mr. Shyam is employed in Private Company: The value of perquisite in this case shall also be
Rs. 10,170. Under the new rules there is no difference between the semi-Govt. and other
employees
Note: The Solution is provided assuming assessee has not opted for section 115BAC. However, even if
the would have opted for section 115BAC, the solution would be same.
Illustration 5 : (ICSI Module)
Mr. Ramamoorthy, an employee of M/s. Gopalkrishnan & Co. of Chennai receives during the previous
year ended March 31, 2020 the following payments:

Particulars (Rs.) (Rs.)


Basic Salary 40,000
Dearness allowance forms part of the retirement benefits 3,000
Leave Salary 5,400
Professional tax paid by employer 1,000
Fair rent of the flat provided by employer 6,000
Rent paid for furniture 1,000
Rent recovered by employer 3,000
Contribution to Statutory Provident Fund 4,000
Employer’s contribution to Statutory Provident Fund 4,000
Compute his taxable income for the Assessment Year 2020-21

Option 1 : assessee has not opted for Section 115BAC Option 2 : assessee has opted for Section 115BAC
Solution: Option 1 : assessee has not opted for Section 115BAC

Solution: Computation of taxable income of Mr. Rama moorty for the Assessment Year 2021-22

Basic Pay 40,000


Dearness allowance 3,000
Leave salary 5,400
Professional tax paid by employer 1,000
Perquisite for House:
15% of salary (Rs. 40,000 + 3,000 + 5,400) 7,260
Add: Furniture rent 1,000
5,260
Less: Rent recovered (-) 3,000
54,660
Less: Professional tax u/s 16 1,000
Unique Academy – 8007916622 CA Saumil Manglani - Contact: 9921051593
4. Salaries 4.35

Less: Standard deduction u/s 16 50,000


Gross Total Income 3,660
Less: Tax deduction under Section 80C (3,660)
Total income Nil
Total tax payable NIL

Note: Assumed that dearness allowance forms part of the salary for the purpose of computation of superannuation
or retirement benefits.
Option 2 : Assessee has opted for Section 115BAC
Computation of taxable income of Mr. Ramamoorty for the Assessment Year 2021-22

Basic Pay 40,000


dearness allowance 3,000
leave salary 5,400
Professional tax paid by employer 1,000
Perquisite for House :
15% of salary (Rs. 40,000 + 3,000 + 5,400) 7,260
add: Furniture rent 1,000 less:
rent recovered (-) 3,000 5,260
54,660
less: Standard deduction under section 16(ia) (Na)
less: Professional tax u/s 16 (Na)
Gross total income 54,660
less: deduction under Section 80C (Na)
total income 54660
total tax payable Nil

Note:
1. assumed that dearness allowance forms part of the salary for the purpose of computation of
superannuation or retirement benefits.
Deduction u/s 16 & 80C is not allowed as the assessee has opted for section 115BAC.

Unique Academy – 8007916622 CA Saumil Manglani - Contact: 9921051593


4. Salaries 4.36

Illustration 6: (ICSI Module)


Raman, an employee of the Gas Supply Ltd., Agra, receives the following emoluments during the
previous year 2019-20.

Particulars (Rs.)
Basic pay 10,000
Project allowance 1,800
Arrears of project allowance of May, 2012 150
Professional tax paid by the employer 200
Rent free furnished house
- Fair rent of the house 2,000
- Rent of furniture 500
Free gas supply 400
Service of sweeper 600
Services of gardener 1,000
Service of cook 800
Free lunch 2,400
Free use of chauffeur driven Fiat car which is used partly for official and partly for private purposes.
He is a member of recognized provident fund to which he contributes Rs.1,500. His employer also
contributes an equal amount. He deposits Rs. 600 per month in 10-year account under the Post
Office Savings Bank. Determine his taxable income and tax payable thereon for the assessment
year 2021- 22
(a) If Raman is a director in the employer company and the rent-free house is owned by it,
(b) If Raman is neither a director nor a shareholder in the employer company and the rent-free house is not
owned by it.

Solution:

Solution:
Option 1 : assessee has not opted for Section 115BAC
His taxable income will be computed as under :

If Raman is a director If Raman is neither


and rent-free house is a director nor a
owned by the company shareholder and
‘A’ rent- free house is
not owned by the
Company

‘B’
(1) (2) (3)
Rs. Rs.
Basic Pay 10,000 10,000
Project allowance 1,800 1,800
arrears of project allowance of May, 2012 150 150
Professional tax paid by the employer 200 200
rent free furnished house :
– 15% of Salary 1,770 1,770

Unique Academy – 8007916622 CA Saumil Manglani - Contact: 9921051593


4. Salaries 4.37
– rent of furniture 500 500
Free gas supply 400 Nil
Service of sweeper 600 Nil
Service of gardener 1,000 Nil
Service of cook 800 Nil
Free lunch Nil Nil
excess of employer’s contribution towards provident fund 300 300
over 12% of salary (1,500 - 12% of’ 10,000)
Gross salary 17,520 14,720
deduction : Standard deduction (Maximum Rs 50,000) 17,520 14,720
Net income from Salary Nil Nil
tax on total income Nil Nil

Notes:
(1) it is assumed that the arrears of project allowance are taxable on receipt basis.
(2) Perquisite in respect of rent Free house is taxable in the hands of all the assessees. in this case
fair market value has no relevancy (w.e.f. ay 2002-03) and assumed that the house is owned by
the employer. Since the house is provided in agra, population is assumed as exceeding 25 lakhs.
Salary for valuation of perquisite is (10,000 + 1,800).
(3) the free sweeper, gardener, cook, lunch, car etc. are not taxable in the second case, because
raman does not fall in the category of specified employee under Section 17(2)(iii) of the Act i.e.,
he is neither a director nor his salary is rs. 50,000 p.a. or more.
(4) Free lunch provided is not taxable to the extent of rs. 50 per day.

(5) Since raman is employed in a Gas supply company, the value of gas supplied is taxable as cost to the
employer. and it is assumed that the cost of supply is same as rs. 400 as given.
Option 2: assessee has opted for Section 115BAC
His taxable income will be computed as under :

If Raman is a director and If Raman is neither


rent-free house is owned by a director nor a
the company not owned by shareholder and
the Company rent- free house is
‘A’ ‘B’

(1) (2) (3)


rs. rs.
Basic Pay 10,000 10,000
Project allowance 1,800 1,800
arrears of project allowance of May, 2012 150 150
Professional tax paid by the employer 200 200
rent free furnished house :
– 15% of Salary 1,770 1,770
– rent of furniture 500 500
Free gas supply 400 Nil
Service of sweeper 600 Nil
Service of gardener 1,000 Nil
Service of cook 800 Nil

Unique Academy – 8007916622 CA Saumil Manglani - Contact: 9921051593


4. Salaries 4.38
Free lunch 2400 2400
excess of employer’s contribution towards provident 300 300
fund over 12% of salary (1,500 - 12% of’ 10,000)

Gross salary 19,920 17,120


deduction : Standard deduction Nil Nil

Net income from Salary 19,920 17,120


tax on total income Nil Nil

Notes:
1. it is assumed that the arrears of project allowance are taxable on receipt basis.
2. Perquisite in respect of Rent Free house is taxable in the hands of all the assessees. in this case
fair market value has no relevancy (w.e.f. ay 2002-03) and assumed that the house is owned by the
employer. Since the house is provided in Agra, population is assumed as exceeding 25 lakhs. Salary
for valuation of perquisite is (10,000 + 1,800).
3. The free sweeper, gardener, cook, lunch, car etc. are not taxable in the second case, because raman
does not fall in the category of specified employee under Section 17(2)(iii) of the Act i.e., he is
neither a director nor his salary is rs. 50,000 p.a. or more.
4. Free lunch provided is taxable as the assessee is opted for Section 115BAC.
5. Since Raman is employed in a Gas supply company, the value of gas supplied is taxable as cost to
the employer. and it is assumed that the cost of supply is same as rs. 400 as given.

6. No deduction is allowed u/s 16 as the assessee is opted for Section 115BAC

Illustration 6: (ICSI Module)


For the financial year 2020-21, ‘A’, a Central Government Officer receives salary of Rs. 77,000 (including
dearness allowance of Rs. 42,000) and entertainment allowance of Rs.18,000. His contribution to
provident fund during this period is Rs. 7,200. In addition, he has purchased National Savings Certificates
(VIII Issue) for Rs. 6,000. He has been provided with accommodation by the Government for which the
rent determined is Rs. 375 per month and this is recovered from A’s salary. Compute A’s tax liability for
the assessment year 2021-22 assuming that he has no other income.
Option 1 : assessee has not opted for Section 115BAC

Option 2 : assessee has opted for Section 115BAC

Solution:
Option 1 : assessee has not opted for Section 115BAC
Name of assessee : Mr. a
assessment year : 2021-22
Status: resident/individual
Statement of assessable
income

Solution:
Name of assessee: Mr. A Assessment Year: 2020-21 Status: Resident/Individual Statement of assessable
income
Salary from Central Government 77,000
Entertainment allowance 18,000
Less: Entertainment Allowance under Section 16(ii) Rs. 5,000 or [1/5th of 5,000 13,000
salary exclusive of any allowance, benefit or perquisite C 35,000)]

Unique Academy – 8007916622 CA Saumil Manglani - Contact: 9921051593


4. Salaries 4.39
Less: Standard Deduction (50,000)

GROSS TOTAL INCOME 40,000


Less : Deduction under Section 80C (7,200 + 6,000) 13,200
Total Income 26,800
Tax liability Nil

Option 2 : assessee has opted for Section 115BAC


Name of assessee : Mr. a assessment year : 2021-22 Status: resident/individual Statement of assessable
income

Salary from Central Government 77,000

entertainment allowance 18,000

less: deduction u/s 16 Nil

Gross Total Income 95,000

less : deduction under Section 80C Nil


total income 95,000

tax liability Nil

Net tax payable Nil

Illustration 7: Mr. X is employed in aBC ltd. getting basic pay rs. 60,000 p.m. and dearness allowance rs.
10,000 p.m. (forming part of salary). employer has paid bonus rs. 20,000 during the year. Commission was
allowed @ 2% of sales turnover of Rs. 50,00,000. The employer and employee both are contributing Rs. 11,000
p.m. (each) to the recognised provident fund. during the year interest of rs. 1,00,000 was credited to the RPF
@ 10% p.a. Compute tax liability of Mr. X for A.Y. 2021-22.
Option 1 : assessee has not opted for Section 115BAC
Option 2 : assessee has opted for Section 115BAC
Solution:
Option 1 : assessee has not opted for Section 115BAC

Particulars Amount (INR)


Basic Pay (60,000 x 12) 7,20,000
dearness allowance (10,000 x 12) 1,20,000
Bonus 20,000
Commission (50,00,000 x 2%) 1,00,000
Employer’s contribution to RPF >12% Salary 19,200
Interest credited in excess of 9.5% p.a.(1,00,000 / 10% x 0.5%) 500
Gross Salary 9,79,700
less: Standard deduction u/s 16 (ia) (50,000)
income under the head Salary 9,29,700
Gross total income 9,29,700
less: deduction u/s 80C (1,32,000)
total income 7,97,700

Unique Academy – 8007916622 CA Saumil Manglani - Contact: 9921051593


4. Salaries 4.40
Computation of tax liability
tax on rs. 7,97,700 at slab rate 72,040
Add: HEC @ 4% 2,882
tax liability rs. 74,922 (Round Off) 74,920

Option 2 : assessee has opted for Section 115BAC

Particulars Amount (INR)


Basic Pay (60,000 x 12) 7,20,000
dearness allowance (10,000 x 12) 1,20,000
Bonus 20,000
Commission (50,00,000 x 2%) 1,00,000
Employer’s contribution to RPF >12% Salary 19,200
Interest credited in excess of 9.5% p.a.(1,00,000 / 10% x 0.5%) 500
Gross Salary 9,79,700
less: Standard deduction u/s 16 (ia) (Nil)
income under the head Salary 9,79,700
Gross total income 9,79,700
less: deduction u/s 80C (Nil)
total income 9,79,700
Computation of tax liability
tax on rs. 9,79,700 at slab rate [Section 115BAC Note 1] 71,955
Add: HEC @ 4% 2,878
tax liability rs. 74,833 (Round Off) 74,830

Note:
Salary for RPF= 7,20,000 + 1,20,000 + 1,00,000 = 9,40,000 @12% = 1,12,800
employer’s contribution = 11,000 x 12 = 1,32,000
1,32,000 – 1,12,800 = 19,200

Unique Academy – 8007916622 CA Saumil Manglani - Contact: 9921051593


4. Salaries 4.41

Questions for Practice

1. Mr. Raj Kumar has the following receipts from his employer:
(1) Basic pay ₹3,000 p.m.
(2) Dearness allowance (D.A.) ₹600 p.m.
(3) Commission ₹6,000p.a.
(4) Motor car for personal use (expenditure met by the employer) ₹ 500 p.m.
(5) House rent allowance ₹ 900 p.m.
Find out the amount of HRA eligible for exemption to Mr. Raj Kumar assuming that he paid a rent of
₹ 1,000 p.m. for his accommodation at Kanpur. DA forms part of salary for retirement benefits.
(Assume 115BAC Not opted)

Solution
HRA received ₹ 10,800
Less: Exempt under section 10(13A) [Note] ₹ 7,680
Taxable HRA ₹ 3,120
Note: Exemption shall be least of the following three limits:

(a) the actual amount received (₹ 900 × 12) = ₹ 10,800

(b) excess of the actual rent paid by the assessee over 10% of his salary
= Rent Paid (-) 10% of salary for the relevant period
= (₹ 1,000×12) (-) 10% of [(₹ 3,000+₹ 600) × 12]
= ₹ 12,000 - ₹ 4,320 = ₹ 7,680

(c) 40% salary as his accommodation is situated at Kanpur


= 40% of [(₹ 3,000+ ₹ 600) × 12] = ₹ 17,280
Note: For the purpose of exemption under section 10(13A), salary includes dearness allowance only
when the terms of employment so provide, but excludes all other allowances and perquisites.

2. Mr. Srikant has two sons. He is in receipt of children education allowance of ₹ 150 p.m. for his elder son and
₹ 70 p.m. for his younger son. Both his sons are going to school. He also receives the following allowances:
Transport allowance : ₹ 1,800 p.m.
Tribal area allowance: ₹ 500 p.m.
Compute his taxable allowances.
Solution
Taxable allowance in the hands of Mr. Srikant is computed as under –
Children Education Allowance:
Elder son [(₹ 150 – ₹ 100) p.m. × 12 months] = ₹ 600
Younger son [(₹ 70 – ₹ 70) p.m. × 12 months] = Nil
Transport allowance (₹1,800 p.m. × 12 months) = ₹ 21,600
Tribal area allowance [(₹ 500 – ₹ 200) p.m. × 12 months] = ₹ 3,600
Taxable allowances 25,800

3. Mr. Sagar retired on 1.10.2020 receiving ₹ 5,000 p.m. as pension. On 1.2.2021, he commuted 60% of his
pension and received ₹ 3,00,000 as commuted pension. You are required to compute his taxable pension
assuming:
(a) He is a government employee.
(b) He is a private sector employee, receiving gratuity of ₹ 5,00,000 at the time of retirement.

Solution
(a) He is a government employee
Uncommuted pension received (October – March) ₹ 24,000

Unique Academy – 8007916622 CA Saumil Manglani - Contact: 9921051593


4. Salaries 4.42

[(₹ 5,000 × 4 months) + (40% of ₹ 5,000 × 2 months)]


Commuted pension received ₹ 3,00,000
Less: Exempt u/s 10(10A) (₹ 3,00,000) NIL
Taxable pension ₹ 24,000

(b) He is a private sector employee, receiving gratuity ₹ 5,00,000 at the time of retirement
Uncommuted pension received (October – March) ₹ 24,000
[(₹ 5,000 × 4 months) + (40% of ₹ 5,000 × 2 months)]
Commuted pension received ₹ 3,00,000
Less: Exempt u/s 10(10A)
1/3 x (3,00,000/60%)x100% (₹ 1,66,667) ₹ 1,33,333
Taxable pension ₹ 1,57,333

4. Mr. Ravi retired on 15.6.2020 after completion of 26 years 8 months of service and received gratuity
of ₹ 6,00,000. At the time of retirement, his salary was:
Basic Salary : ₹ 5,000 p.m.
Dearness Allowance : ₹ 3,000 p.m. (60% of which is for retirement benefits)
Commission : 1% of turnover (turnover in the last 12 months was ₹ 12,00,000)
Bonus : ₹ 12,000 p.a.
Compute his taxable gratuity assuming that he is private sector employee and covered by the Payment of Gratuity Act
1972.
Solution
He is covered by the Payment of Gratuity Act 1972
Gratuity received at the time of retirement ₹ 6,00,000
Less: Exemption under section 10(10) Least of the
following:
i. Gratuity received ₹ 6,00,000
ii. Statutory limit ₹ 20,00,000
iii. 15 days’ salary based on last drawn salary for each
completed year of service or part thereof in excess of 6
months

15/26 x Last drawn Salary x years of service

15/26 x (₹ 5,000 + ₹ 3,000) x 27 = ₹ 1,24,615 ₹ 1,24,615


Taxable Gratuity ₹ 4,75,385

5. Mr. Gupta retired on 1.12.2020 after 20 years 10 months of service, receiving leave salary of ₹ 5,00,000. Other
details of his salary income are:

Basic Salary : ₹5,000 p.m. (₹1,000 was increased w.e.f. 1.4.2020, so previously it was Rs. 4,000 p.m)

Dearness Allowance : ₹ 3,000 p.m. (60% of which is for retirement benefits)

Commission : ₹ 500 p.m.


Bonus : ₹ 1,000 p.m.
Leave availed during service : 480 days He was entitled to 30 days leave every year.
You are required to compute his taxable leave salary assuming:
(a) He is a government employee.

(b) He is a non-government employee.


Solution

(a) He is a government employee


Leave Salary received at the time of retirement ₹ 5,00,000

Unique Academy – 8007916622 CA Saumil Manglani - Contact: 9921051593


4. Salaries 4.43

Less: Exemption under section 10(10AA) ₹ 5,00,000


Taxable Leave salary Nil
(b) He is a non-government employee
Leave Salary received at the time of retirement ₹ 5,00,000
Less: Exempt under section 10(10AA) [See Note below] ₹26,400
Taxable Leave Salary ₹ 4,73,600

Note: Exemption under section 10(10AA) is least of the following:


(i) Leave salary received ₹ 5,00,000
(ii) Statutory limit ₹ 3,00,000
(iii) 10 months’ salary based on average salary of last 10 months
(10x Salary of last 10 months/10 Months) ₹ 66,000
(iv) Cash equivalent of leave standing at the credit of the employee
based on the average salary of last 10 months’ (max. 30 days per
year of service)
Leave Due = Leave allowed – Leave taken
= ( 30 days per year × 20 years ) – 480 days = 120 days

Leave due (in due)/ 30 days ) x Average salary p.m


( 120/30 x ₹ 66,000 / 10) ₹ 26,400

6. Mr. A retires from service on December 31, 2020, after 25 years of service. Following are the
particulars of his income/investments for the previous year 2020-21:

Particulars ₹
Basic pay @ ₹ 16,000 per month for 9 months 1,44,000
Dearness pay (50% forms part of the retirement 72,000
benefits) ₹ 8,000 per month for 9 months
Lumpsum payment received from the 6,00,000
Unrecognized Provident Fund
Deposits in the PPF account 40,000

Out of the amount received from the unrecognized provident fund, the employer’s contribution
was ₹ 2,20,000 and the interest thereon ₹ 50,000. The employee’s contribution was ₹ 2,70,000
and the interest thereon ₹ 60,000. What is the taxable portion of the amount received from the
unrecognized provident fund in the hands of Mr. A for the assessment year 2021-22? (Assume
that the employee has not opted for Section 115BAC)
Solution
Taxable portion of the amount received from the URPF in the hands of Mr. A for the A.Y. 2021-22 is
computed hereunder:

Particulars ₹
Amount taxable under the head “Salaries”:
Employer’s share in the payment received from the 2,20,000
URPF
Interest on the employer’s share 50,000
Total 2,70,000
Amount taxable under the head “Income from
Other Sources” :

Interest on the employee’s share 60,000


Total amount taxable from the amount 3,30,000
received from the fund

Unique Academy – 8007916622 CA Saumil Manglani - Contact: 9921051593


4. Salaries 4.44

Note: Since the employee is not eligible for deduction under section 80C for contribution to URPF at the time
of such contribution, the employee’s share received from the URPF is not taxable at the time of withdrawal
as this amount has already been taxed as his salary income

7. Will your answer be any different if the fund mentioned above was a recognised provident fund?
Solution
Since the fund is a recognized one, and the maturity is taking place after a service of 25 years, the entire
amount received on the maturity of the RPF will be fully exempt from tax.

8. Mr. B is working in XYZ Ltd. and has given the details of his income for the P.Y. 2020-21. You are required to
compute his gross salary from the details given below:
Basic Salary ₹ 10,000 p.m.
D.A. (50% is for retirement benefits) ₹ 8,000 p.m.
Commission as a percentage of turnover 0.1%
Turnover during the year ₹ 50,00,000
Bonus ₹ 40,000
Gratuity ₹ 25,000
His own contribution in the RPF ₹ 20,000
Employer’s contribution to RPF 20% of his basic salary

Interest accrued in the RPF @ 13% p.a. ₹ 13,000

Solution
Computation of Gross Salary of Mr. B for the A.Y.2021-22
Particulars ₹ ₹
Basic Salary [ ₹ 10,000 × 12] 1,20,000
Dearness Allowance [₹ 8,000 × 12] 96,000
Commission on turnover [0.1% × ₹ 50,00,000] 5,000
Bonus 40,000
Gratuity [Note 1] 25,000
Employee’s contribution to RPF [Note 2] -
Employers contribution to RPF [20% of ₹ 1,20,000] 24,000
Less: Exempt [Note 3] 20,760 3,240
Interest accrued in the RPF @ 13% p.a. 13,000
3,500
Less: Exempt @ 9.5% p.a. 9,500
Gross Salary 2,92,740

Note 1: Gratuity received during service is fully taxable.


Note 2: Employee’s contribution to RPF is not taxable. It is eligible for deduction under section 80C.
Note 3: Employers contribution in the RPF is exempt up to 12% of the salary.
i.e., 12% of [Basic Salary + Dearness Allowance forming part of retirement benefits + Commission
based on turnover] = 12% of [₹ 1,20,000 + (50% × ₹ 96,000) + ₹ 5,000] = 12% of ₹ 1,73,000 = ₹
20,760
Mr. D went on a holiday on 25.12.2020 to Delhi with his wife and three children (one son – age
5 years; twin daughters – age 2 years). They went by flight (economy class) and the total cost of
tickets reimbursed by his employer was ₹ 60,000 (₹ 45,000 for adults and ₹ 15,000 for the three
minor children). Compute the amount of LTC exempt. (Assume that the employee has not opted
for Section 115BAC)

Unique Academy – 8007916622 CA Saumil Manglani - Contact: 9921051593


4. Salaries 4.45

Solution
Since the son’s age is more than the twin daughters, Mr. D can avail exemption for all his three children. The
restriction of two children is not applicable to multiple births after one child. The holiday being in India and the
journey being performed by air (economy class), the entire reimbursement met by the employer is fully exempt.

9. In the above question , will there be any difference if among his three children the twins were 5
years old and the son 3 years old? Discuss.
Solution
Since the twins’ age is more than the son, Mr. D cannot avail for exemption for all his three
children. LTC exemption can be availed in respect of only two children.
Taxable
LTC = 15,000 X 1/3 = ₹ 5,000
LTC exempt is only ₹ 55,000 (i.e. ₹ 60,000 – ₹ 5,000)

10. Compute the taxable value of the perquisite in respect of medical facilities received by Mr. G
from his employer during the P.Y. 2020-21:
Medical premium paid for insuring health of Mr. G ₹ 7,000
Treatment of Mr. G by his family doctor ₹ 5,000
Treatment of Mrs. G in a Government hospital ₹ 25,000
Treatment of Mr. G’s grandfather in a private clinic ₹ 12,000
Treatment of Mr. G’s mother (68 years and dependent) by family doctor ₹ 8,000

Treatment of Mr. G’s sister (dependent) in a nursing home ₹ 3,000


Treatment of Mr. G’s brother (independent) ₹6,000
Treatment of Mr. G’s father (75 years and dependent) abroad ₹ 50,000

Expenses of staying abroad of the patient and ₹ 30,000


Limit specifiedby RBI ₹ 75,000
Solution

Computation of taxable value of perquisite in the hands of Mr. G


Particulars ₹ ₹
Treatment of Mrs. G in a Government hospital -
Treatment of Mr. G’s father (75 years and dependent) 50,000
abroad
Expenses of staying abroad of the patient and attendant 30,000

80,000
Less: Exempt up to limit specified by RBI 75,000 5,000
Medical premium paid for insuring health of Mr. G -

Treatment of Mr. G by his family doctor 5,000


Treatment of Mr. G’s mother (dependent) by family 8,000
doctor
Treatment of Mr. G’s sister (dependent) in a nursing 3,000
home
Treatment of Mr. G’s grandfather in a private clinic 12,000

Treatment of Mr. G’s brother (independent) 6,000


Taxable value of perquisite 39,000

Note: Grandfather and independent brother are not included within the meaning of family of Mr. G.
Unique Academy – 8007916622 CA Saumil Manglani - Contact: 9921051593
4. Salaries 4.46

11. Mr. C is a Finance Manager in ABC Ltd. The company has provided him with rent- free
unfurnished accommodation in Mumbai. He gives you the following particulars:
Basic salary ₹ 6,000p.m.
Dearness Allowance ₹ 2,000 p.m. (30% is for retirement benefits)
Bonus ₹ 1,500 p.m.
Even though the company allotted the house to him on 1.4.2020, he occupied the same only from
1.11.2020. Calculate the taxable value of the perquisite for A.Y.2021-22.

Solution
Value of the rent free unfurnished accommodation
= 15% of salary for the relevant period
= 15% of [(₹ 6000 × 5) + (₹ 2,000 × 30% × 5) + (₹ 1,500 × 5)] [See Note below]
= 15% of ₹ 40,500 = ₹ 6,075.
Note: Since, Mr. C occupies the house only from 1.11.2020, we have to include the salary due to
him only in respect of months during which he has occupied the accommodation. Hence salary for
5 months (i.e. from 1.11.2020 to 31.03.2021) will be considered.

12. Using the data given in the previous question, compute the value of the perquisite if Mr. C is
required to pay a rent of ₹ 1,000 p.m. to the company, for the use of this accommodation.
Solution
First of all, we have to see whether there is a concession in the matter of rent. In the case of accommodation
owned by the employer in cities having a population exceeding 25 lakh, there would be deemed to be a
concession in the matter of rent if 15% of salary exceeds rent recoverable from the employee.
In this case, 15% of salary would be ₹ 6,075 (i.e. 15% of ₹ 40,500). The rent paid by the employee is ₹ 5,000
(i.e., ₹ 1,000 x 5). Since 15% of salary exceeds the rent recovered from the employee, there is a deemed
concession in the matter of rent. Once there is a deemed concession, the provisions of Rule 3(1) would be
applicable in computing the taxable perquisite.
Value of the rent-free unfurnished accommodation = ₹ 6,075
Less: Rent paid by the employee (₹ 1,000 × 5) = ₹ 5,000
Perquisite value of unfurnished accommodation ₹1,075
given at concessional rent

13. Using the data given in Question 12, compute the value of the perquisite if ABC Ltd. has taken
this accommodation on a lease rent of ₹ 1,200 p.m. and Mr. C is required to pay a rent of ₹ 1,000
p.m. to the company, for the use of this accommodation.
Solution
Here again, we have to see whether there is a concession in the matter of rent. In the case of
accommodation taken on lease by the employer, there would be deemed to be a concession in the
matter of rent if the rent paid by the employer or 15% of salary, whichever is lower, exceeds rent
recoverable from the employee.
In this case, 15% of salary is ₹ 6,075 (i.e. 15% of ₹ 40,500). Rent paid by the employer is ₹ 6,000
(i.e. ₹ 1,200 x 5). The lower of the two is ₹ 6,000, which exceeds the rent paid by the employee i.e.
₹ 5,000 (₹ 1,000 x 5). Therefore, there is a deemed concession in the matter of rent. Once there is
a deemed concession, the provisions of Rule 3(1) would be applicable in computing the taxable
perquisite.
Value of the rent free unfurnished accommodation [Note] = ₹ 6,000
Less: Rent paid by the employee (₹ 1,000 × 5) = ₹ 5,000
Value of unfurnished accommodation given at concessional rent = ₹ 1,000

Note: Value of the rent free unfurnished accommodation is lower of


(i) Lease rent paid by the company for relevant period = ₹ 1,200 × 5 =₹ 6,000
(ii) 15% of salary for the relevant period (computed earlier) = ₹ 6,075

Unique Academy – 8007916622 CA Saumil Manglani - Contact: 9921051593


4. Salaries 4.47

14. Using the data given in Question 12, compute the value of the perquisite if ABC Ltd. has
provided a television (WDV ₹ 10,000; Cost ₹ 25,000) and two air conditioners. The rent paid by the
company for the air conditioners is ₹ 400 p.m. each. The television was provided on 1.1.2021.
However, Mr. C is required to pay a rent of ₹ 1,000 p.m. to the company, for the use of this furnished
accommodation.
Solution
Here again, we have to see whether there is a concession in the matter of rent. In the case of
accommodation owned by the employer in a city having a population exceeding ₹ 25 lakh, there would
be deemed to be a concession in the matter of rent if 15% of salary exceeds rent recoverable from the
employee. In case of furnished accommodation, the excess of hire charges paid or 10% p.a. of the
cost of furniture, as the case may be, over and above the charges paid or payable by the employee
has to be added to the value arrived at above to determine whether there is a concession in the matter
of rent.
In this case, 15% of salary is ₹ 6,075 (i.e. 15% of ₹ 40,500). The rent paid by the employee is ₹ 5,000
(i.e. ₹ 1,000 x 5). The value of furniture of ₹ 4,625 (see Note below) is to be added to 15% of salary.
The deemed concession in the matter of rent is ₹ 6,075 + ₹ 4,625 - ₹ 5,000 = ₹ 5,700. Once there is
a deemed concession, the provisions of Rule 3(1) would be applicable in computing the taxable
perquisite.
Value of the rent free unfurnished accommodation (computed earlier) = ₹ 6,075
Add: Value of furniture provided by the employer [Note] = ₹ 4,625
Value of rent-free furnished accommodation = ₹ 10,700
Less: Rent paid by the employee (₹ 1,000 × 5) = ₹ 5,000
Value of furnished accommodation given at concessional rent = ₹ 5,700
Note: Value of the furniture provided = (₹ 400 p.m. × 2 × 5 months) + (₹ 25,000 × 10%
p.a. for 3 months) = ₹ 4,000 + ₹ 625 = ₹ 4,625

15. Using the data given in above question, compute the value of the perquisite if Mr. C is a
government employee. The license fees determined by the Government for this accommodation was
₹ 700 p.m.

Solution
In the case of Government employees, the excess of license fees determined by the employer as
increased by the value of furniture and fixture over and above the rent recovered/ recoverable from
the employee and the charges paid or payable for furniture by the employee would be deemed to
be the concession in the matter of rent. Therefore, the deemed concession in the matter of rent is ₹
3,125 [i.e. ₹ 3,500 (license fees: ₹ 700 x 5) + ₹ 4,625 (Value of furniture) – ₹ 5,000 (₹ 1,000 × 5)].
Once there is a deemed concession, the provisions of Rule 3(1) would be applicable in computing
the taxable perquisite.
Value of the rent free unfurnished accommodation (₹ 700 × 5) = ₹ 3,500
Add: Value of furniture provided by the employer (computed earlier = ₹ 4,625
Value of rent-free furnished accommodation = ₹ 8,125
Less: Rent paid by the employee (₹ 1,000 × 5) = ₹ 5,000
Perquisite value of furnished accommodation given at concessional rent = ₹ 3,125

16. Mr. X and Mr. Y are working for M/s. Gama Ltd. As per salary fixation norms, the following
perquisites were offered:
(i) For Mr. X, who engaged a domestic servant for ₹ 500 per month, his employer reimbursed the entire
salary paid to the domestic servant i.e. ₹ 500 per month.
(ii) For Mr. Y, he was provided with a domestic servant @ ₹ 500 per month as part of remuneration
package.
You are required to comment on the taxability of the above in the hands of Mr. X and Mr. Y, who are not
specified employees.

Unique Academy – 8007916622 CA Saumil Manglani - Contact: 9921051593


4. Salaries 4.48

Solution
In the case of Mr. X, it becomes an obligation which the employee would have discharged even if the
employer did not reimburse the same. Hence, the perquisite will be covered under section 17(2)(iv) and
will be taxable in the hands of Mr. X. This is taxable in the case of all employees.
In the case of Mr. Y, it cannot be considered as an obligation which the employee would meet. The
employee might choose not to have a domestic servant. This is taxable only in the case of specified
employees covered by section 17(2)(iii). Hence, there is no perquisite element in the hands of Mr. Y.

17. Mr. X retired from the services of M/s Y Ltd. on 31.01.2021, after completing service of 30 years
and one month. He had joined the company on 1.1.1991 at the age of 30 years and received the
following on his retirement:
(i) Gratuity ₹ 6,00,000. He was covered under the Payment of Gratuity Act, 1972.
(ii) Leave encashment of ₹ 3,30,000 for 330 days leave balance in his account. He was credited
30 days leave for each completed year of service.

(iii) As per the scheme of the company, he was offered a car which was purchased on
30.01.2018 by the company for ₹ 5,00,000. Company has recovered ₹ 2,00,000 from him
for the car. Company depreciates the vehicles at the rate of 15% on Straight Line Method.
(iv) An amount of ₹ 3,00,000 as commutation of pension for 2/3 of his pension commutation.
(v) Company presented him a gift voucher worth ₹ 6,000 on his retirement.
(vi) His colleagues also gifted him a Television (LCD) worth ₹ 50,000 from their own
contribution.
Following are the other particulars:
(i) He has drawn a basic salary of ₹ 20,000 and 50% dearness allowance per month for the
period from 01.04.2020 to 31.01.2021.
(ii) Received pension of ₹ 5,000 per month for the period 01.02.2021 to 31.03.2021 after
commutation of pension.
Compute his gross total income from the above for Assessment Year 2021-22.
Solution
Computation of Gross Total Income of Mr. X for A.Y. 2021-22

Particulars ₹
Basic Salary = ₹ 20,000 x 10 2,00,000
Dearness Allowance = 50% of basic salary 1,00,000
Gift Voucher (See Note - 1) 6,000
Transfer of car (See Note - 2) 56,000
Gratuity (See Note - 3) 80,769
Leave encashment (See Note - 4) 1,30,000
Uncommuted pension (₹ 5000 x 2) 10,000
Commuted pension (See Note - 5) 1,50,000
Less- Standard Deduction (50,000)
Taxable Salary /Gross Total Income 7,82,769

Notes:
(1) As per Rule 3(7)(iv), the value of any gift or voucher or token in lieu of gift received by the employee or
by member of his household not exceeding ₹ 5,000 in aggregate during the previous year is exempt. In
this case, the amount was received on his retirement and the sum exceeds the limit of ₹ 5,000.
Therefore, the entire amount of ₹ 6,000 is liable to tax as perquisite.
Note - An alternate view possible is that only the sum in excess of ₹ 5,000 is taxable in view of the
language of Circular No.15/2001 dated 12.12.2001. Gifts upto ₹ 5,000 in the aggregate per annum
would be exempt, beyond which it would be taxed as a perquisite. As per this view, the value of
perquisite would be ₹ 1,000 and gross taxable income would be ₹ 7,27,769.

Unique Academy – 8007916622 CA Saumil Manglani - Contact: 9921051593


4. Salaries 4.49

(2) Perquisite value of transfer of car: As per Rule 3(7)(viii), the value of benefit to the employee, arising
from the transfer of an asset, being a motor car, by the employer is the actual cost of the motor car to
the employer as reduced by 20% of WDV of such motor car for each completed year during which such
motor car was put to use by the employer. Therefore, the value of perquisite on transfer of motor car, in
this case, would be:
Particulars ₹
Purchase price (30.1.2018) 5,00,000
Less: Depreciation @ 20% 1,00,000
WDV on 29.1.2019 4,00,000
Less: Depreciation @ 20% 80,000
WDV on 29.1.2020 3,20,000
Less: Depreciation @ 20% 64,000
WDV on 29.1.2021 2,56,000
Less: Amount recovered 2,00,000
Value of perquisite 56,000
The rate of 15% as well as the straight line method adopted by the company for depreciation of
vehicle is not relevant for calculation of perquisite value of car in the hands of Mr. X.
(3) Taxable gratuity

Particulars ₹
Gratuity received 6,00,000
Less : Exempt under section 10(10) - Least of the following:
(i) Notified limit = ₹ 20,00,000
(ii) Actual gratuity = ₹ 6,00,000
(iii) 15/26 x last drawn salary x no. of completed
years of services or part in excess of 6 months
15/26 x ₹ 30,000 x 30 = ₹ 5,19,231
5,19,231
Taxable Gratuity 80,769

Note: As per the Payment of Gratuity Act, 1972, D.A. is included in the meaning of salary.
Since in this case, Mr. X is covered under payment of Payment of Gratuity Act, 1972,
D.A. has to be included within the meaning of salary for computation of exemption under
section 10(10).
(4) Taxable leave encashment

Particulars ₹
Leave Salary received 3,30,000
Less: Exempt under section 10(10AA) - Least of the
following:

(i) Notified limit ₹ 3,00,000


(ii) Actual leave salary ₹ 3,30,000
(iii) 10 months x ₹ 20,000 ₹ 2,00,000

(iv) Cash equivalent of ₹ 2,20,000


leave to his credit
(330/30) x 20,000
2,00,000

Taxable Leave encashment 1,30,000

Unique Academy – 8007916622 CA Saumil Manglani - Contact: 9921051593


4. Salaries 4.50

Note – It has been assumed that dearness allowance does not form part of salary for retirement benefits.
In case it is assumed that dearness allowance forms part of pay for retirement benefits, then, the third limit
for exemption under section 10(10AA) in respect of leave encashment would be ₹ 3,00,000 (i.e. 10 x ₹
30,000) and thefourth limit ₹ 3,30,000, in which case, the taxable leave encashment would be ₹ 30,000
(₹3,30,000 - ₹ 3,00,000). In such a case, the gross total income would be ₹ 6,32,769.
(5) Commuted Pension
Since Mr. X is a non-government employee in receipt of gratuity, exemption under section
10(10A) would be available to the extent of 1/3rd of the amount of the pension which he would
have received had he commuted the whole of the pension.

Particulars ₹
Amount received 3,00,000
Exemption under section 10(10A) = 1/3 x (3,00,000 x 3/2)
1,50,000
Taxable amount 1,50,000

The taxability provisions under section 56(2)(x) are not attracted in respect of television
received from colleagues, since television is not included in the definition of property
therein.

18. Shri Bala employed in ABC Co. Ltd. as Finance Manager gives you the list of perquisites
provided by the company to him for the entire financial year 2020-21:

i. Domestic servant was provided at the residence of Bala. Salary of domestic servant is ₹ 1,500
per month. The servant was engaged by him and the salary is reimbursed by the company
(employer).
In case the company has employed the domestic servant, what is the value of perquisite?

i. Free education was provided to his two children Arthy and Ashok in a school maintained and owned by
the company. The cost of such education for Arthy is computed at ₹ 900 per month and for Ashok at ₹
1,200 per month. No amount was recovered by the company for such education facility from Bala.
ii. The employer has provided movable assets such as television, refrigerator and air- conditioner
at the residence of Bala. The actual cost of such assets provided to the employee is ₹ 1,10,000.
iv. A gift voucher worth ₹ 10,000 was given on the occasion of his marriage anniversary. It is
given by the company to all employees above certain grade.
v. Telephone provided at the residence of Shri Bala and the bill aggregating to
₹ 25,000 paid by the employer.
vi. Housing loan @ 6% per annum. Amount outstanding on 1.4.2020 is ₹ 6,00,000. Shri Bala
pays ₹ 12,000 per month towards principal, on 5th of each month.
Compute the chargeable perquisite in the hands of Mr. Bala for the A.Y. 2021-22.
The lending rate of State Bank of India as on 1.4.2020 for housing loan may be taken as
10%.

Solution
Taxability of perquisites provided by ABC Co. Ltd. to Shri Bala
(i) Domestic servant was employed by the employee and the salary of such domestic servant was
paid/ reimbursed by the employer. It is taxable as perquisite for all categories of employees.
Taxable perquisite value = ₹ 1,500 × 12 = ₹ 18,000.
If the company had employed the domestic servant and the facility of such servant is given to the
employee, then the perquisite is taxable only in the case of specified employees. The value of the
taxable perquisite in such a case also would be ₹ 18,000.

(ii) Where the educational institution is owned by the employer, the value of perquisite in respect of
free education facility shall be determined with reference to the reasonable cost of such education
in a similar institution in or near the locality. However, there would be no perquisite if the cost of
such education per child does not exceed ₹ 1,000 per month.
Therefore, there would be no perquisite in respect of cost of free education provided to his child

Unique Academy – 8007916622 CA Saumil Manglani - Contact: 9921051593


4. Salaries 4.51

Arthy, since the cost does not exceed ₹ 1,000 per month.
However, the cost of free education provided to his child Ashok would be taxable, since the cost
exceeds ₹ 1,000 per month. The taxable perquisite value would be ₹ 14,400 (₹ 1,200 × 12).
Note – An alternate view possible is that only the sum in excess of ₹ 1,000 per month is taxable. In
such a case, the value of perquisite would be ₹ 2,400.

(iii) Where the employer has provided movable assets to the employee or any member of his
household, 10% per annum of the actual cost of such asset owned or the amount of hire charges
incurred by the employer shall be the value of perquisite. However, this will not apply to laptops
and computers. In this case, the movable assets are television, refrigerator and air conditioner and
actual cost of such assets is ₹ 1,10,000.
The perquisite value would be 10% of the actual cost i.e., ₹ 11,000, being 10% of ₹ 1,10,000.
(iv) The value of any gift or voucher or token in lieu of gift received by the employee or by member of
his household not exceeding ₹ 5,000 in aggregate during the previous year is exempt. In this case,
the amount was received on the occasion of marriage anniversary and the sum exceeds the limit
of ₹ 5,000.
Therefore, the entire amount of ₹ 10,000 is liable to tax as perquisite.

Note- An alternate view possible is that only the sum in excess of ₹ 5,000 is taxable in view of the
language of Circular No.15/2001 dated 12.12.2001. Gifts upto ₹ 5,000 in the aggregate per annum
would be exempt, beyond which it would be taxed as a perquisite. As per this view, the value of
perquisite would be ₹ 5,000.
(v) Telephone provided at the residence of the employee and payment of bill by the employer is a tax
free perquisite.
(vi) The value of the benefit to the assessee resulting from the provision of interest-free or
concessional loan made available to the employee or any member of his household during the
relevant previous year by the employer or any person on his behalf shall be determined as the sum
equal to the interest computed at the rate charged per annum by the State Bank of India (SBI) as
on the 1st day of the relevant previous year in respect of loans for the same purpose advanced by
it. This rate should be applied on the maximum outstanding monthly balance and the resulting
amount should be reduced by the interest, if any, actually paid by him.
“Maximum outstanding monthly balance” means the aggregate outstanding balance for loan as on
the last day of each month.
The perquisite value for computation is 10% - 6% = 4%
Month Maximum outstanding Perquisite value at
balance as on last date 4% for the month
of (₹)
month (₹)
April, 2020 5,88,000 1,960
May, 2020 5,76,000 1,920
June, 2020 5,64,000 1,880
July, 2020 5,52,000 1,840
August, 2020 5,40,000 1,800
September, 2020 5,28,000 1,760
October, 2020 5,16,000 1,720
November, 2020 5,04,000 1,680
December, 2020 4,92,000 1,640
January, 2021 4,80,000 1,600
February, 2021 4,68,000 1,560
March, 2021 4,56,000 1,520
Total value of this perquisite 20,880

Total value of taxable perquisite


= ₹ 74,280 [i.e. ₹ 18,000 + ₹ 14,400 + ₹ 11,000 + ₹ 10,000 + ₹ 20,880].
Note - In case the alternate views are taken for items (ii) & (iv), the total value of taxable perquisite
would be ₹ 57,280 [i.e., ₹ 18,000 +₹ 2,400 +₹ 11,000 +₹ 5,000 + ₹ 20,880].

Unique Academy – 8007916622 CA Saumil Manglani - Contact: 9921051593


4. Salaries 4.52

19. AB Co. Ltd. allotted 1000 sweat equity shares to Sri Chand in June 2020.The shares were
allotted at ₹ 200 per share as against the fair market value of ₹ 300 per share on the date of exercise
of option by the allottee viz. Sri Chand. The fair market value was computed in accordance with the
method prescribed under the Act.
(i) What is the perquisite value of sweat equity shares allotted to Sri Chand?
(ii) In the case of subsequent sale of those shares by Sri Chand, what would be the cost of
acquisition of those sweat equity shares?

Solution
(i) As per section 17(2)(vi), the value of sweat equity shares chargeable to tax as perquisite shall be
the fair market value of such shares on the date on which the option is exercised by the assessee
as reduced by the amount actually paid by, or recovered from, the assessee in respect of such
shares.
Particulars ₹
Fair market value of 1000 sweat equity shares @ ₹ 300 3,00,000
each
Less: Amount recovered from Sri Chand 1000 shares @ ₹ 2,00,000
200 each
Value of perquisite of sweat equity shares allotted to 1,00,000
Sri Chand

(ii) As per section 49(2AA), where capital gain arises from transfer of sweat equity shares, the cost
of acquisition of such shares shall be the fair market value which has been taken into account for
perquisite valuation under section 17(2)(vi). (The provisions of section 49 are discussed in Unit 4:
Capital Gains of this chapter)
(iii) Therefore, in case of subsequent sale of sweat equity shares by Sri Chand, the cost of
acquisition would be ₹ 3,00,000.

20. X Ltd. provided the following perquisites to its employee Mr. Y for the P.Y. 2020-21 –
(1) Accommodation taken on lease by X Ltd. for ₹ 15,000 p.m. ₹ 5,000 p.m. is recovered from the
salary of Mr. Y.

(2) Furniture, for which the hire charges paid by X Ltd. is ₹ 3,000 p.m. No amount is
recovered from the employee in respect of thesame.

(3) A Santro Car which is owned by X Ltd. and given to Mr. Y to be used both for official and
personal purposes. All running and maintenance expenses are fully met by the employer. He is also
provided with a chauffeur.

(4) A gift voucher of ₹ 10,000 on his birthday.


Compute the value of perquisites chargeable to tax for the A.Y.2021-22, assuming his salary for
perquisite valuation to be ₹ 10 lakh.

Solution
Computation of the value of perquisites chargeable to tax in the hands of Mr. Y for the A.Y.2021-22
Particulars Amount in ₹
1 Value of concessional accommodation
Actual amount of lease rental paid by X Ltd. 1,80,000
15% of salary i.e., 15% of ₹ 10,00,000 Lower of the 1,50,000
above 1,50,000
Less: Rent paid by Mr. Y (₹ 5,000 × 12) 60,000
90,000
2 Add: Hire charges paid by X Ltd. for furniture provided for 36,000 1,26,000
the use of Mr. Y (₹ 3,000 × 12)

Unique Academy – 8007916622 CA Saumil Manglani - Contact: 9921051593


4. Salaries 4.53

Perquisite value of Santro car owned by X Ltd. and 32,400


provided to Mr. Y for his personal and official use [(₹
1,800 + ₹ 900) x 12)]

3 Value of gift voucher 10,000


Value of perquisites chargeable to tax 1,68,400

21. Mr. Goyal receives the following emoluments during the previous year ending 31.03.2020.
Basic pay ₹ 40,000
Dearness Allowance ₹ 15,000
Commission ₹ 10,000
Entertainment allowance ₹ 4,000
Medical expenses reimbursed ₹ 25,000
Professional tax paid ₹ 2,000 (₹ 1,000 was paid by his employer)
Mr. Goyal contributes ₹ 5,000 towards recognized provident fund. He has no other income. Determine the
income from salary for A.Y. 2021-22, if Mr. Goyal is a State Government employee. (Assuming that employee
has not opted for Section 115BAC)
Solution
Computation of salary of Mr. Goyal for the A.Y.2021-22
Particulars ₹ ₹
Basic Salary 40,000
Dearness Allowance 15,000
Commission 10,000
Entertainment Allowance received 4,000
Employee’s contribution to RPF [Note] -
Medical expenses reimbursed 25,000
Professional tax paid by the employer 1,000
Gross Salary 95,000
Less: Deductions under section 16
under section 16(ia) - Standard deduction 50,000
of upto ₹ 50,000
under section 16(ii) - Entertainment
allowance being lower of :
(a) Allowance received 4,000
(b) One fifth of basic salary [1/5 × ₹ 8,000
40,000]
(c) Statutory amount 5,000 4,000
under section 16(iii) - Professional tax 2,000
paid
Income from Salary 39,000

Unique Academy – 8007916622 CA Saumil Manglani - Contact: 9921051593


5. House Property 5.1

Chapter - 5

Unique Academy - 8007916622 CA Saumil Manglani - Contact: 9921051593


5. House Property 5.2

REGULATORY FRAMEWORK
Sections Details
Section 22 Basis of Charge
Section 23(1) annual Value of House Property
Section 23(2) annual Value where property is self-occupied / unoccupied
Section 23(3) annual Value where the property is partly let out and partly self-occupied
Section 23(4) deemed to be let-out property
Section 23(5) Notional income from house property held as stock in trade
Section 24 deduction from Net annual Value
Section 24(a) Standard deduction
Section 24(b) interest on borrowed capital
Section 25 inadmissible deductions
Section 25A treatment of unrealized rent / arrear of rent
Section 26 income from Co-Owned Property
Section 27 deemed Ownership

CHARGEABILITY [SECTION 22]


(i) The process of computation of income under the head “Income from house property” starts
with the determination of annual value of the property. The concept of annual value and
the method of determination is laid down in section 23.
(ii) The annual value of any property comprising of buildings or lands appurtenant
thereto of which the assessee is the owner is chargeable to tax under the head
“Income from house property”.
Exceptions: Annual value of the following properties are chargeable under the head
“Profits and gains of business or profession” -
(i) Portions of property occupied by the assessee for the purpose of any business or
profession carried on by him.
(ii) Properties of an assessee engaged in the business of letting out of properties.

Exceptions Income from letting out a vacant land is chargeable to tax under
the head “Income from Other Sources”

CONDITIONS FOR CHARGEABILITY


(i) Property should consist of any building or land appurtenant thereto.
(ii) Assessee must be the owner of the property
(iii) Use of property - The property may be used for any purpose, but it should not be used by
the owner for the purpose of any business or profession carried on by him, the profit of
which is chargeable to tax.
The income earned by an assessee engaged in the business of letting out of
properties on rent would be taxable as business income(Supreme Court ruling in
Rayala Corporation (P) Ltd. v. Asstt. CIT)
(iv) Property held as stock-in-trade etc.

Unique Academy - 8007916622 CA Saumil Manglani - Contact: 9921051593


5. House Property 5.3
Annual value of house property will be charged under the head “Income from house
property”, where it is held by the assessee as stock-in-trade of a business also.
However, the annual value of property being held as stock in trade would be treated
as NIL for a period of two year from the end of the financial year in which certificate
of completion of construction of the property is obtained from the competent authority,
if such property is not let-out during such period. [Section 23(5)]

NAV = Nil
Example

1 April 20 31st March 21 31st March 23


On 15th
July 20
1st April 23 onwards
Certificate
treated as Deemed
received
Let out if not sold
by the contractor

Note - Even the Income of House property situated outside India is Taxable in
India under the head “Income from House Property.

Calculating GAV and NAV – Section 23 + Deductions u/s 24 – Pro forma

Sr.
Particulars
No. Amount
(i) Municipal value xxx
(ii) Fair rent xxx
(iii) Higher of (i) & (ii)
(iv) Standard rent xxx
(v) Expected rent [Lower of (iii) & (iv)
(vi) Annual/Actual rent received/ receivable xxx
(via) After above step – Deduct “Element of Vacancy”
(vii) GAV [Higher of (v) & (vi)] xxx

Municipal taxes paid/ Sewerage Tax in the current year


(It may be of the previous years as well but paid in the
current year & It must be paid by the owner)

Municipal taxes paid outside India are also allowed as


Less deduction (xxx)
(viiI) NAV xxx
Less Deductions u/s 24
(a) Standard Deduction @ 30% (xxx)
(b) Interest on borrowed Capital (xxx)
(ix) Income from house property xxx

Note – Always before comparing with expected rent - First deduct Unrealized rent from
actual/ annual rent
Unique Academy - 8007916622 CA Saumil Manglani - Contact: 9921051593
5. House Property 5.4

DETERMINATION OF ANNUAL VALUE

The process of determination of Annual Value is exhibited below.


Annual Value: The measure of charging income-tax under this head is the annual value of the
property, i.e., the inherent capacity of a building to yield income. The expression ‘annual value’
has been defined in Section 23(1) of the Income-tax Act.
Provided that the taxes levied by any local authority in respect of the property shall be
deducted (irrespective of the previous year in which the liability to pay such taxes was incurred
by the owner according to the method of accounting regularly employed by him) in determining the
annual value of the property of that previous year in which such taxes are actually paid by him, i.e.,
municipal taxes will be allowed only in the year in which it was paid.

• If assessee have only 2 house properties which are self-occupied then the Net Annual Value
of that property is considered as “Nil”

• Where the property consists of a house or part of a house which is in the occupation of the
owner for the purposes of his own residence; or cannot actually be occupied by the owner by
reason of the fact that owing to his employment, business or profession carried on at any
other place, he has to reside at that other place in a building not belonging to him, the annual
value of such house or part of the house shall be taken to be Nil. However, the above
provisions shall not apply if:

(a) the house or part of the house is actually let during the whole or any part of the previous
year; or

(b) any other benefit therefrom is derived by the owner.

• Where the assessee has more than 2 Self Occupied Property then - Only two houses
(any) will be considered as Self Occupied and

• The annual value of the house or houses, other than the house in respect of which the
assessee has exercised an option to claim benefit of Self Occupied, shall be determined \ as
if such house or houses had been let (Deemed to be let out).

Unrealized Rent: The amount of rent which the owner cannot realize shall be equal to the
amount of rent payable but not paid by a tenant of the assessee and so proved to be lost and
irrevocable only if following conditions are satisfied:

(a) tenancy is bonafide;

(b) the defaulting tenant has vacated, or steps have been taken to compel him to vacate the
property;
(c) the defaulting tenant is not in occupation of any other property of the assessee;
(d) the assessee has taken all reasonable steps to institute legal proceedings for the recovery
of the unpaid rent or satisfied the Assessing Officer that legal proceedings would be
useless.

Unique Academy - 8007916622 CA Saumil Manglani - Contact: 9921051593


5. House Property 5.5

Where the property is let out for the whole year [Section 23(1)]

GROSS ANNUAL VALUE

HIGHER OF THE FOLLOWING

EXPECTED RENT ACTUAL RENT RECEIVED


(cannot exceed standard rent)

Higher of

FAIR RENT MUNCIPAL VALUE

The GAV is the higher of


o Expected Rent (ER) and
o Actual Rent received / receivable (AR)
Note:
✓ The Expected Rent is the higher of Fair Rent (FR) and the Municipal Value (MV), but
capped to Standard Rent (SR).
✓ Fair Rent is the rental fetched by a similar property in the adjoining neighborhood.
✓ The Municipal Value is the value determined by the Municipal Authorities.
✓ The Standard Rent is the rent fixed by the Rent Control Act.
Municipal Taxes: The taxes including service taxes (fire tax, conservancy tax, education, water
tax, etc.) levied by any municipality or local authority in respect of any house property to
the extent to which such taxes are borne and paid by the owner, and include enhanced
municipal tax finally determined on appeal and payable by assessee - Clive Buildings Cola
Ltd. v. CIT (1989) 44 Taxman 160.

However, deduction in respect of municipal taxes will be allowed in determining the annual
value of the property only in the year in which municipal taxes are actually paid by the
owner.

Where the tax on property is enhanced with retrospective effect by municipal or local
authorities and the enhanced tax relating to the prior year is demanded during the assessment
year, the entire demand is deductible in the assessment year [C.I.T. v. L. Kuppu Swamy
Chettiar (1981) 132 ITR 416 (Mad.)].

Even where the property is situated outside the country taxes levied by local authority in
that country is deductible is deciding the annual value of the property. [CIT v. R Venugopala
Riddiar (1965) 58 ITR 439 (Mad.)]

Unique Academy - 8007916622 CA Saumil Manglani - Contact: 9921051593


5. House Property 5.6

Illustration 1
Mr. X is the owner of three houses, which are all let out and not governed by the Rent Control
Act. From the following particulars find out the gross annual value in each case:

Particulars I II III
Municipal Value 30,000 20,000 35,000
Actual (De facto) Rent 32,000 28,000 30,000
Fair Rent 36,000 24,000 32,000

Solution:
Gross Annual Value (GAV): Higher of Expected or Actual Rent
Expected Rent: Higher of Municipal Valuation or Fair Rent
House I: Rs. 36,000
House II: Rs. 24,000
House III: Rs. 35,000
Actual Rent (given) GAV:
House I: Rs. 36,000 House II: Rs. 28,000 House III: Rs. 35,000

Illustration 2
Mr. X is the owner of four houses, which are all let out and are covered by the Rent Control Act.
From the following particulars find out the gross annual value in each case, giving reasons for
your answer:

Particulars I II III IV
Municipal Value 30,000 26,000 35,000 30,000
Actual (De Facto) Rent 40,000 30,000 32,000 32,000
Fair Rent 36,000 28,000 30,000 36,000
Standard Rent 30,000 35,000 36,000 40,000

Solution
As all the houses are covered by the Rent Control Act, their gross annual value will be higher of
expected Rent or Actual Rent. Expected Rent Shall be higher of Municipal Value or Fair rent but
subject to Standard Rent:

Particulars I II III IV
Expected Rent 30,000 28,000 35,000 36,000
Actual (De Facto) Rent 40,000 30,000 32,000 32,000
G.A.V. 40,000 30,000 35,000 36,000

Where let out property is vacant for part of the year [Section 23(1)]
In a scenario of vacancy for a part of the year, it is quite probable that the Actual
Rent received / receivable would fall lower than Expected Rent and in such an
eventuality; therefore the Actual Rent becomes the Gross Annual Value.
Illustration 3:
(i.e. No vacancy but there is unrealized rent)

Mr. A owns two houses. The expected rent of the house one is Rs. 65,000. This house was let

Unique Academy - 8007916622 CA Saumil Manglani - Contact: 9921051593


5. House Property 5.7

out for Rs. 7,500 p.m. But the rent for the months of February and March, 2021 could not
be realized.
The expected rent of another house is Rs. 1,50,000. This house was let out for Rs.12,000 p.m.
But the rent for the last three months could not be realized.
In the both cases, Mr. A fulfills the conditions of Rule 4. You are required to compute the Gross
Annual Value of both the houses.
Solution
House I House II
Expected Rent 65,000 1,50,000
Annual Rent` 90,000 144000
Unrealized Rent 15,000 36,000
Computation of Gross Annual Value
Step 1: Expected Rent 65,000 1,50,000
Step 2: Actual Rent (After deducting unrealized rent) 75,000 N.A.
if higher than Expected Rent then Actual rent
otherwise Expected rent
Step 3: Applicable only in case of vacancy N.A. N.A.
Gross Annual Value 75,000 1,50,000

Illustration 4:
(There is vacancy but no unrealized rent)
Find out the gross annual value in the case of the following properties for the Assessment year
2021-22
P Q R S
Expected Rent (Rs. In ‘ 000) 70 55 85 125
Rent Per Month (if let out) 7 5 8 8
Let out period (in months) 11 0 9 10
Vacancy (in months) 1 12 3 2
Further all the rent were realized for the year by the assessee.

Solution:
Calculation of Gross Annual Value of Mr. X for A.Y 2021-22

P Q R S
Annual Rent (If let out for 12 months) 84 60 96 96
Loss due to vacancy 7 60 24 16
Unrealized rent Nil Nil Nil Nil
Actual Rent (for let out period) 77 Nil 72 88
Calculation of Gross Annual Value
Step 1: Expected Rent 70 55 85 125
Step 2: If actual rent is more than Expected
Rent than Actual rent otherwise expected 77 N.A. N.A. N.A.
Rent

Unique Academy - 8007916622 CA Saumil Manglani - Contact: 9921051593


5. House Property 5.8

Step 3: If property remain vacant then 77 0 72 109


decline due to vacancy shall be
considered
Gross annual value 77 0 72 109

Illustration 5:
(Vacancy and unrealized rent both exist)
Mr. X is the owner of a house property. He lets this property during the previous year 2020-21 for
Rs. 7,000 p.m. The house was occupied from 1.4.2020 to 31.1.2021. From 1.2.2021, it
remained vacant. Mr. X fails to realize Rs. 10,000 from the tenant. The Expected rent of the
house is Rs. 82,000 p.a.
Calculate the Gross Annual Value of the house.

Solution
Rs.
Expected Rent 82,000
Annual Rent (Actual for the whole year - 7000 x 12) less unrealized rent of Rs. 74,000
10,000
Higher of above two 82,000

Less - Loss Due to vacancy (7,000 x 2) (14,000)


Gross Annual Value 68,000

Where property is self-occupied / unoccupied [Section 23(2)]


Where the property consists of a house or part of a house in the occupation of the owner for
his own residence, and is not actually let during any part of the previous year and no other
benefit is derived therefrom by the owner, the annual value of such a house or part of the
house shall be taken to be nil. The only deduction available in respect of such house is
towards interest on borrowed capital in terms of Section 24(1)(vi) but subject to a
ceiling of Rs. 30,000 or Rs. 2,00,000 as the case may be. In other words, to this extent
there could be a loss from such house

The aggregate of amount allowed as deduction for both the houses (SOP) cannot
exceed Rs. 30,000/ Rs. 2,00,000
Concession for two houses only:
Where the assessee has occupied more than two houses for the purposes of residence for
himself and family members, he has to make a choice of any two houses in respect of which
he would like to claim exemption. Other self-occupied houses will be treated as if they were
let out and their annual value will be determined in the same manner as we have discussed
in the case of let out property.
✓ Annual Value would be taken as Nil
✓ It is imperative that the property is self-occupied OR unoccupied for the whole year
✓ This benefit is for two houses

Unique Academy - 8007916622 CA Saumil Manglani - Contact: 9921051593


5. House Property 5.9

✓ This benefit is for Individual / HUF only


✓ No deduction is allowed for Municipal Taxes for such property
Note: the deduction of Rs. 30,000 / Rs. 2,00,000 with respect to interest paid on borrowed capital
u/s 24(b) not allowed in case of Self occupied Property, if assessee opted for section 115BAC of
the income tax act, 1961
AY 21-22
Illustration 6:
Mr. R owns a house which uses for residential purposes throughout the previous year 2020-21.
Municipal Value: Rs. 2,40,000. Fair Rent: Rs. 3,00,000. Compute income from house property
assuming following expenditure are incurred by him:
Municipal taxes paid: Rs. 15,000
Repairs: Rs. 12,000
Depreciation: Rs. 10,000
Interest on borrowed capital: Rs. 2,00,000 (loan taken on 1.1.2005). House was purchased on
1.5.2006.

Option 1 : assessee has not opted for Section 115BaC

Option 2 : assessee has opted for Section 115BaC

Solution:
Option 1 Assessee has not opted for Section 115BAC
income from House Property:
Net annual Value Nil
Less : interest on borrowed capital 2,00,000
(lower of actual interest or 2,00,000; as conditions are satisfied)
loss from House Property (2,00,000)
Option 2 : Assessee has opted for Section 115BAC
income from House Property:
Net annual Value Nil
less : interest on borrowed capital Not Available
income from House Property Nil

Where the property is partly let out and partly self-occupied during the PY [Section
23(3)]
(a) Property let out partially:
When a portion of the house is self-occupied for the full year and a portion is self-
occupied for whole year, the annual value of the house shall be determined as under:

(i) From the full annual value of the house the proportionate annual value for self-
occupied portion for the whole year shall be deducted.

(ii) The balance under (i) shall be the annual value for let out portion for a part of the year.
Illustration 7:
Mr. R. owns a house. The Municipal value of the house is Rs. 50,000. He paid Rs. 8,000 as
local taxes during the year. He uses this house for his residential purposes but lets out half of
the house @ Rs. 3,000 p.m. Compute the annual value of the house.

Unique Academy - 8007916622 CA Saumil Manglani - Contact: 9921051593


5. House Property 5.10
Solution:
Rs.
Annual rent or Municipal valuation (higher) 72,000
Less : Local taxes paid 8,000
Annual value of House Property 64,000
Less: Half of annual value regarding self-occupied portion for the whole 32,000
year
Annual Value of let out portion 16,000

(b) House let out during any part of the previous year and self-occupied for the
remaining part of the year:
In this case the benefit of Section 23(2) i.e. SOP is not available and the income will be
computed as if the property is let out.
Illustration 8:
M is the owner of a house. The municipal value of the house is Rs. 40,000. He paid Rs.
8,000 as local taxes during the year. He was using this house for his residential purposes but
let out w.e.f. 1.1.2020@ Rs. 4,000 p.m. Compute the annual value of the house for PY 20-21
Solution
Rs.
Annual rent or municipal valuation (whichever is higher) 48,000
Less: Local taxes 8,000
Annual value of the house 40,000
(No benefit shall be given for self-occupied period as the house did not remain
vacant during the previous year)
Note: If fair rent is not given, then assume actual rent as fair rent.
Deemed to be let-out property [Section 23(4)]
✓ Assessee given the choice of any two houses to be construed as self-occupied and for
that the Annual Value would be NIL
✓ For others, they would be treated as deemed to be let out
✓ The assessee is allowed by the Income Tax Act; the flexibility to change the option to
suit his needs / benefits
✓ In such as case, therefore, the Expected Rent becomes the Gross Annual Value
✓ Municipal Taxes paid by the owner for the whole year allowed as a deduction
Deductions from Net Annual Value (Section 24)
Standard Deduction: 30% of Net Annual Value
a. This is not available when the Annual Value is NIL
b. This is a flat deduction irrespective of the actual expenditure incurred

Unique Academy - 8007916622 CA Saumil Manglani - Contact: 9921051593


5. House Property 5.11

Interest on Borrowed Capital is also allowed.


Deductions
Allowed from NAV

Let out/deemed Self-occupied


Iet out properties
properties

Interest on
borrowed
Standard Interest on
capital u/s 24(b)
deduction borrowed
u/s 24(a) capital u/s
24(b)
loan for repair, loan for
30% renewal or acquisition or
Fully reconstruction construction of
Allowed of house house property
property

loan taken loan taken


Maximum before 1.4.99 on or after
Rs.30,000 1.4.99

Maximum
Rs.30,000
acquisition or construction
completed within 5 years
from the end of the FY in
which the capital was
borrowed
+
certificate from lender
specifying interest payable

No Yes

Maximum Maximum
Rs.30,000 Rs.2,00,000

1. Interest allowable on accrual basis


2. New loan taken for repayment of old loan then interest on new loan is allowed.

Pre-Construction period interest


Interest payable on borrowed capital for the period prior to the previous year in which the property
has been acquired or constructed (Pre-construction interest), can be claimed as deduction
over a period of 5 years in equal annual installments commencing from the year of
acquisition or completion of construction.

Note – Section 115BAC - Interest Deduction not allowed

AY 21-22

Unique Academy - 8007916622 CA Saumil Manglani - Contact: 9921051593


5. House Property 5.12

Maximum interest allowed in one year


1. Cons. & Purchase -> Current year Cons. + Pre-cons. int. = Maximum Rs.
2,00,000
2. Only repairs - Max 30,000
3. Repairs + cons - 2,00,000

In short maximum Rs. 2 lacs only will be allowed in any year.

Summary on Allowability
Let out / Deemed to be let out property
1) Standard deduction of 30% of NAV is fully allowed [Section 24(a)]
2) Interest on borrowed capital is fully allowed [Section 24(b)]

Self-occupied properties
1) Since the Annual Value is nil, there is no Standard deduction available
2) In case the capital is borrowed – Refer flow diagram above.
Illustration 09:
Smt. Shanti Devi has a house property in Kolkata. The Municipal Valuation for the same is INR
10,00,000. The Fair Rental for the property is INR 750,000. The Standard Rent per the Rent
Control Act is INR 800,000. She let out the property until 30th Nov’20 for a monthly rent of Rs.
75,000 per month. Thereafter, the tenant vacated the property and she used the house for self-
occupation. Rent for the months of Oct & Nov 20 couldn’t be realized despite all efforts, and all
the conditions for unrealized rent were satisfied. She paid Municipal Taxes @ 12% during the
year. She also paid Interest of INR 25,000 during the year for amount borrowed for repairs.
Compute the Income from House Property for AY 2021-22.
Solution:
Computation of GAV INR INR
ER
Higher of:
1) Fair Rent 7,50,000
2) Municipal Value 10,00,000
Limited to Standard Rent 8,00,000
Annual Rent 6,00,000
Less: Unrealized Rent 1,50,000
4,50,000
GAV (partly let out and partly self-occupied) 8,00,000
Less: Municipal Taxes paid by the owner during the 1,20,000
PY
NAV 6,80,000
Less: Deductions u/s 24
30% NAV 2,04,000
Interest on borrowed capital 25,000 2,29,000
Income from House Property 4,51,000

Unique Academy - 8007916622 CA Saumil Manglani - Contact: 9921051593


5. House Property 5.13

Inadmissible Deductions (Section 25)

• Interest under the Act, which is payable outside India, shall not be allowed as a
deduction, if tax has not been deducted from such Interest and there is no
person in India, who could be treated as an agent.

Unrealized/ Arrears of Rent received (Section 25A)

• Arrears of Rent and the unrealized rent received subsequently from a tenant by
an assessee, shall be deemed to be the income from House Property in the FY
in which such rental is received and shall be included in the Income from
House Property of that year; irrespective of whether he is the owner of the
property any more or not, in that FY.
• 30% of such arrears or unrealized rent received subsequently is allowed as a
deduction.

Income from Co-Owned Property (Section 26)

• The share of each co-owner should be determined in accordance with Section 22 –


25 and included in the respective individual assessments
• In a scenario, where the house property owned by co-owners is self-occupied by
them, the AV for each of them will be construed as NIL. Each Co-Owner shall be
allowed a deduction of INR 30000 / INR 200,000 as the case may be vis-à-vis
Interest on Borrowed Capital.
• In a scenario, where the house property owned by the co-owners is let out, the
income from the property will be computed as if the property is owned by one
owner, and thereafter such computed income would be apportioned amongst each
of them per their respective share.

Deemed Ownership

Unique Academy - 8007916622 CA Saumil Manglani - Contact: 9921051593


5. House Property 5.14

Unique Academy - 8007916622 CA Saumil Manglani - Contact: 9921051593


5. House Property 5.15

Illustration 10:
Two sisters, Seema and Rashmi, are co-owners of a house property, with 50% share each in
the property. The property was constructed prior to 1st April 1999. The property has 7 equal
units and is situated in Bangalore. During the FY 2020-21, each co-owner occupied one unit
each and the balance were let out @ a rental of INR 20,000 per unit per month. The Municipal
Valuation (MV) was INR 7,00,000 and the Municipal Taxes were @ 10% of the MV. Interest
payable on loan taken for construction was INR 400,000. One of the let-out units was vacant for
6 months in the year.
Compute the Income from House Property for each of the sisters assuming they are not opted
for section 115BAC.

Solution:
Computation of GAV INR INR
Estimated
Rent Higher
of:
1) Fair Rent -
2) Municipal Value 5,00,000
Limited to Standard Rent 5,00,000
Annual Rent 12,00,000
Less: Unrealized Rent 1,20,000
10,80,000
GAV (partly let out and partly self-occupied) 10,80,000
Less: Municipal Taxes paid by the owner during the 50,000
PY
NAV 10,30,000
Less: Deductions u/s 24
30% NAV 3,09,000
Interest on borrowed capital 2,85,714
5,94,714
Income from House Property 4,35,286
Share of each Co-owner 2,17,643
Loss from House Property (self-occupied portions) -30,000
Income from House Property (each co-owner) 1,87,643
Notes:
1) Observe that the computation has been done for the 5 let out and 2 self-occupied
portions separately and commensurately
2) Note that the Interest on Borrowed Capital for let out proportions is fully allowable as
deduction without any upper limit
3) Note that the AV for the Self Occupied Portion is NIL and the Interest on Borrowed
Capital is restricted to INR 30,000 for each co-owner

Unique Academy - 8007916622 CA Saumil Manglani - Contact: 9921051593


5. House Property 5.16

Illustration 11:
Mr. X is the owner of four houses. The following particulars are available:

House 1 House 2 House 3 House 4


Municipal valuation 16,000 20,000 24,000 5,600
Rent (Actual) — 14,000 20,000 6,800
Municipal taxes 400 1,000 1,200 300
Repairs and collection charges 200 2,500 1,040 460
Interest on mortgage — — — 1,000
Ground rent — 100 — 60
Fire premium 140 — 200 —
Annual charges — — 360 —

House No. 1 is self-occupied.


House No. 2 is let out for business, construction was completed on 1.3.91 and
consists of two residential units.
House No. 3 is 3/4 used for own business 1/4 let out to the manager of the business.
House No. 4 is let out for residential purposes.
His other income is Rs. 30,000. Find out the income of X from house property for the
assessment year 2021-22 assuming they are not opted for section 115BAC.

Solution:
House No. 1
Rs.
Municipal valuation 16,00
Annual value deemed to be NIL
House No. 2

Fair rental value 20,000


Less: Municipal taxes 1,000
Net annual value 19,000
Less: 30% of Net Annual Value 5,700
13,300
House No. 3
Since the house is used for own business, the income from this house is not taxable under the
head ‘Income from house property’ but will be assessed under ‘Profit and gains of business or
profession’. 1/4 of the house occupied by the Manager is presumed to be incidental to the
business and hence not assessable under the head ‘Income from house property’.
House No. 4

Unique Academy - 8007916622 CA Saumil Manglani - Contact: 9921051593


5. House Property 5.17
Rent Received 6,800
Less: Municipal taxes 300
Net annual Value 6,500
Less: 30% of Net Annual Value 1,950
4,550
Income from House Property: Rs. NIL + Rs. 13,300 + Rs. 4,550 = Rs. 17,850. It is presumed that
House No. 4 has not been mortgaged for purposes of acquiring or repairs on the house property.

Illustration 12:
For the assessment year 2021-22 Sonu submits the following information:

Income from business (speculative) – Rs. 40,000


House I House II
Property Income
Rs. Rs.
Municipal valuation 35,000 80,000
Rent received 38,000 68,000
Municipal taxes paid by tenant 3,000 4,000
Repairs paid by tenant 500 18,000
Land revenue paid 2,000 16,000
Insurance premium paid 500 2,000
Interest on borrowed capital for payment of 200 400
municipal tax of house property
Let out for Let out for
Nature of occupation residence business
Date of completion of construction 1.4.1996 1.7.1994
Determine the taxable income of Sonu for the assessment year 2021-22.
Solution:
Computation of Taxable Income of Sonu for Assessment Year 2021-22
House I
Gross Annual Value 38000
Less: Municipal Taxes - not deductible since paid by tenant NIL
Net Annual Value 38000
Less: 30% of Net Annual Value Taxable Income 11,400
26,600
House II
Gross Annual Value 80000
Less: Taxes - not deductible, paid by tenant NIL
Net Annual Value 80,000
Less: 30% of Net Annual Value Taxable Income 24,000
56,000
Total Income = Rs. 26,600 + Rs. 56,000 + Rs. 40,000 = Rs. 1,22,600.
Note: Interest on borrowed capital for payment of municipal tax is not allowed as deduction
under Section 24 of the Act.

Unique Academy - 8007916622 CA Saumil Manglani - Contact: 9921051593


5. House Property 5.18

llustration 13:
Mr. X has taken a loan of Rs. 5,00,000 on 01.10.1999 @ 10% p.a. for construction of a house which was
completed on 01.10.2019 and the house remained self-occupied throughout the previous year 2020-21. the
assessee has income under the head salary rs. 4,00,000. Mr X has paid life insurance premium of rs.
20,000. Compute tax liability for assessment year 2021-22.
Option 1 : assessee has not opted for Section 115BAC
Option 2 : assessee has opted for Section 115BAC
Solution: Option 1 Assessee has not opted for Section 115BAC
Net annual Value Nil
less: interest on capital borrowed u/s 24(b) (30,000)
loss under the head House Property (30,000)
income under the head Salary 4,00,000
Gross total income 3,70,000
less: deduction u/s 80C (20,000)
Total income 3,50,000

Computation of tax liability tax on Rs. 3,50,000 at slab rate 5,000


less: rebate u/s 87A (5,000)
Tax liability Nil
Working Note:
Current period interest From 01.04.2020 to 31.03.2021
5,00,000 x 10% = 50,000
Prior period interest From 01.10.1999 to 31.03.2019
5,00,000 x 10% x 234/12 = 9,75,000
instalment = 9,75,000 /5 = 1,95,000
total interest = rs. 50,000 + rs.1,95000 =2,45,000 Subject to maximum rs. 30,000
Solution: Option 2 Assessee has opted for Section 115BAC
Net annual Value Nil
less: interest on capital borrowed u/s 24(b) (Not Available)
income under the head House Property Nil
income under the head Salary 4,00,000
Gross total income 4,00,000
less: deduction u/s 80C (Not Available)
total income 4,00,000
Computation of Tax Liability
tax on rs. 4,00,000 at slab rate 7,500
less: rebate u/s 87A (7,500)

tax liability Nil

Unique Academy - 8007916622 CA Saumil Manglani - Contact: 9921051593


5. House Property 5.19
HOUSE PROPERTY INCOMES – EXEMPTED FROM TAX

Rent from Farm buildings around Agricultural Land - 10(1)

Income from HP of Local Authorities - 10(20)

Income from HP of Political Parties - 10(13A)

Property belonging to an approved scientific research association -


10(21)

educational organizations, medical institutions - 10(23C)

property subjected to charitable or religious purpose - 11

Certified trade union - 10(24)

annual value of one palace possessed by an ex-ruler of Indian states


where other places comes under taxation - 10(19A)

NAV of 2 SOP's - 23(2)

Property used for own's business or profession - 22

Impact of Section 115BAC under the head House Property [Amendment vide
Finance Act, 2020]
AY 21-22

Sr. Nature of Exemption/Deduction Relating to New System of Tax Existing system


No. Head House Property u/s 115BAC of Tax
1. deduction of Municipal tax from GaV allowed allowed
2. Standard deduction u/s 24(a) from NaV allowed allowed
3. interest deduction u/s 24(b) from NaV
(a) let out properties u/s 23(1) allowed allowed
(b) Self residential Property u/s 23(2) Not allowed allowed
(c) Property which is stock in trade u/s 23(5) allowed allowed

Unique Academy - 8007916622 CA Saumil Manglani - Contact: 9921051593


5. House Property 5.20
Case Law

1. Would income from letting out of properties by a company, whose main object as
per its memorandum of association is to acquire and let out properties, be taxable as
its business income, or as income from house property, considering the fact that the
entire income of the company as per its return of income was only from letting out of
properties?
Chennai Properties and Investments Ltd. v. CIT (2015) (SC)
The Supreme Court opined that the judgment in Karanpura Development Co. Ltd.’s case
squarely applied to the facts of the present case, where letting of the properties is in fact the
business of the assessee. The main objective of the company as per its memorandum of
association is to acquire and hold properties in Chennai and let out these properties.
Therefore, holding of the properties and earning income by letting out these properties is the
main objective of the company. Further, in the return of income filed by the company and
accepted by the AO, the entire income of the company comprised of income from letting out
of such properties. The Supreme Court, accordingly, held that the assessee had rightly
disclosed the income derived from letting out of such properties under the head
“Profits and gains of business or profession”.

2.Can benefit of self-occupation of house property under section 23(2) be denied to a


Firm on the ground that it, being a fictional entity, cannot occupy a house property?
And Whether HUF can claim such benefit?
CIT v. Hariprasad Bhojnagarwala (2012) (Guj.)
The Gujarat High Court observed that a firm, which is a fictional entity, cannot physically
reside in a house property and therefore a firm cannot claim the benefit of this
provision, which is available to an individual owner who can actually occupy the house.

However, the HUF is a group of individuals related to each other i.e., a family comprising
of a group of natural persons. The said family can reside in the house, which belongs to the
HUF. Since a HUF cannot consist of artificial persons, it cannot be said to be a fictional
entity.
Therefore, the Court held that the HUF is entitled to claim benefit of self-occupation of house property
under section 23(2).

3. Letting out is subservient and incidental to the main business


As per CIT v. Delhi Cloth & General Mills Co. ltd., if an assessee constructs residential quarter’s &
lets them out to his employees & letting out of residential quarter’s is only related to business, i.e. it
is not main business of assessee, then income is taxable as business income & not income from
house property.
In the same way it was held in CIT v. National News prints & Paper Mills ltd., that if the assessee
makes its accommodation available to Govt. for locating a branch of Nationalised bank, post
office, police station, central excise office etc., with the aim of carrying on its business efficiently and
smoothly, Rent collected is taxable as business income and not as house property income

Unique Academy - 8007916622 CA Saumil Manglani - Contact: 9921051593


6. PGBP 6.1

CHAPTER - 6
Profits & Gains from Business & Profession

Unique Academy - 8007916622 CA Saumil Manglani - Contact: 9921051593


6. PGBP 6.2
Particulars (Rs)
Net profit as per statement of profit and loss A
Add: Expenses debited to statement of profit and loss but not allowable B
• Depreciation as per books of accounts
• 30% of sum payable to residents on which tax is not deducted at
source or after deduction has been remitted on or before the due date
u/s 139(1), would be disallowed u/s 40(a)(ia) [The same is allowable in
the year in which the tax is deducted and remitted]
• Income-tax paid disallowed u/s40(a)(ii)
• Any expenditure incurred, in respect of which payment is made for
goods services or facilities to a related person, to the extent the same
is excessive or unreasonable, in the opinion of the A.O., having regard
to its FMV [disallowed u/s 40A (2)]
• Any expenditure incurred in respect of which payment or aggregate of
payments to a person exceeding Rs. 10,000 in a single day is made
otherwise than by way of A/c payee cheque/bank draft or use of
ECS through bank A/c [disallowed u/s 40A (3)]
• Certain sums payable by the assessee which have not been paid during
the relevant P.Y. in which the liability was incurred or on or before the
due date for filing return u/s 139(1) in respect of that P.Y. [disallowed
u/s 43B]
• Personal expenses [not allowable as per section 37]
• Capital expenditure [not allowable as per section 37]
• Repairs of capital nature [not allowable as per sections 30 & 31]
• Amortization of preliminary expenditure u/s 35D/expenditure incurred
under voluntary retirement scheme u/s 35DDA [4/5th of such
expenditure to be added back]
• Fine or penalty paid for infringement or breach of law [However, penalty
in the nature of damages for delay in completion of a contract, being
compensatory in nature, is allowable]
• All expenses related to income which is not taxable under this head e.g.
municipal taxes in respect of house property.
• Any sum paid by the assessee as an employer by way of

Unique Academy - 8007916622 CA Saumil Manglani - Contact: 9921051593


6. PGBP 6.3
contribution to pension scheme u/s 80CCD exceeding 10% of the
salary of the employee
(A-B) C
Less: Expenditure allowable as deduction but not debited to statement of D
profit and loss
• Depreciation u/s 32 [computed as per Rule 5 of Income-tax Rules,
1962]
• Additional depreciation @20% of actual cost of new P & M acquired
by an assessee engaged in the business of manufacture or production
of any article or thing generation, transmission or distribution of power
(10% of actual cost, if put to use for less than 180 days in the year of
acquisition)/ 35% of actual cost (17.5% if put to use for less than 180
days in the year of acquisition), if the manufacturing undertaking is set
up in a notified backward area in the State of A.P./Bihar/Telangana
/West Bengal on or after 1.4.2015 [Balance additional depreciation can
be claimed in the next year]
• Deduction @ 15% of actual cost of new P&M u/s 32AD in case of a
manufacturing undertaking/enterprise set up in a notified backward area
in the State of A.P./Bihar/Telangana/West Bengal on or after 1.4.2015.
• Weighted deduction for expenditure on/contribution for research u/s
35(1) (ii), 35(2AA), 35(2AB) in excess of the amount already debited to
statement of profit & loss
• Investment-linked tax deduction in respect of specified businesses
u/s 35AD

(C-D) E
Less: Income credited in the statement of profit and loss but not F
taxable/taxable under any other head
• Dividend income taxable under IOS
• Agricultural income exempt u/s 10(1)
• Interest on securities taxable under the head "Income from other
sources"
• Profit on sale of capital asset taxable under the head "Capital Gains"
• Rent from house property taxable under the "Income from house
property"

Unique Academy - 8007916622 CA Saumil Manglani - Contact: 9921051593


6. PGBP 6.4
• Interest on savings bank account/FD taxable under the head "Income
from other sources"
• Winnings from lotteries, house races, etc. taxable under the head
"Income from other sources"
• Gifts exempt or taxable under the head "Income from other sources"
• Income-tax refund not taxable
• Interest on income-tax refund taxable under the head "Income from
other sources"

(E-F) G
Add: Deemed Income H
• Bad debt allowed as deduction u/s 36(1)(vii) in an earlier PY, now
recovered [deemed as income u/s41(4)]
• Remission or cessation of a trading liability [deemed as income u/s
41(1)]

(G+H) I

2. BUSINESS’ OR ‘PROFESSION

• ‘Business - Section 2(13) - Business includes any trade, commerce or


manufacture or any adventure or concern in the nature of trade, commerce or
manufacture.

• The concept of business presupposes the carrying on of any activity for


profit.

• Length of time of running business does not matter.

• The expression ‘Profession’ has been defined in Section 2(36) of the Act to
include any vocation.
For instance, an auditor carrying on his practice, the lawyer or a doctor, a painter,
an actor, an architect or sculptor, would be persons carrying on a profession and
not a business.
• The common feature in the case of both profession as well as business is that
the object of carrying them out is to derive income or to make profit.

Unique Academy - 8007916622 CA Saumil Manglani - Contact: 9921051593


6. PGBP 6.5

Regulatory Framework

Sections (Income Tax Act, Details


1961)
Section 28 income chargeable to tax under the head Business or Profession
Section 43(5) Speculation transaction
Section 145 Method of accounting
Section 145A Method of accounting in certain cases
Section 145B taxability of Certain income
Section 30-37 admissible deduction
Section 2(11) Block of assets
Section 43(1) actual Cost
Section 43(6) Written down Value (WDV)
Section 40 expenses disallowed
Section 43B disallowance of unpaid Statutory liability
Section 43A Changes in rate of exchange
Section 41 Deemed Profits
Section 43CA transfer of immovable Property
Section 43D Special provision in case of income of Public Financial institutions, etc.
Section 44AE Special provisions for computing profits and gains of business of plying,
hiring or leasing goods carriages
Section 44 Special provisions related to insurance Business
Section 44AA Compulsory maintenance of Books of account
Section 44AB Compulsory audit of Books of account
Section 44AD Special provision for computing profits and gains of business on
Presumptive Basis
Section 44ADA Presumptive taxation for Professionals

INCOME CHARGEABLE TO TAX UNDER THE HEAD BUSINESS OR


PROFESSION (SECTION 28)

The various items of income chargeable to tax as income under the head ‘profits
and gains of business or profession’ are as under:
(i) Income from business or profession
(ii) Any compensation or other payment due to or received by:
(a) Any person, by whatever name called, managing the whole or
substantially the whole of -
(i) the affairs of an Indian company or
(ii) the affairs in India of any other company
at or in connection with the termination of his management or office or the
modification of any of the terms and conditions relating thereto;

Unique Academy - 8007916622 CA Saumil Manglani - Contact: 9921051593


6. PGBP 6.6

(b) any person, by whatever name called, holding an agency in India for
any part of the activities relating to the business of any other person, at or in
connection with the termination of the agency or the modification of any
of the terms and conditions relating thereto;
(c) any person, by whatever name called, at or in connection with the
termination or modification of the terms and conditions, of any contract
relating to his business, whether revenue or capital. (AY 19 – 20)
(iii) Income from specific services performed for its members by a trade,
professional or business: Income derived by any trade, professional or similar
associations from specific services rendered by them to their members.
It may be noted that this forms an exception to the general principle of mutuality that
no one can make profit out of himself. It governs the assessment of income of
mutual associations such as chambers of commerce, stock brokers’ associations
etc.
Therefore any surplus arising to the mutual associations such as Labour Welfare
Association, Chamber of Commerce etc. by performing specific services to its
members is deemed as income earned as carrying on business in respect of these
services and accordingly chargeable to tax.
Trade association means an association of businessmen for the protection
and advancement of their common interest e.g. a Chamber of Commerce.
Section 28(iii) does not apply to other social associations e.g. a sports club
or cricket club etc.
(iv) Incentives received or receivable by assessee carrying on export
business:
(a) Profit on sale of import license
(b) Cash assistance against exports under any scheme of GOI
(c) Customs duty or excise re-paid or repayable as drawback:
(d) Profit on transfer of Duty Entitlement Pass Book Scheme or Duty-Free
Replenishment Certificate
(v) Value of any benefit or perquisite: The value of any benefit or perquisite
whether convertible into money or not, arising from business or the exercise of any
profession.
(vi) Sum due to, or received by, a partner of a firm: Any interest, salary,
bonus, commission or remuneration, by whatever name called, due to or received
by a partner of a firm from such firm will be deemed to be income from business.
(vii) Any sum whether received or receivable, in cash or kind, under an
agreement:
(a) for not carrying out any activity in relation to any business or
profession; or
(b) for not sharing any know-how, patent, copyright, trade mark, license,
franchise or any other business or commercial right of similar nature or
information or technique likely to assist in the manufacture or processing of
goods or provision for services.
Unique Academy - 8007916622 CA Saumil Manglani - Contact: 9921051593
6. PGBP 6.7
(viii) Any sum received under a Keyman insurance policy: Any sum
received under a Keyman insurance policy including the sum allocated by way of
bonus on such policy will be taxable as income from business.
(ix) Fair market value of inventory on its conversion as capital asset
(x) Sum received on account of capital asset referred under section
35AD

3. SPECULATION BUSINESS
✓ Section 43(5) → “speculative transaction” as “a transaction in which a contract for the
purchase or sale of any commodity including stocks and shares is periodically or
ultimately settled otherwise than by the actual delivery or transfer of the commodity or
scrips”.
✓ A company carrying on such business will be considered to engage in a speculative business
which will be treated as a separate business.
✓ However, the following will not be considered as engaging in speculative business –
a. Where a company whose gross total income consists mainly of income which is
chargeable under the heads “Interest on securities”, “Income from house property”,
“Capital gains” and “Income from other sources”, or
b. A company, the principal business of which is:

the business of trading in shares or


banking business or
the granting of loans and advances
Accordingly, if these companies carry on the business of purchase and sale of shares of
other companies, the income from such business is not treated as income from
speculative business.

4. Key points for consideration

Method of Accounting [Section 145]

Income earned in cash or in kind

Continuation of Busines/Profession

Key Points
for Ownership of Business is not necessary for Taxability
Consideration

Business may be Legal or Illegal

Profit motive is not the sole consideration


for taxability

Computation of income separately for each business

Unique Academy - 8007916622 CA Saumil Manglani - Contact: 9921051593


6. PGBP 6.8

Method of Accounting (Section 145)


An assessee may adopt either the cash system of accounting or the mercantile
system of accounting.
An assessee may also follow the hybrid system of accounting for his business or
profession. The hybrid system is the combination of the cash system and the
mercantile system. This is mostly done in the case of professional men who follow
cash system for their receipts and mercantile system for their payments.
A. Method of accounting – Section 145
✓ Income chargeable under the head PGBP or IOS

✓ Computed in accordance with either cash or mercantile system of accounting

✓ If AO is not satisfied about the

➢ correctness or c

➢ completeness or

➢ method of accounting has not been regularly followed by the assessee, or

➢ ICDS Not followed,

✓ The AO may make a Best Judgement assessment as provided in section 144.


B. The CG has notified 10 ICDS for the purpose of computation of income under the head PGBP or IOS
and not for maintaining books of accounts.
Some key features of ICDS are as under :
(i) ICDS applies to all tax payers except Individual and HUF who are not covered under the tax audit
under section 44AB.

(ii) ICDS applies only to tax payers following mercantile system of accounting.
(iii) ICDS shall apply irrespective of the accounting standards adopted by companies i.e., either
Accounting Standards or Ind-AS.

(iv) ICDS shall also apply to the persons computing income under the relevant presumptive
taxation scheme.
For example, for computing presumptive income of a partnership firm under section 44AD of the Act, the
provisions of ICDS on Construction Contract or Revenue recognition shall apply for determining the
receipts or turnover, as the case may be.
(v) The provisions of ICDS shall not apply for computation of MAT.
(vi) However it shall apply for computation of AMT as AMT is computed on adjusted total income
which is derived by making specified adjustments to total income computed as per the regular provisions
of the Act.

(vii) I.T ACT/ IT Rules Vs ICDS → Act/Rules shall prevail

(viii) judicial pronouncements/ judgements Vs ICDS – ICDS Shall prevail

C. Method of accounting in certain cases [Section 145A] - For PGBP


(i) Inventory – Value at Actual Cost or NRV whichever is lower.
(ii) Valuation of purchase and sale of goods or services and of inventory to

Unique Academy - 8007916622 CA Saumil Manglani - Contact: 9921051593


6. PGBP 6.9
include the amount of any tax, duty, cess or fee (by whatever name called) actually paid or incurred
by the assessee to bring the goods or services to the place of its location and condition as on the
date of valuation.
(iii) the inventory being securities not listed on a recognised stock exchange, or listed but not quoted on
a recognised stock exchange, shall be valued at actual cost as initially recognized in accordance with
the ICDS.
(iv) the inventory being securities listed and quoted on a recognised stock exchange, shall be valued at
lower of actual cost or net realisable value in accordance with the ICDS.
Provided that the inventory being securities held by a scheduled bank or public financial institution shall
be valued in accordance with the ICDS after taking into account the extant guidelines issued by the RBI
Guidelines in this regard.

D. Taxability of certain income [Section 145B]

• Interest on compensation or Enhanced compensation – Taxable on Receipt basis

• Subsidy or grant from Government - Taxable on Receipt basis

• Claim for escalation of price in a contract or export incentives → taxable when reasonable certainty
of its realization is achieved.

5. COMPUTATION OF PROFITS OF BUSINESS OR PROFESSION (Section 29)


According to section 29, the profits and gains of any business or profession are
to be computed in accordance with the provisions contained in sections 30 to
43D.
Specific expenses given in section 30 to 36 & General expenses are allowed in
section 37.

General Commercial Principles are as under:


Computation of profits and
gains from business or
Profits should be computedprofession
according(section
to the29)method of accounting regularly
employed by the assessee, provided that actual profit can be ascertained
by this method, whether on receipt basis or accrual basis.
Only those expenses and losses Expenses or
are allowed as deductions which were
Admissible
incurred or sustained payments not Profits
Inadmissibleduring the relevant previous year and related to
business.
deductions deductible in chargeable to Other
deductions certain
[ 30 These
to 37] losses(section
and expenses tax (section provisions
should be incidental to the operation of the
40) circumstances 41) during the course
business. For example, embezzlement by an employee
(section 40A)
of business is a loss incidental to the business.
If a business has been discontinued before the commencement of the
previous year, it’s expenses cannot be allowed as deduction against the
income of any other running business of the assessee.
There are some essential expenses, though neither expressly allowed nor
disallowed, but are deductible while computing the profits of business or
profession on the basis of general commercial principles provided that
these are not expenses or losses of a capital nature or personal nature.
Any expenditure incurred in consideration of commercial expediency is
allowed as deduction.
Deduction can be made from the income of that business only for which the
expenses were incurred. The expenses of one business cannot be charged
against the income of any other business.

Unique Academy - 8007916622 CA Saumil Manglani - Contact: 9921051593


6. PGBP 6.10

6. COMPUTATION OF INCOME UNDER THE HEAD “PROFITS AND GAINS


FROM BUSINESS OR PROFESSION”

Net Profit as per profit and loss account xxx


Add: Inadmissible expenses debited to profit and loss account xxx
Add: Deemed incomes not credited to profit and loss account xxx
Less: Deductible expenses not debited to profit and loss account (xxx)
Less: Incomes chargeable under other heads credited to Profit & Loss A/c (xxx)

7. Rent, rates, repairs and insurance for buildings (Section 30)


This section allows the deduction in respect of rent, rates, repairs and insurance
for buildings that are used by the assessee for his business / profession.
• If the property is partly used, the deduction will be proportionate to the use.
• In case the property is sub-let, the differential, i.e., rent paid minus rent
recovered would be taxable.
• No notional rent is allowable for owned properties.
• Repairs undertaken, whether as an owner / tenant, are allowed.
• Municipal taxes, rates, insurance incurred by the assessee for the property is
also allowed.

8. Repairs and insurance for Plant & Machinery, Furniture (Section 31)
This section allows deduction in respect of expenses on current repairs and
insurance of Plant & Machinery, & furniture used for business / profession.
• Allowable in full, even if used for part of the year
• Current repairs which are of capital nature aren’t allowed
• Insurance premia paid to insure the assets against risks of losses owing to
damage / destruction,
provided that the assets are used for business / profession
are allowed, only if these premiums are paid/payable during the Previous Year.
9. Deduction of expenses on the basis of usage
Section 38 provides allowing expenses on proportionate basis depending upon
how much percentage has been spent for business & how much for non-business
purpose.

10. Depreciation (Section 32)

This section provides for compulsory deduction on account of depreciation, that


is, diminution in the value of assets.
The provisions for allowing depreciation are contained in Section 32 and are
regulated under Rule 5 of the Income- tax Rules. The rates of depreciation are
Unique Academy - 8007916622 CA Saumil Manglani - Contact: 9921051593
6. PGBP 6.11
also provided in the Income-tax Rules.
Conditions for claiming depreciation
• Asset must be used for the business
• Asset should be owned by the Assessee. Registered ownership not required.
• Depreciation is allowed on Block of assets
• Asset must be Put to Use.
It’s not just use that’s important, it’s owned and used that’s mandatory, i.e., the assets should be
owned and used by the assessee in the Previous Year. In this context, even if owned by assessee
and then leased out and used by the lessee, even then depreciation can be claimed by the
assessee.
Meaning of Block of Assets
The depreciation is provided in respect of “Block of assets”. As per Section 2(11) Block of assets
means “a group of assets falling within a class of assets, being tangible assets such as buildings,
machinery, plant or furniture and intangible assets, being know-how, patents, copyrights,
trademarks, licenses, Franchises or any other business or commercial rights of similar nature, in
respect of which the same percentage of depreciation is prescribed”.
There are four classes of the assets which are further categorized into ten Blocks of assets
according to different rates of assets prescribed as under:
S. No. Class of asset Block of asset
1. Building 3 blocks (5%, 10% and 40%)
2. Furniture & Fixture 1 block (10%)
3. Plant and machinery 5 blocks (15%, 20%, 30%, 40%, 45%)
4. intangible assets 1 block (25%)
Written Down Value of Assets (W.D.V.) [Section 43(6)]

(1) W.D.V. of the block of assets on 1st April of the previous year xxx
(2) Add: Actual cost of assets acquired during the previous year xxx
(3) Total (1) + (2) xxx
(4) Less: Money receivable in respect of any asset falling within the block which
is sold, discarded, demolished or destroyed during that previous year xxx
(5) W.D.V at the end of the year (on which depreciation is allowable) [(3) – (4)]
(6) Depreciation at the prescribed rate
(Rate of Depreciation × WDV arrived at in (5) above)

NOTE –V.V. Imp. - Asset is acquired in the previous year & Put to use for less than 180 days
(up to 179 days) during the same PY – only half depreciation is allowed.

Check Number of days of put to use only if acquisition and put to use are of same PY. If the
year of acquisition and year of put to use are different then full depreciation will be allowed.
Conditions in Detail:
In order that the depreciation is allowable, the following conditions must be
fulfilled:
(a) Classification of Assets – Few Pointers
✓ Roads within a factory compound form part of building and are
entitled to depreciation.
✓ Similarly, residential quarters provided to the employees are
incidental to the carrying on the business. Therefore, the roads to

Unique Academy - 8007916622 CA Saumil Manglani - Contact: 9921051593


6. PGBP 6.12
such residential quarters are also entitled to depreciation.
However, the M.P. High Court has held that expenditure incurred on
construction of metal roads for approach to trenches to dump the waste and
night soil, is capital expenditure. Moreover, such roads are not plant and
machinery. Hence, the assessee is not entitled to depreciation on the cost of
the metal roads [Indore Municipal Corporation v. C.I.T. (1981) 132 I.T.R. p. 540
(MP)].
(b) Ownership v/s lease:
Note : As per the CBDT circular, irrespective of the accounting treatment, the lessor
shall be entitled to claim depreciation on leased assets, whether the lease is an
operating lease or a financial lease.
Note: As per the landmark judgment of CIT V/s Annamalai Finance Ltd.
(Madras HC) it is the end use of the specified asset which is relevant for
determining the percentage of depreciation. For example, in case of business of
leasing out vehicle, if lessee is using the vehicle for running them on hire,
depreciation shall be allowable at Higher rate of 30% instead of 15% to the
lessor.

In the case of assets acquired on hire-purchase


a. Plant and machinery taken on hire, the assessee would not be the owner
thereof and consequently would not be entitled for depreciation in respect of
the same.
b. But if the plant and machinery had been acquired on instalment basis, the
assessee becomes the owner of the assets the moment the purchase or sale
is concluded and consequently is entitled to depreciation although a part or
whole of the price is payable in future.
(c) Used for the purpose of Business or Profession:.
(d) No deduction on sold assets: No depreciation is allowable in respect
of the depreciable asset if the asset concerned is sold, destroyed, discarded or
demolished in the same year in which it was acquired.
(e) Cash Payments - Where an assessee incurs any expenditure for
acquisition of depreciable asset in respect of which a payment (or aggregate
of payments made to a person in a day), otherwise than by an account
payee cheque/draft or use of ECS through a bank account, exceeds Rs.
10,000 such payment shall not be eligible for normal/ additional
depreciation.
Also, such Payment will be ignored for the purpose of computation of Actual
Cost of such asset under section 43(1). [Amendment vide Finance Act, 2017]
w.e.f. AY 2018-19]
(f) Depreciation in case of amalgamation/Demerger etc. - Depreciation is
allowable in the hands of the predecessor and successor, in case of succession
of a firm, proprietary concern, by a company, or predecessor company and
successor LLP, in case of conversion of Pvt. Ltd. Co. to LLP., in case of
amalgamation or demerger, and that shall not exceed the depreciation that
would have been allowed if the succession, demerger, amalgamation had not
taken place and such depreciation shall be apportioned between them
proportionate to use (on the basis of number of days).

Unique Academy - 8007916622 CA Saumil Manglani - Contact: 9921051593


6. PGBP 6.13
Depreciation Rates
Note –
• No depreciation allowed on land
• Depreciation not allowed on live stocks or tea bushes.

Rate of
Nature of Assets Depreciation
(WDV)
Buildings
Residential 5%
General 10%
Temporary Structure 40%
Furniture & Fittings 10%
Plant & Machinery
General 15%
Motors cars other those used in a business of running them on hire 15%
Motors cars acquired on or after 23rd August 19 but upto 31st March 20 30%
and is put to use upto 31st March 2020.
Motor buses, lorries, vans and taxis used in a business of running them on 30%
hire acquired on or after 23rd August 19 but upto 31st March 20 and is 45%
put to use upto 31st March 2020

Books owned by assesses carrying on a profession


40%
Books owned by assesses carrying on a business
15%
Books owned by assesses carrying business of running libraries
40%
ships
20%
Airplanes
40%
Air Pollution control Equipments, Water Pollution Control Equipments
40%
Computers including Computer Software (Operating System only)
40%
Oil wells (P&M)
15%
Windmills installed up to 31.3.14 (पुराने) (P&M) 15%
Windmills installed up to 1.4.14 onwards (नए) (P&M) 40%
Intangible Assets
Software, knowhow, patents, copy-rights, trade marks, licences, franchises 25%
or any other business or commercial rights of similar nature
Illustration (For Amalgamation, Demerger etc.)
X Ltd. has a block of assets (P&M), carrying 15% depreciation, WDV on 1st Apr 2020 is
INR 75,00,000. It purchased subsequently on 1st Dec 2020, another machinery for INR
25,00,000 and put to use on the same day. X Ltd. was amalgamated with Y Ltd. effective 1st
Feb 2021.
Compute the depreciation allowable for X Ltd. and Y Ltd. for the AY 2021-22.
Solution
Block Days Allowable Depreciation
75,00,000 365 Full 11,25,000

Unique Academy - 8007916622 CA Saumil Manglani - Contact: 9921051593


6. PGBP 6.14
25,00,000 121 Half 1,87,500
Total Depreciation 13,12,500
Predecessor 306 On the Opening P&M 9,43,151
Successor 59 1,81,849
Predecessor 62 On the Acquired P&M 96,074
Successor 59 91,426

Notes:
1. The depreciation for the asset in existence and used for the whole year is proportionately
divided basis the number of days used by the predecessor until amalgamation date and the
successor post amalgamation date.
2. The depreciation for the asset acquired during the year and used for the remaining part of the
year is proportionately divided basis the number of days used by the predecessor until
amalgamation date and the successor post amalgamation date, beginning the date of acquisition.
Actual Cost [Section 43(1)]
✓ The actual cost of an asset to the assessee is normally the amount incurred by
him to make the asset ready for the purpose of its use in the business.
✓ It is important to note that where, an assessee, in acquisition of an asset, makes
payment(s) in a day otherwise than by an a/c payee cheque / demand draft /
ECS, which is > INR 10000, such expenditure would not be a part of the actual
cost.
The provisions of Section 43(1) of the Act clarify that the actual cost of
depreciable asset should be determined in the following circumstances as
indicated below:
a. If an asset was first used for scientific research and then used
for business later, the actual cost would be Nil.
b. If the asset is acquired as a gift / inheritance, the actual cost is
the WDV of the previous owner.
c. If an asset was once in use, then transferred and later re-
acquired, the actual cost would be the WDV at the time of transfer
OR the price of re-acquisition, whichever is lower.
d. Asset acquired at a higher price with a view to claim higher
depreciation.
For instance, if ‘X’ transfers his machinery on 1.1.1990 to ‘Y’ for a sum of Rs.
6.00 lakhs while the actual cost of the asset and the written down value thereof
on that day to ‘X’ are Rs. 3.00 lakhs and Rs. 1.00 lakh respectively, it may be
inferred that the transfer by ‘X’ to ‘Y’ is made with idea to enable ‘Y’ to claim
depreciation on Rs. 6.00 Lakhs while the market value of the asset on the date
of sale by ‘X’ to ‘Y’ may be Rs. 4.00 lakhs only. In such a case, the Assessing
Officer would be entitled to allow depreciation to ‘Y’ on the basis of the cost
which may be determined by him to be Rs. 4.00 lakhs instead of Rs. 6.00 lakhs
as claimed by ‘Y’.
e. Any amount of interest paid / payable as interest till date of put to use is
capitalized and after that date is put into P&L.
Note: interest paid before the commencement of the production on amounts
borrowed by the assessee for acquisition and installation of the plant and
machinery shall form part of the actual cost u/s 43(1), as decided by the Supreme
Unique Academy - 8007916622 CA Saumil Manglani - Contact: 9921051593
6. PGBP 6.15
Court in Challapalli Sugars Limited Vs CIT.
f. Where a portion of the cost of an asset acquired by the assessee has
been met directly or indirectly by the Central Government or a State
Government or any authority established under any law or by any
other person, in the form of a subsidy or grant or reimbursement (by
whatever name called), then, so much of the cost as is relatable to such
subsidy or grant or reimbursement shall not be included in the actual
cost of the asset to the assessee.
g. Capital Asset on which deduction has been allowed or allowable u/s
35AD shall be treated as nil.
h. Building used for private purpose by the assessee and subsequently brought
into business. – Cost will be the actual cost as reduced by the notional
depreciation.
i. Inventory converted into capital asset and used for business or
profession: The fair market value of such inventory as on the date of
its conversion into capital asset shall be the actual cost of such capital
asset to the assessee [Explanation 1A]) (AY 19-20)

Unabsorbed Depreciation
• It’s the depreciation that couldn’t be consumed fully, that is, the profits were not
sufficient to absorb it.
• First adjust depreciation under the head PGBP
• Next - Set off against other heads except Salary
• Next – Can be carried forward for indefinite number of years
• In the subsequent years – First set off under the head PGBP then against other
heads except Salary.
• Set off will be allowed even if the same business to which it relates is no
longer in existence in the year in which the set off takes place.
• Unabsorbed depreciation can be carried forward by the same Assessee.

Depreciation on Straight line basis


In the case of Power Units [Section 2(1)(i)] (optional to power generating
units)
• An undertaking engaged in generation or generation and distribution of
power can claim depreciation on straight line basis on the actual cost of
individual asset.

• Alternatively, such undertaking can claim depreciation, at its option,


according to written down value method like any other assessee.
Unique Academy - 8007916622 CA Saumil Manglani - Contact: 9921051593
6. PGBP 6.16
• The option for this purpose shall be exercised before the due date of
furnishing return of income.

• Once this option is exercised, it shall be final and shall apply to all the
subsequent years.
Terminal depreciation
If any asset, on which depreciation is claimed on basis of SLM, is sold and the
amount by which money payable together with scrap value, fall short of WDV of
such asset, depreciation shall be allowed equal to such deficiency in the year of
sale.
Balancing Charge [Section 41(2)]
If any asset, on which depreciation is claimed on basis of SLM, asset is sold and
the amount by which moneys payable together with scrap value, exceeds WDV
of such asset, then the least of the following shall be taxable under the head
PGBP.
(i) difference between the actual cost and WDV
(ii) difference between aggregate of moneys payable and WDV
If sale value is more than the original cost of the asset then the amount which is
over and above the original cost is taxable under the head Capital Gains.

10. Additional Depreciation [Section 32(1)]


✓ The additional depreciation is available to assessee engaged in the business of
manufacture or production of any article or thing or engaged in the business of
generation or generation and distribution or transmission of power at the rate of
20% of actual cost of eligible new machinery or plant (other than ships and
aircrafts acquired and installed in a previous year).

✓ However, in case of Investment is made in New Plant and Machinery in Notified Backward areas
in state of Bihar, West Bengal, Andhra Pradesh or Telangana on or after 1/4/15 but upto
31.3.2020 , Additional depreciation shall be allowed @ 35% of Actual cost.

Additional Depreciation shall not be allowed if -

(a) any machinery or plant which, before its installation by the assessee, was
used either within or outside India by any other person; or

(b) any machinery or plant installed in any office premises or any residential
accommodation, including accommodation in the nature of a guest house; or

(c) any office appliances or road transport vehicles; or

(d) any machinery or plant, the whole of the actual cost of which is allowed as a
deduction (whether by way of depreciation or otherwise) in computing the
income chargeable under the head “Profits and gains of business or profession” of
any one previous year.
Note: The funda of 180 days is also checked here.

Unique Academy - 8007916622 CA Saumil Manglani - Contact: 9921051593


6. PGBP 6.17

Note – Additional Depreciation is not allowed under section 115BAC


Illustration 1 AY 21-22

Mr. X, a proprietor engaged in manufacturing business, furnishes the following particulars:

Particulars Rs.
(1) Opening WDV of plant and machinery as on 1.4.2020 30,00,000
(2) New plant and machinery purchased and put to use on 20,00,000
08.06.2020
(3) New plant and machinery acquired and put to use 8,00,000
on15.12.2020
(4) Computer acquired and installed in the office premises on 3,00,000
2.1.2021

Compute the amount of depreciation and additional depreciation as per the Income- tax Act,
1961 for the A.Y. 2021 - 22. Assume that all the assets were purchased by way of account payee
cheque and the assessee has not opted for section 115BAC.
Solution
Computation of depreciation and additional depreciation for A.Y. 2021-22

Plant & Machinery Computer


Particulars (15%) (40%)
Normal depreciation

@15% on Rs. 50,00,000 [See Working Notes 1& 2] 7,50,000 -


@7.5% (50% of 15%, since put to use for less than
60,000 -
180 days) on Rs. 8,00,000
@20% (50% of 40%, since put to use for less than
- 60,000
180 days) on Rs. 3,00,000
Additional Depreciation
@20% on Rs. 20,00,000 (new plant and machinery 4,00,000 -
put to use for more than 180 days)
@10% (50% of 20%, since put to use for less than 180
days) on Rs. 8,00,000 80,000 -
Total depreciation 12,90,000 60,000
Working Notes:

(1) Computation of written down value of Plant & Machinery as on 31.03.2021


Particulars Plant & Computer
Machinery (Rs.)
(Rs.)
Written down value as on 1.4.2020 30,00,000 -
Add: Plant & Machinery purchased on 08.6.2020 20,00,000 -
Add: Plant & Machinery acquired on 15.12.2020 8,00,000 -
Computer acquired and installed in the office - 3,00,000
premises
Written down value as on 31.03.2021 58,00,000 3,00,000

Unique Academy - 8007916622 CA Saumil Manglani - Contact: 9921051593


6. PGBP 6.18

Composition of plant and machinery included in the WDV as on 31.3.2021.


Plant & Computer
Particulars Machinery (Rs. ) (Rs.)

Plant and machinery put to use for 180 days or more 50,00,000
[Rs. 30,00,000 (Opening WDV) + Rs. 20,00,000
(purchased
Illustration 2
Mr. Gopi carrying on business as proprietor converted the same into a limited company by name
Gopi Pipes (P) Ltd. from 01-07-2020. The details of the assets are given below:

Rs.
Block - I WDV of plant & machinery (rate of depreciation @ 15%) on 01.04.2020 12,00,000

Block - II WDV of building (rate of depreciation @ 10%) on 01.04.2020 25,00,000

The company Gopi Pipes (P) Ltd. acquired plant and machinery in December 2020 for Rs.
10,00,000. It has been doing the business from 01-07-2020.
Compute the quantum of depreciation to be claimed by Mr. Gopi and successor Gopi Pipes (P)
Ltd. for the assessment year 2021-22. Assume that plant and machinery were purchased by way
of account payee cheque.
Note: Ignore additional depreciation.

Solution
Computation of depreciation allowable to Mr. Gopi for A.Y. 2021-22

Particulars Rs. Rs.


Block 1 Plant and Machinery (15% rate)
WDV as on 1.4.2020 12,00,000
Depreciation@15% 1,80,000
Block 2 Building (10% rate)
WDV as on 1.4.2020 25,00,000

Depreciation@10% 2,50,000
Total depreciation for the year 4,30,000

Proportionate depreciation allowable to Mr. Gopi


1,07,205
for 91 days (i.e., from 1.4.2020 to 30.6.2020) [i.e.,
91/365 x Rs. 4,30,000)
Computation of depreciation allowable to Gopi Pipes (P) Ltd. for A.Y.2021-22

Particulars Rs.
(i) Depreciation on building and plant and machinery Proportionately for 3,22,795
274 days (i.e. from 1.7.2020 to 31.3.2021) (274/365 x Rs. 4,30,000)
(ii) Depreciation@ 50% of 15% on Rs. 10 lakhs, being the value of 75,000
plant and machinery purchased after conversion, which was put to use
for less than 180 days during the P.Y. 2020-21
Depreciation allowable to Gopi Pipes (P) Ltd. 3,97,795
Unique Academy - 8007916622 CA Saumil Manglani - Contact: 9921051593
6. PGBP 6.19
Note: In the case of conversion of sole proprietary concern into a company, the depreciation should
be first calculated for the whole year as if no succession had taken place. Thereafter, the depreciation
should be apportioned between the sole proprietary concern and the company in the ratio of the
number of days for which the assets were used by them. It is assumed that in this case, the conditions
specified in section 47(xiv) are satisfied.
Illustration 3
Sai Ltd. has a block of assets carrying 15% rate of depreciation, whose written down value on
01.04.2020 was Rs. 40 lacs. It purchased another asset (second-hand plant and machinery) of the
same block on 01.11.2020 for Rs. 14.40 lacs and put to use on the same day. Sai Ltd. was
amalgamated with Shirdi Ltd. with effect from 01.01.2021.

You are required to compute the depreciation allowable to Sai Ltd. & Shirdi Ltd. for the previous year
ended on 31.03.2021 assuming that the assets were transferred to Shirdi Ltd. at Rs. 60 lacs. Also
assume that the plant and machinery were purchased by way of account payee cheque.
Solution –
Statement showing computation of depreciation allowable to Sai Ltd. & Shirdi Ltd. for A.Y.
2021-22
Particulars Rs.
Written down value (WDV) as on 1.4.2020 40,00,000
Addition during the year (used for less than 180 days) 14,40,000
Total 54,40,000
Depreciation on Rs. 40,00,000 @ 15% 6,00,000
Depreciation on Rs. 14,40,000 @ 7.5% 1,08,000
Total depreciation for the year 7,08,000
Apportionment between two companies:
(a) Amalgamating company, Sai Ltd.
Rs. 6,00,000 × 275/365 4,52,055
Rs. 1,08,000 × 61/151 43,629
4,95,684
(b) Amalgamated company, Shirdi Ltd.
Rs. 6,00,000 × 90/365 1,47,945
Rs. 1,08,000 × 90/151 64,371
2,12,316
Notes:

(i) The aggregate deduction, in respect of depreciation allowable to the amalgamating company
and amalgamated company in the case of amalgamation shall not exceed in any case, the
deduction calculated at the prescribed rates as if the amalgamation had not taken place. Such
deduction shall be apportioned between the amalgamating company and the amalgamated
company in the ratio of the number of days for which the assets were used by them.

(ii) The price at which the assets were transferred, i.e., Rs. 60 lacs, has no implication in
computing eligible depreciation.
Illustration 4
A newly qualified Chartered Accountant Mr. Dhaval, commenced practice and has acquired the
following assets in his office during F.Y. 2020-21 at the cost shown against each item. Calculate
the amount of depreciation that can be claimed from his professional income for A.Y.2021-22.
Assume that all the assets were purchased by way of account payee cheque.

Unique Academy - 8007916622 CA Saumil Manglani - Contact: 9921051593


6. PGBP 6.20
Sl. Description Date of Date when Amount
No. acquisition put to use Rs.
1. Computer including computer software 27 Sept., 20 1 Oct., 20 35,000

2. Computer UPS 2 Oct., 20 8 Oct., 20 8,500


3. Computer printer 1 Oct., 20 1 Oct., 20 12,500
4. Books (other than annual publications 1 Apr., 20 1 Apr., 20 13,000
are of Rs. 12,000)
5. Office furniture 1 Apr., 20 1 Apr., 20 3,00,000
(Acquired from a practicing C.A.)
6. Laptop 26 Sep., 20 8 Oct., 20 43,000

Solution –

Computation of depreciation allowable for A.Y.2021-22


Asset Rate Depreciation
(Rs.)
Block 1 Furniture [See working note below] 10% 30,000
Block 2 Plant (Computer including computer software,
Computer UPS, Laptop, Printers and Books)
[See working note below] 40% 34,500
Total depreciation allowable 64,500

Working Note:

Computation of depreciation

Block of Assets Rs.


Block 1: Furniture – [Rate of depreciation - 10%]
Put to use for more than 180 days [Rs. 3,00,000@10%] 30,000
Block 2: Plant [Rate of depreciation- 40%]
(a) Computer including computer software (put to use for more than 180 days) [Rs. 14,000
35,000 @ 40%]
(b) Computer UPS (put to use for less than 180 days) [Rs. 8,500@ 20%] 1,700
[See note below]
(c) Computer Printer (put to use for more than 180 days) [Rs. 12,500 @ 40%] 5,000
(d) Laptop (put to use for less than 180 days) [Rs. 43,000 @ 20%] [See note below] 8,600

(e) Books (being annual publications or other than annual publications) (Put to use for
more than 180 days) [Rs.13,000 @ 40%] 5,200
34,500

Note - Where an asset is acquired by the assessee during the previous year and is put to use for
the purposes of business or profession for a period of less than 180 days, the deduction on
account of depreciation would be restricted to 50% of the prescribed rate. In this case, since Mr.
Dhaval commenced his practice in the P.Y. 2020-21 and acquired the assets during the same
Unique Academy - 8007916622 CA Saumil Manglani - Contact: 9921051593
6. PGBP 6.21
year, the restriction of depreciation to 50% of the prescribed rate would apply to those assets
which have been put to use for less than 180 days in that year, namely, laptop and computer
UPS.
Illustration 5
Mr. Gamma, a proprietor started a business of manufacture of tyres and tubes for motor vehicles
on 1.1.2020. The manufacturing unit was set up on 1.5.2020. He commenced his manufacturing
operations on 1.6.2020. The total cost of the plant and machinery installed in the unit is Rs. 120
crores. The said plant and machinery included second hand plant and machinery bought for Rs.
20 crore and new plant and machinery for scientific research relating to the business of the
assessee acquired at a cost of Rs. 15 crores.

Compute the amount of depreciation allowable under section 32 of the Income-tax Act, 1961 in
respect of the assessment year 2021-22. Assume that all the assets were purchased by way of
account payee cheque.

Solution
Computation of depreciation allowable for the A.Y. 2021-22 in the hands of Mr. Gamma
Particulars Rs. in crore
Total cost of plant and machinery 120.00
Less: Used for Scientific Research (Note 1) 15.00
105.00
Normal Depreciation at 15% on Rs. 105 crore 15.75
Additional Depreciation:
Cost of plant and machinery 120
20
Less: Second hand plant and machinery (Note 2)

Plant and machinery used for scientific research, the whole of the actual 15 (35)
cost of which is allowable as deduction under section 35(1)(iv) read with
section 35(2) (ia) (Note 2)
Additional Depreciation at 20% 85 17

Depreciation allowable for A.Y.2021-22 32.75

1. As per section 35(2)(iv), no depreciation shall be allowed in respect of plant and machinery
purchased for scientific research relating to assessee’s business, since deduction is allowable
under section 35 in respect of such capital expenditure.
2. Additional depreciation is not allowed on Second hand plant and machinery
New plant and machinery purchased for scientific research relating to assessee’s business in
respect of which the whole of the capital expenditure can be claimed as deduction under section
35(1)(iv) read with section 35(2) (ia) & (iv).

Illustration 6

Mr. X, set up a manufacturing unit in Warangal in the state of Telangana on 01.06.2019. It invested
Rs. 30 crores in new plant and machinery on 1.6.2019. Further, he invested Rs. 25 crores in the
plant and machinery on 01.11.2019, out of which Rs. 5 crores was second hand plant and
machinery. Compute the depreciation allowable under section 32. Is Mr. X entitled for any other
benefit in respect of such investment? If so, what is the benefit available?

Unique Academy - 8007916622 CA Saumil Manglani - Contact: 9921051593


6. PGBP 6.22
Solution
Computation of depreciation under section 32 for Mr. X for A.Y. 2020-21

Particulars Rs. (in


crores)
Plant and machinery acquired on 01.06.2019 30.000
Plant and machinery acquired on 01.11.2019 25.000
WDV as on 31.03.2020 55.000
Less: Depreciation @ 15% on Rs. 30 crore 4.500
Depreciation @ 7.5% (50% of 15%) on Rs. 25 crore 1.875
Additional Depreciation@35% on Rs. 30 crore 10.500
Additional Depreciation@17.5% (50% of 35%) on
3.500 20.375
Rs. 20 crore
WDV as on 01.04.2020 34.625

Computation of deduction under section 32AD for Mr. X for A.Y. 2020-21

Particulars Rs. (in


crores)
Deduction under section 32AD @ 15% on Rs. 50 crore 7.50
Total benefit 7.50

11. Short Term Capital Gain/ Loss on block of Capital Assets

• If the net consideration of an asset out of the block is less than the
remaining balance of the block, there would be no capital gain.
• If the net consideration of an asset is more than remaining balance of
the block, then the excess shall be deemed to be short term capital gain.
• If all the assets of the block are sold in the previous year and the net
consideration is less than the remaining balance of the block then the
loss shall be deemed to be short term capital loss
• . If all the assets of the block are sold in the previous year and the net
consideration is more than the remaining balance of the block then the
gain shall be deemed short term capital gain.
Note Example Below

Unique Academy - 8007916622 CA Saumil Manglani - Contact: 9921051593


6. PGBP 6.23

Illustration 7
Mr. Mohan, engaged in the business of generation of power, furnishes the following details for FY
2020-21. He has opted for WDV method, you are required to compute the allowable depreciation
for FY 2020-21.
The Opening Block as on 1st Apr 2020 was INR 950,000 (15% block). He acquired second hand
machinery for INR 250,000 on 30th Nov’20. He also acquired Power Generation Machinery on 1st
Aug 2020 for INR 10,00,000. He invested in an AC for Office for INR 200,000 on 9th Sep’20 and a
pollution control equipment for INR 250000 on 30th Jun’20. He bought additional power generation
machinery for INR 500,000 on 1st Feb’21. Also, he sold assets valued INR 400,000 for INR
350,000 during the year.
Calculation –
INR Block Date INR Block Date
Opening WDV 9,50,000 15% Air Pollution Ctrl 2,50,000 40% 30th
Equipment Jun 20
Acq. Second 2,50,000 15% 30th
hand Nov’20
Power 10,00,000 15% 1st Aug
Generation 20
Machinery
AC for Office 2,00,000 15% 9th Sep
Power 5,00,000 15% 1st Feb
Generation 21
Machinery
Sale Value 3,50,000
Depreciation
On the 18,750
ones put to
use for <
180
day
37,500

On the 2,70,000 On the Pollution Control 1,00,000


Equipment
Remaining
Block,
which
comes to
INR
18,00,000
Depreciati 3,26,250
on
allowable
Addl 2,00,000 Addl Depreciation 50,000
allowable on the Poll
Depreciati Control Equipment @
on
Unique Academy - 8007916622 CA Saumil Manglani - Contact: 9921051593
6. PGBP 6.24
allowable
on Power
Generation
Machinery
@ 20
Power 50,000
Generation
Machinery
@ 20%
which was
used for <
180 days
Total 5,76,250 Total Depreciation 1,50,000
Allowable
Depreciation
Allowable
Notes:
a) Power generation equipment, pollution control equipment can enjoy additional
depreciation.
b) No depreciation is allowable on second hand machinery or office appliances like AC.
12. Investment in New Plant or Machinery in Notified Backward Areas in Certain
States (Section 32AD)
Additional investment allowance of an amount equal to 15% of the cost of new
asset acquired and installed by an assessee is allowed, if
(a) He sets up an undertaking or enterprise for manufacture or production of any
article or thing on or after 1st April, 2015 in any notified backward areas in the
State of Andhra Pradesh, Bihar, West Bengal and the State of Telangana;
and
(b) The new assets are acquired and installed from the 1st April, 2015 to 31st
March, 2020.
(c) Transfer of the plant or machinery for a period of 5 years has also been
restricted. However, this restriction shall not apply in a case of amalgamation or
demerger but shall continue to apply to the amalgamated company or resulting
company, as the case may be. AY 21-22
Note – This Investment allowance is not allowed under section 115BA

Section 33AB (Tea, Coffee, Rubber Section 33ABA (Site Restoration Fund
Development Account) Account)
For Assessee engaged in tea, coffee, rubber For Assessee engaged in production of
plantation Petroleum, Natural Gas in India
Deposit amount in NABARD or any approved Deposit amount in SBI Account or any
account approved site restoration account
Amount to be deposited within 6 months from Amount to be deposited before end of PY
the Year end or Due date of filing return
(Earlier)

Unique Academy - 8007916622 CA Saumil Manglani - Contact: 9921051593


6. PGBP
Amount of deduction (Lower of) – Amount of deduction (Lower of) – 6.25
• Amount deposited or • Amount deposited or
• 40% of profits • 20% of profits

Lock in Period → Asset Acquisition → Year End + 8 Years


Amount van be withdrawn because of –
a. Closure of Business
Taxable
b. Dissolution of Firm
c. Death of assessee
d. Partition of Firm
e. Liquidation of a Company
f. For use
under specific
scheme

Assessee needs to get accounts audited & & file return of Income duly signed and verified by
an accountant.
The audit report is to be furnished at least 1 month prior to the due date for furnishing the return
of income under section 139(1).

Note – This deduction is not allowed under section 115BAC AY 21-22

14. Scientific Research (Section 35)

In house Research

Scientific
Research

Research - Maximum 3
years before date of Research after
commencement commencement
(100%)

Manufacturing
Capital exp. Company -
Revenue exp. Others
expl. to Research
35(1)(i)
35(2)(ia) approved
35(2AB)
Revenue -
Only Salary Capital - 100%
100%
(Excluding All exp. allowed Land - Not
perquisites )& Except Land Revenue - 35(1)(i) allowed
Material allowed
100% Capital-100 % 35(1)(iv)
Land - Not alowed
AY 21-22

Unique Academy - 8007916622 CA Saumil Manglani - Contact: 9921051593


6. PGBP 6.26

Contribution to outsiders

Research

Scientific Social & Statistical

To approved To approved Research


To approved Research To IIT, National
Indian co. association, Institute,
association, Institute, Lab. College, University
College, University engaged in R&D
35(2AA) 35(1)(iii)
35(1)(ii) 35(1)(iia)

100% 100% 100% 100%


Note – Section 115BAC – Deduction of in house research allowed & and contribution to
outsiders is not allowed. AY 21-22

Further Amendments in section 35 AY 21-22

• Provided also that every notification under clause (ii), (iii) or (iia)
• on or before the date on which this proviso has come into force,
• shall be deemed to have been withdrawn
• unless such research association, university, college or other institution
• makes an intimation to the prescribed income-tax authority
• within 3 months from the date on which this proviso has come into force,
• and subject to such intimation the notification shall be valid for a period of five consecutive assessment
years
• beginning with the assessment year commencing from AY 22-23.

• In other words – The benefits given to research institutes in the past will become
ineffective if such research institutions do not intimate the Income tax authorities that
they have availed the benefit of such notifications within3 months from the date this
proviso came into effect.
• And instead of life time approval now this benefit shall be applicable for 5 years only
starting from AY 22-23 and after every 5 years approvals would have to be taken.

Provided also that any further notifications which will be issued issued will have the effect for maximum 5
years.

Unique Academy - 8007916622 CA Saumil Manglani - Contact: 9921051593


6. PGBP 6.27

Following sub-section (1A) shall be w.e.f. 1-4-2021 :


• Notwithstanding anything contained in sub-section (1),
✓ these outside agencies (donee) under section 35(1)(ii), (iia), (iii) are
✓ not entitled to deduction under section 35(1)
✓ unless they prepare and deliver the Statement of donation receipts by donee to cross
check claim of donation by donor for such period as may be prescribed to the said prescribed
income-tax authority in such form, verified in such manner and within such time, as may be
prescribed.
✓ And furnishes a certificate to the donor, specifying the amount of donation within such time
from the date of receipt of sum.
Note: Any failure by the done in preparing and delivering the Statement of donation to the income tax
Authority or in furnishing certificate to the donor within the time prescribed shall

• Section 234G - attract fee @Rs. 200 for every day during which the failure continues
+
• Section 271K - Penalty for a sum not less than rs. 10,000 which may be extended to rs. 1,00,000
Illustration 8
Binod furnishes the following particulars for PY 2020-21. You are required to arrive at the deduction
allowable u/s 35 for AY 2021-22, while computing the Income under the head “Profits / Gains from
Business / Profession”.
1) Amount paid to M/s ABC Ltd., a company registered in India, which has as its main object,
scientific research and development, as approved by the prescribed authority INR 600,000
2) Amount Paid to IIT Mumbai, for an approved scientific research programme INR 375,000
3) Revenue Expenditure on In-house R&D facility as approved by prescribed authority INR 450,000
4) Capital Expenditure on In-house R&D facility as approved by prescribed authority INR
12,00,000. This includes cost of Land INR 450,000
5) Amount paid to Indian Institute of Science, Bangalore, for Scientific Research INR 10,00,000
Assume – Assessee has opted for Section 115BAC
Solution
Expenditure INR Allowable Sec Deduction
Amount paid to M/s ABC Ltd., a company 6,00,000 100% 35(1) (iia) -
registered in India, which has as its main Section
object, scientific research and development, 115BAC
as approved by the prescribed
authority
Amount Paid to IIT Mumbai, for an approved 3,75,000 100% 35(2AA) -
scientific research programme Section
115BAC
Revenue Expenditure on In-house R&D facility 4,50,000 100% 35(1)(i) 4,50,000
As approved by prescribed authority
Capital Expenditure on In-house R&D facility 7,50,000 100% 35(1)(iv) 7,50,000
as approved by prescribed authority
Amount paid to Indian Institute of Science, 10,00,000 100% 35(1)(ii) -
Bangalore for Scientific Research Section 115BAC
Totals 31,75,000 12,00,000
Unique Academy - 8007916622 CA Saumil Manglani - Contact: 9921051593
6. PGBP 6.28

15. Amortization of Spectrum Fees for Purchase of Spectrum for telecom


services 35ABA/ Telecom License 35ABB

The section provides:


i. Any capital expenditure incurred and actually paid by the assessee on
acquisition of any right to use spectrum for Telecom services by paying spectrum
fee will be allowed as deduction in equal instalments.
Deduction will start from the year in which payment is made (or the year of
commencement of business, whichever is later) and ending with the year in
which spectrum comes to an end, irrespective of the previous year in which
liability for expenditure was incurred according to the method of accounting
followed.
Where the spectrum is transferred and the proceeds of transfer are less than
the expenditure remaining unallowed, a deduction equal to the expenditure
remaining unallowed as reduced by the proceeds of transfer, shall be allowed
in the year of transfer of spectrum.
ii. If spectrum is transferred and proceeds of transfer exceed the amount of
expenditure remaining unallowed, the excess amount (to the extent it does not
exceed deduction claimed u/s 35ABA) shall be chargeable to tax as PGBP in the
previous year in which such spectrum has been transferred.
iii. Unallowed expenditure in a case where a part of spectrum is transferred would
be amortized.
16. Expenditure of capital nature incurred in respect of specified business (Section
35AD)
An assessee (if he opts) shall be allowed a deduction of capital nature
expenditure incurred for any specified business carried on by him during the
AY 21-22 previous year in which such expenditure is incurred by him. This section talks
about investment linked incentives for specified businesses as under:
A) Setting up & operating cold chain facilities for specified products
B) Setting up & operating warehousing facilities for agricultural produce
C) Laying and operating a cross-country natural gas or crude or petroleum
oil pipeline network for storage and distribution, as a part of the network
D) Building and operating a 2-star hotel or above, anywhere in India
E) Building and operating a hospital with > 100 beds, anywhere in India
F) Developing & building a housing project under a scheme for slum
redevelopment / affordable housing
G) Production of fertilizers in India
H) Setting up and operating an Inland Container Depot OR a Container
Freight Station, under Customs Act, 1962
I) Bee-keeping and production of honey and beeswax
J) Setting up and operating a warehousing facility for storage of sugar
K) Laying and operating a slurry pipeline for transportation of iron-ore

Unique Academy - 8007916622 CA Saumil Manglani - Contact: 9921051593


6. PGBP 6.29
L) Setting up and operating a semi-conductor wafer fabrication
manufacturing unit
M) Developing / Maintaining & Operating / Developing & Maintaining &
Operating a new infrastructure facility in India
Other Pointers
✓ 100% of the capital expenditure incurred during the Previous Year will be
allowed.
✓ The expenditure incurred prior to the commencement of the
business, would be allowed as a deduction in the year of
commencement of business, and should also be capitalized in the
books of the assessee on the commencement of operations.
✓ The loss from specified businesses can be set off ONLY against
profits of specified businesses but can be carried forward
indefinitely for set off against one or more specified businesses.
✓ The losses of specified business can be carried forward only if the Tax
return is filed within the due date.
✓ Accounts needs to be audited by a Chartered Accountant and
✓ That any asset in respect of which the deduction is claimed, can be used
ONLY for the specified businesses for a period of 8 years beginning
with the PY in which the asset was acquired / constructed.
Exceptions
• Expenditure on acquisition of land, Financial instruments OR goodwill will
not be allowed as a deduction
• Any expenditure where the aggregate of payments made to a person
exceeding INR 10,000 per day, otherwise than by account payee cheque /
DD / ECS/ bank account or through such other electronic mode as may be
prescribed would not be eligible for a deduction.

• No deduction in respect of the expenditure under section 35AD (1) shall be


AY 21-22 allowed to the assessee under any other section in any previous year or
under this section in any other previous year, if the deduction has been
claimed or opted by the assessee and allowed to him under this section.

AY 21-22 Note – Section 115BAC – Deduction of section 35AD is not allowed.


Illustration 9
Mr. A commenced operations of the businesses of setting up a warehousing facility for storage of
food grains, sugar and edible oil on 1.4.2020. He incurred capital expenditure of Rs. 80 lakhs, Rs.
60 lakh and Rs. 50 lakhs, respectively, on purchase of land and building during the period January,
2020 to March, 2020 exclusively for the above businesses, and capitalized the same in its books
of account as on 1st April, 2020. The cost of land included in the above figures is Rs. 50 lakhs,
Rs. 40 lakh and Rs. 30 lakhs, respectively. Further, during the P.Y.2020-21, he incurred capital
expenditure of Rs. 20 lakhs, Rs. 15 lakh & Rs. 10 lakhs, respectively, for extension/ reconstruction
of the building purchased and used exclusively for the above businesses.
Compute the income under the head “Profits and gains of business or profession” for the
Unique Academy - 8007916622 CA Saumil Manglani - Contact: 9921051593
6. PGBP 6.30
A.Y.2021-22 and the loss to be carried forward, assuming that Mr. A has fulfilled all the
conditions specified for claim of deduction under section 35AD and has not claimed any
deduction under Chapter VI-A under the heading “C – Deductions in respect of certain incomes”.
The profits from the business of setting up a warehousing facility for storage of food grains, sugar and
edible oil (before claiming deduction under section 35AD and section 32) for the A.Y. 2021- 22 is Rs. 16
lakhs, Rs. 14 lakhs and Rs. 31 lakhs, respectively. Also, assume in respect of expenditure incurred, the
payments are made by account payee cheque or use of ECS through bank account.
Solution
Computation of profits and gains of business or profession for A.Y.2021-22

Particulars Rs. (in lakhs)


Profit from business of setting up of warehouse for storage of edible oil 31
(before providing for depreciation under section 32)
Less: Depreciation under section 32
10% of Rs. 30 lakhs, being (Rs. 50 lakh – Rs. 30 lakhs + Rs.10 lakh) 3
Income chargeable under “Profits and gains from business or 28
profession”
Computation of income/loss from specified business under section 35AD

Particulars Food Sugar Total


Grains.
(A) Profits from the specified business of setting up a
warehousing facility (before providing deduction under
section 35AD) 16 14 30

Less: Deduction under section 35AD


(B) Capital expenditure incurred prior to 1.4.2020 (i.e., prior to
commencement of business) and capitalized in the books
of account as on 1.4.2020 (excluding the expenditure
incurred on acquisition of land) = Rs. 30 lakhs (Rs. 80 lakh 30 20 50
– Rs. 50 lakh) and Rs. 20 lakhs (Rs. 60 lakh – Rs.40 lakh)
(C) Capital expenditure incurred during the P.Y. 2020-21 20 15 35
(D) Total capital expenditure (B + C) 50 35 85
(E) Deduction under section 35AD
100% of capital expenditure (food grains/sugar) 50 35 85
Total deduction u/s 35AD for A.Y.2021-22 50 35 85
(F) Loss from the specified business of setting up and
operating a warehousing facility (after providing for
deduction under section 35AD) to be carried forward (34) (21) (55)
as per section 73A (A-E)

Notes:

(i) Deduction of 100% for sugar storage ware house is allowed.


(iii) Deduction for warehousing facility for storage of edible oils is not allowed.
Mr. A can, however, claim depreciation@10% under section 32 in respect of the capital
expenditure incurred on buildings. It is presumed that the buildings were put to use for more than
180 days during the P.Y.2020-21.

Unique Academy - 8007916622 CA Saumil Manglani - Contact: 9921051593


6. PGBP 6.31

Loss from a specified business can be set-off only against profits from another specified business.
Therefore, the loss of Rs. 55 lakhs from the specified businesses of setting up and operating a
warehousing facility for storage of food grains and sugar cannot be set-off against the profits of Rs.
28 lakhs from the business of setting and operating a warehousing facility for storage of edible oils,
since the same is not a specified business. Such loss can, however, be carried forward indefinitely
for set-off against profits of the same or any other specified business.
17. Expenditure on Agricultural extension project (Section 35CCC)
Where an assessee incurs any expenditure on agricultural extension project
notified by the Board then, there shall be allowed a deduction of a sum equal to
100% of such expenditure
Note – Deduction of Land & Building is not allowed.
AY 21-22 Note – This deduction is not allowed u/s 115BAC
18. Expenditure on skill development project (Section 35CCD)
Where a company incurs any expenditure (not being expenditure in the nature
of cost of any land or building) on any skill development project notified by the
Board then, there shall be allowed a deduction of a sum equal to100% of
such expenditure AY 21-22

19. Amortization of Preliminary Expenses (Section 35D)


Under Section 35D, Indian companies and other non-corporate taxpayers’
resident in India would be entitled to amortization of certain preliminary
expenses incurred by them.
The expenditure which qualifies for amortization should have been incurred
by the assessee before the commencement of his business.
If, however, the expenditure is incurred after the commencement of
business, it is essential that the expenditure should be in connection with the
extension or expansion of the undertaking of the assessee or in connection
with the setting up of a new unit by the assessee.
The amount qualifying for amortization would be allowable as a deduction in 5
equal instalments beginning with the previous year in which the business of
the assessee actually commences or the previous year in which the
extension of the present undertaking is completed or the new unit
commences production or operation, as the case may be.
• Eligible expenses - The following expenditure are eligible for amortization:
(i) Expenditure in connection with - (Mnemonics – MELFP)
(a) feasibility report
(b) project report;
(c) market survey
(d) engineering services
(e) legal charges for drafting any agreement between
(Above work must be carried out by the Assessee himself or by a concern
which is approved by the Board)
(ii) Where the Assessee is a company, in addition to the above,
expenditure incurred –
(f) by way of legal charges for drafting the MOA - AOA

Unique Academy - 8007916622 CA Saumil Manglani - Contact: 9921051593


6. PGBP 6.32

(g) on printing MOA – AOA


(h) by way of fees for registering the company under the Companies
Act
in connection with the issue, for public subscription, of the shares in or
debentures of the company, being underwriting commission, brokerage and
charges for drafting, printing and advertisement of the prospectus;

Whichever
Amount of 5% of cost of

is higher
deduction project
5% of the cost of
1/5th of 5% of capital
the project
qualifying limit employed
In case of other for each of the
resident non- five successive In case of Indian
corporate assesses years companies

Where
• Cost pf project = FA shown in the books on last day of the PY in which the
business commences
• Capital employed means Issued share capital + Debentures + Long term
borrowings (Min. 5 years and If borrowed from outside India – Min. 7 yrs.)
• Note: The audit report is to be furnished at least 1 month prior to the due date for
AY 21-22 furnishing the return of income under section 139(1).

20. Amortization of Expenditure in the case of Voluntary Retirement Scheme


(Section 35DDA)
The object of this section is to provide amortization of one-fifth every year
from the year in which the expenditure is incurred, of expenditure by way of
payment of any sum to an employee in connection with his voluntary
retirement.
21. Other Deductions [Section 36]

Type & Section Deductions


Insurance Premia paid u/s Premia paid on insurance policy to cover risk of damage /
36(1)(i) destruction to stock / stores of the business.
Premia paid by employer for Premia paid by employer by any mode other than by cash,
health insurance of on health insurance of its employees.
employees u/s 36 (1) (ib)
Bonus & Commission u/s Deductible in full as long as the bonus / commission shall not
36 (1)(ii) be payable to them as profits / dividends, if it had not been paid
as bonus / commission

Unique Academy - 8007916622 CA Saumil Manglani - Contact: 9921051593


6. PGBP 6.33

Interest on Borrowed Capital Deduction allowed for any interest paid in respect of
u/s 36(1)(iii) capital borrowed for business.
In case the capital is borrowed for acquiring an asset, the
interest is capitalized from the date of borrowing until the
date when the asset is put to use.
Post the “put to use” date, it cannot be capitalized anymore
and then such interest becomes an allowable deduction
Discount on Zero Coupon Difference between the issue and the redemption values,
Bonds u/s 36(1) (iiia) as these are issued at a discount and redeemed at par.
Available to Infra. Companies / funds / Scheduled Banks,
starting from the date of issue of the bond, ending with the
maturity / redemption.
Contribution to Provident & Allowable if the fund is settled upon a trust, it should
Provident & Another funds be recognized / approved, and the contributions
u/s 36(1)(iv) & (v) should be periodic, and as long as the fund is for the
benefit of the employees
Employer’s contribution to Deduction is restricted to 10% of salary of employee in PY.
the a/c of the employee Salary, here, would include ONLY Basic & DA (if the terms of
under a pension scheme employment provide).
referred to in Section
80CCD [Section 36(1) (va)]
Employee’s Contribution to Will be allowed as a deduction ONLY if the employee
Welfare Funds [Section contributions have been credited to the employees’ account by
36(1) (va)] the taxpayer in the fund, on or before the due date.

Bad Debts u/s 36(1)(vii) & Allowable if the debts debts written off as irrecoverable in
36(2) the accounts of the assessee pertain to the business /
profession carried on during the PY and as long as the debt
was considered in the income for the PY in which it was
earned. If on the final settlement, the amount recovered on
any debt falls short of the total debt minus the debt allowed,
the deficiency will be allowed as a deduction in the year of
recovery and if the amount so recovered is more than the
amount due after the allowance has been made, the excess
will be chargeable to tax in the year of recovery.
Expenses on family planning If the expenditure is capital in nature, allowable in five equal
[Section 36(1)(ix)] instalments beginning the PY in which it was incurred and if
revenue in nature, it shall be fully allowable in the PY in which it
was incurred. The deduction is allowable to corporate
assessees ONLY.
Securities Transaction Tax Allowable in respect of transactions entered in the course of
[Section 36(1)(xv)] business, as long as the income from the taxable securities’
transactions, in respect of which it was incurred, is included
under the heads “Profits / Gains from Business / Profession”
Commodities Transaction Allowable in respect of transactions entered in the course of
Tax [Section 36(1)(xvi)] business, as long as the income from the taxable commodities’
transactions, in respect of which it was incurred, is included
under the heads “Profits / Gains from Business / Profession”

Unique Academy - 8007916622 CA Saumil Manglani - Contact: 9921051593


6. PGBP 6.34

22. Other Expenses (Section 37)


Section 37(1) of the Income-tax Act provides for allowance in respect of any other item of
expenditure not covered by any of the provisions contained in Sections 30 to 36 discussed
above and is limited to the amount actually expended during the Previous Year.
This deduction is subject to the following conditions:

Note:
1) Corporate Social Responsibility (CSR) expenditure is not construed
to have been incurred for the purposes of business / profession and hence
will be disallowed, and will be allowed aptly under the relevant Sec’s 30-36
2) Any advertisement expenditure in souvenirs of political parties,
representing contributions for political purposes, would be disallowed.
Some of the examples of allowable expenses under Section 37(1) are:
(i) Expenditure incurred on raising loans or issuing debentures but not on
issuing share capital.
(ii) Legal expenses incurred:
(a) to avoid a business liability, e.g. for alleged breach of a trading
contract;
(b) to defend the assessee’s title to his assets, e.g. land, building, etc.;
(c) to secure the termination of a disadvantageous trading relationship, e.g.
removal of an undesirable employee;
(d) by a director of a company in defending a suit brought to challenge the
validity of his election to the directorship;
(e) to protect the capital asset of the business which has already been
acquired;
(f) by a company in resisting a winding up petition by some shareholders;
(g) for defending monopoly rights;

Unique Academy - 8007916622 CA Saumil Manglani - Contact: 9921051593


6. PGBP 6.35

However, the expenses incurred in criminal proceedings are not allowable.


Legal expenses relating to acquisition of capital asset for a business are capital
expenses and as such, not allowable.
In this connection the Supreme Court held that-where litigation expenses are
incurred by the assessee for the purpose of creating, curing or completing his title
to the capital, then the expenses incurred must be considered as capital
expenditure. But if the litigation expenses are incurred to protect the business of
the assessee, they must be considered as a revenue expenditure. [Dalmia Jain
& Co. v. C.I.T. (1971) 81 ITR p. 754 (S.C.)].
Expenditure for prosecuting civil proceedings is deductible provided the
expenditure was laid out for the purpose of the business wholly and exclusively
i.e. to promote the interest of the business. [Sree Meenakshi Mills v. C.I.T. (1976)
63 ITR, p. 207 (S.C.)]
(iii) Bonus to employees under an industrial award.
(iv) Expenses for the installation of new telephone.
(v) Sales tax is an admissible deduction but not estate duty.
(vi) Interest on unpaid purchase price of goods or capital assets.
(viii) Subscriptions given are allowed if their payment is compulsory or
commercially expedient and of benefit to the payer.
(ix) Expenses incurred on the occasion of festival or customary days are
allowed up to a reasonable amount keeping in view the size of the business
and subject to the satisfaction of the Assessing Officer that the expenses
are not expenses of personal, social or religions nature.
(x) Recurring expenses incurred on imparting basic training to apprentices under
the Apprentices Act, 1961 is deductible.
(xi) Initial expenditure on the first installation of fluorescent tube lights is
treated as Capital expenditure and hence not deductible but all subsequent
expenditure for replacement of tubes is treated as revenue expenditure and
hence deductible.
(xii) Loss through embezzlement by an employee is deductible.
(xiii) Professional tax paid by a person carrying on business or trade is
allowed as deduction.
(xiv) Annual listing fee paid to stock exchange is allowed as deduction.
(xvi) Brokerage paid for raising loan to finance business.
(xvii) Stamp and registration charges for the purpose of entering into agreement
for obtaining overdraft facilities.
(xviii) Security deposited with postal authorities deductible as business
expenditure.
However, when the amount is returned by postal authorities, when the
refund shall be treated as an income of the assessee of the year in which
the amount is refunded.
(xix) Compensation payable as a result of negligence in carrying on a business
Unique Academy - 8007916622 CA Saumil Manglani - Contact: 9921051593
6. PGBP 6.36
or termination of an employee, director or agent.
(xx) Compensation to an employee for injury sustained or accident met with while
on duty.
(xxi) Royalty paid for mining, patents or copyrights.
(xxiii) Penalty paid by the assessee for saving from confiscation of the
goods which he has purchased from a third-party without knowing that they
had been illegally imported.
23. Certain Allowable Losses
Losses which are directly incidental to the business or profession of the assessee
are allowable. Following are some examples of such losses:
1. Robbery or Dacoity: Loss caused by robbery or dacoity is not deductible. But, if
it is incidental to business it will be allowed as a deduction and this depends upon
the specific circumstances and conditions.
For example, if cash is sent for disbursement at different centers by a sugar
factory in rural area, it is incidental to business and is, therefore, allowed.
Any loss due to robbery in a bank will be allowed as the bank is under an
obligation to maintain some cash outside the storeroom for payments.
2. Embezzlement, Theft, etc.: The loss of money due to embezzlement by an
employee handling the funds of the business while discharging his official duties
is allowed as deduction. It is deductible when discovered.
When an employee goes to bank to deposit the cash or takes cash with him for
disbursement and he takes away the money for his own use, even then, the loss
is allowable.
Theft by a cashier, who is in charge of cash is also an allowable loss. A theft
committed either by an employee or by someone else by breaking open into the
business premises after office hours, is also allowable.
3. Loss due to Non-recovery of advances: Is Allowable
Few more examples –
Particulars Deduction u/s 37 (1)
Penalties imposed for infraction of law Not allowed
Penalty paid on failure to deduct TDS Not allowed
Interest paid in respect of delayed payment on Not allowed
income tax
Any interest/penalty paid under Income Tax Not allowed
Interest paid under GST Law Allowed
Demurrage paid to port authorities in connection Allowed as it is not a fine paid for
with release of confiscated goods infraction of flaw
Interest paid under Employees Provident Fund Allowed
& Misc. provision Act 1952
Penalty paid by the assessee contractor for non- Allowed as it is not a fine paid for
completion of contract within stipulated time infraction of law

Unique Academy - 8007916622 CA Saumil Manglani - Contact: 9921051593


6. PGBP 6.37

24. EXPENSES DISALLOWED (SECTION 40)

Amounts not deductible


Section Particulars
40(a)(i) Any interest, royalty, fees for technical services or other sum chargeable payable
• outside India or
• in India
to a non-resident or to a foreign company, on which TDS has not been deducted
or
after deduction not been paid on or before the due date specified under section 139(1).
40(a)(ia) 30% of any sum payable to a resident on which
(i) TDS has not been deducted
or,
(ii) after deduction has not been paid on or before the due date for filing of return of
income under section 139(1).
However, in both the above cases 40(a)(i) & (ia), if TDS has been deducted in any
subsequent year or deducted, but paid after due date specified u/s 139(1), 100% / 30%
of such amount disallowed will be allowed in such subsequent year.

For both the above two clauses – (a) (i) and (a) (ia)
Provided further that where an Payer fails to deduct the TDS but the Payee whether
Resident or Non-Resident has –
• Considered the income in his calculation
• Paid the tax on such income
• Filed the return &
• Certified the same from a CA

Then Payer will not be considered as an assessee in default, means –


a. Amount of expenditure will be allowed in the year in which the return has been filed
by the payee
b. But will have to pay interest for non-deduction of TDS till the date return if filed by
the payee (whether Resident or Non-Resident)

40(a)(iii) Any payment chargeable under the head “Salaries”, if it is


• payable outside India or
• to a non-resident, if tax has not been paid thereon nor deducted therefrom till
due date of TDS
40(a)(ii) Any sum paid on account of income-tax
Other point – Income tax refund is not taxable but interest from IT dept. is taxable under
IOS
40(a)(v) Tax paid by the employer on non-monetary perquisites provided to its employees,
which is exempt under section 10(10CC) in the hands of the employee.

Unique Academy - 8007916622 CA Saumil Manglani - Contact: 9921051593


6. PGBP 6.38
Illustration 10
Delta Ltd. credited the following amounts to the account of resident payees in the month of
March, 2020 without deduction of tax at source. What would be the consequence of non-
deduction of tax at source by Delta Ltd. on these amounts during the financial year 2019-20,
assuming that the resident payees in all the cases mentioned below, have not paid the tax, if
any, which was required to be deducted by Delta Ltd.?

Particulars Amount in
Rs.
(1) Salary to its employees (credited and paid in March, 2020) 12,00,000
(2) Directors’ remuneration (credited in March, 2020 and paid in April, 2020) 28,000

Would your answer change if Delta Ltd. has deducted tax on directors’ remuneration in April,
2020 at the time of payment and remitted the same in July, 2020?
Solution

Non-deduction of tax at source on any sum payable to a resident on which tax is deductible at
source as per the provisions of Chapter XVII-B would attract disallowance under section
40(a)(ia).
Therefore, non-deduction of tax at source on any sum paid by way of salary on which tax is
deductible under section 192 or any sum credited or paid by way of directors’ remuneration on
which tax is deductible under section 194J, would attract disallowance@30% under section
40(a)(ia). Whereas in case of salary, tax has to be deducted under section 192 at the time of
payment, in case of directors’ remuneration, tax has to be deducted at the time of credit of such
sum to the account of the payee or at the time of payment, whichever is earlier. Therefore, in
both the cases i.e., salary and directors’ remuneration, tax is deductible in the P.Y.2019-20,
since salary was paid in that year and directors’ remuneration was credited in that year.
Therefore, the amount to be disallowed under section 40(a)(ia) while computing business
income for A.Y.2020-21 is as follows –

Particulars Amount paid in Rs. Disallowance u/s


40(a)(ia) @30%

(1) Salary 12,00,000 3,60,000


[tax is deductible under section 192]
(2) Directors’ remuneration 28,000 8,400
[tax is deductible under section
194J without any threshold limit]
Disallowance under section 40(a)(ia) 3,68,400

25. Maximum Permissible Remuneration to Partner in Firm [Sec. 40(b)]


To allow remuneration the following specific conditions, as prescribed by section
40(b), should be satisfied:
1. Remuneration should be paid only to a working partner.
2. Remuneration must be authorized by the partnership deed.
3. Remuneration should not pertain to period perior to partnbership deed.
Unique Academy - 8007916622 CA Saumil Manglani - Contact: 9921051593
6. PGBP 6.39
4. Remuneration should not exceed the permissible limit.
If the above conditions are satisfied remuneration to partners is allowable
deduction in the hands of the firm. However, the maximum amount of such
payment to “all” the partners during the previous year should not nxceed the
limits given below :
Book Profit Maximum amount deductible in respect of
remuneration to partners u/s 40(b)
If book profit is negative Rs. 1,50,000
In case book profit is positive
On first Rs. 3 lakh of book profit Rs. 1,50,000 or 90% of book profit, whichever
On the balance of the book profit is more
60% of book profit
Any remuneration above this limit is not allowed as deduction in the hands of firm and
also not taxable in the hands of partner.
Computation of Book Profit for Remuneration u/s 40(b)

Rs.
NP as per P/L A/c (before Income Tax) xx
Less: Income under all other head (except PGBP) xx
Add: Remuneration to Partner appearing in P/L xx
Add: Excessive Interest of Partner on Capital xx
Less: B/F Depreciation (not b/f loss) xx
Book Profit xx
Maximum Permissible Interest on Capital to Partner in Firm [Section 40(b)]
The following specific onditions should be fulfilled to obtain deduction of interest paid to the
partners :
1. Payment of interest should be authorized by the partnership deed.
2. Payment of interest should pertain to the period after the partnership deed.
3. Rate of interest should not exceed 12%.
Any interest exceeding this limit is not allowed as deduction to firm and also not
taxable in the hands of partner.

PAID BY THE FIRM TO ITS PARTNERS


Allowability of Remuneration Allowability of Interest
1. To working partner. 1. To working/non-working partner.
2. To an individual only. 2. To any partner.
3. Should be authorized by partnership 3. Should be authorized by partnership deed.
deed.
4. In the partnership deed : 4. Rate of interest should be specified in the
- either specify the amount of partnership deed.
remuneration payable to each partner
or
- lay down the manner of quantification of
remuneration to each partner.
5. Remuneration should not be 5. Interest should not be retrospective.
retrospective.

Unique Academy - 8007916622 CA Saumil Manglani - Contact: 9921051593


6. PGBP 6.40
Illustration 10
Rao & Jain, a partnership firm consisting of two partners, reports a net profit of
Rs. 7,00,000 before deduction of the following items:

(1) Salary of Rs. 20,000 each per month payable to two working partners of the firm (as
authorized by the deed of partnership).
(2) Depreciation on plant and machinery under section 32 (computed) Rs. 1,50,000.
(3) Interest on capital at 15% per annum (as per the deed of partnership). The amount of capital
eligible for interest Rs. 5,00,000.
Compute:

(i) Book-profit of the firm under section 40(b) of the Income-tax Act, 1961.
(ii) Allowable working partner salary for the assessment year 2021-22 as per section 40(b).
Solution
(i) As per Explanation 3 to section 40(b), “book profit” shall mean the net profit as per the profit
and loss account for the relevant previous year computed in the manner laid down in Chapter
IV-D as increased by the aggregate amount of the remuneration paid or payable to the
partners of the firm if the same has been already deducted while computing the net profit.
In the present case, the net profit given is before deduction of depreciation on plant and
machinery, interest on capital of partners and salary to the working partners. Therefore, the
book profit shall be as follows:
Computation of Book Profit of the firm under section 40(b)
Particulars Rs. Rs.
Net Profit (before deduction of depreciation, salary and 7,00,000
interest)
Less: Depreciation under section 32 1,50,000
Interest @ 12% p.a. [being the maximum allowable as
per section 40(b)] (Rs. 5,00,000 × 12%) 60,000 2,10,000

Book Profit 4,90,000


(ii) Salary actually paid to working partners = Rs.20,000 × 2 × 12 = Rs.4,80,000.

The maximum allowable working partners’ salary for the A.Y. 2021- 22 in this
case would be:
Particulars Rs.
On the first Rs. 3,00,000 of book profit [(Rs. 1,50,000 or 90% of 2,70,000
Rs. 3,00,000) whichever is more]
On the balance of book profit [60% of (Rs. 4,90,000 - Rs. 3,00,000)] 1,14,000
Maximum allowable partners’ salary 3,84,000

Hence, allowable working partners’ salary for the A.Y.2021-22 as per the provisions of
section 40(b)(v) is Rs. 3,84,000.
26. EXPENSESOR PAYMENT NOT DEDUCTIBLE IN CERTAIN
CIRCUMSTANCES [SECTION 40A]
1. Payment to Relatives and Associates.
Section 40A (2) provides → any expenditure in respect of which a payment
Unique Academy - 8007916622 CA Saumil Manglani - Contact: 9921051593
6. PGBP 6.41
has been or is to be made to a specified person [See column (2) of Table
below) so much of the expenditure as is considered to be excessive or
unreasonable shall be disallowed by the Assessing Officer. While doing so he
shall have due regard to:

(a) the fair market value of the goods, service of facilities for which the payment is
made; or
(b) the legitimate needs of the business or profession carried on by the assessee;
or
(c) the benefit derived by or accruing to the assessee from such a payment.
Note – The section is applicable to expenses only and not on selling at
lower prices.

Assessee Specified Person


(1) (2)
Individual 1. Any relative of the individual assessee
2. Any person who carries on a business or profession, if
• the individual assessee has a substantial interest in the business of that
person or
• any relative of the individual assessee has a substantial interest in the
business of that person
Company, 1. Any director of the company, partner of the firm or member of the family or
Firm,HUF association or any relative of such director, partner or member or
or AOP 2. In case of a company assessee, any individual who has substantial interest
in the business or profession of the company or any relative of such individual
or
3. Any person who carries on a business or profession, in which the Company/
Firm/ HUF/ AOP or director of the company, partner of the firm or member of
the family or association or any relative of such director, partner or member
has substantial interest in the business of that person
All The following are specified persons:
assessees Person who has substantial Other related persons of such person,
interest in the assessee’s who has a substantial interest in the
business assessee’s business
Company/ AOP/Firm • Any director of such company, partner
/HUF of
such firm or the member of such family
or association or
• any relative of such director, partner or
member or
• Any other company carrying on
business or profession in which the
first mentioned company has a
substantial interest

Unique Academy - 8007916622 CA Saumil Manglani - Contact: 9921051593


6. PGBP 6.42
• Company/ Firm/ AOP/
a director, partner or member HUF of which he is a
director, partner or
member or
• Any other
director/ partner/
member of the
such
Company/Firm/
AOP/ HUF or
• Any relative of such director, partner
or member

Relative in relation to an Individual means the spouse, brother or sister or any lineal
ascendant or descendant of that individual [Section 2(41)].
Substantial interest in a business or profession
A person shall be deemed to have a substantial interest in a business or profession if -
- in a case where the business or profession is carried on by a company, such person
is, at any time during the previous year, the beneficial owner of equity shares carrying
not less than 20% of the voting power and

- in any other case, such person is, at any time during the previous year, beneficially entitled to
not less than 20% of the profits of such business or profession.
27. Cash Payments in excess of Rs. 10,000 - section 40A (3)

• Where the assessee incurs any expenditure,


• in respect of which payment or aggregate of payments made
• to a person
• in a day
• otherwise than by an account payee cheque drawn on a bank or by an account payee
bank draft or use of electronic system through bank account or through such other
electronic mode as may be prescribed
• exceeds Rs. 10,000,
• such expenditure shall not be allowed as a deduction.
• Example:

• If, in respect of an expenditure of Rs. 32,000 incurred by X Ltd., 4 cash payments of


Rs. 8,000 are made on a particular day to one Mr. Y – one in the morning at 10 a.m.,
one at 12 noon, one at 3 p.m. and one at 6 p.m., the entire expenditure of Rs. 32,000
would be disallowed under section 40A (3), since the aggregate of cash payments
made during a day to Mr. Y exceeds Rs. 10,000.

• Cash Payment made in excess of Rs. 10,000 deemed to be the income of the
subsequent year, if expenditure has been allowed as deduction in any previous year on
due basis

• Increase in limit of cash payment, where payment made to transport operator:


Payment or aggregate of payments up to Rs. 35,000 in a day can be made to a transport operator for
plying, hiring or leasing goods carriages otherwise than by way of account payee cheque or account
payee bank draft or use of electronic clearing system through a bank account or through such other
electronic mode as may be prescribed. In all other cases, the limit would continue to be Rs. 10,000.

Unique Academy - 8007916622 CA Saumil Manglani - Contact: 9921051593


6. PGBP 6.43

Exceptions – Cash payments more than 10,000 allowed [Rule 6DD]:


(a) where the payment is made to -
(i) the Reserve Bank of India or any banking company;
(ii) the State Bank of India or any subsidiary bank;
(iii) any co-operative bank or land mortgage bank;
(iv) any primary agricultural credit society or any primary credit
society;
(v) the Life Insurance Corporation of India;
(b) where the payment is made to the Government and, under the rules framed
by it, such payment is required to be made in legal tender;
(c) where the payment is made by -
(i) any letter of credit arrangements through a bank;
(ii) a mail or telegraphic transfer through a bank;
(iii) a book adjustment from any account in a bank to any other
account in that or any other bank;
(iv) a bill of exchange made payable only to a bank;
(v) a credit card;
(vi) a debit cards.
(d) where the payment is made by way of adjustment against the amount of any
liability incurred by the payee for any goods supplied or services rendered by
the assessee to such payee;
(e) where the payment is made for the purchase of -
(i) agricultural or forest produce; or
(ii) the produce of animal husbandry (including livestock, meat,
hides and skins) or dairy or poultry farming; or
(iii) fish or fish products; or
(iv) the products of horticulture or apiculture,
to the cultivator, grower or producer of such articles, produce or
products;
(f) where the payment is made for the purchase of the products manufactured
or processed without the aid of power in a cottage industry, to the producer of
such products;
(g) where the payment is made in a village or town, which on the date of such
payment is not served by any bank, to any person who ordinarily resides, or is
carrying on any business, profession or vocation, in any such village or town;
(h) where any payment is made to an employee of the assessee or the heir of
any such employee, on or in connection with the retirement, retrenchment,
resignation, discharge or death of such employee, on account of gratuity,
retrenchment compensation or similar terminal benefit and the aggregate of
such sums payable to the employee or his heir does not exceed fifty thousand
rupees;
(i) where the payment is made by an assessee by way of salary to his employee

Unique Academy - 8007916622 CA Saumil Manglani - Contact: 9921051593


6. PGBP 6.44
after deducting the income-tax from salary in accordance with the provisions of
section 192 of the Act, and when such employee -
(i) is temporarily posted for a continuous period of fifteen days or more
in a place other than his normal place of duty or on a ship; and
(ii) does not maintain any account in any bank at such place or ship;
(j) where the payment was required to be made on a day on which the banks
were closed either on account of holiday or strike;
(k) where the payment is made by any person to his agent who is required to
make payment in cash for goods or services on behalf of such person;
(l) where the payment is made by an authorized dealer or a money changer
against purchase of foreign currency or travelers’ cheques in the normal course
of his business.

28. Provision for Gratuity [Section 40A (7)]:


No deduction shall be allowed for provision made by the assessee for the
payment of gratuity to his employees on their retirement or termination of their
employment for any reason.
However, any provision made by the assessee for the payment of a sum by
way of any contribution towards an approved gratuity fund or for the purpose of
payment of any gratuity that has become payable during the previous year
shall be allowed.
29. Restriction on contribution by employers to non-statutory funds [Sections 40A
(9), (10) and (11)]:

30. Disallowance of unpaid statutory liability (Section 43B)

a) Any sum payable by way of tax, duty, cess or fee. in the P.Y.
Deduction in
b) Any sum payable as an employer by way of in which the
respect of such In the P.Y. of
contribution to any PF or superannuation fund or liability to pay
sums shown in such sum actual
gratuity fund etc. the table was payment
c) Any sum payable to an employee as bonus or incurred
commissions for services redndered.
d) Any sum payable as interest on any loan or
borrowing from any public financial institution or a
State financial corporation or a State industrial
investment corporation.
In any other case
da any sum payable by the assessee as interest
on any loan or borrowing from a deposit
If payment was made
taking non-banking financial company
- in the same P.Y.
or systemically important non-deposit (or)
taking non-banking financial company, in - on or before the due date of
accordance with the terms and conditions filing
of the agreement governing such loan or return u/s 1391)
borrowing
(Clarified – If already expenditure claimed
on accrual basis then can’t again claim
deduction under this clause)
Unique Academy - 8007916622 CA Saumil Manglani - Contact: 9921051593
6. PGBP 6.45
e) Any sum payable as interest on any loan or
advance from a scheduled bank or co-operative “Deposit taking non-banking financial company”
bank.
means a NBFC which is accepting or holding
f) Any sum payable as an employer in lieu of any public deposits and is registered with the RBI
leave at the credit of his employee.
g) Any sum payable to the Indian Railways for use of “Systemically important non-deposit taking
Railway assets. non-banking financial company” means a
NBFC which is not accepting or holding
public deposits and having total assets of
not less than five hundred crore rupees as
per the last audited balance sheet and is
registered with RBI

Where there is default in the payment of such interest, such interest can be converted in
to a loan. Such conversion of the unpaid interest in to loan, by itself, does not constitute
the payment, for purposes of Section 43B. This shall be allowed proportionately, as and
when these are paid.
It must also be noted that where the assessee has not paid any tax, duty, Cess, or fee by whatever name
called, under any law for the time being in force, or any sum payable by the assessee as an employer by
way of contribution to Provident / Super-annuity / Gratuity fund, on or before the “due date” but if he
deposits such sums before the due date for furnishing the return u/s 139(1), no disallowance can be
made u/s 43B.
Poem
नही दिया Railways or सरकारी Tax - Bonus – Commission भी नहीीं बाींटा
PF – Leave Encashment को भी कर दिया काटा
Bank- NBFC – Financial institutions के ब्याज भुगतान में चूक की नही
दमलेगा इन सब का deduction लगेगा section 43B

Return ki due date तक pay िकय तो इस ही साल में


दमल जायेगा नही तो बेटा दजस साल pay करोगे उस साल में
दमल जायेगा

Illustration 11
Hari, an individual, carried on the business of purchase and sale of agricultural commodities like
paddy, wheat, etc. He borrowed loans from Andhra Pradesh State Financial Corporation
(APSFC) and Indian Bank and has not paid interest as detailed hereunder:

Rs.
(i) Andhra Pradesh State Financial Corporation (P.Y. 2019-20 15,00,000
& 2020-21)
(ii) Indian Bank (P.Y. 2020-21) 30,00,000
45,00,000
Both APSFC and Indian Bank, while restructuring the loan facilities of Hari during the year 20-21,
converted the above interest payable by Hari to them as a loan repayable in 60 equal instalments.
During the year ended 31.3.2021, Hari paid 5 instalments to APSFC and 3 instalments to Indian
Bank.
Hari claimed the entire interest of Rs. 45,00,000 as an expenditure while computing the income from
business of purchase and sale of agricultural commodities. Discuss whether his claim is valid and if not,
what is the amount of interest, if any, allowable.
Unique Academy - 8007916622 CA Saumil Manglani - Contact: 9921051593
6. PGBP 6.46

Solution
According to section 43B, any interest payable on the term loans to specified financial institutions
and any interest payable on any loans and advances to, inter alia, scheduled banks shall be
allowed only in the year of payment of such interest irrespective of the method of accounting
followed by the assessee. Where there is default in the payment of interest by the assessee, such
unpaid interest may be converted into loan. Such conversion of unpaid interest into loan shall not
be construed as payment of interest for the purpose of section 43B. The amount of unpaid interest
so converted as loan shall be allowed as deduction only in the year in which the converted loan is
actually paid.
In the given case of Hari, the unpaid interest of Rs. 15,00,000 dues to APSFC and of Rs.
30,00,000 dues to Indian Bank was converted into loan. Such conversion would not amount to
payment of interest and would not, therefore, be eligible for deduction in the year of such
conversion. Hence, claim of Hari that the entire interest of Rs. 45,00,000 is to be allowed as
deduction in the year of conversion is not tenable. The deduction shall be allowed only to the
extent of repayment made during the financial year. Accordingly, the amount of interest eligible
for deduction for the A.Y.2021-22 shall be calculated as follows:

Interest Number of Amount per Instalments Interest


outstanding Instalments instalment paid allowable
(Rs.)
APSFC 15 lakhs 60 25,000 5 1,25,000
Indian 30 lakhs 60 50,000 3 1,50,000
Bank
Total amount eligible for deduction 2,75,000

31. Profits Chargeable to tax (Section 41)


(i) Remission or cessation of trading liability/expenditure -
Suppose an allowance or deduction has been made in any assessment year in
respect of loss, expenditure or trading liability incurred by A. Subsequently, if
A has obtained, whether in cash or in any manner whatsoever, any amount in
respect of such loss or expenditure of some benefit in respect of such trading
liability by way of remission or cessation thereof, the amount obtained by A, or
the value of benefit accruing to him shall be taxed as income of that previous
year.
It does not matter whether the business or profession in respect of which the
allowance or deduction has been made is in existence in that year or not.
It is possible that after the above allowance in respect of loss, expenditure, or
trading liability has been given to A, he could have been succeeded in his
business by another person. In such a case, the successor will be liable to
be taxed in respect of any such benefit received by him during a subsequent
previous year.
Remission or cessation of a trading liability includes remission or cessation of
liability by a unilateral act of the assessee by way of writing off such liability in his
accounts.

Unique Academy - 8007916622 CA Saumil Manglani - Contact: 9921051593


6. PGBP 6.47
(ii) Brought forward losses of defunct business –
In cases where a receipt is deemed to be profit of a business under section 41
relating to a business that had ceased to exist and there is an unabsorbed loss,
not being a speculation loss, which arose in that business during the previous year
in which it had ceased to exist, it would be set off against income that is
chargeable under this section.

32. STAMP DUTY VALUE OF LAND AND BUILDING TO BE TAKEN AS THE FULL VALUE OF
CONSIDERATION IN RESPECT OF TRANSFER, EVEN IF THE SAME ARE HELD
BY THE TRANSFEROR AS STOCK-IN-TRADE [SECTION 43CA]

Section 43CA has been inserted as an anti-avoidance measure to provide that where the
consideration for the transfer of an asset (other than capital asset), being land or building or
both, is less than the stamp duty value, the value so adopted or assessed or assessable (i.e.,
the stamp duty value) shall be deemed to be the full value of the consideration for the purposes of
computing income under the head “Profits and gains of business of profession”.
However, if the stamp duty value does not exceed 110% of the consideration received
or accruing then, such consideration shall be deemed to be the full value of AY 21-22
consideration for the purpose of computing profits and gains from transfer of such
asset. (AY 19-20)
Further, where the date of an agreement fixing the value of consideration for the transfer of
the asset and the date of registration of the transfer of the asset are not same, the stamp
duty value may be taken as on the date of the agreement for transfer instead of on the date
of registration for such transfer, provided at least a part of the consideration has been received
by way of an account payee cheque/ account payee bank draft or use of ECS through a
bank account or through such other electronic mode as may be prescribed on or before the date of the
agreement.

(ii) The Assessing Officer may refer the valuation of the asset to a valuation officer as
defined in section 2(r) of the Wealth-tax Act, 1957 in the following cases -
(1) Where the assessee claims before any Assessing Officer that the value adopted or
assessed or assessable by the authority for payment of stamp duty exceeds the fair
market value of the property as on the date of transfer and
(2) the value so adopted or assessed or assessable by such authority has not been disputed
in any appeal or revision or no reference has been made before any other authority,
court or High Court.
Where the value ascertained by the Valuation Officer exceeds the value adopted or assessed or
assessable by the Stamp Valuation Authority, the value adopted or assessed or assessable shall be taken
as the full value of the consideration received or accruing as a result of the transfer.

Unique Academy - 8007916622 CA Saumil Manglani - Contact: 9921051593


6. PGBP 6.48
Example

Case Date of Actual Stamp duty Stamp duty Full value Remark
transfer of consider- value on the value on the of
land/ action date of date of consider
building agreement registration -tions
held as
stock-
in-trade Rs. in lakhs

1 1/5/2020 100 120 210 120 Stamp duty


(Rs. 10 lakhs (1/9/2019) (1/5/2020) Value on date
received by agreement the to
A/c payee be adopted as full
cheque on value of consideration
1/9/2019) since the stamp duty
value exceeds 110%
of consideration i.e.,
Rs. 110 lakhs.

2 1/5/2020 100 104 210 210 Stamp duty


(Rs. 10 lakhs (1/9/2019) (1/5/2020) Value on the date of
received by Registration to be
cash on adopted as full value of
1/9/2019) Consideration since
part of consideration is
Received by cash and
such stamp duty Value
exceeds 110% of
consideration i.e., Rs.
110 lakhs.

3 31/1/2021 100 110 210 100 Actual sales


(Rs. 10 lakhs (1/9/20 31/1/2021 consideration would
received by 19) be the full value of
A/c payee consideration, since
cheque on stamp duty value
1/9/2019) on the date of
agreement
(which has to
adopted as full
value of consideration
since part of
consideration is
received by account
payee cheque) does
not exceed 110% of
actual consideration
4 31/3/2021 100 120 210 210 Stamp duty
(Full amount (1/5/2020) (31/3/2021) value of the date of
received on registration would be
the date of the full value of
registration) consideration since the
stamp duty value
Exceeds 110% of
consideration i.e., Rs.
110 lakhs.

Unique Academy - 8007916622 CA Saumil Manglani - Contact: 9921051593


6. PGBP 6.49

Demand booster for Residential Real Estate Income Tax relief for Developers & Home Buyers
in Atmanirbhar Bharat Package 3.0

AY 21-22

• Economic slowdown has led to decline in prices of residential unit


• Presently Section 43CA of IT Act restricts differential between circle rate & agreement value @ 10% –
Prices may actually be lower than this.
• Decided to increase the differential from 10% to 20% (under section 43CA) for the period from
12th November 2020 to 30th June 2021 for only primary sale of residential units of value up to Rs
2 crores.
• Consequential Relief up to 20% shall also be allowed to buyers of these units under section
56(2)(x) of IT Act for the said period. - IOS
This measure will reduce hardships faced by both home-buyers and developers and help in clearing the
unsold inventory

33. Compulsory maintenance of books of accounts – Section 44AA

Books to
maintain

(a) Specified Other than (a)


professionals (b) Indi - HUF and (b)

Gross receipts > G.R upto 1.5


lacs in all 3 Existing Newly Set up Existing Newly set up
1.5 lacs in all 3
preceeding PY's preceeding PY's

TI > 2.5 lacs or Likely to TI . 1.2 lacs or


T.over/sakes > exceed the T Over/ Sales Likely to exceed
cash book, Journal ,
Such books to 25 lacs in any 3 limits of > 10 lacs in the limits of
Ledger, ccopes of
enable AO to preceeding PY previous any 3 previous block
bills issued . 25rs,
compute block preceeding
Original bills, Pay taxable PY
vouchers . 50rs income

Notified professions: The professions notified so far are as the profession of authorized representative; the
profession of film artist (actor, camera man, director, music director, art director, editor, singer, lyricist, story writer,
screen play writer, dialogue writer and dress designer); the profession of company secretary; and
information technology professionals.

Note - The books of accounts and other documents shall be kept and maintained for a period of 6 years
from the end of relevant assessment year.

Unique Academy - 8007916622 CA Saumil Manglani - Contact: 9921051593


6. PGBP 6.50

34. Section 44AB - Audit of accounts (+ element of books of accounts)


Section 44AB - Audit of accounts (+ element of books of accounts)

Audit

44AE + 44AD(4) & 44ADA


Business Profession Claiming
+ Claims lower profits
lower profits

cash receipts Audit + Income Income doesnt


Gross Rec. >50
upto 5% and cash applicable (In exceeds exceed exemption
Lacs limit
payments upto this case exemption limit
5% of total monetary
AY 21-22 limits doesnt
apply)
Books maintain + Check 44AA & 44AB
Yes No audit applicable individually
TO during PY > 5 TO During PY > 1
Crores Crore

The assessee needs to furnish audit report 1 month before the due date of return filing specified in section
139(1) –
S. Type of Assessee Due Date u/s 44AB Due Date u/s 139(1)
No. for furnishing Tax for furnishing
Audit report Return of Income
1. Where the assessee is required to 31st October of the 30th November of the
furnish a report of a CA u/s 92E relevant assessment relevant assessment
relating to international transaction or year year
specified domestic transaction
(transfer Pricing cases)
2. Any other case 30th September of the 31st October of the
relevant assessment relevant assessment
year year

Illustration 12
Vinod is a person carrying on profession as film artist. His gross receipts from profession
are as under: Rs.
Financial year 2017-18 1,15,000
Financial year 2018-19 1,80,000
Financial year 2019-20 2,10,000

What is his obligation regarding maintenance of books of accounts for Assessment Year
2021-22 under section 44AA of Income-tax Act, 1961?
Solution

Section 44AA (1) requires every person carrying on any profession, notified by the Board in
the Official Gazette (in addition to the professions already specified therein), to maintain
such books of account and other documents as may enable the Assessing Officer to
Unique Academy - 8007916622 CA Saumil Manglani - Contact: 9921051593
6. PGBP 6.51
compute his total income in accordance with the provisions of the Income-tax Act, 1961.
As per Rule 6F, a person carrying on a notified profession shall be required to maintain
specified books of accounts:
a. if his gross receipts in all the three years immediately preceding the relevant previous
year has exceeded Rs.1,50,000; or
b. if it is a new profession which is setup in the relevant previous year, it is likely to exceed
Rs.1,50,000 in that previous year.
In the present case, Vinod is a person carrying on profession as film artist, which is a notified
profession. Since his gross receipts have not exceeded Rs. 1,50,000 in financial year 2017-
18, the requirement under section 44AA to compulsorily maintain the prescribed books of
account is not applicable to him.
Mr. Vinod, however, required to maintain such books of accounts as would enable the Assessing
Officer to compute his total income.

35. Presumptive Income


Particulars Section 44AD Section 44ADA Section 44AE
1) Eligible Resident individual, HUF Resident assessee An assessee
Assessee Partnership firm (but not LLP) engaged in any owning not more
engaged in eligible business and profession specified u/s than 10 goods
who has not claimed deduction 44AA (1), namely, legal, carriage at any time
under section 10AA or Chapter medical, engineering, during the P.Y.
VIA under the heading "C"- architectural profession
Deductions in respect of or profession of
certain accountancy or
incomes" technical consultancy or
Non-applicability of section interior decoration or
44AD - notified profession
A person carrying on profession (authorised
specified u/s 44AA (1); representative, film
A person earning income in artist, company
the nature of commission or secretary, profession of
brokerage; agency business. information technology).

2) Eligible business/ Any business, other than Any profession specified Business of Plying,
profession business referred to in section under section 44AA (1), hiring or leasing
44AE, whose total whose total gross goods carriages.
turnover/gross receipts in the receipts < Rs. 50 lakhs
P.Y. < Rs. 200 lakhs in the relevant P.Y.
3) Presumptive 8% of total turnover/gross 50% of total gross *
income receipts or a sum higher than receipts of such Given separately
the aforesaid sum as received profession or a sum after this table
in cash/ crossed/ bearer higher than the because of AY 19-
cheque. aforesaid sum claimed 20
6% of total turnover/gross to have been earned by
receipts in respect of the the assessee.
amount of total turnover/gross
receipts received by A/c payee
cheque/bank draft/ECS during
the P.Y. or before due date of
filing of return
Unique Academy - 8007916622 CA Saumil Manglani - Contact: 9921051593
6. PGBP 6.52
u/s 139(1) in respect of that
P.Y.

4) Non- Deductions allowable under sections 30 to 38 shall be deemed to have been


allowability given full effect to and no further deduction shall be allowed.
of Even in case of a firm, salary and Even in case of a firm, In case of a
deductions interest paid to partners is not salary and interest firm, salary and
while deductible. paid to partners is not interest paid to
computing deductible. partners is
presumptive deductible
income subject to the
conditions and
limits in section
40(b)

5) Written WDV of any asset of an eligible business/profession shall be deemed to


down have been calculated as if the eligible assessee had claimed and had been
value of actually allowed depreciation for each of the relevant assessment years.
asset
6) Requirement After declaring profits on presumptive If the assessee claims If the assessee
of basis u/s. 44AD, say, for A.Y. 2018- his profits to be lower claims his
maintenance 19, non-declaration of profits on than the profits profits to be
of presumptive basis for any of the 5 computed by applying lower than the
books of successive AYs thereafter (i.e., from the presumptive rate, he profits
account A.Y. 2019-20 to A.Y. 2023-24), say, has to maintain books of computed by
u/s 44AA for A.Y. account and other applying the
and audit 2020-21, would disentitle the documents u/s 44AA (1) presumptive
u/s 44AB assessee from claiming profits on and get his accounts rate, he has to
presumptive basis for five successive audited u/s 44AB, if his maintain books
AYs subsequent to the AY relevant to total income > basic of account u/s
the PY of such non- declaration (i.e. exemption limit for that 44AA (2) and
from A.Y. 2021-22 to A.Y. 2025-26). year. get his
In such a case, the assessee would accounts
have to maintain books of account audited u/s
and other documents u/s 44AA (2) 44AB.
and get his accounts audited u/s
44AB, if his total income > basic
exemption limit in those years.

7 Advance Tax Single Instalment – 15th March Single Instalment – 15th 4 Instalments –
Instalment March 15th June
15th September
15th December
15th March

a. Light Weight Vehicles – Rs. 7,500 per month per vehicle for a month or for a part of the
month
b. Heavy Weight Vehicle - Rs. 1,000 per ton per vehicle for a month or for a part of the
month

Unique Academy - 8007916622 CA Saumil Manglani - Contact: 9921051593


6. PGBP 6.53
Regarding section 44AE - Meaning of certain terms –

S.No Term Meaning


(1) Heavy goods Any goods carriage, the gross vehicle weight of which > 12,000
vehicle kilograms.
(2) Gross vehicle Total weight of the vehicle and load certified and registered by the
weight registering authority as permissible for that vehicle.

(3) Unladen The weight of a vehicle or trailer including all equipment ordinarily
weight used with the vehicle or trailer when working but excluding the weight
of driver or attendant
and
where alternative parts or bodies are used the unladen weight of the
vehicle means the weight of the vehicle with the heaviest such
alternative body or part

Illustration 13 (Section 44AE)


Mr. X commenced the business of operating goods vehicles on 1.4.2020. He purchased the
following vehicles during the P.Y.2020-21. Compute his income under section 44AE for
A.Y.2020-21.

Gross Vehicle Number Date of


Weight (in purchase
kilograms)
(1) 7,000 2 10.04.2020
(2) 6,500 1 15.03.2021
(3) 10,000 3 16.07.2020
(4) 11,000 1 02.01.2021
(5) 15,000 2 29.08.2020
(6) 15,000 1 23.02.2021
Would your answer change if the goods vehicles purchased in April, 2020 were put to use only
in July, 2020?

Solution –
Since Mr. X does not own more than 10 vehicles at any time during the previous year 2020-21,
he is eligible to opt for presumptive taxation scheme under section 44AE. Rs.1,000 per ton of
gross vehicle weight or unladen weight per month or part of the month for each heavy goods
vehicle and Rs.7,500 per month or part of month for each goods carriage other than heavy
goods vehicle, owned by him would be deemed as his profits and gains from such goods
carriage.
Heavy goods vehicle means any goods carriage, the gross vehicle weight of which exceeds
12,000 kg.
(1) (2) (3) (4)
Number Date of No. of months No. of months × No. of vehicles
of purchase for which [(1) × (3)]
Vehicles vehicle is
owned
For Heavy goods vehicle

Unique Academy - 8007916622 CA Saumil Manglani - Contact: 9921051593


6. PGBP 6.54
2 29.08.2020 8 16
1 23.02.2021 2 2
18
For goods vehicle other than heavy goods vehicle
2 10.4.2020 12 24
1 15.3.2021 1 1
3 16.7.2020 9 27
1 2. 1. 2021 3 3
55

The presumptive income of Mr. X under section 44AE for A.Y.2021-22 would be -
Rs. 6,82,500, i.e., 55 × Rs. 7,500, being for other than heavy goods vehicle (+) 18 x
Rs.1,000 x 15 ton being for heavy goods vehicle.
The answer would remain the same even if the two vehicles purchased in April, 2 2020
were put to use only in July, 2020, since the presumptive income has to be calculated per
month or part of the month for which the vehicle is owned by Mr. X.
Illustration 14
Mr. Praveen engaged in retail trade, reports a turnover of Rs. 1,98,50,000 for the financial year
2020-21. His income from the said business as per books of account is Rs. 13,20,000 computed as
per the provisions of Chapter IV-D “Profits and gains from business or Profession” of the Income-
tax Act, 1961. Retail trade is the only source of income for Mr. Praveen. A.Y. 2020-21 was the first
year for which he declared his business income in accordance with the provisions of presumptive
taxation under section 44AD.
(i) Is Mr. Praveen also eligible to opt for presumptive determination of his income chargeable to
tax for the assessment year 2021-22?
(ii) If so, determine his income from retail trade as per the applicable presumptive provision
assuming that whole of the turnover represents cash receipts.
(iii) In case Mr. Praveen does not opt for presumptive taxation of income from retail trade, what are
his obligations under the Income-tax Act, 1961?
(iv) What is the due date for filing his return of income under both the options?
Solution
(i) Yes. Since his total turnover for the F.Y.2020-21 is below Rs. 200 lakhs, he is eligible to opt for
presumptive taxation scheme under section 44AD in respect of his retail trade business.
(ii) His income from retail trade, applying the presumptive tax provisions under section 44AD,
would be Rs. 15,88,000, being 8% of Rs. 1,98,50,000.
Mr. Praveen had declared profit for the previous year 2019-20 in accordance with the presumptive
provisions and if he does not opt for presumptive provisions for any of the five consecutive
assessment years i.e., A.Y. 2021-22 to A.Y. 2025-26, he would not be eligible to claim the benefit of
presumptive taxation for next five assessment years
Consequently, Mr. Praveen is required to maintain the books of accounts and get them audited under section
44AB, since his income exceeds the basic exemption limit.
(iii) In case he opts for the presumptive taxation scheme under section 44AD, the due date
would be 31st July, 2021.
(iv) In case he does not opt for presumptive taxation scheme, he is required to get his books of account
audited, in which case the due date for filing of return of income would be 31st October, 2021.

Unique Academy - 8007916622 CA Saumil Manglani - Contact: 9921051593


6. PGBP 6.55
36. Other electronic modes have been notified as per new Rule 6ABBA

For various sections under the income tax Act like 35AD,40A,43CA,44AD, 50C/ 269SU etc. the following shall
be the other electronic modes –
(a) Credit Card;
(b) Debit Card;
(c) Net Banking;
(d) IMPS (Immediate Payment Service);
(e) UPI (Unified Payment Interface);
(f) RTGS (Real Time Gross Settlement);
(g) NEFT (National Electronic Funds Transfer), and
(h) BHIM (Bharat Interface for Money) Aadhar Pay”;

37. Case Laws

1. “Actual write off” of individual debtor’s account is not necessary under 36(1)(vii) Bad
Debt, of the Income-tax Act, 1961
Vijaya Bank v. Commissioner of Income Tax [2010] [323 ITR 166]

Supreme Court referring to its judgement in Southern Technologies Limited v. Joint CIT
held that in order to understand the term “write-off” one has to see how the write off has
been effected. If an assessee debits an amount of doubtful debtors to profit and loss
account and credits the asset account (i.e., sundry debtors) it would constitute an actual
write off of a debt.
On the contrary, if the amount is credited to “current liabilities and provisions”, then it would be
a provision. In the latter case the assessee would not be entitled to the
Reference may also be made to the Supreme Court decision in TRF Limited vs CIT10 wherein
it was held that bad debts need not be proven to be irrecoverable under section 36(1)(vii). It is
sufficient if they are written off.

2. Remission of a liability under section 41(1) of the Income Tax Act


Commissioner of Income-tax v. Smt. Sita Devi Juneja [2010] [325 ITR 593, Punjab and
Haryana High Court]
The High Court held that merely because liability was outstanding for the last six years, it
could not be presumed that the said liability had ceased to exist. It was also conceded that
there was no bilateral act between the assessee and the creditors, which indicated that the said
liability had ceased to exist. In absence of any bilateral act, the said liability could not have
been treated as ceased.
3. Is interest income on margin money deposited with bank for obtaining bank guarantee
to carry on business, taxable as business income?
CIT v. K and Co. (2014) (Del)
The High Court held that the interest income received on funds kept as margin money for
obtaining the bank guarantee would be taxable under the head “Profits and gains of business or
profession”.

Unique Academy - 8007916622 CA Saumil Manglani - Contact: 9921051593


6. PGBP 6.56

4. Is
expenditure incurred for construction of transmission lines by the assessee for
supply of power to UPPCL by the assessee deductible as revenue expenditure?
Addtl. CIT v. Dharmpur Sugar Mill (P) Ltd (2015 Allahabad HC)
Following the principle of law laid down by the Supreme Court in Empire Jute Mills’ case, the
Allahabad High Court, in this case, held that the expenditure which was incurred by the
assessee in the laying of transmission lines was clearly on the revenue account. The
transmission lines, upon erection, vested absolutely in UPPCL. The expenditure which was
incurred by the assessee was for aiding efficient conduct of its business since the assessee
had to supply electricity to its sole consumer UPPCL. This was not an advantage of a capital
nature.

5. What is the nature of expenditure incurred on glow-sign boards displayed at dealer


outlets - capital or revenue?
CIT v. Orient Ceramics and Industries Ltd. (2013) (Delhi)
The Delhi High Court held that such expenditure on glow sign boards displayed at dealer outlets
was revenue in nature.

6. Would the expenditure incurred on issue and collection of convertible debentures be


treated as revenue expenditure or capital expenditure?
CIT v. ITC Hotels Ltd. (2011) (Kar.)
The Karnataka High Court held that the expenditure incurred on the issue and collection of
debentures shall be treated as revenue expenditure even in case of convertible debentures,
i.e., the debentures which had to be converted into shares at a later date.

7.Can the commission paid to doctors by a diagnostic centre for referring patients for
diagnosis be allowed as a business expenditure under section 37 or would it be treated
as illegal and against public policy to attract disallowance?
CIT v. Kap Scan and Diagnostic Centre P. Ltd. (2012) (P&H)
The demanding as well as paying of such commission is bad in law. It is not a fair practice and
is opposed to public policy and should be discouraged. Thus, the High Court held that
commission paid to doctors for referring patients for diagnosis is not allowable as a business
expenditure.

Unique Academy - 8007916622 CA Saumil Manglani - Contact: 9921051593


6. PGBP 6.57

8. In a case where payment of bonus due to employees is paid to a trust and such amount
is subsequently paid to the employees before the stipulated due date, would the same be
allowable under section 36(1)(ii) while computing business income?
Shasun Chemicals & Drugs Ltd v. CIT (2016 - SC)
The Apex Court held that section 36(1) contains various kinds of expenses which are
allowable s deduction while computing the business income. The amount paid by way
of bonus is one such expenditure which is allowable as deduction under section
36(1)(ii).
Note: In this case, the Supreme Court has held that the bonus was allowable as
deduction under section 36(1)(ii), even though it was initially remitted to the trust
created for this purpose, from which the payment was ultimately made to the
employees before the due date. The Supreme Court has applied the concept of
“substance over form” in allowing the deduction of bonus paid under section
36(1)(ii) by considering that the payment of bonus was ultimately made to employees
before the stipulated due date.

9.Income from Rent out Business is ‘Profits & gains of business’ not ‘Income from
House property. M/s. Rayala Corporation Pvt. Ltd vs. Assistant Commissioner of Income
Tax
In a recent case between M/s. Rayala Corporation Pvt. Ltd vs. Assistant Commissioner of
Income Tax, the Supreme Court of India has declared that, Income which was arises from Rent
out business should be taxed under the Head “Profits and gains of business or profession’ not
‘Income from House property.
The appellant-assessee, a private limited company, is having house property, which has been
rented and the assessee is receiving income from the said property by way of rent.
The division bench comprising of Justice Anil R Dave and Justice L Nageshwar Rao has relied
the case Chennai Properties and Investments Ltd. v. Commissioner of Income Tax [2015] 373
ITR 673 (SC) that if an assessee is having his house property and by way of business he is
giving the property on rent and if he is receiving rent from the said property as his business
income, the said income, even if in the nature of rent, should be treated as “Business Income”
because the assessee is having a business of renting his property and the rent which he
receives is in the nature of his business income.

10. CIT v. Mahindra and Mahindra Ltd. [2018] 404 ITR 1 (SC) – Newly added in ICSI Module
Issue - Whether the waiver in respect of loan taken for purchase of plant & Machinery and tooling
equipment , would the same be taxable in the hands Mahindra and Mahindra Ltd. under section
28(iv) or 41(1)
Conclusion – Such waiver would not be taxable under both the sections. because section 28(iv)
applies if, income arises from business or profession and the benefit received is in non-monetary form
and 41(1) applies when assess claims an allowance or deduction and debits the amount to the
trading account or to the profit and loss account. Both the elements were missing in this case.

Unique Academy - 8007916622 CA Saumil Manglani - Contact: 9921051593


6. PGBP 6.58

QUESTIONS
Question 1
Ms. Priya is engaged in the business of generation and distribution of power and opts the WDV
method for claiming Depreciation. She has an opening block of INR 50,00,000. She acquired new
machinery for INR 25,00,000 on 15th Nov 2020. She also imported a new machinery from Zurich
for INR 10,00,000 on 14th Apr 2020. This machine was used there earlier and she is the first user
in India. Additionally, she bought computers for INR 500,000 on 9th Sep 2020.
You are required to compute the allowable depreciation under Income Tax Act, 1961 for AY 2021-
22.
Solution
Items Date INR Category Rate Depreciation
Opening WDV 1st Apr 2019 50,00,000 Full 15% 7,50,000
New Machinery 15th Nov 2019 25,00,000 Half 15% 1,87,500
Imported Machinery 14th Apr 2019 10,00,000 Full 15% 1,50,000
Computers 9th Sep 2019 5,00,000 Full 40% 2,00,000
Total Depreciation 12,87,500
Addl Depreciation
New Machinery 25,00,000 Half 20% 2,50,000
Computers 5,00,000 Full 20% 1,00,000
Additional Depreciation 3,50,000
Annual Depreciation 16,37,500

Note: The machinery that was imported was first used therein at Zurich, earlier before being imported in to
India and hence no depreciation will be allowable on the same.
Question 2
Examine whether the following expenses incurred by Ms. Priyanka, a dealer in Securities, will be allowable?
a) Expenses on CSR Activities INR 750,000

b) Setting up a cold chain facility for specified products, INR 10,00,000

c) Interest on loan paid to Mr. Shyam, INR 100,000 on which no TDS was affected. Her sales for the PY was
INR 5 Crores

d) Securities Transaction Tax paid INR 50,000


Solution
a) Expenses on CSR activities are not allowable as a deduction u/s 37 and hence the entire INR 750,000
would be disallowed
b) This is a specified business for which she can claim 100% deduction u/s 35AD, hence entire INR
10,00,000 will be allowable
c) The turnover of Ms. Priyanka exceeds the threshold of INR 100,00,000 and hence she was required to
deduct tax at source. Since she didn’t, 30% of the interest, i.e., 30% of INR 100,000, that is INR 30,000
would be disallowed u/s 40(a)(ia) and the balance INR 70,000 would be allowable for the assessee u/s
36(1)(iii)
d) Securities Transaction Tax of INR 50,000 would be an allowable expense, assuming that income from
such source, has been included under the head “Profits / Gains from Business / Profession”.

Question 3
Mr. Kundan Lal, a trader at Kolkata, submits the P&L as under, for FY 2020-21

Unique Academy - 8007916622 CA Saumil Manglani - Contact: 9921051593


6. PGBP 6.59
Profit & Loss Account for Year Ended 31st March, 2021
Particulars INR Particulars INR
To Opening Stock 1,00,000 By Sales 1,25,00,000
To Purchases 1,20,00,000 By Closing Stock 2,00,000
To Gross Profit 6,00,000
Total 1,27,00,000 Total 1,27,00,000
To Rent, Rates, Taxes 1,08,000 By Gross Profit 6,00,000
To Salaries 1,25,000 By Interest Income 5,000
To Interest on loan 25,000
To Depreciation 2,25,000
To Printing & 25,000
Stationery
To Postage & 1,750
Telegram
To Loss on Sale of Shares 12,190
(Short Term)
To General Expenses 17,060

To Net Profit 66,000

Total 6,05,000 Total 6,05,000

Additional Information:
a) Closing Stock & Opening Stock was under-valued by 10%
b) Salary includes INR 20,000 paid to a relative which was considered unreasonable
c) The whole amount of Printing & Stationery was paid in Cash at one go
d) WDV of the Plant & Machinery on 1st Apr was INR 12,00,000. Additions of INR
5,00,000 were made on 1st June 2020 and on 1st Oct 2020, Machinery was sold
for INR 12,57,993
e) Rent & Rates included outstanding GST Liability for Mar’21, of INR 27,000, duly
paid within 7th April 2021
f) A donation of INR 12,000 was made to a public charitable trust during the year
You are required to:
a) Calculate the Profits / Gains from Business Profession
b) Advise whether he should opt for the Presumptive scheme u/s 44AD
You can assume that the entire amount of turnover was received by account payee cheque.
Solution

Particulars INR
Net Profit 66,000
Add: UV Closing Stock 2,22,222
Salary (Relative) 20,000

Unique Academy - 8007916622 CA Saumil Manglani - Contact: 9921051593


6. PGBP 6.60
Printing & Stationery (Cash > 20000) 25,000
Depreciation 2,25,000
Donation 12,000
Loss on Sale of Shares 12,190
5,82,412
Less: UV Opening Stock 1,11,111
Allowable Depreciation 66,301
Interest Income 5,000
Profits / Gains from Business / Profession 4,00,000
Under Presumptive Taxation 7,50,000

Since the tax liability on presumptive basis, i.e. 6% of Gross Receipts (INR 125,00,000 *6%) = INR
750,000 is higher than the computed Profits / Gains from Business Profession, he shouldn’t adopt
for presumptive basis. However, since his turnover is > INR 1 Crore, audit u/s 44AB would be
mandatory, if he doesn’t adopt presumptive basis.
Notes:
a) Under-valued closing stock added to Profits (100/90*200,000)
b) Under-valued opening stock reduced (100/90*100,000)
c) Salary to relative, to the extent considered reasonable, added back
d) Since the cash payment was > INR 10,000, entire amount disallowed u/s 40 A (3)
e) Depreciation added back and allowed as under:

Particulars INR
Opening WDV 12,00,000
Additions 5,00,000
Disposals 12,57,993
Closing WDV 4,42,007
Depreciation @ 15% | 66,3011
f) Since the unpaid GST Liability was paid before the due date and before the date of filing
return of income u/s 139(1), it is allowable.
Question 4
Net profit as per profit and loss account of X is Rs. 6,86,000 for the year ending 31st March, 2021. The
following information is noted from his accounts:
(a) Advertisement expenditure debited to profit and loss account include the following:
(i) expenditure incurred outside india: rs. 46,000 (permitted by rBi);
(ii) articles presented by way of advertisement (60 articles cost of each being rs. 900; and 36
articles cost of each being rs. 1,700);
(iii) rs 16,000 being cost of advertisement which appeared in a newspaper owned by a political party;
(iv) rs. 11,400 being capital expenditure on advertisement;
(v) rs. 12,000 paid in cash; and
(vi) rs. 7,000 paid to a concern in which X has substantial interest (amount is excessive to the extent of rs.
1,400).
(b) Out of salary to employees of Rs. 8,70,000 debited to the profit and loss account:
rs. 40,000 is employees’ contribution to recognised provident fund, rs. 37,500 of which is
(i)
credited in the employees’ account in the relevant fund before the ‘due date’;
Unique Academy - 8007916622 CA Saumil Manglani - Contact: 9921051593
6. PGBP 6.61
(ii) rs. 46,000 is bonus which is paid on 13th November, 2021;
(iii) rs. 36,000 is commission which is paid on 1st december, 2021;
(iv) rs. 20,000 is incentive to workers which is paid on 10th december, 2021;
(v) rs. 40,000 is paid outside india in respect of which tax is not deducted at source;
(vi) rs. 6,000 being capital expenditure for promoting family planning amongst employees; and
(vii) rs. 40,000 being entertainment allowance given to employees.
(c) Entertainment expenditure debited to profit and loss account is Rs. 9,000.
Determine the net income of X for the assessment year 2021-22.

Solution:
Calculation of Net income of X for assessment year 2021-22

Net Profit as per Profit and Loss Account 6,86,000


add: inadmissible items:
Cash paid for advertisement expenses (Note 3) 12000
Cost of advertisement which appeared in a newspaper owned by a political party 16000
excessive amount paid to a concern in which X has substantial interest 1400
employee contribution to recognised provident fund 2500
(to the extent not credited in the employees’ account in the relevant fund before the
‘due date’)
Bonus being paid to employees after the ‘due date’ of filing the return 46000
Commission being paid to employees after the ‘due date’ of filing the return 36000
Salary paid outside india in respect of which tax is not deducted at source 40000
Capital expenditure for promoting family planning amongst employees 6000
(allowed only to a corporate assessee)
Capital expenditure on advertisements 11400
Net income 8,57,300
Notes:
1. restrictions on advertisement and entertainment abolished.
2. With the abolition of Section 37(3), which inter alia governed the deductibility of advertising
expenses, advertising too has come within the fold of the omnibus Section 37(1) which
specifically frowns on capital expenditure. the Himachal Pradesh High Court verdict in Mohan
Meakin Breweries ltd. v. Cit (1979) 118 itr 101 allowing capital expenditure on advertising
therefore has ceased to have the force of law as it was rendered in the context of Section
37(3).
3. advertisement expenses of rs. 12,000 (i.e. exceeding the limit of rs. 10,000) is paid in cash, hence
disallowed under section 40a(3).
4. The ‘due date’ for filing return where the assessee is a person (other than a company) who is
required to get his accounts audited under the income-tax act or any other law is September 30;
and where the assessee is a person deriving income from business and who is not required to get
his accounts audited, the ‘due date’ is July, 31. under the provisions of Section 43B of the act -
Bonus rs. 46,000 paid on 13th Nov., 2021 and Commission rs. 36,000 paid on 1st dec., 2021 are
not admissible since the payments are made after the above mentioned ‘due date’.
5. incentive to workers which is paid on 10th december, 2021 is admissible on ‘due basis’
Question 5
Mr. Sivam, a retail trader of Cochin gives the following Trading and Profit and Loss Account for the year
Unique Academy - 8007916622 CA Saumil Manglani - Contact: 9921051593
6. PGBP 6.62
ended 31st March, 21:
Trading and Profit and Loss Account for the year ended 31.03.2021
Particulars Rs. Particulars Rs.
To Opening stock 90,000 By Sales 1,12,11,500
To Purchases 1,10,04,000 By Closing stock 1,86,100
To Gross Profit 3,03,600 -
1,13,97,600 1,13,97,600
To Salary 60,000 By Gross profit b/d 3,03,600
To Rent and rates 36,000 By Income from UTI 2,400
To Interest on loan 15,000
To Depreciation 1,05,000
To Printing & stationery 23,200
To Postage & telegram 1,640
To Loss on sale of shares (Short term) 8,100
To Other general 7,060
expenses
To Net Profit 50,000
3,06,000 3,06,000
Additional Information:

(i) It was found that some stocks were omitted to be included in both the Opening and Closing Stock,
the values of which were:
Opening stock Rs. 9,000
Closing stock Rs. 18,000
(ii) Salary includes Rs. 10,000 paid to his brother, which is unreasonable to the extent of Rs. 2,000.
(iii) The whole amount of printing and stationery was paid in cash by way of one time payment.
(iv) The depreciation provided in the Profit and Loss Account Rs. 1,05,000 was based on the following
information:
The written down value of plant and machinery is Rs. 4,20,000 as on 01.04.2020. A new plant falling
under the same block of depreciation was bought on 1.7.2020 for Rs. 70,000. Two old plants were
sold on 1.10.2020 for Rs. 50,000.

(v) Rent and rates includes GST liability of Rs. 3,400 paid on 7.4.2021.
(vi) Other general expenses include Rs. 2,000 paid as donation to a Public Charitable Trust.
You are required to compute the profits and gains of Mr. Sivam under presumptive taxation under section
44AD and profits and gains as per normal provisions of the Act. Assume that the whole of the amount of
turnover received by account payee cheque or use of electronic clearing system through bank account
during the previous year.

Answer
Computation of business income of Mr. Sivam for the A.Y. 2021-22

Particulars Rs. Rs.


Net Profit as per profit and loss account 50,000
Add: Inadmissible expenses/ losses
Under valuation of closing stock 18,000
Salary paid to brother – unreasonable [Section 40A (2)] 2,000

Unique Academy - 8007916622 CA Saumil Manglani - Contact: 9921051593


6. PGBP 6.63
Printing and stationery -whole amount of printing& stationary 23,200
paid in cash would be disallowed, since such amount exceeds Rs.
10,000 [Section 40A (3)]
Depreciation (considered separately) 1,05,000
Short term capital loss on shares 8,100
Donation to public charitable trust 2,000 1,58,300
Less: Deductions items: 2,08,300
Under valuation of opening stock 9,000
Income from UTI [Taxable under IOS] 2,400 11,400
Business income before depreciation 1,96,900
Less: Depreciation (See Note 1) 66,000
1,30,900
Computation of business income as per section 44AD -

As per section 44AD, where the amount of turnover is received, inter alia, by way of account payee
cheque or use of electronic clearing system through bank, the presumptive business income would be
6% of turnover, i.e., Rs. 1,12,11,500 x 6 /100 = Rs.6,72,690
Notes:

1. Calculation of depreciation
Particulars Rs.

WDV of the block of plant & machinery as on 1.4.2020 4,20,000


Add: Cost of new plant & machinery 70,000
4,90,000
Less: Sale proceeds of assets sold 50,000
WDV of the block of plant & machinery as on 31.3.2021 4,40,000
Depreciation @ 15% 66,000
No additional depreciation is allowable as the assessee is not
engaged in manufacture or production of any article.

2. Since GST liability has been paid before the due date of filing return of income under section 139(1),
the same is deductible.
Question 6
Mr. Raju, a manufacturer at Chennai, gives the following Manufacturing, Trading and Profit & Loss
Account for the year ended 31.03.2021:

Manufacturing, Trading and Profit & Loss Account for the year ended
31.03.2021
Particulars Rs. Particulars Rs.
To Opening Stock 71,000 By Sales 2,32,00,000
To Purchase of Raw 2,16,99,000 By Closing 2,00,000
Materials stock

To Manufacturing Wages & 5,70,000


Expenses
To Gross Profit 10,60,000

Unique Academy - 8007916622 CA Saumil Manglani - Contact: 9921051593


6. PGBP 6.64
2,34,00,000 2,34,00,000
To Administrative charges 3,26,000 By Gross Profit 10,60,000
To GST penalty 5,000 By Dividend 15,000
from domestic
companies
To GST paid 1,10,000 By Income from 1,80,000
agriculture (net)
To General Expenses 54,000
To Interest to Bank (On 60,000
machinery term loan)
To Depreciation 2,00,000
To Net Profit 5,00,000
12,55,000 12,55,000

Following are the further information relating to the financial year 2020-21:
(i) Administrative charges include Rs. 46,000 paid as commission to brother of the assessee. The
commission amount at the market rate is Rs. 36,000.
(ii) The assessee paid Rs. 33,000 in cash to a transport carrier on 29.12.2020. This amount is included in
manufacturing expenses. (Assume that the provisions relating to TDS are not applicable to this
payment)
(iii) A sum of Rs. 4,000 per month was paid as salary to a staff throughout the year and this has not been
recorded in the books of account.
(iv) Bank term loan interest actually paid upto 31.03.2021 was Rs. 20,000 and the balance was paid in
November 2021.
(v) Housing loan principal repaid during the year was Rs. 50,000 and it relates to residential property
occupied by him. Interest on housing loan was Rs. 23,000. Housing loan was taken from Canara
Bank. These amounts were not dealt with in the profit and loss account given above.
(vi) Depreciation allowable under the Act is to be computed on the basis of following information:

Plant & Machinery (Depreciation rate @ 15%) Rs.


Opening WDV (as on 01.04.2020) 12,00,000
Additions during the year (used for more than 180 days) 2,00,000
Total additions during the year 4,00,000
Note: Ignore additional depreciation under section 32(1)
(iia)

Compute the total income of Mr. Raju for the assessment year 2021-22.

Note: Ignore application of section 14A for disallowance of expenditures in respect of any exempt income.

Answer
Computation of total income of Mr. Raju for the A.Y. 2021-22
Particulars R R
s s
. .
Profits and gains of business or profession
Net profit as per profit and loss account 5,00
,000
Add: Excess commission paid to brother disallowed under 10,000
section 40A (2)

Unique Academy - 8007916622 CA Saumil Manglani - Contact: 9921051593


6. PGBP 6.65
Disallowance under section 40A (3) is not attracted since the limit for one- Nil
time cash payment is Rs. 35,000 in respect of payment to transport
operators. Therefore, amount of Rs. 33,000 paid in cash to a transport
carrier is allowable as deduction.

Salary paid to staff not recorded in the books (Assuming that the 48,000
expenditure is in the nature of unexplained expenditure and hence, is
deemed to be income as per section 69C and would be taxable @ 60%
under section 115BBE – no deduction allowable in respect of such
expenditure) [See Note 1 below]
Bank term loan interest paid after the due date of filing 40,000
of return under section 139(1) – disallowed as per section 43B
State GST penalty paid disallowed [See Note 2 below] 5,000
Depreciation debited to profit and loss account 2,00,000 3,03,000
Less: Dividend from domestic companies – Taxable under IOS 15,000 8,03,000
Income from agriculture [Exempt under section 10(1)] 1,80,000
Depreciation under the Income-tax Act, 1961
(As per working note) 2,25,000 4,20,000
Income from house property 3,83,000
Annual value of self-occupied property Nil
Less Deduction under section 24(b) – interest on housing loan 23,000 (23,000)
Income From Other Sources
Dividend from domestic companies 15,000
Gross Total Income 3,75,000
Less: Deduction under section 80C in respect of Principal
repayment of housing loan 50,000
Total Income 3,25,000
Working Note:
Computation of depreciation under the Income-tax Act, 1961

Particulars Rs.
Depreciation@15% on Rs. 14 lakhs (Opening WDV of Rs. 12 lakh plus
assets purchased during the year and used for more than 180 days 2,10,000
Rs. 2 lakh)
Depreciation @7.5% on Rs. 2 lakh (Assets used for less than 180 days) 15,000
2,25,000
Notes (Alternate views):

1. It is also possible to take a view that the salary not recorded in the books of account was an erroneous
omission and that the assessee has offered satisfactory explanation for the same. In such a case, the
same should not be added back as unexplained expenditure, but would be allowable as deduction
while computing profits and gains of business and profession.
2. Where the imposition of penalty is not for delay in payment of sales tax or VAT or GST but for
contravention of provisions of the Sales Tax Act or VAT Act or GST Law, the levy is not
compensatory and therefore, not deductible. However, if the levy is compensatory in nature, it would
be fully allowable. Where it is a composite levy, the portion which is compensatory is allowable and
that portion which is penal is to be disallowed.
Since the question only mentions “GST penalty paid” and the reason for levy of penalty is not given,
it has been assumed that the levy is not compensatory and therefore, not deductible. It is, however,
possible to assume that such levy is compensatory in nature and hence, allowable as deduction. In
such a case, the total income would be Rs.3,20,000.
Unique Academy - 8007916622 CA Saumil Manglani - Contact: 9921051593
7. Capital Gains 7.1

Chapter - 7

Unique Academy - 8007916622 CA Saumil Manglani - 9921051593


7. Capital Gains 7.2

REGULATORY FRAMEWORK

Sections (Income Tax Act,


Details
1961)

Section 2(14) Capital assets


Section 2(42A) Short term Capital assets
Section 2(42B) Short term Capital Gains
Section 2(47) Transfer
Section 45(5A) Taxability of capital gains in case of Specified Agreement
Section 46 Capital Gains on distribution of assets by Companies in liquidation
Section 46A Capital Gains on Buy Back of Shares or Specified Securities
Section 48 Mode of Computation of Capital Gains
Section 55 Cost of improvement
Section 49 Ascertainment of Cost in Specified Circumstances
Section 50 Capital Gains in respect of depreciable assets
Section 50B Capital Gains in respect of Slump Sale
Section 2(42C) Meaning of Slump Sale
Section 50C Computation of Capital Gain in real estate transaction
Section 50CA Capital Gain on transfer of unlisted Shares in a Company
Section 50D Fair Market Value to be Full Value of Consideration in Certain Cases
Section 55A Reference to Valuation Officer
Section 51 Advance Money received
Section 54 Profit on sale of property used for residence
Section 54B Transfer of land used for agricultural purposes
Section 54D Compulsory acquisition of lands and buildings
Section 54EC No tax on long-term capital gains if investments made in specified bonds
Section 54EE Tax incentives for Start-ups
Capital gain on the transfer of certain capital assets not to be charged in
Section 54F
case of investment in residential house
Exemption of capital gain on transfer of assets of shifting of industrial
Section 54GA
undertaking from urban area to a Special economic Zone
Section 54GB Capital Gain on transfer of residential Property (a house or a plot of land)
Extension of time for acquiring New asset or depositing or investing
Section 54H
amount of Capital Gain
Section 112A Tax on long-term capital gains in case of specified securities

Impact of Section 115BAC

Under the head Capital Gains, all exemptions and deductions are allowed even under the
New tax system. So computation of Capital Gain Income will not be effected under the new
tax system.
AY 21-22

Unique Academy - 8007916622 CA Saumil Manglani - 9921051593


7. Capital Gains 7.3
1. CAPITAL
GAINS
Sections 45 to 55A of the Income-tax Act, 1961 deal with capital gains.
Section 45 of the Act, provides that any profits or gains arising from the transfer
of a capital asset effected in the previous year shall, save as otherwise provided
in various sections of Sec. 54, be chargeable to income-tax under the head
“Capital Gains” and shall be deemed to be the income of the previous year in
which the transfer took place.

Doubts may arise as to whether “Capital Gains” being a capital receipt can be
brought to tax as income. The constitutional validity of the provisions of
the Act relating to capital gains was challenged in Navin Chandra
Mafatlal v. C.I.T. (1955) 27 ITR 245. The Supreme Court while upholding the
competence of parliament in legislating with regard to capital gains as part of
income, observed that the term income should be given the widest connotation
so as to include capital gains within its scope.

However, all capital profits do not necessarily constitute capital gains. For
instance, profits on re-issue of forfeited shares, profits on redemption of
debentures, premium on issue of shares, are capital profits and not
capital gains, hence, not liable to tax.

The requisites of a charge to income-tax, of capital gains under Section 45(1)


are:
(i) There must be a capital asset.
(ii) The capital asset must have been transferred.
(iii) The transfer must have been effected in the previous year.
(iv) There must be a gain arising on such transfer of a capital asset. These
requisites are briefly analyzed below.
(v) Such capital gain should not be exempt under Sections 54, 54B, 54D, 54EC,
54EE, 54ED, 54F, 54G or 54GA

Unique Academy - 8007916622 CA Saumil Manglani - 9921051593


7. Capital Gains 7.4

2. CAPITAL ASSET [Section 2(14)]

Capital Assets means

any securities held by a foreign


property of any kind held by an But does not
Institutional Investor which has
assessee, include
invested in such securities

whether or not in accordance with the


connected with his regulations made under the
business or profession; SEBI Act, 1992,

any stock-in-trade, consumable stores or raw-materials held for


the purposes of his business or profession;
personal effects that is to say, movable property (including
wearing apparel and furniture but excluding jewellery) held for
personal use by the assessee or any member of his family
dependent on him;
Rural agriculture land excludes :
any area within the jurisdiction of a municipality or a
cantonment board and which has a population of not less
than ten thousand; or
any area within the distance, measured aerially from the
jurisdiction of a municipality or a cantonment board –
not being more than two kilometers, from the local
limits of any municipality or cantonment board and
which has a population of more than ten thousand but
not exceeding one lakh
not being more than six kilometers, from the local limits
of any municipality or cantonment board and which
has a population of more than one lakh but not
exceeding ten lakh
not being more than eight kilometers, from the local
limits of any municipality or cantonment board and
which has a population of more than ten lakh.
6 per cent Gold Bonds, 1977 or 7 per cent Gold Bonds,
1980 or national Defense Gold Bonds, 1980 issued by the
Central Government;
Special Bearer Bonds 1991 issued by the Central Govt.
Gold Deposit Bonds issued under the Gold Deposit Scheme,
1999

The Supreme Court in the case of Vodafone International Holdings B.V vs. union of India
[2012] 204 Taxman 408 held that influence/persuasion of a parent company over its
subsidiary could not be construed as a right in the legal sense.
To supersede this ruling with retrospective effect from 1st April 1962, an explanation has been
inserted to clarify that “property” includes and shall be deemed to have always included any
rights in or in relation to an Indian company, including rights of management or control or any
other rights whatsoever.

Unique Academy - 8007916622 CA Saumil Manglani - 9921051593


7. Capital Gains 7.5
3. SHORT-TERM & LONG-TERM ASSETS

• Section 2(42A) defines short term capital asset as a capital asset held by the
assessee for not more than 36 months immediately preceding the date of
transfer. Therefore, an asset which is held by the assessee for period of > 36
months immediately preceding the date of transfer is a long- term capital asset.

• However, a security (other than a unit) listed in a recognised stock exchange or a unit
of an equity oriented fund, or of UTI or a Zero-Coupon Bond, will be considered as a
long-term asset if it is held for period of > 12 months immediately preceding the date
of transfer.

• A share of a company not being a share which is listed on a recognised stock


exchange in India, would have a holding period of 24 months.

• Capital asset, being Immoveable property (land or building or both) is transferred on


or after April 1, 2017, then it will be treated as Long Term Capital Asset if it is held
for more than 24 months immediately prior to the date of its transfer. [Amendment
vide Finance Act, 2017 w.e.f. AY 2018-19]

• Assets other than short-term capital assets are known as ‘long-term capital assets’
and the gains arising therefrom are known as ‘long-term capital gains’.

In determining the period for which a capital asset is held by an assessee, the following
must be noted:

(i) In the case of shares held in a company in liquidation, the period subsequent to the
date on which the company goes into liquidation shall be excluded;

(ii) In the case of the shares in an Indian Company which become the property of the
assessee in a scheme of amalgamation, the period for which the shares in the
amalgamating company were held by the assessee shall be included;

(iii) In the case of a capital asset, being a share or shares in an Indian company, which
becomes the property of the assessee in consideration of a demerger, there shall be
included the period for which the share or shares held in the demerged
company were held by the assessee;

(iv) Where preference shares are converted into equity shares, the period of holding
shall be considered from the date of acquisition of preference shares. Cost of
acquisition of preference shares shall be taken as cost of

acquisition of equity shares in the hands of assessee. [Amendment vide Finance Act,
2017 w.e.f. AY 2018-19]

Unique Academy - 8007916622 CA Saumil Manglani - 9921051593


7. Capital Gains 7.6
4. TRANSFER

The essential requirement for the incidence of tax on capital gains is the
transfer of a “capital asset”. The Act contains an inclusive definition of
“transfer”, and hence, it includes:

The distribution of capital assets on the dissolution of a firm, body of individuals or other
association of persons, is also regarded as transfer liable to capital gains tax. For the purposes
of computing capital gain in such cases, the fair-market value of the capital asset on the date of
such distribution will be deemed to be the full value of consideration received or
accruing as a result of transfer of the capital asset.

5. What doesn’t constitute Transfer?


Section 47 specifies certain transactions which will not be regarded as a transfer, as below:
Transactions not regarded as transfer [Section 47]: Some Examples
1. Any distribution of capital assets on the total or partial partition of a HUF

2. Any transfer of capital asset under a gift or will or an irrevocable trust

3. Any transfer of capital asset by a holding company to its 100% subsidiary Indian company or by
a subsidiary company to its 100% holding Indian company

4. Any transfer of capital asset by amalgamating company to amalgamated company if


amalgamated company is an Indian company

5. Any transfer of capital asset by demerged company to resulting company if resulting company
is an Indian company

6. Any transfer or issue of shares by the resulting company, in a scheme of demerger to the
shareholders of the demerged company

7. Any transfer by a shareholder in a scheme of amalgamation of shares held by him in the


amalgamating company

Unique Academy - 8007916622 CA Saumil Manglani - 9921051593


7. Capital Gains 7.7
8. Any transfer, made outside India, of a capital asset being rupee denominated bond of an
Indian company issued outside India, by a non-resident to another non- resident

9. Any transfer of a capital asset, being a Government Security carrying a periodic payment
of interest, made outside India through an intermediary dealing in settlement of securities,
by a non-resident to another non-resident

10. Any transfer by an individual of sovereign gold bonds issued by RBI by way of redemption

11. Any transfer by way of conversion of bonds, debentures, debenture stock, deposit
certificates of a company, into shares or debentures of that company.
12. Any transfer by way of conversion of preference shares of a company into equity shares
of that company
13. Any transfer of specified capital asset the University or the National Museum, National Art
Gallery, National Archives or any other public museum or institution notified by the Central
Government to be of national importance or to be of renown throughout any State – work of art,
archaeological, scientific or art, collection, book, manuscript, drawing, painting, photograph or
print.
14. Any transfer of a capital asset in a transaction of reverse mortgage under a scheme made
and notified by the Central Government

15. Transfer by a unit holder under consolidation plans / schemes of Mutual Fund

Note – Regarding Cost of assets


For points 1 -2 - 3 – 4 – 5 - Cost of asset is to be considered as cost to the previous owner

For points 7 -11- 12-15 - Cost of the previous asset is to be treated as the cost of the newly acquired asset.

Cost of acquisition of shares received in the resulting company, in the scheme of demerger: In the
case of a demerger, the cost of acquisition of the shares in the resulting company shall be the amount
which bears to the cost of acquisition of shares held by the assessee in the demerged company the same
proportion as the net book value of the assets transferred in a demerger bears to the net worth of the
demerged company immediately before such demerger [Section 49(2C)].

B
Cost of acquisition of shares in the resulting company = A x
C

A = Cost of acquisition of shares held in the demerged company B = Net book value of the assets
transferred in a demerger
C = Net worth of the demerged company i.e. the aggregate of the paid up share capital and general reserves
as appearing in the books of account of the demerged company immediately before the demerger.

Cost of acquisition of the shares held in the demerged company: The cost of acquisition of the original
shares held by the shareholder in the demerged company shall be deemed to have been reduced by the
amount as so arrived under the sub-section (2C) [Section 49(2D)].

Unique Academy - 8007916622 CA Saumil Manglani - 9921051593


7. Capital Gains 7.8

6. Year of Chargeability as “Capital Gains”

Sec. Profits or gains arising from the P.Y. in which Deemed Full Value of
following transactions chargeable as income is consideration (FVC)
income chargeable to tax u/s 48

45(1A) Money or other asset received under an The P.Y. in which such The value of money or
insurance on account of damage / money or other asset the FMV of other asset on
destruction of any capital asset, as a result is received. the date of receipt.
of, flood, typhoon,hurricane, cyclone,
earthquake, riot, civil disturbance
accidental fire, explosion, enemy
actionetc.
45(2) Transfer by way of conversion of a The P.Y. in which such The FMV of the capital
capital asset into stock-in-trade (SIT) of SIT is sold or asset on the date of
a business carried on by him otherwise such conversion.
transferred.

Components of income Manner of


arising on subsequent Computation of capital
sale of stock-in-trade gains and business
income

FMV on the date of


conversion (-)
Cost/Indexed Cost of
acquisition/
improvement

Capital Gains
Indexation benefit
would be considered in
relation to the year of
conversion of capital
Conversion of capital asset into stock-in-
asset into stock-in- trade
trade

Sale price of stock-in-


trade (-) FMV on the
Business date of conversion
Income

Unique Academy - 8007916622 CA Saumil Manglani - 9921051593


7. Capital Gains 7.9

Sec. Profits or gains arising from P.Y. in which Deemed Full Value of
the following transactions income is consideration (FVC)
chargeable as income chargeable to tax for computation of
CG u/s 48
45(3) Transfer of a capital asset by a The P.Y. in which The amount recorded
person to a firm or other AOPs or such transfer in the books of
BOIs in which he is or becomes a takes place. account of the firm,
partner or member, by way of capital AOPs or BOIs as the
contribution or otherwise. value of the capital
asset.
45(4) Transfer of a capital asset by way of The P.Y. in which The FMV of the
distribution of capital assets on the such transfer capital asset on the
dissolution of a firm or other AOPs or takes place. date of such transfer.
BOIs or otherwise, is chargeable to
taxes the income of the firm, AOPs or
BOIs.
45(5) Transfer of capital asset by way of The P.Y. in which Compensation or
compulsory acquisition under any the consideration consideration
law, or a transfer, the consideration or part thereof is determined or
for which was determined or first received. approved in the first
approved by the Central instance by the
Government or RBI. Central Govt. or RBI.
If the compensation or consideration The P.Y. in which Amount by which the
is further enhanced by any court, the amount was compensation/
Tribunal or other authority, the received by the consideration is
enhanced amount deemed to be assessee. enhanced or further
the income. enhanced. For this
However, compensation received in purpose, cost of
pursuance of an interim order of a acquisition and cost of
court/Tribunal deemed to be income improvement shall be
of the P.Y. in which the final order is taken as ‘Nil’.
made.
45(5A) Transfer of a capital asset, being land The P.Y. in which The stamp duty value
or building or both, by an individual the certificate of of his share in the
or HUF, who enters into a specified completion for the project, being L or B
agreement for development of a whole or part of or both, on the date of
project, provided he does not the project is issued issue of completion
transfer his share in project on or by the competent certificate
before the date of issuance of authority (+)
completion certificate. Consideration
received in cash, if
any

Unique Academy - 8007916622 CA Saumil Manglani - 9921051593


7. Capital Gains 7.10

7. Section 45(2A) – Transfer of security in Demat Form -

• Beneficial owner of shares/ securities is chargeable to tax


• FIFO method is applied in case of demat holdings.
• FIFO method is applied account wise

8. Section 46A – Buy Back by company of its own shares/ Securities.

Taxation provisions in respect of buyback effected on or after 5.7.2019


1.

(1) (2) (3) (4)


Taxability in the Buyback of Buyback of Buyback of
hands of shares by shares by a specified
domestic company, other securities by any
companies(Listed/ than a domestic company
Unlisted) company

Company Subject to additional Not taxable to company.


income – tax
@23.296%.
(115QA)
Shareholder/ Income arising to Taxable to Shareholders/ Security Holders
holder of shareholders as capital gains u/s 46A.
specified exempt under
securities section 10(34A)

Unique Academy - 8007916622 CA Saumil Manglani - 9921051593


7. Capital Gains 7.11
[Provided that the provisions of this sub-section shall not apply to such buy-back of shares (being the
shares listed on a recognized stock exchange), in respect of which public announcement has been
made before 5th day of July, 2019]

Case Study
ABC Ltd.; a domestic company purchases its own unlisted shares, on 14th Oct’20. The consideration
for the buyback amounted to INR 50,00,000, which was paid the very same day. The amount received
by the company 2 years ago, for the issue of such shares was INR 27,00,000.
The tax on such buy back was deposited by the company to the credit of the Central Government on
27th Feb’21. You are required to compute the tax and the interest payable.
Solution
Buy Back Consideration 50,00,000

Less: Cost of Shares 27,00,000

net paid for buy-back 23,00,000

Tax payable 4,60,000

SC 55,200

Cess (on Tax + SC) @ 4% 20,608

Total additional tax payable 5,35,808

Additional Tax Payable (rounded off) 5,35,810

Interest payable 21,432

Note: The tax had to be deposited to the credit of the Central Government within 14 days of the payment of
buy-back consideration, i.e., on or before 28th Oct’20. However, the tax was deposited on 27th Feb, and hence
interest for 4 months @ 1% per month is applicable.
9. MODE OF COMPUTATION

Section 48 of the Act provides that the income chargeable under the head
“capital gains” shall be computed by deducting from the full value of
consideration received or accruing as a result of the transfer of the capital
asset the following amounts –

(i) the expenditure incurred wholly and exclusively in connection with such
transfer;
(ii) the cost of acquisition of the capital asset and the cost of any
improvement thereto -
The Finance Act, 1997 has with effect from 1.4.1998 denied the benefit of
indexation of cost of bonds and debentures other than indexed bonds
issued by the government & sovereign Gold Bonds.
Provided also that where shares, debentures or warrants referred to in the
proviso to Clause (iii) of Section 47 are transferred under a gift or an
irrevocable trust, the market value on the date of such transfer shall be
deemed to be the full value of consideration received or accruing as a result
of transfer for the purposes of this section.
The RBI has recently permitted Indian corporate to issue rupee
denominated bonds outside India as a measure to enable Indian
corporate to raise funds from outside India.
Unique Academy - 8007916622 CA Saumil Manglani - 9921051593
7. Capital Gains 7.12

For this purpose: a. “indexed cost of acquisition” means an amount which bears to the cost
of acquisition the same proportion as Cost Inflation Index for the year in which the asset is
transferred bears to the Cost Inflation Index for the first year in which the asset was held by the
assessee or for the year beginning on the 1st day of April 2001 whichever is later;
b. “indexed cost of improvement” means an amount which bears to the cost of
improvement the same proportion as Cost Inflation Index for the year in which the
asset is transferred bears to the Cost Inflation Index for the year in which the
improvement to the asset took place; and
c. “cost inflation index”, in relation to a previous year, means such Index as the
Central Government may, having regard to seventy-five per cent of average rise
in the Consumer Price Index (urban) the immediately preceding previous year to
such previous year, by notification in the Official Gazette, specify in this behalf.
(Refer to CII Table below)
Financial Cost Inf0lation Financial Cost Inflation
Year Index Year Index
2001-02 100 2010-11 167
2002-03 105 2011-12 184
2003-04 109 2012-13 200
2004-05 113 2013-14 220
2005-06 117 2014-15 240
2006-07 122 2015-16 254
2007-08 129 2016-17 264
2008-09 137 2017-18 272
2009-10 148 2018-19 280
2019-20 289 2020-21 301

Cost of acquisition shall have to be adjusted by the Cost Inflation Index


to arrive at the indexed cost of acquisition.
Note:
• If any assesseee/previous owner has acquired capital asset prior to
1-4- 2001 then he will have option to choose actual cost of acquisition
or FMV as on 1-4-2001 as his cost of acquisition.
• Cost of improvement incurred by assessee or previous owner prior to
1-4-2001 shall be taken as NIL. [Amendment vide Finance Act, 2017
w.e.f. AY 2018-19.]
• In case of goodwill, trademark or other intangible assets, the option to
take cost of acquisition or market price which ever is higher is not
available (as on 01.04.2001), irrespective its purchased or self-
generated.
In case of a capital asset being land or building or both, the fair market value of such asset on the 1st day of
April, 2001, shall not exceed the stamp duty value, wherever available, of such asset as on the 1st day of April,
2001.
AY 21-22

Unique Academy - 8007916622 CA Saumil Manglani - 9921051593


7. Capital Gains 7.13

10. Cost of Acquisition of certain assets

Cost of Acquisition Cost of


Capital Asset
Acquired Self - Generated Improvement
Goodwill of a Purchase Price Nil Nil
Business (Not
Profession)
For Cost of Acquisition -> In above cases, FMV as on 1.4.2001 is not available
Note – For Cost of improvement -
a. The Cost of improvement before 1.4.2001 is to be ignored
b. As the cost of improvement before 1.4.2001 is to be ignored, the question of considering
FMV for cost of improvement as on 1.4.2001 does not arise
c. In case of Bonus and Right Shares

Bonus Shares Cost of acquisition


If bonus shares are allotted before 1.4.2001 Fair Market Value on
1.4.2001
If bonus shares are allotted on or after 1.4.2001 Nil
Rights Shares Cost of Acquisition Period of Holding

Original shares (which forms the basis of Amount actually From date of
entitlement of rights shares) paid for acquiring allotment
the original shares
Rights entitlement (which is renounced by the Nil From Date of offer by
assessee in favor of a person) company to date of
renouncement
Rights shares acquired by the assessee Amount actually From date of
paid for acquiring allotment
the rights shares
Rights shares which are purchased by the Purchase price paid From date of
person in whose favor the assessee has to the renouncer of allotment
renounced the rights entitlement rights entitlement +
amount paid to the
company which has
allotted the rights
shares.

Section 49(9)]
Cost of acquisition of a capital asset which was used by the assessee as an inventory
Where the capital gain arises from the transfer of a capital asset which was used by the assessee
as inventory earlier before its conversion into capital asset, the cost of acquisition of such
capital asset shall be the fair market value of the inventory as on the date on such conversion
determined in the prescribed manner.

Unique Academy - 8007916622 CA Saumil Manglani - 9921051593


7. Capital Gains 7.14

Section 2(42A)(ba)
Period of Holding of a capital asset which was used by the assessee as an inventory

Where inventory of business Period from the date of


is converted into or treated as conversion or treatment as
a capital asset by the a capital asset shall be
assessee considered.
Illustration 1:
Mr. Nagendra Kumar converts his capital asset acquired for an amount of INR 125000 in
2005-06, into stock in trade in the FY 2016-17. He thereafter sells this asset for INR
10,00,000 in 2020-21.FMV as on the date of conversion is Rs. 7.5 lacs
Please advise on the taxability.
Solution
It is to be noted that when the owner of a capital asset, converts it in to stock in trade, the
“capital gains” arises in the year of such conversion, that is regarded as transfer, i.e., in the
year in which the transfer (conversion) was effected. Therefore, in this case the Capital Gains
arises in FY 2016-17. However, the same will be taxable only in the year the asset was sold,
and along with it shall be chargeable to tax, the relevant income under the head “Profits / Gains
from Business / Profession”.
Particulars INR
FMV of asset on date of conversion 7,50,000
Indexed Cost of acquisition 2,82,051
Capital Gain arises in FY 2016-17 4,67,949
finally sold for 10,00,000
Cost of Acquisition 7,50,000
Business Income arises in FY 2019-20 2,50,000

Note:
1) Indexed Cost of Acquisition is INR 125000/117*264 (refer to the CII table for the indices of
respective years)
2) The Capital Gain arises in FY 2016-17
3) The Capital Gain is taxable only in FY 2020-21 when the asset is sold, along with the
Business Income
Illustration 2:
Mr. Srinivasan, purchases 2000 equity shares in ABC Ltd., for INR 50 per share (Brokerage 1%),
in Feb 1997. He gets 200 Bonus shares in Sep 2000. He again gets 2200 bonus shares in Sep
2007. FMV of the Shares on 1st Apr’01 was INR 125.
In Jan’21, he sells all the shares for INR 500 per share (Brokerage 2%).
Compute the Capital Gains Tax in the hands of Srinivasan in FY 2020-21.

Unique Academy - 8007916622 CA Saumil Manglani - 9921051593


7. Capital Gains 7.15

Solution
Cost of Acquisition Nos. Per Share Total INR
Original 2,000 50.50 101000
Bonus Shares prior to 1st 200 - 25000
Apr’01
Bonus Share post 1st Apr’01 2,200 - -
full value of Consideration 4,400 490.00 21,56,000
Indexed Cost of Acquisition
Original (2000 x 125 x ( 301 7,52,500
)/100)
Bonus (200 x 125 x 301 7,5,250 8,27,750
/100)
Capital Gains (Long Term) 13,28,250

Note:
– The brokerage is netted against the costs and sales vales
– The Cost of Acquisition of Bonus Shares acquired prior to 1st Apr’01 is the FMV on 1st
Apr’01 and for the ones acquired post 1st Apr’01 is NIL
– For the Original Shares, since the acquisition cost (INR 50) is less than the FMV (INR
125) as on 1st Apr’01, the FMV as on 1st Apr’01 is considered for computing the indexed
cost of acquisition
11. Cost of Improvement
Section 55 mentions that in relation to a capital asset, being goodwill, or a right, the cost of
improvement will be taken as NIL.
For any other capital asset:
a) Cost of improvement, prior to 1st Apr’ 01 shall be Nil
b) Cost of improvement shall be all expenditure of a capital nature, incurred in making
additions / alterations on or after 01.04.2001.
Illustration 3:
M & sons, HUF, had purchased a land for INR 150,000 in 2002-03. In the PY 2006-07, a
partition takes place and the Coparcener, Mr. B, gets this plot, valued at INR 200,000. In PY
2007-08, he incurs expenses of INR 250,000 on the plot towards fencing of the plot of land.
Mr. B then sells this plot at INR 15,00,000 in PY 2020-21. You are required to compute the
capital gains for AY 2021-22.
Solution:

Cost of acquisition 1,50,000


Cost of Improvement 2,50,000
full Value of Consideration 15,00,000
Indexed Cost
Acquisition 4,30,000

Improvement 5,83,333

Capital Gains 4,86,667

Unique Academy - 8007916622 CA Saumil Manglani - 9921051593


7. Capital Gains 7.16
Note:
a) Although the cost of acquisition for the land, in case of partition of HUF would be the cost to
the previous owner, the year would be the year in which he gets the asset upon partition,
that is FY 2006-07

b) Indexed Cost of Acquisition therefore is INR (150,000/105) * 301 = 430000


c) Indexed Cost of Improvement therefore is INR (250,000/129) * 301 = 583333
Illustration 4:
Mr. X purchases a property for INR 50000 on 3rd May 1975. The following expenses were
incurred by him: Improvement of property in 1998-99 INR 250,000
o Construction of two floors in 2002-03 INR 800,000

o Reconstruction and refurbishment of property in 2012-13 INR 15,00,000


FMV of property on 1st Apr’01 is 10,00,000. He sells the house on 9th Sep’20 for INR 80,00,000
and incurs INR 250,000 on transfer. Compute the Capital Gains taxable in his hands in AY
2021-22.
Solution:
INR
Full Value of Consideration 77,50,000
Less Indexed Cost of Acquisition[(FMV as 30,10,000
on 01.04.2001 or cost of acquisition
whichever is higher i.e. 10,00,000*301/100]
Indexed Cost of Improvement 22,93,333
[800000*301/105]
- Indexed Cost of Improvement 22,57,500
[1500000*301/200]
Capital Gains 1,89,167

Note:
The improvement to the property only after 1st Apr’01 are considered. The costs are
indexed using the indices from the CII table.
12. Capital Gains for Depreciable Assets (Section 50)

– If the Full Value of Consideration is > than the WDV of the Block of Assets, then the
differential is the Short-Term Capital Gain (STCG)
– If the full Value of Consideration falls short of the WDV and the Block continues to exist,
the differential is the WDV and if it doesn’t exist, the differential becomes the Short-
Term Capital Loss (STCL)
13. Capital Gains in respect of Slump Sale (Section 50B)

Meaning of slump sale [Section 2(42C)]


Slump sale’ means the transfer of one or more undertakings as a result of the sale for a lump
sum consideration without values being assigned to the individual assets and liabilities in such
sales. In other words it is a sale where the assessee transfers one or more undertaking as a
whole including all the assets and liabilities as a going concern. The consideration is fixed for
the whole undertaking and received by the transferor it is not fixed for each of the asset of the
undertaking as a whole by way of such sale.Thus it may be noted that the undertaking as a
Unique Academy - 8007916622 CA Saumil Manglani - 9921051593
7. Capital Gains 7.17
whole or the division transferred shall be a capital asset.
– Any gains arising from the slump sale of one / more undertakings held for more than 36
months, shall be chargeable to tax as Long-Term Capital Gains in the Previous Year in
which the slump sale was effected
– Any gains arising from the slump sale of one / more undertakings held for less than 36
months, shall be chargeable to tax as Short-Term Capital Gains in the Previous Year in
which the slump sale was effected
– The Net Worth would be used for the purposes of Cost of Acquisition / Improvement of
the unit(s)
– Every assessee, in the case of slump sale, shall furnish a report of a Chartered accountant before the
specified date referred to in section 44AB indicating the computation of the net worth of the undertaking or
division, as the case may be, and certifying that the net worth of the undertaking or division, as the case
AY 21-22 may be, has been correctly arrived at in accordance with the provisions of this section. [Amended by
Finance Act, 2020]

Illustration 5:
Mohan is the proprietor of Photo film Agencies which has 2 units, one for printing and the
other for binding. He transferred, by way of slump sale, one of the units (unit 2) on 1st Apr’20,
for a total consideration of INR 50,00,000. expenses on sale were 0.5%. This unit was started
in the year 2012-13.

Appended below is the Balance Sheet


Particulars Unit 1 Unit 2
Building 7,50,000 15,00,000
Machinery 5,00,000 10,00,000
Debtors 2,50,000 5,00,000
Other Assets 1,00,000 2,00,000
16,00,000 32,00,000
Capital 40,00,000
Revaluation Reserve 2,50,000
Bank Loan 3,50,000
Creditors 2,00,000
48,00,000

The Revaluation Reserve was created by upward revaluation of Buildings in unit 2. Other
Assets of unit 2 include, INR 100,000 of Patents acquired on 1st Jul’16, on which no
depreciation has been charged. 75% of Creditors and 25% of Bank Loan is for unit 2.
Compute the Capital Gains on the slump sale for AY 2021-22.
Solution:
The Capital Gains from Slump Sale are as under:
full Value of Consideration 50,00,000
expenses on Sale 25,000
net Sale Consideration 49,75,000
net Worth 26,44,141
Long Term Capital Gains 23,30,859
Unique Academy - 8007916622 CA Saumil Manglani - 9921051593
7. Capital Gains 7.18

Working Notes
1. The first note details the computation of net worth for unit 2

Net Worth
Buildings 15,00,000
Less : Reval Reserve 2,50,000
net for 12,50,000
Buildings
Machinery 10,00,000
Debtors 5,00,000
Other Assets 1,00,000
Patents 31,641
Total Assets 28,81,641
Less: Creditors 1,50,000
Bank Loan 87,500
Total Liabilities 2,37,500
net Worth 26,44,141
2. The second note details the valuation of WDV of Patents
Patents
Op Block 1,00,000
Dep Y1 25,000
WDV End of Year 1 75,000
Dep Y2 18,750
WDV End of Year 2 56,250
Dep Y3 14,063
WDV End of Year 3 42,188
Dep Y4 10,547
WDV End of Year 4 31,641

14. Computation of Capital Gain in Real Estate Transaction [Section 50C]


Section 50C makes a special provision for determining the full value of consideration in
cases of transfer of immovable property. It provides that where the consideration declared
to be received or accruing as a result of the transfer of land or building or both, is less
than the value adopted or assessed by any authority of a State Government (i.e. “stamp
valuation authority”) for the purpose of payment of stamp duty in respect of such transfer,
the value so adopted or assessed shall be deemed to be the full value of the
consideration, and capital gains shall be computed on the basis of AYsuch
21-22 consideration
under Section 48 of the Income-tax Act.
However, where the stamp duty value does not exceed 110% of the sale
consideration received or accruing as a result of the transfer, the consideration so
received or accruing shall be deemed to be the full value of the consideration.
(Amended by the Finance Act, 2020)

Unique Academy - 8007916622 CA Saumil Manglani - 9921051593


7. Capital Gains 7.19

Rationalization of section 50C in case sale consideration is fixed


under agreement executed prior to the date of registration of
immovable property (w.e.f. AY 2017-18)
– Section 50C of the Act has been amended in line with section 43CA to
provide that where the date of the agreement fixing the amount of
consideration and the date of registration for the transfer of the capital
asset are not the same, the value adopted or assessed or assessable by
the stamp valuation authority on the date of agreement may be taken for
the purposes of computing full value of consideration for such transfer.
– It is further provided that this provision shall apply only in a case where the
amount of consideration referred to therein, or a part thereof, has been
received by way of an account payee cheque or account payee bank draft
or by use of electronic clearing system through a bank account or through
such other electronic mode as may be prescribed, on or before the date of
the agreement of transfer

15.Capital Gain on Transfer of Unlisted Shares in a Company [Section 50CA]


If an assessee transfers shares in a company (other than quoted shares)
at less than the fair market value, In such case, the FMV of such shares
shall deemed to be the full value of consideration.

The above provision should not apply if the transfer of such unquoted
shares have taken by the approval of Tribunal on application moved by
CG and the old BOD have been removed and new management has been
appointed.
16.Fair Market Value to be Full Value of Consideration in Certain Cases (Section 50D)
Where the consideration in respect of transfer of an asset is not
determinable under the existing provisions of the Income-tax Act, then, as
the machinery provision fails, the gains arising from the transfer of such
assets is not taxable.
– Section 50D has been inserted to provide that fair market value of the
asset on the date of transfer shall be deemed to be the full value of
consideration if actual consideration is not attributable or determinable.
17.REFERENCE TO VALUATION OFFICER (SECTION 55A)

With a view to ascertaining the fair market value of a capital asset, the concerned
Assessing Officer may refer the valuation of the capital asset to a Valuation Officer
appointed by the Income-tax Department in the following cases:

(a) Where the value of the asset as claimed by the assessee is in accordance with the estimate
made by a registered valuer but the Assessing Officer is of the opinion that the value so
claimed is less than its fair market value the Assessing Officer is enabled to make a
reference to the Valuation Officer where in his opinion the value declared by the assessee is
at variance from the fair market value [Section 55A(a)].

Unique Academy - 8007916622 CA Saumil Manglani - 9921051593


7. Capital Gains 7.20

(b) Where the Assessing Officer is of the opinion that the fair market value of the asset
exceeds the value of the asset by more than Rs. 25,000 or 15 per cent of the value
claimed by the assessee whichever is less [Section 55A(b)(i) read with Rule 111AA].
(c) Where the Assessing Officer is of the opinion that, having regard to the nature of an asset
and relevant circumstances, it is necessary so to make a reference to the Valuation
Officer [Section 55A(b)(ii)].
Illustration 6:
A has the following incomes for the previous year 2020-21:
Rs.
Business income (--) 30,000 (viz. business loss of 30,000)
Short-term capital gains 6,000
Long-term capital gains 1,90,000
A deposits Rs. 9,000 in public provident fund account. You are required to find out his tax liability
for the assessment year 2021-22.
Solution:
Computation of net Income of Mr. A for the assessment year 2021-22
Rs. Rs.
Business Income (-) 30,000
Capital gain
– Short-term 6,000
– Long-term 1,90,000 1,96,000
1,66,000
Computation of Tax Liability for the assessment year 2020-21
Tax on Total income nil
Notes :
1. If the net income (other than long-term capital gain) is below the amount of first
slab which is not taxable (i.e. Rs. 2,50,000), then the long-term capital gain is
to be reduced by the amount by which the total income (other than long-term
capital gain) falls short of the maximum amount which is not chargeable tax.
2. In this case, net income (other than long-term capital gain is (-) Rs. 24,000
which will be allowed for carried forward i.e. not to be deducted from Rs.
2,50,000. Therefore, long-term capital gain shall be reduced by Rs. 2,50,000.
Thus, no tax shall be leviable on long-term capital gain.

Unique Academy - 8007916622 CA Saumil Manglani - 9921051593


7. Capital Gains 7.21
18. ADVANCE MONEY RECEIVED [SECTION 51]

Tax treatment of advance money


forfeited on failure of negotiations
for transfer of a capital asset
[Sections 51 & 56(2)(ix)]

If advance was If advance was


received and received and
forfeited before forfeited on or
after 1-4-2014
1-4-2014

Advance forfeited to be
deducted while determining Advance forfeited to be taxed
Cost of acquisition for under 56(2)(ix) as
computing capital gains income from other sources

Taxability is postponed to the year


Tax liability is attracted in the
of actual transfer of capital asset.
year of forfeiture of advance

Note – IMP

✓ The advance forfeited upto 31.3.2014 is to be adjusted from the actual cost if the amount
forfeited by the Assessee and the person selling the asset are the same.
✓ If advance has been received and retained by the previous owner and not the assessee himself, then
the same will not go to reduce the cost of acquisition of the assessee

19. EXEMPTION OF CAPITAL GAINS

Unique Academy - 8007916622 CA Saumil Manglani - 9921051593


7. Capital Gains 7.22

Relevant for Section 54 mentioned in the next page

'Provided that where the amount of the capital gain does not exceed two crore rupees, the
assessee may, at his option, purchase or construct two residential houses in India, and where
such option has been exercised,—
(a) the provisions of this sub-section shall have effect as if for the words "one
residential house in India", the words "two residential houses in India" had been substituted;
(b) any reference in this sub-section and sub-section (2) to "new asset" shall be
construed as a reference to the two residential houses in India:

Provided further that where during any assessment year, the assessee has exercised the option
referred to in the first proviso, he shall not be subsequently entitled to exercise the option for the
same or any other assessment year.

Unique Academy - 8007916622 CA Saumil Manglani - 9921051593


7. Capital Gains 7.23

S.No. Particulars Section 54 Section 54B Section 54D Section 54EC 54EE Section 54F
1 Eligible Assessee Individual/HUF Individual/HUF Any assessee Any assessee Any assessee Individual/HUF
2 Asset transferred Residential House (LTCA) Urban Agricultural Land Land & building Long term Landor Any LTCA Any LTCA other than
forming part of an Residential House
Building or both
industrial undertaking (AY 19-20)
3 Other Conditions Income from such house Land has been used for L & B have been used for - - Assessee should not own
should be chargeable agricultural purposes by business of undertaking more than one residential
under the head "Income assessee or his parents for at least 2 years house on the date of
from house property". or HUF for 2 years immediately preceding transfer. He should not
immediately preceding the date of transfer purchase within 2 years
the date of transfer. or construct within 3 years
The transfer should be by after the date of transfer,
way of compulsory another residential house.
acquisition of the
industrial undertaking
4 Qualifying asset i.e. asset One Residential House Land for being used for Land or Building or right Bonds of NHAI or RECL Unit issued before One Residential House
in which capital gains has situated in India agricultural purposes. in land or building of or any other bond notified 1.4.2019 of Specified situated in India.
to be invested purchased by assessee industrial undertaking by Central Govt. (PFC, Fund notified by Central
or Legal Representatives IRFC) (Redeemable after Govt.
5 years)
5 Time limits for purchase / Purchase within 1 year Purchase with period of 2 Purchase construct within Purchase with a period of Purchase within a period Purchase within 1 years
construction before or 2 years after the years after the date of 3 years after the date of 6 months after the date of of 6 months after the date before or 2 years after the
date of transfer transfer. transfer. for shifting or re- transfer of such transfer. date of transfer or
(or) establishing the existing Construct within 3 years
construct with 3 years undertaking or setting up after the date of transfer.
after the date of transfer a new industrial
undertaking
6 Amount of Exemption Cost of new Residential Cost of new Agricultural Cost of new asset or CG, CG or amount invested in CG or amount invested in Cost of new Residential
House or CG, whichever Land or CG, whichever is whichever is lower. specified bonds, notified units of specified House > Net sale
is lower, is exempt. lower, is exempt. whichever is lower. fund, whichever is lower. consideration of original
Maximum permissible Maximum permissible asset, entire CG is
investment out of CG investment in such units exempt.
arising in any FY is Rs. 50 out of CG arising in any Cost of new Residential
lakhs, whether such FY is Rs. 50 lakhs, House < Net sale
investment is made in the whether such investment consideration of original
current FY or next FY or is made in the current FY asset, proportionate CG is
both. or next FY or both. exempt.
7 Lock in period of not 3 years from date of 3 years from date of 3 years from date of 5 years form the date of 3 years from date of 3 years from date of
transferring the newly acquisition/ purchase acquisition acquisition acquisition (Don’t transfer acquisition acquisition/ purchase
acquired asset or convert into money)

Unique Academy - 8007916622 CA Saumil Manglani - 9921051593


7. Capital Gains 7.24

S.no. Particulars 54G 54GA 54GB


1 Eligible Assessee Any assessee being an Any assessee being an Individual/ HUF
Industrial undertaking Industrial undertaking
2 Asset transferred Land, Building, P&M Land, Building, P&M Long term residential
property (house or plot of
land)
3 Other Conditions Transferred from Urban Transferred from For eligible start up – Long
area to Non-Urban area Urban area to SEZ term residential property
should be transferred upto
31.3.21

4 Qualifying asset Land, Building, P&M + Land, Building, P&M + Subscribe to > 25% of equity
i.e. asset in which Shifting charges Shifting charges shares of start- up company
capital gains has upto 139(1)
to be invested & then Company should use
the amount for purchasing
P&M within one year from
the date of subscription in
equity shares.
5 Time limits for 1 year before or 3 years 1 year before or 3 years New assets should be
purchase / after date of transfer after date of transfer purchased by due date of
construction return filing
6 Amount of Cost of new asset or CG, Cost of new asset or Amount of CG x (Amount
Exemption whichever is lower. CG, whichever is lower. Invested/ Net Consideration)
7 Lock in period of 3 years from date of 3 years from date of 5 years from date of
not transferring acquisition/ purchase acquisition/ purchase acquisition/ purchase
the newly (Shares & P&M)
acquired asset
However in case of a new
asset, being computer or
computer software the lock
in period is 3 years

Unique Academy - 8007916622 CA Saumil Manglani - 9921051593


7. Capital Gains 7.25

25%

Unique Academy - 8007916622 CA Saumil Manglani - 9921051593


7. Capital Gains 7.26

20. Few other pointers


Capital Gains Account Scheme (CGAS)
Under sections 54, 54B, 54D, 54F, 54G and 54GA, capital gains is exempt to the extent of
investment of such gains/ net consideration (in the case of section 54F) in specified assets within

the specified time. If such investment is not made before the date of filing of return of income,
then the capital gain or net consideration (in case of exemption under section 54F) has to be
deposited under the CGAS.
Time limit
Such deposit in CGAS should be made before filing the return of income or
on or before the due date of filing the return of income, whichever is earlier.
Consequences if the amount deposited in CGAS is not utilized within the stipulated time
of 2 years / 3 years
If the amount deposited is not utilized for the specified purpose within the stipulated
period, then the unutilized amount shall be charged as capital gain of the previous
year in which the specified period expires. In the case of section 54F, proportionate
amount will be taxable.
Consequence of breaching the lock in period i.e. if the assets are sold before the
stipulated time limit
Under section 54,54B, 54D, 54G, 54GA – In these sections the cost of acquisition of new
assets shall be reduced by the amount of capital gains exempt earlier under respective sections.
Under Section 54F –
Consequences where the assessee “purchases” any other residential house within a
period of 2 years or “constructs” any other residential house within a period of 3 years
from the date of transfer of original asset:
The capital gains exempt earlier under section 54F shall be deemed to be taxable as
long-term capital gains in the previous year in which such residential house is purchased
or constructed.
Consequences if the new house is transferred within a period of 3 years from the date of
its purchase
• Capital gains would arise on transfer of the new house; and
• The capital gains exempt earlier under section 54F would be taxable
as long-term capital gains.
Under Section 54EC/EE - Capital Gains exempt earlier shall be shall be LTCG of the PY in
which the asset is transferred.

Extension of Time for Acquiring New Asset or Depositing or Investing Amount of Capital
Gain (Section 54H)
This section states that where the transfer of the original asset is by way of compulsory
acquisition under any law and the amount of compensation awarded for such acquisition is not

Unique Academy - 8007916622 CA Saumil Manglani - 9921051593


7. Capital Gains 7.27

received by the assessee on the date of such transfer, the period of acquiring the new asset
by the assessee referred to in Sections 54, 54B, 54D, 54EC and 54F or for depositing or
investing the amount of capital gain shall be extended. This extended period shall be
reckoned from the date of receipt of such compensation.
Illustration 7

Mr. Khan purchased a residential house in the previous year 2005-06 for rs. 2 crores. The house
property is sold for Rs. 10 crores in the previous year 2020-21 and the capital gain is invested in
two residential house properties worth Rs. 4 crores each. Can he claim the benefit of section 54
in respect of both houses?
Solution :
Exemption under section 54 can be claimed in respect of capital gains arising on transfer of
capital asset, being long-term residential house property. With effect from Assessment Year
2020-21, an assessee has an option to make investment in two residential house properties in
india to claim section 54 exemption. this option can be exercised by the assessee only once in
his lifetime provided the amount of long-term capital gain does not exceed rs. 2 crores. Since,
the gain arising in hands of Mr. Khan is rs. 4.86 crores which is more than rs. 2 crore, he cannot
claim the benefit of section 54 by making investment in both the house properties. However he
can claim the benefit only in respect of one residential property invested.
Computation of Long Term Capital Gains (LTCG)

Sale Consideration 10 Crores

less indexed Cost of acquisition (2*301/117) 5.14 Crores

long term Capital Gains (LTCG) 4.86 Crores

less deduction u/s 54 4 Crores

taxable LTCG 0.86 Crores

21. Tax on long-term capital gains in case of specified securities [Section


- 112A (Finance Act of 2018]

1. Section 112A (Very Important) [New Section Inserted]


a. Applicability
Applicable on sale of equity share listed on a recognized Stock exchange or unit of
equity- oriented fund or unit of business trust, where such sale transaction is
chargeable to securities transaction tax (STT).
b. Conditions:
The conditions for availing the benefit of this concessional rate are–
(a) In case of equity share in a company, STT has been paid on acquisition
and transfer of such capital asset
(b) In case of unit of an equity-oriented fund or unit of business trust, STT
has been paid on transfer of such capital asset.
Further, LTCG arising from transaction undertaken on a recognized stock
exchange located in an International Financial Service Centre (IFSC)
would be taxable at a concessional rate of 10%, where the consideration

Unique Academy - 8007916622 CA Saumil Manglani - 9921051593


7. Capital Gains 7.28

for transfer is received or receivable in foreign currency, even though STT is


not leviable in respect of such transaction.
c. Position before amendment
Prior to 01.04.2018 any LTCG on sale of such specified securities was exempt
under Section 10(38). This exemption has been withdrawn by the finance Act,
2018 w.e.f. Assessment Year 2019-20 and a new section 112A is introduced in the
Income-tax Act.
d. Amount chargeable to tax as per the Finance Act 2018
As per this new section, where the total income of an assessee, includes any
LTCG income [which was earlier exempt under section 10(38) upto 31.03.2018]
shall now be taxed at the rate of 10% on such capital gains exceeding Rs.
1,00,000 i.e. tax will be applicable only on the amount which is over & above
Rs. 1 LacsNote

1. Point of taxation - Section 112A is applicable if Specified long term Capital Asset
is sold on or after 1.4.2018.
If Capital Asset is sold on or after 1st Feb 2018 but upto 31st March 2018, full
exemption u/s 10(38) will be available

e. Cost of Acquisition

The cost of acquisitions for computing LTCG in respect of a listed equity share
acquired by the assessee before February 1, 2018, shall be deemed to be the
“higher” of following:

The actual cost of acquisition of such In case the specified asset


has been acquired before
or 1.4.2001, then FMV as on
1.4.2001 can be considered
as the Cost of Acquisition.

Lower of following:

Fair market value of such shares as on January 31,


2018;

or

Actual sales consideration accruing on its transfer

Please remember – Through above comparison we are just ascertaining the cost. For
calculation of Capital Gains, the cost has to be deducted from the Sale Consideration.

Unique Academy - 8007916622 CA Saumil Manglani - 9921051593


7. Capital Gains 7.29

Illustration
A B C D E F
Cases Actual FMV Sale Lower of Cost of Capital
Cost as on Value FMV & acquisition Gain/ Loss
31.1.18 Sale Value (Higher of
(C – E)
(B&C) D & A)

I 100 200 250 200 200 50


II 100 200 150 150 150 Nil
III 100 50 150 50 100 50
IV 100 200 50 50 100 (50)
[This Capital
Loss can be
set off & C/f
normally]

Meaning of Fair Market value - The highest price of the capital asset quoted on such
exchange on the said date.
f. Few important points
• The benefit of indexation shall not be allowed on such LTCG.
• Deductions under chapter VIA (section 80C to 80U) not to be allowed from such
LTCG.
• Rebate of tax under section 87A not to be allowed from the tax payable on such
LTCG.
• Section 112A is available to all assessee’s including Non-Residents and Foreign
Institutional Investors.
• Section 112A is available to those assessee’s who holds the shares/ units as
capital assets and not as Stock in Trade.
• Losses u/s 112A can be normally carried forward/ set off.
• Losses (LT/ST) can be set off against the LTCG referred to u/s 112A.
Question (Focus Indexation)
Mr. Raman is a salaried employee. In the month of January, 2015 he purchased 100 shares of X
Ltd. @ Rs. 1,400 per share from Bombay Stock Exchange.
These shares were sold through BSE in April, 2020 @ Rs. 2,600 per share. The highest price of X
Ltd. share quoted on the stock exchange on January 31, 2018 was Rs. 1,800 per share. What will
be the nature of capital gain in this case?
Solution: Shares were purchased in January, 2015 and were sold in April, 2020, i.e., sold after
holding them for a period of more than 12 months and, hence, the gain will be long-term capital
gain (LTCG).
In the given case, shares are sold after holding them for a period of more than 12 months, shares
are sold through recognized stock exchange and the transaction is liable to STT. Therefore,
section 112A is applicable in this case.
Unique Academy - 8007916622 CA Saumil Manglani - 9921051593
7. Capital Gains 7.30

The cost of acquisition of X Ltd. Shares shall be higher of:


a) Cost of acquisition i.e., Rs. 1,40,000 (1,400 × 100)
b) Lower of:*
i). Highest price quoted as on 31-1-218 i.e., 1,80,000 (1,800 × 100);
ii). Sales consideration i.e., 2,60,000 (2,600 × 100)
Thus, the cost of acquisition of shares shall be Rs. 1,80,000. Accordingly, Long-term capital gains
in hands of Mr. Raman would be Rs. 80,000 (i.e., 2,60,000 – 1,80,000). Since long-term capital
gains doesn’t exceed Rs. 1,00,000, nothing is taxable in hands of Mr. Raman.
g. Adjustment of Unexhausted Basic Exemption Limit: In the case of resident individuals or
HUF, if the basic exemption is not fully exhausted by any other income, then such long-term
capital gain exceeding Rs. 1 lakh will be reduced by the unexhausted basic exemption limit
and only the balance would be taxed at 10%.

However, the benefit of adjustment of unexhausted basic exemption limit is not available in the
case of non-residents.

Question (ICAI Module)


Mr. Mithun purchased 100 equity shares of M/s Goodmoney Co. Ltd. on 01-04-2006 at rate of
Rs. 1,000 per share in public issue of the company by paying securities transaction tax.
Company allotted bonus shares in the ratio of 1:1 on 01.12.2019. He has also received dividend
of Rs. 10 per share on 01.05.2020.
He has sold all the shares on 01.10.2020 at the rate of Rs. 4,000 per share through a recognized
stock exchange and paid brokerage of 1% and securities transaction tax of 0.02% to celebrate
his 75th birthday. The cost inflation Index are as follows:
Financial Year Cost Inflation Index
2006-07 122
2020-21 301

Compute his total income and tax liability for Assessment Year 2021-22, assuming that he is
having no income other than given above. Fair market value of shares of M/s Goodmoney Co.
Ltd. on 31.1.2018 is Rs. 2,000.
Answer
Computation of total income and tax liability of Mr. Mithun for A.Y. 2021-22
Particulars Rs.
Long term capital gains on sale of original shares
Gross sale consideration (100 x Rs. 4,000) 4,00,000
Less: Brokerage @1% 4,000
Net sale consideration 3,96,000
Less: Cost of acquisition (100 x Rs. 2,000) (Refer note 2) 2,00,000
Long term capital gains 1,96,000

Short term capital gains on sale of bonus shares


Gross sale consideration (100 x Rs. 4,000) 4,00,000
Less : Brokerage @ 1% 4,000
Unique Academy - 8007916622 CA Saumil Manglani - 9921051593
7. Capital Gains 7.31
Net sale consideration 3,96,000
Less: Cost of acquisition of bonus shares NIL
Short term Capital Gains 3,96,000
Dividend Income – IOS (200 shares x 10 rs.) 2,000
Total Income 5,94,000
Tax Liability
15% of (Rs. 3,96,000 - Rs. 2,98,000, being unexhausted basic 14,700
exemption limit)
10% of (Rs. 1,96,000 - Rs.1,00,000) 9,600
24,300
Add : Health and education cess @4% 972
Tax payable 25,270

Notes:
(1) Long-term capital gains exceeding Rs. 1 lakh on sale of original shares through a
recognized stock exchange (STT paid at the time of acquisition and sale) is taxable
under section 112A at a concessional rate of 10%, without indexation benefit.
(2) Cost of acquisition of such equity shares acquired before 1.2.2018 is higher of

- Cost of acquisition i.e., Rs. 1,000 per share and


- lower of
Fair market value of such asset i.e., Rs. 2,000 per share
and Full value of consideration i.e., Rs. 4,000 per share.
So, the cost of acquisition of original share is Rs. 2,000 per share.
(2) Since bonus shares are held for less than 12 months before sale, the gain arising there
from is a short-term capital gain chargeable to tax@15% as per section 111A after
adjusting the unexhausted basic exemption limit. Since Mr. Mithun is over 60 years of
age, he is entitled for a higher basic exemption limit of Rs. 3,00,000 for A.Y. 2021-22.
(3) Brokerage paid is allowable since it is an expenditure incurred wholly and exclusively in
connection with the transfer. Hence, it qualifies for deduction under section 48(i).
(4) Cost of bonus shares will be Nil as such shares are allotted after 1.04.2001.

(5) Securities transaction tax is not allowable as deduction.


Illustration
Suppose the assessee is a Resident Individual aged 50 years, The assessee’s income
consists of PGBP income of Rs 2, 00,000 & sec. 112A income of Rs. 1, 80,000. Find the
amount of tax payable.
Answer
• In this case the Basic Exemption limit will be Rs. 2, 50,000.
• Therefore for tax calculation Rs 50,000 of sec. 112A will be adjusted to exhaust the
difference of Rs. 2, 50,000 & Rs. 2 Lacs (PGBP Income).
• So tax payable = Rs. 1, 80,000 of 112A – Rs. 50,000(adjusted) – Rs. 1,00,000 (as tax is
levied on the amount exceeding Rs. 1,00,000)
i.e. Rs. 30,000 * 10% = Rs. 3000 + 4% HEC = Rs. 3120.

Unique Academy - 8007916622 CA Saumil Manglani - 9921051593


7. Capital Gains 7.32

22. TAX RATES

Short-term Capital Gains (STCG)


– STCG is clubbed with Total Income and therefore charged to tax at normal rates
– However, STCG on transfer of listed equity shares / unit of an equity-oriented
fund / unit of a business trust, where STT has been paid, STCG is taxable @
15% under section 111A.

Long-term Capital Gains (LTCG)

Long term Capital gain


Section 112

Resident N.R. + Foreign


Indi/HUF AOP/BOI/ Firm/Cos. Companies

Normal assets
Assets, other Unlisted
than unlisted securities or
@ 20% Securities or shares of Pvt.
shares of Cos.
ZCB & Listed
securities PVT. Cos.
(other than @ 10%
units) @ 20% (without benefit)
of indexation &
Foreign
lower of
currency
Fluctuation
20% or 10%
without
indexation
.
Note:

• If STT has been paid for listed shares or a unit of equity-oriented fund / business
trust, the LTCG is taxable @10%, if such LTCG is > Rs. 1,00,000 under section 112A.

• STT is not allowed as a deduction in the computation of Capital Gains.

• No deduction under Chapter VI-A against incomes which are taxable at a lower
rate.
Adjustment of LTCG u/s 112, u/s 112A and STCG u/s111A against the basic exemption
limit
Only a resident individual/HUF can adjust the basic exemption limit (i.e. Rs. 2,50,000 or
3,00,000 or Rs. 5,00,000 limits) against LTCG u/s 112, u/s 112A(if amount > 1,00,000)
and
STCG u/s 111A. Thus, a non-resident individual/ HUF cannot adjust their basic
exemption limit (Rs. 2,50,000) against such capital gains.

Unique Academy - 8007916622 CA Saumil Manglani - 9921051593


7. Capital Gains 7.33

Case Study 1

Mr. Kapoor (age 67 years and resident) is a retired person earning total pension of Rs.
1,00,000. He purchased gold in December, 2010 and sold the same in April, 2020.
Taxable LTCG amounted to Rs. 2,80,000. What will be his tax liability for the A.Y. 2021-
22?

Option 1 : assessee has not opted for Section 115BaC


Option 2 : assessee has opted for Section 115BaC

Solution : Computation of tax liability for the A.Y. 2021- 22 is as under :


Option 1 : Assessee has not opted for Section 115BAC
Rs. 1,00,000
Gross Salary (pension income)
Less: Standard deduction u/s 16 (ia) Rs. 50,000

Income under the head Salary Rs. 50,000

LTCG on sale gold Rs. 2,80,000

Total income Rs. 3,30,000

Tax on LTCG u/s 112 on Rs. 30,000 @ 20%* Rs. 6,000

Less : Rebate as per section 87A Rs. 6,000


Tax after Rebate Nil

* Resident individual above 60 years but below 80 years of age has basic exemption limit
of Rs. 3,00,000. Which can be adjusted against LTCG of Rs. 2,80,000 but after the
adjustment of salary income of Rs. 50,000. Hence, the balance LTCG taxable will come
to Rs. 30,000 @ 20%.

Option 2 : Assessee has opted for Section 115BAC


Computation of tax liability for the a/y is as under :
Gross Salary (pension income) less: rs. 1,00,000

Standard deduction u/s 16 (ia) N/a


income under the head Salary rs. 1,00,000

LTCG on sale gold rs. 2,80,000

total income rs. 3,80,000


Tax on LTCG U/s 112 on Rs. 1,30,000 @ 20%* Rs. 26,000

less : rebate as per section 87A rs. 12,500

tax after rebate rs. 13,500


Add: Health and Education Cess @4% on Rs. 13500 Rs. 540
tax liability rs. 13,940

* u/s 115BaC Basic exemption limit is 2,50,000 irrespective of age. unutilised exemption limit of rs.2.50,000
– 1,00,000 i.e. Rs. 1,50,000, can be adjusted against LTCG of rs. 2,80,000. Hence, the balance LTCG taxable
will come to Rs.1,30,000 @ 20%.

Unique Academy - 8007916622 CA Saumil Manglani - 9921051593


7. Capital Gains 7.34

Case Study 2

Mr. Gagan (age 69 years and non-resident) is a retired person earning total pension of Rs.
1,00,000 from Indian employer. He purchased a piece of land in Delhi in December, 2010 and
sold the same in April, 2020. Taxable LTCG amounted to Rs. 2,30,000. What will be his tax
liability for the year 2020-21?

Option 1 : assessee has not opted for Section 115BAC


Option 2 : assessee has opted for Section 115BAC
Solution : Computation of tax liability for the A.Y. 2021 - 22 is as under :
Option 1 : assessee has not opted for Section 115BAC

Gross Salary (pension income) Rs. 1,00,000


Less: Standard deduction u/s 16 (ia) Rs. 50,000
Income under the head Salary Rs. 50,000
LTCG on sale gold Rs. 2,30,000

Total income Rs. 2,80,000


Tax on LTCG u/s 112 on Rs. 2,30,000 @ 20%* Rs. 46,000
Less : Rebate as per section 87A (Not available to non resident) NIL
Tax after Rebate Rs. 46,000
Add : Health & education cess @ 4% Rs. 1,840
Net tax payable Rs. 47,840
*Non-Resident individual (of any age) has basic exemption limit of Rs. 2,50,000. Which
cannot be adjusted against LTCG of Rs. 2,30,000 but the same can be adjusted against
salary income of Rs. 50,000. Hence, the whole amount of LTCG is taxable @ 20%.

Solution : Option 2 : Assessee has opted for Section 115BAC


Computation of tax liability for the a.yr. 2021-22 is as under :
Gross Salary (pension income) less: rs. 1,00,000

Standard deduction u/s 16 (ia) N/a


income under the head Salary rs. 1,00,000
ltCG on sale gold rs. 2,30,000

total income rs. 3,30,000

Tax on LTCG U/s 112 on Rs. 2,30,000 @ 20%* Rs. 46,000


less : rebate as per section 87a (Not available to non-resident) Nil

tax after rebate rs. 46,000

Add : Health & education cess @ 4% Rs. 1840


Net tax payable rs. 47840
*Non-resident individual (of any age) has basic exemption limit of rs. 2,50,000. Which cannot be adjusted against
LTCG of rs. 2,30,000 but the same can be adjusted against salary income of rs. 1,00,000. Hence, the whole amount
of LTCG is taxable @ 20%.

Unique Academy - 8007916622 CA Saumil Manglani - 9921051593


7. Capital Gains 7.35

Case Study 3

Priyanka furnishes the following data for the PY ended 31st Mar’21.
– She had unlisted shares of XYZ Ltd., 100,000 in number, which she sold on 30th Jun’20
for INR 750 per share.
– The above shares were acquired as under:
– Gift from father: 50000 shares on 3rd May’ 99 (FMV on 1st Apr’01 is INR 300)
– Bonus Shares on 21st Jun’ 09: 20000 shares
– Purchased 30,000 shares on 1st Jan’11 @ INR 525 per share
Thereafter, she invested the proceeds to buy a residential house at INR 4,00,00,000 on 3rd
May’21 and she was already owing a residential house prior to the purchase of this one. You are
required to compute the Capital Gains.
Solution :
Particulars Number Rate per Amount Date
Share
Full Value of 1,00,000 750 7,50,00,000 30th Jun’20
Consideration
Less: Indexed COA
Gift from father 50,000 300 4,51,50,000 3rd May’99
Bonus 20,000 0 - 21st Jun’09
Purchased 30,000 525 2,83,87,725 1st Jan’11
Total Indexed COA 7,35,37,725

LTCG 14,62,275

New Residential House 4,00,00,000


LTCG exempt 7,79,880

Taxable LTCG 6,82,395

Note:
a) The bonus shares were granted after 1st Apr’01 and hence the cost of acquisition is NIL
b) For the shares gifted by her father prior to 1st Apr’01, the FMV as on 1st Apr’01 would be
considered as the Cost of Acquisition
c) Indexed COA for the gifted shares therefore is 50000*300/100*301
d) Indexed COA for the acquired shares therefore is 30000*525/167*301
e) In the case above the necessary conditions u/s 54F have been fulfilled and
therefore she is entitled to a proportionate exemption from LTCG (i.e. INR 4
Crores/INR 7.50 Crores * 14,62,275)
Case Study 4

Mrs. Shanti Devi, a resident individual, sold her residential property on 18th Jul20 for
INR 75,00,000. She had purchased the same for INR 25,00,000 on 3rd May 2006. She
paid INR 250,000 towards brokerage for the sale. The stamp duty valuation was INR

Unique Academy - 8007916622 CA Saumil Manglani - 9921051593


7. Capital Gains 7.36

100,00,000.
She bought another property for INR 20,00,000 on 14th Dec’20 and deposited another
INR 5,00,000 on 21st Jun’21 in the capital gain deposit scheme with SBI for construction
of an additional floor in the property.
She also deposited INR 13,50,000 on 30th Nov’20 in the NHAI Bonds.
Compute the Capital Gains and Tax Liability. Assuming not opted for section 115BAC

Solution :
Particulars INR
Full Value of Consideration 1,00,00,000
Brokerage 2,50,000
net Sale Consideration 97,50,000
Indexed Cost ( 25 Lacs/ 122 (59,22,131)
*289)
Gains before Exemptions 38,27,869
exemption u/s 54 (20 + 5) 25,00,000
exemption u/s 54EC 13,50,000
Capital Loss (Total Income) (22,131)
Tax Payable –
Rebate u/s 87A –
Tax Payable –
Cess –
Total Tax Liability –
Notes:

As Per Section 50C, in case the actual sale consideration is less than the stamp duty
value, the stamp duty valuation would be the full value of the consideration. B) Indexed
COA = 25,00,000 / 122*. C) Exemptions u/s 54 are towards purchase of another house
within 2 years of date of transfer and Deposit in Capital Gains Accounts Scheme (CGAS)
on or before the due date of filing return of Income. D) Exemption u/s 54EC is towards
investment in specified bonds (NHAI) within 6 months from date of transfer. E) Since the
Taxable LTCG, which is less than basic exemption limit of Rs. 2,50,000, therefore no tax
will be payable.

Case Study 5

Mr. X purchased house 01.04.2001 rs. 2,00,000 and incurred rs. 3,00,000 on improvement on
01.07.2002 and it was received by his son Mr. y on 01.07.2011 and Mr. y incurred rs. 4,00,000
on improvement 01.07.2013 and house was sold by him on 01.07.2020 for rs. 1,00,00,000.
entiled to deduction u/s 80C of rs,1,00,000. Compute tax liability of Mr. y
Option 1 : assessee has not opted for Section 115BaC
Option 2 : assessee has opted for Section 115BaC
Solution : Option 1 : Assessee has not opted for Section 115BAC
Unique Academy - 8007916622 CA Saumil Manglani - 9921051593
7. Capital Gains 7.37
Full value of consideration 1,00,00,000
less: indexed cost of acquisition = 2,00,000 / 100 x 301 (6,02,000)
less: indexed Cost of improvement = 3,00,000 / 105 x 301 (8,60,000)
less: indexed Cost of improvement = 4,00,000 / 220 x 301 (5,47,273)
long term Capital Gains 79,90,727

Gross total income 79,90,727


less: deduction u/s 80C (1,00,000)
Total Income (rounded off u/s 288A) 78,90,730

Computation of tax liability


Tax on LTCG Rs. 76,40,730 (78,90,730 – 2,50,000) @ 20% 15,28,146
Add: Surcharge @ 10% 1,52,815
tax before health & education cess 16,80,961
Add: HEC @ 4% 67,238
tax liability 17,48,200

Option 2 : Assessee has opted for Section 115BAC

Full value of consideration 1,00,00,000


less: indexed cost of acquisition = 2,00,000 / 100 x 301 (6,02,000)
less: indexed Cost of improvement = 3,00,000 / 105 x 301 (8,60,000)
less: indexed Cost of improvement = 4,00,000 / 220 x 301 (5,47,273)
long term Capital Gains 79,90,727
Gross total income 79,90,727
less: deduction u/s 80C N/a
Total Income (rounded off u/s 288A) 79,90,730

Computation of tax liability


Tax on LTCG Rs. 77,40,730 (79,90,730 – 2,50,000) @ 20% 15,48,146
Add: Surcharge @ 10% 1,54,815
tax before health & education cess 17,02,961
Add: HEC @ 4% 68,118
tax liability (rOuNd OFF) 17,71,080

23.Case Laws
1. Whether, for the purpose of computing the period of holding of the property, the date of
allotment letter issued by the builder of the flat or the date of registration of the property
has to be considered for determining the nature of capital asset – long-term or short-term?
CIT v. S.R. Jeyashankar (2015 Madras HC)

In effect, the P&H HC (in a similar case) held that the allottee gets the title to the property on
issuance of allotment letter and payment in installments is only a consequential act upon which
delivery of possession to the property flows. The Madras HC also noted that the Punjab &

Unique Academy - 8007916622 CA Saumil Manglani - 9921051593


7. Capital Gains 7.38
Haryana HC had taken a similar view in Vinod Kumar Jain’s case Accordingly, the Madras HC held
that the assessee had rightly claimed the benefit of long-term capital gain, since the holding period
exceeded 36 months (i.e., from 22.02.2005, being the date of agreement, to 10.04.2008, being the
date of sale of property).
2. What would be the period of holding to determine whether the capital gains on
renunciation of right to subscribe for additional shares is short-term or long-term?
Navin Jindal v. ACIT (2010) (SC)
For determining whether the capital gains on renunciation of right to subscribe for additional
shares is short term or long-term, the period of holding would be from the date on which such right to
subscribe for additional shares comes into existence upto the date of renunciation of such right.

3. Whether indexation benefit in respect of the gifted asset shall apply from the year in
which the asset was first held by the assessee or from the year in which the same was
first acquired by the previous owner?
CIT v. Manjula J. Shah (2013) (Bom.)
The indexed cost of acquisition in case of gifted asset has to be computed with reference to the year
in which the previous owner first held the asset and not the year in which the assessee became the
owner of the asset.

4.Would an assessee be entitled to exemption under section 54 in respect of purchase of two


flats, adjacent to each other and having a common meeting point? CIT v. Syed Ali Adil
(2013) (A.P.)
The Andhra Pradesh High Court, on the basis of the above rulings of the Karnataka High Court, held
that in this case, the assessee was entitled to investment in both the flats purchased by him, since
they were adjacent to each other and had a common meeting point, thus, making it a single
residential unit.
5. Can exemption under section 54EC be denied on account of the bonds being issued after
six months of the date of transfer even though the payment for the bonds was made by the
assessee within the six month period?
Hindustan Unilever Ltd. v. DCIT (2010) (Bom.)
For the purpose of the provisions of section 54eC, the date of investment by the assessee must be
regarded as the date on which payment is made. The High Court, therefore, held that if such
payment is within a period of six months from the date of transfer, the assessee would be eligible to
claim exemption under section 54eC.
6.Sale of a Running Business with all Assets and Liabilities is a Slump Sale, would not
attract Sec 50(2) of Income Tax Act
CIT v. Equinox Solution Pvt. Ltd
Concurring with the orders of the lower authorities, the bench comprising Justice R.K agarwal and
Justice Ajay Manohar Sapre held that the transaction was rightly treated as slum sale under
section 48. “Section 50 (2) applies to a case where any block of assets are transferred by the
assessee but where the entire running business with assets and liabilities is sold by the assessee
in one go, such sale, in our view, cannot be considered as “short-term capital assets”. In other
words, the provisions of Section 50 (2) of the Income Tax Act would apply to a case where the
assessee transfers one or more block of assets, which he was using in running of his business.
Such is not the case here because in this case, the assessee sold the entire business as a
running concern.”
7. Shankar Dalal & others v/s CIT (Bom.)
Provisions of local land laws should be considered while determining the land as agricultural land.
the Court held that if activities performed by assessee on the land are recognised as ‘agricultural’ activities
under the local land law, then the land can be treated as agricultural land, even if actual growing does
not take place.
Unique Academy - 8007916622 CA Saumil Manglani - 9921051593
7. Capital Gains 7.39

Illustrations for Practice

ILLUSTRATION 1
A is the owner of a car. On 1-4-2020, he starts a business of purchase and sale of motor cars. He
treats the above car as part of the stock-in-trade of his new business. He sells the same on 31-3-
2021 and gets a profit of Rs. 1 lakh. Discuss the tax implication in his hands under the head “Capital
gains”.
SOLUTION
Since car is a personal asset, conversion or treatment of the same as the stock-in- trade of his
business will not be trapped by the provisions of section 45(2). Hence, A is not liable to capital gains
tax.
ILLUSTRATION 2

X converts his capital asset (acquired on June 10, 2002 for Rs. 60,000) into stock-in- trade on March
10, 2020. The fair market value on the date of the above conversion was Rs. 5,50,000. He
subsequently sells the stock-in-trade so converted for Rs. 6,00,000 on June 10, 2020. Discuss the
year of chargeability of capital gain.
SOLUTION
Since the capital asset is converted into stock-in-trade during the previous year relevant to the A.Y.
2020- 21, it will be a transfer under section 2(47) during the P.Y. 2019-20. However, the profits or
gains arising from the above conversion will be chargeable to tax during the A.Y. 2021-22, since the
stock-in-trade has been sold only on June 10, 2020. For this purpose, the fair market value on the
date of such conversion (i.e. 10th March, 2020) will be the full value of consideration.
ILLUSTRATION 3
M held 2000 shares in a company ABC Ltd. This company amalgamated with another company
during the previous year ending 31-3-2021. Under the scheme of amalgamation, M was allotted 1000
shares in the new company. The market value of shares allotted is higher by Rs. 50,000 than the
value of holding in ABC Ltd. The Assessing Officer proposes to treat the transaction as an exchange
and to tax Rs. 50,000 as capital gain. Is he justified?
SOLUTION
In the above example, assuming that the amalgamated company is an Indian company, the transaction
is squarely covered under the provisions explaining “transactions not regarded as transfer” and the
proposal of the Assessing Officer to treat the transaction as a transfer is not justified.
ILLUSTRATION 4
In which of the following situations capital gains tax liability does not arise?
(i) Mr. A purchased gold in 1970 for Rs. 25,000. In the P.Y. 2020-21, he gifted it to his son at
the time of marriage. Fair market value (FMV) of the gold on the day the gift was made was
Rs. 1,00,000.
(ii) A house property is purchased by a Hindu undivided family in 1945 for
Rs. 20,000. It is given to one of the family members in the P.Y. 2020-21 at the time of
partition of the family. FMV on the day of partition was Rs. 12,00,000.
(iii) Mr. B purchased 50 convertible debentures for Rs. 40,000 in 1995 which are converted in
to 500 shares worth Rs. 85,000 in November 2020 by the company.

Unique Academy - 8007916622 CA Saumil Manglani - 9921051593


7. Capital Gains 7.40

SOLUTION
We know that capital gains arise only when we transfer a capital asset. The liability of
capital gains tax in the situations given above is discussed as follows:
(i) As per the provisions of section 47(iii), transfer of a capital asset under a gift is not regarded
as transfer for the purpose of capital gains. Therefore, capital gains tax liability does not arise
in the given situation.
(ii) As per the provisions of section 47(i), transfer of a capital asset (being in kind) on the total or
partial partition of Hindu undivided family is not regarded as transfer for the purpose of capital
gains. Therefore, capital gains tax liability does not arise in the given situation.
(iii) As per the provisions of section 47(x), transfer by way of conversion of bonds or debentures,
debenture stock or deposit certificates in any form of a company into shares or debentures of
that company is not regarded as transfer for the purpose of capital gains. Therefore, capital
gains tax liability does not arise in the given situation.
ILLUSTRATION 5
Mr. Abhishek a senior citizen, mortgaged his residential house with a bank, under a notified
reverse mortgage scheme. He was getting loan from bank in monthly installments. Mr. Abhishek
did not repay the loan on maturity and hence gave possession of the house to the bank, to
discharge his loan. How will the treatment of long-term capital gain be on such reverse mortgage
transaction?
SOLUTION
Section 47(xvi) provides that any transfer of a capital asset in a transaction of reverse mortgage
under a scheme made and notified by the Central Government shall not be considered as a
transfer for the purpose of capital gain.
Accordingly, the mortgaging of residential house with bank by Mr. Abhishek will not be regarded
as a transfer. Therefore, no capital gain will be charged on such transaction.
Further, section 10(43) provides that the amount received by the senior citizen as a loan, either
in lump sum or in installment, in a transaction of reverse mortgage would be exempt from
income-tax. Therefore, the monthly installment amounts received by Mr. Abhishek would not be
taxable.
However, capital gains tax liability would be attracted at the stage of alienation of the mortgaged
property by the bank for the purposes of recovering the loan.
ILLUSTRATION 6
Ms. Usha purchases 1,000 equity shares in X Ltd., an unlisted company, at a cost of
Rs. 30 per share (brokerage 1%) in January 1996. She gets 100 bonus shares in August 2000. She
again gets 1100 bonus shares by virtue of her holding on February 2006. Fair market value of the
shares of X Ltd. on April 1, 2001 is Rs. 80.
On 1st January 2021, she transfers all her shares @ Rs. 200 per share (brokerage 2%). Compute
the capital gains taxable in the hands of Ms. Usha for the A.Y.2021-22 Cost Inflation Index for F.Y.
2001-02: 100, F.Y.2005-06: 117& F.Y.2020-21 = 301.

Unique Academy - 8007916622 CA Saumil Manglani - 9921051593


7. Capital Gains 7.41

SOLUTION
Computation of capital gains for the A.Y. 2020-21
Particulars Rs.
1000 Original shares
Sale proceeds (1000 × Rs. 200) 2,00,000
Less : Brokerage paid (2% of Rs. 2,00,000) (4,000)
Net sale consideration 1,96,000
Less : Indexed cost of acquisition [Rs. 80 × 1000 × 301/100] 2,40,800
Long term capital loss (A) (44,800)
100 Bonus shares
Sale proceeds (100 × Rs. 200) 20,000
Less : Brokerage paid (2% of Rs. 20,000) (400)
Net sale consideration 19,600
Less : Indexed cost of acquisition [Rs. 80 × 100 ×301/100] [See 24,080
Note below]
Long term capital loss (B) (4,480)
1100 Bonus shares
Sale proceeds (1100 × Rs. 200) 2,20,000
Less: Brokerage paid (2% of Rs. 2,20,000) (4,400)
Net sale consideration 2,15,600
Less: Cost of acquisition NIL
Long term capital gain (C) 2,15,600
Long term capital gain (A+B+C) 1,65,780
Note: Cost of acquisition of bonus shares acquired before 1.4.2001 is the FMV as
on 1.4.2001 (being the higher of the cost or the FMV as on 1.4.2001).
ILLUSTRATION 7
On January 31, 2021, Mr. A has transferred self-generated goodwill of his profession for a sale
consideration of Rs. 70,000 and incurred expenses of Rs. 5,000 for such transfer. You are required
to compute the capital gains chargeable to tax in the hands of Mr. A for the A.Y. 2021-22.
SOLUTION
The transfer of self-generated goodwill of profession is not chargeable to tax. It is based upon the
Supreme Court’s ruling in CIT vs. B.C. Srinivasa Shetty.
ILLUSTRATION 8
Mr. R holds 1000 shares in Star Minus Ltd., an unlisted company, acquired in the year 2001-02 at a
cost of Rs. 75,000. He has been offered right shares by the company in the month of August,
2020 at Rs. 160 per share, in the ratio of 2 for every 5 held. He retains 50% of the rights and
renounces the balance right shares in favour of Mr. Q for Rs. 30 per share in September 2020. All
the shares are sold by Mr. R for Rs. 300 per share in January 2021 and Mr. Q sells his shares in
December 2020 at Rs. 280 per share.

Unique Academy - 8007916622 CA Saumil Manglani - 9921051593


7. Capital Gains 7.42

What are the capital gains taxable in the hands of Mr. R and Mr. Q?
Financial Cost Inflation
year Index
2001-02 100
2020-21 301

Computation of capital gains in the hands of Mr. R for the A.Y.2021-22


Particulars Rs.
1000 Original shares
Sale proceeds (1000 × Rs. 300) 3,00,000
Less : Indexed cost of acquisition [Rs. 75,000 × 301/100] 2,25,750
Long term capital gain (A) 74,250
200 Right shares
Sale proceeds (200 × Rs. 300) 60,000
Less : Cost of acquisition [Rs. 160 × 200] [Note 1] 32,000
Short term capital gain (B) 28,000

Sale of Right Entitlement


Sale proceeds (200 × Rs. 30) 6,000
Less : Cost of acquisition [Note 2] NIL
Short term capital gain (C) 6,000
Capital Gains (A+B+C) 108250
Note 1: Since the holding period of right shares is not more than 24 months, they are short
term capital assets and hence cost of acquisition will not be indexed.
Note 2: The cost of the rights renounced in favour of another person for a consideration is taken
to be nil. The consideration so received is taxed as short- term capital gains in full. The period
of holding is taken from the date of the rights offer to the date of the renouncement.
Computation of capital gains in the hands of Mr. Q for the A.Y.2021-22
Particulars Rs.

200 shares
Sale proceeds (200 × Rs. 280) 56,000
Less: Cost of acquisition [200 shares × (Rs. 30 + Rs. 160)] 38,000
[See Note below]
Short-term capital gain 18,000
Note: The cost of the rights is the amount paid to Mr. R as well as the amount paid to
the company. Since the holding period of these shares is not more than 24 months,
they are short term capital assets.

Unique Academy - 8007916622 CA Saumil Manglani - 9921051593


7. Capital Gains 7.43

ILLUSTRATION 9
X & sons, HUF, purchased a land for Rs. 1,20,000 in the P.Y. 2002-03. In the P.Y. 2006- 07, a
partition takes place when Mr. A, a coparcener, is allotted this plot valued at Rs. 1,50,000. In
P.Y. 2007-08, he had incurred expenses of Rs. 2,35,000 towards fencing of the plot. Mr. A sells
this plot of land for Rs. 15,00,000 in P.Y. 2020-22 after incurring expenses to the extent of Rs.
20,000. You are required to compute the capital gain for the A.Y.2021-22.
Financial Cost Inflation
year Index
2002-03 105
2006-07 122
2007-08 129
2019-20 301
Computation of taxable capital gains for the A.Y.2021-22
Particulars Rs. Rs.
Sale consideration 15,00,000
Less: Expenses incurred for transfer 20,000
14,80,000
Less: (i) Indexed cost of acquisition (Rs. (296066)
1,20,000 x 301 /122)
(ii) Indexed cost of improvement (Rs. (548333)
2,35,000 x 301/129)
Long term capital gains 635601
Note - As per the view expressed by Bombay High Court in CIT v. Manjula J. Shah 16
Taxman 42, in case the cost of acquisition of the capital asset in the hands of the
assessee is taken to be cost of such asset in the hands of the previous owner, the
indexation benefit would be available from the year in which the capital asset is
acquired by the previous owner. If this view is considered, the indexed cost of
acquisition would have to be calculated by considering the Cost Inflation Index of
F.Y.2002-03.

ILLUSTRATION 10
Mr. Dinesh received a vacant site as gift from his friend in November 2005. The site was
acquired by his friend for Rs. 7,00,000 in April 2002. Dinesh constructed a residential building
during the year 2010-11 in the said site for Rs. 15,00,000. He carried out some further
extension of the construction in the year 2012-13 for Rs. 5,00,000.
Dinesh sold the residential building for Rs. 55,00,000 in January 2021 but the State stamp
valuation authority adopted Rs. 65,00,000 as value for the purpose of stamp duty.
Compute his long-term capital gain, for the assessment year 2021-22 based on the above
information. The cost inflation indices are as follows:

Unique Academy - 8007916622 CA Saumil Manglani - 9921051593


7. Capital Gains 7.44
Financial Year Cost inflation index
2002-03 105
2005-06 117
2010-11 167
2012-13 200
2020-21 301

SOLUTION
Computation of long term capital gain of Mr. Dinesh for the A.Y. 2021-22
Particulars Rs. Rs.

Full value of consideration (Note 1) 65,00,000


Less: Indexed cost of acquisition-land (Rs. (20,06,667)
7,00,000 x 301 /105) (Note 2 & 3)
Indexed Cost of acquisition-building
(Rs. 15,00,000 x 301/ 167) (Note 3) ( 2703592 )
Indexed Cost of improvement-
building (Rs. 5,00,000x 301 /200) (752500)

Long-term capital gain 10,37,241

Notes:
1. As per section 50C, where the consideration received or accruing as a result of transfer
of a capital asset, being land or building or both, is less than the value adopted by the
Stamp Valuation Authority, such value adopted by the Stamp Valuation Authority shall be
deemed to be the full value of the consideration received or accruing as a result of such
transfer. Accordingly, full value of consideration will be Rs. 65 lakhs in this case since the
stamp duty value exceeds 110% of the sales consideration.
2. Since Dinesh has acquired the asset by way of gift, therefore, as per section 49(1), cost
of the asset to Dinesh shall be deemed to be cost for which the previous owner acquired
the asset i.e., Rs. 3,00,000, in this case.
3. Indexation benefit is available since both land and building are long-term capital assets.
However, as per the definition of indexed cost of acquisition under clause (iii) of
Explanation below section 48, indexation benefit for land will be available only from the
previous year in which Mr. Dinesh first held the land i.e., P.Y. 2005-06.
Alternative view: In the case of CIT v. Manjula J. Shah 16 Taxmann 42 (Bom.), the
Bombay High court held that indexation cost of acquisition in case of gifted asset can be
computed with reference to the year in which the previous owner first held the asset.
As per this view, the indexation cost of acquisition of land would be Rs. xxxx and long term
capital gain would be Rs. xxxx

Unique Academy - 8007916622 CA Saumil Manglani - 9921051593


7. Capital Gains 7.45

ILLUSTRATION 11
The following particulars, compute the taxable capital gains of Mr. D for A.Y.2021-22
Cost of Jewelry [Purchased in F.Y.2004-05] Rs. 4,52,000
Sale price of Jewelry sold in January 2021 Rs. 13,50,000
Expenses on transfer Rs. 7,000
Residential house purchased in March 2021 Rs. 5,00,000
The cost inflation Index are as follows:
Financial Year Cost Inflation
Index
2004-05 113
2020-21 301
SOLUTION
Computation of taxable capital gains for A.Y.2021-22
Particulars Rs.
Gross consideration 13,50,000
Less: Expenses on transfer (7,000)
Net consideration 13,43,000
Less: Indexed cost of acquisition (Rs. 4,52,000 × 301/113) (13,20,893)
Gain before exemption 22107
Less: Exemption under section 54F (Rs. (8230)
22,107× Rs. 5,00,000/ Rs. 13,43,000)
Taxable capital gains 13,877
ILLUSTRATION 12
Calculate the income-tax liability for the assessment year 2021-22 in the following cases:
Mr. A Mrs. B (age Mr. C Mr. D
62)
(age (age 81) (age 82)
45)
Status Resident Non- resident Resident Non- resident

Total 2,40,000 2,80,000 5,90,000 4,80,000


income
other than
long- term
capital gain
Long- 15,000 10,000 60,000 Nil
term from from sale of from sale of
capital sale of listed equity agricultural
gain vacant shares (STT land in rural
site paid on sale and area
purchase of
shares)

Unique Academy - 8007916622 CA Saumil Manglani - 9921051593


7. Capital Gains 7.46

SOLUTION
Computation of income-tax liability for the A.Y.2020-21
Particulars Mr. A Mrs. B (age Mr. C Mr. D
(age 62) (age 81) (age 82)
45)
Residential Resident Non-resident Resident Non- resident
Status
Applicable Rs. Rs. 2,50,000 Rs. 5,00,000 Rs. 2,50,000
basic 2,50,000
exemption limit
Asset sold Vacant Listed equity Rural -
site shares (STT paid agricultural
on both sale and land
purchase of
shares)
Long-term Rs. Rs. 10,000 Rs. 60,000 -
capital gain (on 15,000 [exempt u/s 112A (Exempt – not
sale of above [Taxable since it is less a capital
asset) @20% u/s than Rs. asset)
112] 1,00,000]
Other income Rs. Rs. 2,80,000 Rs. 5,90,000 Rs. 4,80,000
2,40,000

Tax liability

On LTCG (after Rs. 1,000 - - -


adjusting Basic
Exemption limit)
On Other income Nil Rs. 1,500 Rs. 18,000 Rs. 11,500

Less: Rebate u/s Rs. 1,000


87A

Add: Health and Nil Rs. 60 Rs. 720 Rs. 460


education cess
@4%
Total tax liability Nil Rs. 1,560 Rs. 18,720 Rs. 11,960

Notes:
1. Since Mrs. B and Mr. D are non-residents, they cannot avail the higher basic
exemption limit of Rs. 3,00,000 and Rs. 5,00,000 for persons over the age of 60
years and 80 years, respectively.
2. Since Mr. A is a resident whose total income does not exceed Rs.5 lakhs, he is
eligible for rebate of Rs. 12,500 or the actual tax payable, whichever is lower,
under section 87A.

Unique Academy - 8007916622 CA Saumil Manglani - 9921051593

You might also like